Download as pdf or txt
Download as pdf or txt
You are on page 1of 488

ASSURANCE SERVICES

1. Which of the following statements best describes assurance services?


A. Independent professional services that are intended to enhance the credibility of
information to meet the needs of an intended user.
B. Services designed to express an opinion on the fairness of historical financial statements
based on the results of an audit
C. The preparation of the financial statements or the collection, classification and
summarization of other financial information.
D. Services designed for the improvement of operations resulting in better outcomes.
Assurance engagements performed by professional accountants are intended to enhance the
credibility of information about subject matter by evaluating whether the subject matter
conforms in all material respects with suitable criteria, thereby improving the likelihood that the
information will meet the needs of the intended user.
Services performed by professional accountants that are not assurance engagements include the
following:

1. Agreed-upon procedures

2. Compilation of financial or other information

3. Preparation of tax returns where no conclusion is expressed, tax consulting

4. Management consulting

5. Other advisory services

2. Which of the following is not an assurance service?


A. examination of prospective financial information
B. Audit of historical financial statements
C. Review of financial statements
D. Compilation of financial information.

3. Which of the following professional services would be considered an assurance engagement?


A. A management consulting engagement to provide IT advice to
a client
B. An engagement to report on compliance with statutory
requirements.
C. An income tax engagement to prepare tax returns.
D. A compilation of financial statements from a client’s accounting record

4. Which of the following best describes the objective of an assurance engagement?


A. Improve the company outcomes
B. Compare the company’s information and policies with those of other entities
C. Enhance the credibility of information in order to improve the likelihood that the information
will meet the needs of intended user.
D. Assist in preparing the company’s financial statements.

The objective of an assurance engagement is for a professional accountant to evaluate or


measure a subject matter that is the responsibility of another party against identified suitable
criteria, and to express a conclusion that provides the internal users with a level of assurance
with the subject matter
An assurance engagement is intended to enhance the credibility of information about a subject
matter by evaluating whether the subject matter conforms in all material respects with suitable
criteria, thereby improving the likelihood that the information will meet the needs of the an
intended user

5. Assurance services differ from consulting services in that they:


I. Focus on providing advice
II. Involve monitoring of one party by another
A. I only C. Both I and II
B. II only D. Either 1 and II

Assurance services differ from consulting services in that they:

1. Focus on enhancing the credibility of information rather that providing advice

2. Typically involve situations in which one party wants to monitor another


Consulting services are usually two-party arrangements that focus on providing advice on how to
use the information for better outcomes.
6. How many separate parties are involved in an assurance engagement?

A. 2 C. 4

B. 3 D. 5

7. An assurance engagement should have which of the following

elements?

Subject Matter Criteria

A. Yes No

B. No Yes

C. Yes Yes

D. No No

An assurance engagement should have the following elements:

1. A three-party relationship involving:

A. a professional accountant

B. a responsible party

C. intended users

2. A subject matter

3. Suitable criteria

4. Sufficient appropriate evidence

5. An assurance report

8. The Philippines Framework for Assurance Engagements


A. Contains basic principles, essential procedures and related guidance for the performance of
assurance engagements.
B. Define and describes the elements and objectives of an assurance engagements and identifies
engagements to which PSAs, PSREs and PSAEs apply.
C. Provides a frame of reference of CPAs in public practice when performing audits, review and
compilations of historical financial information.
D. Establishes standards and provides procedural requirements for the performance of assurance
engagements.

The Framework defines and describes the elements and objective of an assurance engagement
and identifies engagements to which Philippine Standards of Auditing (PSAs), Philippines
Standards on Review Engagements (PSREs) and Philippine Standards on Assurance
Engagements (PSAEs) apply.
The Framework does not itself establish standards or provide procedural requirements for the
performance of assurance engagement. PSAs, PSREs and PSAEs contain basic principles,
essential procedures, and related guidance, consistent with the concepts in the Framework, for
the p [performance of assurance engagements.

9. CPAs in public practice who perform assurance engagements are governed by the following,
except:
A. Philippine Framework for Assurance Engagements
B. Code of Ethics for Professional Accountants in the
Philippines
C. Philippine Standards in Related Services
D. Philippine Standards in Quality Control

10. in an assurance engagements, the responsible party and the internal users:
A. Should be from different entities
B. Should be from the same entity
C. May be from the same entity or different entities
D. Are both responsible for determining the nature, timing and extent of the procedures to be
performed?

According to the Philippine Framework for Assurance Engagements, the responsible party and
the intended users may be from different entities or the same entity.
Answer D is incorrect because the practitioner is responsible for determining the nature, timing
and extent of procedures to be performed.

11. The subject matter of an assurance engagement may include:


Financial Information Internal Control Compliance with
Regulation

A. Yes Yes Yes

B. No No No

C. Yes No Yes

D. No Yes No

According to the Philippine Framework for Assurance Engagement, an assurance engagement’s


subject matter may include the following:

• Financial performance or conditions such as historical or perspective financial position,


financial performance and cash flows
• Non-financial performance or conditions, for example, performance of an entity
• Physical characteristics, for example, an entities internal control or IT system
• Behavior, such as corporate governance, compliance with regulation, human resource
practices.

12. For assurance engagement regarding historical financial information, reasonable


assurance engagements are called:
A. Audit C. Compilation
B. Review D. Examination

13. When performing as assurance service, professional accountants use standards or


benchmarks to evaluate or measure the subject matter of an assurance engagement. These are
referred to in the Framework as
A. Criteria C. Conditions
B. Norms D. Gauges

Criteria are the standards or benchmarks used to evaluate or measure the subject matter of an
assurance engagement. These are important because they establish and inform the intended users
of the basis against which the subject matter has been evaluated or measured in forming the
conclusion.

14. The criteria against which the subject matter of the assurance engagements is to be
evaluated or measured should process which of the following characteristics?

Relevant Concise Neutral


A. Yes No Yes

B. No Yes No

C. Yes No No

D. No Yes Yes

15. Relevant criteria contribute to conclusions that are


A. Free from bias
B. Clear and comprehensive
C. Subject to different interpretations
D. Useful for decision making

According to the Philippine Framework for Assurance Engagements, suitable criteria should
have the following characteristics:
1. Relevance

• Relevant criteria contribute to conclusions that assist decision-making by the intended


user

2. Completeness

• Criteria are sufficiently complete when relevant factors that could affect the conclusions
in the context of the engagement are not omitted.
• Complete criteria include, where relevant, bench-marks for presentation and disclosure
3. Reliability

• Reliable criteria allow reasonable consistent evaluation or measurement of the subject


matter including where relevant, presentation disclosure, when used in similar
circumstance by similar qualified practitioners.
4. Neutrality

• Neutral criteria contribute to conclusions that are free from bias.


5. Understandability

• Understandable criteria contribute to conclusions that are clear, comprehensive, and not
subject to significantly different interpretations.
16. Criteria that are embodied in laws or regulations or issued by authorized or recognized body
of experts that follow a transparent due process are called
A. Suitable Criteria
B. Established Criteria
C. Specifically developed criteria
D. General criteria

The Framework states that criteria can either be established or specifically developed.
Established criteria are those that are embodied in laws or regulations, or issued by authorized or
recognized bodies. Specifically developed criteria are those designed for the purpose of the
engagement.
17. in an assurance engagements, the person or persons, either as individuals or representative
of an entity, responsible for the subject matter is the A. Intended user
B. Responsible party
C. Professional Accountant
D. Client

The responsible party is the one responsible for the subject matter of an assurance engagement.
For example, an entity’s management is responsible for the preparation and presentation of
financial statements or the establishment and implementation of internal control.
The responsible party may or may not be the party who engages the professional accountant.

18. In an assurance engagement, the person or class of persons for whom the professional
account prepares the report for a specific user or purpose is the
A. Intended user
B. Responsible party
C. Management
D. Client

The intended user is the person or a class of persons for whom the professional accountant
prepares the report for a specific use or purpose.
19. In an assurance engagement, the outcome of the evaluation or measurement of a subject
matter against criteria is called
A. Subject matter information
B. Subject matter
C. Assurance
D. Conclusion
The term “subject matter information” is used in the Framework for Assurance Engagements to
mean the outcome of the evaluation or measurement of a subject matter.
According to the Framework, it is the subject matter information about which the practitioner
gathers sufficient appropriate evidences to provide a reasonable basis for expressing a conclusion
in an assurance report.
20. In some assurance engagements, the evaluation or measurement of the subject matter is
performed by the responsible party, and the subject matter information is in the form of an
assertion by the responsible party that is made available to the untended users. It is called:
A. Direct reporting engagements
B. Assertion-based engagements
C. Non-assurance engagements
D. Recurring engagements

21. The following are characteristics of “direct reporting” assurance engagements, except
A. The subject matter information is in the form of an assertion by the responsible party that
is made available to the intended users.
B. The subject matter information is provided to the intended users in the assurance report
C. The practitioner either directly performs the evaluation or measurement of the subject
matter our obtains a representation from the responsible party that has performed the evaluation
or measurement
D. The representation of the responsible party that has performed the evaluation or
measurement of the subject matter is not available to the intended users.

22. What type of assurance engagement is involved when the practitioner expresses a positive
form of conclusion?
A. Limited assurance engagement
B. Positive assurance engagements
C. Reasonable assurance engagements
D. Absolute assurance engagements

According to the Framework, the objective of the reasonable assurance engagement is the
reduction in the assurance engagement risk to an acceptable low level in the circumstances
of the engagements the basis for a positive form of expression of the practitioner’s
conclusion.

23. What type of assurance engagements is involved when the practitioner expresses a negative
form of conclusion?
A. Reasonable Assurance engagements
B. Negative assurance engagements
C. Assertion-based assurance engagements
D. Limited assurance engagements

According to the Framework, the objective of the limited assurance engagement is the reduction
in the assurance engagement risk to the level that is acceptable in the circumstances of the
engagement, but where the risk is greater than for reasonable assurance engagement, as the basis
for a negative form of expression of the practitioner’s conclusion.

24. A practitioner’s assurance report contains the following conclusions:


“Based on our work described in this report, noting has come to our attention that causes us to
believe that internal control is not effective, in all material aspects based on ABC criteria”

A. Limited assurance engagements


B. Reasonable assurance engagements
C. Negative assurance engagements
D. Positive assurance engagements

25. In assertion-based engagements, the evaluation or measurement of the subject matter against
criteria is performed by the
A. Intended users C. Practitioner
B. Responsible party D. AASC
In assertion-based assurance engagements, the evaluation or measurement of the subject matter
against criteria is performed by the responsible party and the subject matter in formation
(outcome) is in the form of an assertion by the responsible party that is made available to the
intended users.

26. The following statements relate to the three parties involved in an assurance engagement. Which
is correct?
A. The responsible party and the intended users should be from different entities.
B. A practitioner should decline a proposed assurance engagement when the subject matter requires
specialized skills and knowledge beyond those ordinarily possessed by the
practitioner.
C. A responsible party is the person who is responsible for the subject matter or the subject matter
information
D. The responsible party, not intended users, determines the nature of the procedures to be
performed

According to the Framework, the responsible party is the person (or persons) who:

• In a direct reporting engagement, is the repos bile for the subject matter
• In an assertion-based engagement, is responsible for the subject matter information (the
assertion) and may be responsible for the subject matter.

27. A proposed assurance engagements can be accepted when the practitioner’s preliminary
knowledge about the engagement circumstances indicates that relevant ethical requirements will
be satisfied and:
I. The subject matter of the engagement is appropriate
II. The criteria to be used are suitable and are available to the intended users
III. The practitioner has access to sufficient appropriate evidence to support the conclusion.
IV. The conclusion is to be contained in a written report
V. There is a rational purpose for the engagement

A. I, II, III
B. I, II, IV, V
C. I, II, III, IV
D. I, II, III, IV, V

A proposed assurance engagement can be accepted when the practitioner’s preliminary


knowledge of the engagement circumstances indicates that relevant ethical requirements such as
independence and professional competence will be met and the engagement exhibits all of the
characteristics described in statements I to V.

28. A practitioner should accept an assurance engagements only if


A. The subject matter is in the form of financial information
B. The criteria to be used are not available to the intended
users.
C. The practitioner’s conclusion is to be contained in a written report
D. The subject matter is the responsibility of either the intended users or the practitioner.

29. Which of the following statements is true concerning evidence in assurance engagement?
A. Sufficiency is the measure of the quantity of evidence
B. Appropriateness is the measure of the quantity of evidence, that is, its reliability and
persuasiveness
C. The reliability of evidence is influenced not by its nature but by its source
D. Obtaining more evidence may compensate for its poor quality

Sufficiency is the measure of the quantity if evidence. The quantity of evidence needed is
affected by the quality of such evidence (the higher the quality, the less may be required.)
However, merely obtain more evidence may not compensate for its poor quality.
Appropriateness is the measure of the quality of evidence, that is, its relevance and the reliability.
The reliability of evidences influent by its source and by its nature.

30. Assurance engagement risk is the risk


A. That the practitioner expresses an inappropriate conclusion when the subject matter information
is materially misstated
B. Of expressing an inappropriate conclusion when the subject matter information is not materially
misstated
C. Through the loss from litigation, adverse publicity or other events rising in connection with a
subject matter reported on
D. Of expressing an inappropriate conclusion when the subject matter of information is either
materially misstated or not materially misstated

31. The following are components of assurance engagement risk,


except
A. Inherent risk C. Detection risk
B. Control risk D. Business risk

Assurance engagement risk has the following components:


1. The risk that the subject matter information is materially misstated. This consist of:

A. Inherent Risk- the susceptibility of the subject matter information to a material


misstatement, assuming that there are no related controls.

B. Control Risk- the risk that a misstatement will occur will not be prevented, or detected
and corrected, on a timely basis by related internal controls.
2. Detection Risk- the risk that the practitioner will not detect a material misstatement that exists.

32. An unqualified conclusion is not appropriate for either reasonable or limited assurance
engagement when
A. Circumstances prevent the practitioner from obtaining evidence required to reduce
assurance engagement risk to the appropriate level
B. The responsible party or the engaging party imposes the restriction that prevents the
practitioner from obtaining evidence required to reduce assurance engagement risk to the
appropriate level C. Both A and B
D. Neither A nor B

According to the Framework, an unqualified conclusion is not appropriate for either type of
assurance engagement in the case of a material limitation on the scope of the practitioner’s work,
whether imposed by the engagement circumstances or the engaging party or the responsible
party.

33. The following statements relate to the performance of an assurance engagement other than an
audit or review of historical financial information covered by PSAs and PSREs. Which is
incorrect?
A. Those persons who are to perform the engagement should collectively possess the necessary
professional competence
B. The practitioner is precluded from using the work of persons from other professional disciplines
C. The practitioner should consider materiality and assurance engagement risk when planning and
performing an assurance
engagement
D. The assurance report should be in writing and should contain a clear expressions of the
practitioner’s conclusion about the subject matter information

The subject matter and related criteria of some assurance engagements may include aspects
inquiring specialized knowledge and skills in the accumulation and evaluation of evidence. The
standards allow a practitioner to engage persons from other professional disciplines, referred to
as experts.

34. Reducing assurance engagement risk to zero is very rarely attainable or cost beneficial as a
result of the following factors except
A. The use of selective testing
B. The fact that much of the evidence available to the practitioner is persuasive rather that
conclusive
C. The practitioner may not have required assurance knowledge and skills to gather and
evaluate evidence
D. The use of judgment in gathering and evaluating evidence and forming conclusions based on
that evidence

35. After an accepting an assurance engagement, a practitioner is not allowed to change the
engagement to a non-assurance engagement, or from a reasonable assurance engagement to a
limited assurance engagement except when there is reasonable justification for the change.
Which of the following ordinarily will justify a request for the change in the engagement?
I. A change in circumstances that affects the intended users’ requirements
II. A misunderstanding concerning the nature of the engagement

A. I only C. Both I and II


B. II only D. Neither I nor II
B. AUDITING AND RELATED SERVICES
36. Which of the following standards are to be applied, as appropriate, in the audit or historical
financial information?
A. PSREs c. PSRSs
B. PSAEs D. PSAs

The Philippine Standards of Auditing (PSAs) are to be applied, as appropriate, in the audit of
historical financial information.

37. Which of the following standards are to be applied to compilation engagement, engagements to
apply agreed-upon procedures of information, and other related services engagements as
specified by the AASC?

A. PSRSs C. PSAEs
B. PSAs D. PSREs

The Philippine Standards on Related Services (PSRSs) are to be applied to compilation


engagement, engagements to apply agreed-upon procedures of information, and other related
services engagements as specified by the AASC

38. The Philippine Standards on Review Engagements (PSREs) are to be applied in


A. The audit of historical financial information
B. Assurance engagements dealing with the subject matters other that the historical financial
information
C. The review of historical financial information
D. The review of both historical and prospective financial
information

39. PSRE 2400 (Engagement to Review Financial Statements), as amended by the AASC in
February 2008, applies to
A. Reviews of any historical financial information of an audit client
B. Reviews of any historical financial information by a
practitioner other than the entity’s auditor.
C. Reviews of historical financial or other information by a
practitioner other than the entity’s auditor
D. Reviews of historical financial or other information of an audit client

PSRE 2400 (Engagement to Review Financial Statements), and PSRE 2410 (Review on Interim
Financial Information Performed by the Independent Auditor of the Entity) were amended by the
AASC in February 2008. The objective of the amendment made is to clarify to which
engagements each of the standards is to be applied.
The effect of the amendments is summarized as follows:

• PSRE 2400 applies to reviews of historical financial information by a practitioner other


than the entity’s auditor.
• PSRE 2410 applies to reviews of historical financial information by the entity’s auditor.
• Reviews of other historical information fall under PSAs 3000 (Revised), Assurance
Engagement other than Audits and Reviews of Historical Financial Information.
40. The Philippine Standards on Assurance Engagements (PSAEs) are to be applied in
A. Assurance engagement dealing with the subject matters other than historical financial
information
B. Compilation engagement and agreements to apply agreed-upon procedures to information
C. The audit or review of historical financial information
D. Assurance engagement dealing with historical financial information

41. The Philippine Standards on Quality Control (PSQSs) are to be applied to


A. Assurance engagements only
B. Review engagements only
C. Compilation and review engagements only
D. All services that fall under the AASC’s engagement standards

PSAs, PSREs, PSAEs, and PSRSs are collectively referred to as the AASC’s Engagement
Standards. PSQSs are to be applied for all services under these Engagement Standards.

42. These statements are issued by the AASC to provide interpretive guidance and practical
assistance to auditors in the implementations of PSAs and to promote good practice

A. PREPSs C.
PAEPs
D.PRPS
B. PAPSs Ps

The AASC issues Practical Statements to provide interpretative guidance and practical assistance
to practitioners in implementing the Engagement Standards and to promote good practice. The
following are the AASC engagement standards and the related Practice Statements.

Engagement Standards Practice Statements

1. Philippine Standards Philippine Auditing Practice on Auditing (PSAs)

Statements (PAPSs)

2. Philippine Standards on Philippine Review Engagement


Review Engagements (PSREs) Practice Statements (PREPSs)

3. Philippine Standards on Philippine Assurance Engagements


Assurance Engagements (PAEPSs) Practice Statements (PAEPSs)

4. Philippine Standards on Philippine Related Services


Related Services (PSRSs) Practice Statements (PRSPSs)

43. The auditor’s satisfaction as to reliability of an assertion being made by one party for use by
another party is called
A. Opinion C. Examination
B. Assurance D. Verification

The term “assurance means” means the practitioner’s satisfaction as to the reliability of an
assertion being made by one party for use by another party. The procedures performed and the
evidence collected by the practitioner.

44. What level of assurance is provided by the auditor in an audit engagement?


A. Absolute C. Moderate
B. High, but not absolute D. No assurance
In an audit engagement, the auditor provides a high, but not absolute level of assurance that the
financial statements are free of material misstatement. That expressed positively in the audit
report as the reasonable assurance.

45. What level of assurance is provided by the practitioner in a review engagement?


A. No assurance C. Reasonable
B. High, but not absolute D. Moderate

In a review engagement, the practitioner provides a moderate level of assurance that the
information subject to review is free of material misstatements. This is expressed in the form of a
negative (also limited) assurance.

46. For the purpose of expressing negative assurance in the review report, the practitioner should
obtain sufficient appropriate evidence primarily through
A. Inquiry and confirmation
B. Analytical procedures and substantive tests of details of transactions and account balances
C. Confirmation and test of controls
D. Inquiry and analytical procedures

47. In reviewing a company’s financial statements, a practitioner is required to


A. Send bank confirmations
B. Obtain knowledge of the client’s industry
C. Obtain a signed engagement letter from the client
D. Observe client’s physical inventory

48. In reviewing engagement, the practitioner performs which of the following?

Obtain an understanding Test of internal Test of


Of Internal Control controls Transactions

A. Yes Yes No

B. Yes No Yes
C. No Yes Yes

D. No No No

49. A practitioner’s review of an entity’s financial statements does not provide assurance that
he/she will become aware of all significant matters that would be disclosed in an audit. However,
if the practitioner has become aware that information coming to his/her attention may be
materially misstated, the practitioner should
A. Carry out additional or more extensive procedures as are necessary to achieve limited
assurance
B. Withdraw immediately from the engagement
C. Perform a complete audit and issue a modified auditor’s
report
D. Downgrade the engagement to a compilation and issue the appropriate report

According to PSRE 2400, if the practitioner has reason to believe that the information subject to
review may be materially misstated, he/she should carry out additional or more extensive
procedures as are necessary to be able to express negative assurance or to confirm that a
modified report is required.

50. The following statements relate to a review of financial statements. Which is incorrect?
A. The objective of a review of financial statements is to enable a practitioner to state whether
anything has come to the practitioner’s attention that causes the practitioner to believe that the
financial statements are not prepared in accordance with an identified financial reporting
framework
B. A review comprises inquiry and analytical procedures which are designed to review the
reliability of an assertion that is the responsibility of one party for use by another party
C. A review ordinarily involves an assessment of accounting and internal control systems
D. The level of assurance provided in review report is less than that given in an audit report

While a review involves the application of audit skills and techniques and the gathering of
evidence, it does not ordinarily involve an assessment of accounting and internal control
systems, test of recorded and of responses to inquiries by obtaining corroborating evidence
through inspection, observation, confirmation and computations which are procedures ordinarily
performed during an audit.

51. The following statement relate to a review of an interim financial information performed by
the entity’s independent auditor. Which is incorrect?
A. Similar to a financial statement audit, a review of interim financial information is
designed to obtain reasonable assurance that the interim financial information is free from
material misstatement
B. A review of interim financial information does not provide a basis for expressing an
opinion whether the financial information is presented fairly, in all material respects, in
accordance with an applicable financial reporting framework.
C. In a review of interim financial information. The auditor should have an understanding of
the entity and its environment, including its internal control
D. A review of interim financial information may bring significant matters affecting the
interim financial information to the auditor’s attention, but it does not provide all of the evidence
that would be required in an audit

In contrast to an audit, a review of interim financial information is to enable the auditor to


express a conclusion whether, on the basis of the review, anything has come to the auditor’s
attention that causes the auditor to believe that the interim financial information is not prepared,
in all material aspects, in accordance with an applicable financial reporting framework.

52. In a compilation engagement, the accountant is engaged to use accounting expertise to


collect, classify, and summarize financial information. What type of assurance is provided by the
accountant when he/she performs this engagement?
A. Positive assurance
B. Negative assurance
C. No assurance
D. Limited assurance

A compilation engagement ordinarily entails reducing detailed data to a manageable and


understandable form without a requirement to test the assertions underlying that information. The
procedures employed are not designed and do not enable the accountant to express any assurance
on the financial information.
However, the accountant’s involvement provides some benefit to users of complied financial
information because the work has been performed with due professional skill and care.
53. Which of the following statements concerning compilation engagement is incorrect?
A. In a compilation engagement, the accountant us engaged to use accounting expertise as
opposed to auditing expertise to collect, classify and summarize financial information
B. The procedures employed in the compilation engagement enable the accountant to
express a moderate level of assurance in a compiled financial information
C. Users of the compiled financial information derived some benefit as a result of the
accountant’s involvement because the service has been performed with due professional skills
and care.
D. A compilation engagement ordinarily engaged reducing detailed data to a manageable
and understandable form without a requirement to test the assertions underlying that
information.

The procedures employed in a compilation engagement are not designed and do not enable the
accountant to express any assurance in the financial information.

54. When performing a compilation engagement, the accountant is required to


A. Assess internal controls
B. Make inquiries of management to assess the reliability and completeness of the information
provided
C. Verify matters and explanations
D. Obtain a general knowledge of the business and operations of the entity

According to PSRS 4410 (Engagements to obtain a Compile Financial Information), “The


accountant should obtain a general knowledge of the business and operation of the entity and
should be familiar with the accounting principles and practices of the industry in which the entity
operates and with the form and content of the financial information that is appropriate in the
circumstances.”
The standards further provides that, “The accountant ordinarily obtains knowledge of these
smatters through experience with the entity or the inquiry of the entity’s personnel.”
PSRS 4410, par. 13, provides that the accountant is not ordinarily required to:

a) Make inquiries to the management to assess the reliability and completeness of the
information provided;
b) Assess internal controls;
c) Verify any matters; or
d) Verify any explanations.

55. Each page of the financial information complied by the accountant should include the
following reference, except
A. “Unaudited”
B. “Compiled without Audit or Review”
C. “Refer to Compilation Report”
D. “Compiled, Negative Assurance Expressed”

According to PSRS 4410 (Engagements to compile Financial Information), the financial


information compiled by the accountant should contain a reference such as:

• Unaudited
• Compiled without Audit or Review
• Refer to Compilation Report on each page of the financial information or on the front of
the complete set of financial statements.

56. An accountant who performs a compilation engagement


A. Should read the compiled information and consider whether it appears to be appropriate in form
and free from obvious material misstatements.
B. Should use his/her auditing expertise in testing the assertions underlying the compiled financial
information
C. Include his/her report a listing of the specific procedures performed
D. Need not obtain an acknowledgement form the management of its responsibility of its
appropriate presentation of the financial information

57. What assurance is provided by the author in the agreedupon procedures engagement?
A. Reasonable C. Moderate
B. Absolute D. No assurance

In an agreed-upon procedures engagement, the auditor simply provides a report of the factual
findings and expressed no assurance in his/her report. Users of the report make an assessment of
the procedures and findings reported by the auditor and draw their own conclusions from the
auditor’s work.
58. In an engagement to perform agreed- upon procedures, an auditor is engage to
A. Carry out those procedures of an audit nature to which the auditor, the entity and any appropriate
third party have agreed and to report factual findings.
B. Use accounting expertise as opposed to auditing expertise to collect, summarize and classify
financial information
C. Provide a moderate level of assurance that the information is free from any material
misstatement
D. Provide a high, but not absolute, level of assurance that the information is free of material
misstatement

In an engagement to perform agreed- upon procedures, an auditor is engage to carry out those
procedures, an auditor is engaged to carry out those procedures of an audit nature which the
auditor and the entity and any appropriate parties have agreed and to report factual findings.
The report contains no assurance and is restricted to those parties that have agreed to the
procedures to be performed, since others, unaware of the reasons for other procedures, may
misinterpret the results. Users of the report must form their own conclusions from the auditor’s
work.

59. A report may be based upon applying agreed-upon procedures to specified elements,
accounts or items of a financial statement. The users of the report should participate in
establishing the procedure to be performed. If the auditor cannot discuss the procedures with all
the parties who will receive the report, he/she may
I. Discuss the procedures to be applied with appropriate representatives of the parties
involved.
II. Review relevant correspondence from the parties involved.
III. Distribute a draft of the type of report that will be issued to the parties involved.

A. I and II only C. II and III


only
B. I and III only D. I, II and III

PSRS 4400 (Engagements on Agreed- upon Procedures) states, “in certain circumstances for
example, when the procedures have been agreed to between the regulator, industry
representatives and representatives of the accounting profession, the auditor may not be able to
discuss the procedures with all the parties who will receive the report . In such cases, the auditor
may consider, for example, discussing the procedures to be applied with appropriate
representatives of the parties involved, reviewing relevant correspondence from such parties or
sending them a draft of the type of report that will be issued.”

60. An auditor may accept an engagement to perform specifies procedures on the specific subject
matter of specified elements, accounts or items of a financial statement if
A. The report does not list the procedures performed
B. The financial statements are prepared in accordance with a special purpose framework
C. Use of the report is restricted
D. The auditor is also the entity’s continuing auditor
PSRS 4400 states that the report is restricted to those parties that have agreed to the procedures
to be performed, since others, unaware of the reasons for other procedures, may misinterpret the
results.
Answer A is incorrect because the report should include a listing of the specific procedures
performed.
Answer B is incorrect because the financial statements need not be prepared in accordance with a
special purpose
framework.
Answer D is incorrect because the auditor need not be the entity’s continuing auditor.

61. Reports on agreed- upon procedure are intended to be distributed


A. To only involved parties, who are aware of the reasons for the procedures
B. Only to the stockholders of the entity
C. To any part to whom the client wishes
D. Only to the entity’s management

62. An engagement to perform an agreed-upon procedures may involve the auditor in performing
certain procedures
concerning
I. Individual items of the financial data
II. A single financial statement
III. A complete set of financial statement

A. I and II only C. I and III only


B. II and III only D. I, II and III

63. The report on an agreed0upon procedures engagement should


contain
A. Identification of the purpose for which the agreed-upon procedures were performed
B. An expression of positive assurance based on the specific procedures performed
C. A statement that the auditor is independent of the entity
D. A general description of the procedures performed

According to PSRS 4400, the report on an agreed- upon procedures engagement needs to be
described the purpose and the agreed-upon procedures of the engagement in sufficient detail to
enable the users of the report to understand the nature and extent of the work performed.
Answer B is incorrect because the report should include a statement that the procedures
performed do not constitute either an audit or a review and, as such, no assurance is
expressed.
Answer C is incorrect because the report should contain the statement that the auditor is not
independent of the entity if such is the case.
Answer D is incorrect because the report should include a listing of the specific procedures
performed.

64. Which of the following engagement does not require compliance with independence
requirements?
A. Compilation of financial information
B. Review of financial statements
C. Examination of prospective financial information
D. Audit of financial statement

Independent is not a requirement for compilation and agreeupon procedures engagement.


However where the accountant or auditor is not independent, a statement to that effect would be
made in the report.
65. Which of the following services, if any, may a practitioner who is not independent provide?
A. Compilations but not reviews
B. Reviews but not compilations
C. Reviews but not financial statement audits
D. Agreed-upon procedures but not compilations

66. A practitioner is associated with financial information when


I. The practitioner attaches a report to that financial information
II. The practitioner consents to the use of his/her name in a professional connection

A. I only C. Either I or II
B. II only D. Neither I or II
CHAPTER 2
THE ACCOUNTANCY PROFESSION

A. Philippine Accountancy Act of 2004

1. Republic Act 9298 is known as the


A. Revised Accountancy Law
B. Revised Accountancy Act
C. Philippine Accountancy Act of 2004
D. Philippine Accountancy Law of 2004

2. Which of the following is not an objective of the Philippine Accountancy Act of 2004?
A. The standardization and regulation of accounting education.
B. The examination for registration of certified public
accountants.
C. The supervision, control, and regulation of the practice of accountancy in the Philippines.
D. The development and improvement of accounting standards that will be generally
accepted in the Philippines
3. The practice of Accountancy includes

I. Practice of Public Accountancy


II. Practice in Commerce and Industry
III. Practice in Education/Academe
A. I and II only
B. II and III only
C. I and III only
D. I,II, and III

Section 4 of RA 9298 provides that the practice of accountancy shall include, but not limited
to, the following:
a) Practice of Public Accountancy
b) Practice in Commerce and Industry
c) Practice in Education/Academe
d) Practice in the Government

4. A CPA is in public accounting practice when he/she


A. Represents his/her employer before government agencies on tax and other matters related
to accounting.
B. Represents his/her clients before government agencies on tax and other matters related to
accounting.
C. Teaches accounting, auditing, management advisory services, accounting aspect of
finance, business law, taxation, and other technically related subjects.
D. Holds, or is appointed to, a position in an accounting professional group in government or
in a government-owned and/or controlled corporation where decision making requires
professional knowledge in the science of accounting.

5. Section 4 of the Rules and Regulations Implementing RA 9298 (IRR) provides that any
position in any business or company in the private sector which requires supervising the
recording of financial transactions, preparation of financial statements, coordinating with
the external auditors of the audit of such financial statements, and other related functions
should be occupied by a duly registered CPA. It provides further that the business or
company where such position exists has a
A. Paid-up capital of at least P5,000,000 and/or an annual revenue of at least P10,000,000.
B. Paid-up capital of at least P10,000,000 and/or an annual revenue of at least P5,000,000.
C. Paid-up capital and/or an annual revenue of at least P10,000,000.
D. Paid-up capital and/or an annual revenue of at least P5,000,000.

Section 4 of the IRR states that:


a) The business or company where such position exists has a paid-up capital of at least P5
million and/or annual revenue of at least P10 million.
b) The provision shall apply only to persons to be employed after the effectivity of the IRR.
c) The provision shall not result to deprivation of the employment of incumbents to the
position.
6. Which of the following statements concerning the practice of accountancy in the
academe/education is incorrect?
A. Members of the Integrated Bar of the Philippines are prohibited from teaching business
law and taxation subjects.
B. Members of the Integrated Bar of the Philippines may be allowed to teach business law
and taxation subjects.
C. The position of either the Dean or the Department Chairman or its equivalent that
supervises the Bachelor of Science in Accountancy program of an educational
institutional is deemed to be in practice of accountancy in the
academe/education.
D. The position of either the Dean or the Department Chairman or its equivalent that
supervises the Bachelor of Science in Accountancy program of an educational institution
must be occupied only by a duly registered CPA.

7. A CPA is in the practice of accountancy in commerce and industry when he/she

A B C D
1. Is involved in decision making requiring
professional knowledge
in the science of accounting Yes No No Ye
2. Represents his/her employer before s
government agencies on tax and other
matters related to
accounting Yes No Yes N
3. Renders professional services as a CPA to o
more than one client on
a fee basis. No Yes No Ye
s

Section 4 (b) of the IRR states that Practice in Commerce and


Industry- shall constitute in a person,
i. Involved in decision making requiring professional knowledge in the science of
accounting, as well as the accounting aspects of finance and taxation, or
ii. When he/she represents his/her employer before government agencies on tax and
other matters related to accounting, or
iii. When such employment or position requires that the holder thereof must be a
certified public accountant. A CPA is in public accountancy when he/she renders
his/her professional services as a CPA to more than one client on a fee basis.

8. Section 5 of the Accountancy Act of 2004 states that the Board of Accountancy shall be
composed of a chairman and
A. 2 members B. 4 members C. 6
members
D. 8 members

9. The members of the Board of Accountancy shall be appointed by the


A. Philippine Institute of CPAs (PICPA)
B. Professional Regulation Commission (PRC0
C. President of the Philippines
D. Association of CPAs in Public Practice (ACAPP)

10. If the PICPA fails to submit to the PRC its own nominees within 60 days prior to the
expiry of the term of an incumbent chairman or member of the Board of Accountancy
(BOA), the PRC in consultation with the BOA shall submit to the president a list of how
may nominees for each vacant position?
A. 2
B. 3
C. 4
D. 5

11. A member of the BOA shall, at the time of his/her appointment, possess which of the
following qualifications? A. Must be a natural-born citizen of the Philippines.
B. Must be a Filipino citizen.
C. Must be a Filipino citizen and a resident of the
Philippines.
D. Must be a natural-born citizen and a resident of the Philippines.

According to Section 6 of the Accountancy Act of 2004, am member of the Board shall, at
the time of his/her appointment, possess the following qualifications:
a) Must be a natural-born citizen and a resident of the
Philippines;
b) Must be a duly registered Certified Public Accountant with at least ten (10) years of
work experience in any scope of practice of accountancy.
c) Must be of good moral character and must not have been convicted of crimes
involving moral turpitude; and
d) Must not have any pecuniary interest, directly or indirectly, in any school, college,
university, or institution conferring an academic degree necessary for admission to the
practice of accountancy or where review classes in preparation for the licensure
examination are being offered or conducted, nor shall he/she be a member of the
faculty or administration thereof at the time of his/her appointment to the Board.

12. The Chairman and members of the Board of Accountancy shall hold office for a term of
A. 2 years B. 3 years C. 4 years
D. 5 years

13. According to Section 7 of the IRR, no person who has served two successive complete
terms as chairman or member of the Board of Accountancy shall be eligible for
reappointment as
chairman or member until the lapse of
A. 1 year
B. 2 years C. 3 years
D. 4 years

14. According to Section 7 of the IRR, no person shall serve in the Board of Accountancy for
more than
A. 3 years
B. 5 years
C. 10 years
D. 12 years

15. Which of the following is a function of the Board of Accountancy?


I. To prescribed and adopt the rules and regulations necessary for carrying out the
provisions of the Philippine Accountancy Act of 2004.
II. To supervise the regulation, licensure, and practice of accountancy in the Philippines.
III. To issue, suspend, revoke, or reinstate the certificate of registration for the practice of
the accountancy profession.

A. I and II only
B. II and III only
C. I and III only
D. I, II, and III

Section 9 of the Accountancy Act of 2004 states that the Board of Accountancy shall exercise
the following specific powers, functions and responsibilities:

a) To prescribe and adopt rules and regulations necessary for carrying out the provisions
of the Act;
b) To supervise the registration, licensure and practice of accountancy in the Philippines;
c) To administer oaths in connection with the administration of the Act;
d) To issue, suspend, revoke, or reinstate the Certificate of Registration for the practice
of the accountancy profession;
e) To adopt an official seal of the Board;
f) To prescribe and/or adopt a Code of Ethics for the practice of accountancy;
g) To monitor the conditions affecting the practice of accountancy and adopt such
measures, rules and regulations and best practices as may be deemed proper for the
enhancement and maintenance of high professional, ethical, accounting and auditing
standards.
h) To conduct an oversight into the quality of audits of financial statements through a
review of the quality control measures instituted by auditors in order to ensure
compliance with the accounting and auditing standards and practices;
i) To investigate violations of the Act and the IRR;
j) To make such investigations as it deems necessary to determine whether any person
has violated any provision of the Act, any accounting or auditing standards or rules
duly promulgated by the Board as part of the rules governing the practice of
accountancy;
k) To issue a cease or desist order to any person, association, partnership or corporation
engaged in violation of any provision of the Act, any accounting or auditing standards
or rules duly promulgated by the Board as part of the rules governing the practice of
accountancy in the Philippines;
l) To punish for contempt of the Board, both direct and indirect, in accordance with the
pertinent provisions of and penalties prescribed by the Rules of the Court;
m) To prepare, adopt, issue or amend the syllabi of the subjects for examinations in
consultation with the academe, determine and prepare questions for the licensure
examination which shall directly be within the scope of the syllabi of the subjects for
examinations as well as administer, correct and release the results of the licensure
examinations;
n) To ensure, in coordination with the Commission on Higher Education (CHED) or
other authorized government offices that all higher educational instruction and
offering of accountancy comply with the policies, standards and requirements of the
course prescribed by CHED or other authorized government offices in the areas of
curriculum, faculty, library, and facilities; and
o) To exercise such other powers as may be provided by law as well as those which may
be implied from, which are necessary or incidental tot the carrying out of, the express
powers granted to the Board to achieve the objectives and purpose of the Act.

16. The Board of Accountancy shall elect a vice-chairman from among its members for a
term of
A. Two (2) years
B. One (1) year
C. Three (3) years
D. Five (5) years

17. According to Section 9(A) of the IRR, the Commission, upon the recommendation of the
Board, shall create an accounting
standard setting body to be known as the A. Accounting Standards
Council
B. Financial Reporting Standards Council
C. Accounting Standards Board
D. Financial Reporting Standards Board

18. According to Section 9(A) of the IRR, the Commission, upon the recommendation of the
Board, shall create an auditing standard setting body to be known as the A. Auditing and
Assurance Standards Council (AASC)
B. Auditing Standards and Practices Council (ASPC)
C. Auditing Standards Board D. Auditing Standards Council

19. This standard setting body shall have a chairman who had been or presently a senior
accounting practitioner in any of the scope of accounting practice.
A. FRSC
B. AASC
C. PICPA
D. ACPAPP

20. This standard setting body shall have a chairman who had been or presently a senior
accounting practitioner in public accountancy. A. AASC
B. FRSC
C. ACPAE
D. BOA

21. The Chairman and the members of the FRSC and AASC shall have a term of A. 3 years
B. 5 years C. 6 years
D. 7 years

22. Which of the following is not represented in the AASC?


A. Bangko Sentral ng Pilipinas
B. Board of Accountancy
C. Bureau of Internal Revenue
D. Securities and Exchange Commission

23. Who has the power to suspend or remove any member of the Board of Accountancy?
A. The Chairman of the FRSC B. The Chairman of
the PRC
C. The Chairman of the AASC
D. The President of the Philippines

24. An applicant for the CPA licensure examination should be

I. A Filipino citizen
II. Of good moral character
III. A holder of the degree of Bachelor of Science in
Accountancy

A. I and II only
B. I and III only
C. II and III only
D. I, II, and III Section 14 of the IRR states that any person applying for examination shall
be establish the following requisites to the satisfaction of the Board that he/she:
a) Is a Filipino citizen;
b) Is of good moral character;
c) Is a holder of the degree of Bachelor of Science in Accountancy conferred by a
school, college, academy or institute duly recognized and/or accredited by the
CHED or other authorized government offices;
d) Has not been convicted of any criminal offense involving moral turpitude.

25. The following documents shall be submitted by applicants for the CPA licensure
examination, except
A. Certificate of Live Birth in National Statistics Office (NSO) security paper.
B. Marriage contract in NSO security paper for married male applicants.
C. NBI clearance.
D. Transcript of records with indication therein of date of graduation and Special Order
number unless it is not required.

Section 14 of the IRR requires applicants to submit the following:


a) Birth certificate in NSO security paper;
b) Marriage contract in NSO security paper for married female applicants;
c) College diploma;
d) Transcript of records;
e) NBI clearance;
f) Other documents that the Board may require.

26. Section 16 of the IRR states that to be qualified as having passed the licensure
examination for accountants, a candidate must obtain a
A. General average of seventy-five percent (75%), with no grades lower than sixty percent
(60%) in any given subject.
B. General average of seventy-five percent (75%), with no grades lower than sixty-five
percent (65%) in any given subject.
C. General average of seventy-five percent (70%), with no grades lower than sixty percent
(60%) in any given subject.
D. General average of seventy-five percent (75%), with no grades lower than sixty percent
(60%) in any given subject.
27. A candidate who obtains the rating of seventy-five percent (75%) and above in at least a
majority of the subjects shall receive a conditional credit for the subjects passed. He/she
shall take an examination in the remaining subjects within how many years from the
preceding examination?
A. 1 B. 2 C. 3
D. 5

28. Any candidate who fails in two (2) complete CPA board examinations shall be
disqualified from taking another set of examinations unless he/she has completed at least
how many units of subjects given in the licensure examination?
A. 4
B. 8
C. 16
D. 24

29. The Board of Accountancy shall submit to the PRC the ratings obtained by each
candidate within how many calendar days after the examination?
A. 1 B. 2
C. 5
D. 10

30. The Certificate of Registration issued to successful


examinees
A. Is renewable every three years.
B. Is renewable every five years.
C. Shall remain in full force and effect until withdrawn, suspended or revoked in accordance
with RA9298.
D. Shall bear the signature of the PRC Chairperson and the two PRC Commissioners.

According to Section 20 of the IRR, the Certificate of Registration shall:


a) Bear the signature of the Chairperson of the PRC and the Chairman and Members of
the Board of Accountancy, stamped with the official seal of the PRC and of the Board
of Accountancy, indicating that the person named therein is entitled to the practice of
the profession with all the privileges appurtenant thereto;
b) Remain in full force and effect until withdrawn, suspended or revoked in accordance
with RA 9298.

31. The Professional Identification Card issued to successful


examinees
A. Is renewable every three years.
B. Is renewable every five years.
C. Shall remain in full force and effect until withdrawn, suspended or revoked in accordance
with RA 9298.
D. Shall bear the signature of the PRC Chairperson and the Chairman and Members of the
Board of Accountancy

Section 20 of the IRR provides that a Professional Identification Card bearing the registration
number, date of issuance, expiry date, duly signed by the Chairperson of the PRC, shall be
issued to every registrant. It provides further that the reissuance or renewal of the said card
shall be subject to payment of the annual registration fees for another and every after three
(3) years thereafter.

32. The Board of Accountancy may reinstate the validity of a revoked Certificate of
Registration after the expiration of how many years from the date of revocation?
A. 1 B. 2 C. 3
D. 5

Section 25 of the IRR provides that the Board of Accountancy may, after the expiration of
two (2) years from the date of revocation of a Certificate of Registration and upon
application and for reasons deemed proper and sufficient, and after being convinced of
applicant’s remorse and rehabilitation, reinstate the validity of a revoked Certificate of
Registration and in so doing, may, in its discretion, exempt the applicant from taking another
examination.

33. The following statements relate to the practice of public accountancy. Which is incorrect?
A. Single practitioners and partners of partnerships organized for the practice of public
accountancy shall be registered CPAs in the Philippines.
B. A partnership engaged in the practice of public accountancy may be carried on in the
form of a general partnership (GP) but not a limited liability partnership (LLP).
C. A CPA is in public accounting practice when he/she represents his/her clients before
government agencies on tax and other matters related to accounting.
D. The Securities and Exchange Commission shall not register any corporation organized for
the practice of public accountancy.

Section 28 of the IRR provides that partnership engaged in in the practice of public
accountancy may be carried on in the form of a general partnership (GP) or a limited liability
partnership (LLP).

34. Which of the following statements concerning ownership of working papers is incorrect?
A. All working papers made by a CPA and his/her staff in the course of an examination
remain the property of such CPA in the absence of any agreement (written or oral)
between the CPA and the client to the contrary.
B. Working papers include schedules and memoranda prepared and submitted by the client
of the CPA.
C. All working papers, except reports submitted by a CPA to his/her client shall be treated
confidential and privileged.
D. Working papers shall be treated confidential and privileged and remain the property of
the CPA unless such documents are required to be produced through subpoena issued by
any court, tribunal, or government regulatory or administrative body.

35. All registered CPAs shall obtain and use a seal which shall be circular in form with a
smaller circle within bearing the registrant’s name, registration number and title. Which
of the following is correct?
A. Engraved in the lower portion of the space between the circles is the CPA’s name.
B. Engraved in the upper portion of the space between the circles is the CPA registration
number.
C. Engraved in the middle of the smaller circle are the letters “CPA”.
D. Engraved in the middle of the smaller circle are the CPA’s name and registration number.
E. Engraved in the middle of the smaller circle is the CPA’s name

Section 33 of the IRR states that the seal of a CPA shall be circular in form with a smaller
circle within. In the upper portion of the space between the circles shall be engraved the
name of the individual CPA, firm or partnership as the case maybe, the lower portion thereof
shall be engraved the CPA registration number of the individual CPA, proprietor of the firm
and the signing partner of the partnership, and in the middle of the smaller circle shall be
engraved the letters “CPA”.

The auditor’s reports shall be stamped with said seal, indicating therein his/her current
Professional Tax Receipt (PTR) number, date/place of payment when filed with government
authorities or when used professionally.

36. Section 36 (Penal Provision) of RA 9298 states that any person who shall violate any of
the provisions of the Act or any of its implementing rules and regulations shall, upon
conviction, be punished by
A. A fine of not less than P50,000
B. Imprisonment for a period not exceeding two (2) years.
C. A fine of not less than P50,000 or by imprisonment for a period not exceeding two (2)
years.
D. A fine of not less than P50,000 or by imprisonment for a period not exceeding two (2)
years, or both.

37. Which of the following is the accredited national professional organization of CPAs
(APO)?
A. PICPA
B. ASC
C. AASB
D. FRSC

38. Tanya, Sam, and Jervi, CPAs, a newly formed partnership, applied for registration with
the Commission and the Board which was approved on June 30, 2012. The partnership
should apply for renewal of its registration on or before
A. September 30,2014
B. September 30,2015
C. December 30,2016
D. June 30,2015

39. Which of the following statements concerning the use of firm or partnership name is
incorrect?
A. In the case of an individual CPA, he/she shall do business under his/her registered name
with the BOA and the PRC and as printed in his/her CPA certificate (for example, Juan
Puruntong, CPA).
B. In the case of a firm, it shall do business under its duly registered and authorized firm
name appearing in the registration documents issued by the Department of Trade and
Industry (DTI) and other government offices and such firm name shall include the real
name of the sole proprietor as printed in his/her CPA certificate (for example, Amulfo
Gumamela and Associates).
C. In the case of registered partnership, it shall do business under its name as indicated in its
current Articles of Partnership and Certificate of Registration issued by the Securities and
Exchange Commission (SEC) (for example, Tanya, Sam, and Jervi, CPAs).
D. A CPA shall practice only under an individual, firm, or partnership name in accordance
with Philippine law and shall not include any fictitious name but may indicate
specialization.

40. A partner surviving the death or withdrawal of all the other partners in a partnership may
continue to practice under the partnership name for a period of not more than ___ years
after becoming a sole proprietor.
A. 1 B. 2 C. 3
D. 4

41. The death or disability of an individual CPA and/or the dissolution and liquidation of a
firm or partnership of CPAs shall be reported to the BOA not late than ____ days from
the date of such death, dissolution or liquidation.
A. 15 B. 30 C. 60
D. 90

42. As defined in Annex “C” of the IRR, this refers to the inculcation, assimilation and
acquisition of knowledge, skills, proficiency and ethical and moral values, after the initial
registration of a professional that raise and enhance the professional’s technical skills and
competence.
A. Professional Development
B. Continuing Professional Education
C. Continuing Professional Development
D. Professional Growth and Development

43. The PRC CPE Council shall assist the BOA in implementing its CPE program. Which of
the following statements is incorrect concerning the Council’s compostion?
A. The PRC CPE Council shall be composed of a chairperson and two (2) members.
B. The Chairperson shall be chosen from among the members of the BOA by the PRC.
C. The first member shall be the President or, in his/her absence or incapacity, any officer
chosen by the Board of Directors of PICPA.
D. The second member shall be the President or, in his/her absence or incapacity, any officer
of the organization of Deans or Department Heads of schools, colleges, or universities
offering the degree requiring licensure examination.

44. The following statements relate to CPE credit units. Which is incorrect?
A. The total CPE credit units for registered accounting professionals shall be sixty (60)
credit units for three (3) years, provided that a minimum of fifteen (15) credit units shall
be earned in each year.
B. Any excess credit units in one year may be carried over to the succeeding years within
the three-year period.
C. Excess credit units earned may be carried over to the next three-year period except credit
units earned for doctoral and master’s degrees.
D. One credit hour of CPE program, activity or source shall be equivalent to one (1) credit
unit.

45. The following statements relate to the exemption from CPE requirements. Which is false?
A. A registered professional shall be permanently exempted from CPE requirements upon
reaching the age of 60 years old.
B. A registered professional applying for permanent CPE exemption is required to submit an
authentic or authenticated copy of his/her birth certificate or, if not available, his/her
voter’s ID or driver’s license.
C. A registered professional who is working or practicing his/her profession abroad shall be
temporarily exempted from compliance with CPE requirements during the period of
his/her stay abroad provided that he/she has been out of the country for at least two years
immediately prior to the date of renewal.
D. A registered professional who is furthering his/her studies abroad shall be temporarily
exempted from compliance with CPE requirements during the period of his/her stay
abroad provided that he/she has been out of the country for at least two years
immediately prior to the date of renewal.
A registered professional shall be permanently exempted from CPE requirements upon the
reaching the age of 65 years old.

46. Which of the following statements concerning a CPA’s disclosure of confidential client
information is ordinarily
correct?
A. Disclosure may be made to any party on consent of the client.
B. Disclosure should not be made even if such disclosure will protect the CPA’s professional
interests in legal proceedings.
C. Disclosure should be made only if there is a legal or professional duty to make disclosure.
D. Disclosure may be made to any government agency without subpoena.

47. Listed below are names of four CPA firms and pertinent facts relative to each firm.
Unless otherwise indicated, the individuals named are CPAs and partners, and there are
no other partners. Which is a violation of the Implementing Rules and Regulations of RA
9298?
A. Tin, Ton and Tan, CPAs (Tin died about five years ago; Ton and Tan are continuing the
firm.)
B. Poe and Que, CPAs (The name of Cua, a third partner, is omitted from the partnership
name.)
C. Joni and Jona, CPAs (Joni died about three years ago; Jona is continuing the firm as a
sole proprietor.)
D. Elias and Co., CPAs (The firm has ten other partners who are all CPAs).

Quality Assurance Review (QAR)


(Revised Rules and Regulations for the Conduct by the Professional Regulatory Board of
Accountancy of Oversight into the Quality of Audits of Financial Statements)

BOA Resolution No. 23, Series 2010

48. Which of the following has the power to conduct an oversight into the quality of audits of
financial statements through the review of the quality control measures instituted by
auditors?
A. Bureau of Internal Revenue
B. Securities and Exchange Commission
C. Board of Accountancy
D. Insurance Commission

Under RA 9298, the Board of Accountancy (BOA) has the power to conduct an oversight
into the quality of audits of financial statements through a review of the quality control
measures instituted by auditors in order to ensure compliance with the accounting and
auditing standards and practices.
49. In the exercise of its power to conduct an oversight into the quality of audits, the BOA
organized the _______ Department of the Philippine Institute of Certified Public
Accountants (PICPA) to conduct an independent study, appraisal, or review of the quality
of audit of financial
statements.
A. Quality Assurance Review
B. Quality Control Review
C. Independent Quality Assurance Review
D. Independent Quality Control Review

Section 2 of the Revised Rules and Regulations for the Conduct by the Professional
Regulatory Board of Accountancy of Oversight into the Quality of Audits of Financial
Statements defines Quality Assurance Review as “a study, appraisal, or review by an
independent Quality Assurance Review Department organized by the Board, of the quality of
audit of financial statements through a review of the quality control measures instituted by
member-CPA Practitioners engaged in the practice of public accountancy to ascertain
compliance with prescribed professional, ethical and technical standards of public practice.”

50. The operation of the Quality Assurance Review Department


(QARD) of the PICPA shall be supervised by a/an
A. Executive Committee B. Supervisory Board
C. Oversight Committee
D. Accountability Board

Section 5 provides, “The Executive Committee shall have the full power and authority to set
policies and to supervise the operation of QARD.”

51. Which of the following is not a responsibility of the Executive Committee?


A. To set policies that will ensure effective implementation of the quality assurance review
program.
B. To monitor the quality of audits through its QARD.
C. To recommend to the BOA a rolling Quality Assurance Review (QAR) plan for 3 years.
D. To select the CPA practitioners to be reviews for the year, in accordance with the three-
year plan.

Listed under Section 5 are the following duties and responsibilities of the Executive
Committee:
a) To set policies that will ensure effective implementation of the quality assurance
review program;
b) To maintain independence of the QARD;
c) Through its QARD, to monitor the quality of audits;
d) To hire the Chief Inspector and Chief of Administration of QARD including other
personnel thereof;
e) To recommend to the Board a rolling Quality Assurance Review (QAR) plan for three
(3) years, which may be approved by the Board without referral to the Commission.
f) To implement the approved QAR plan through the QARD;
g) To obtain independent technical advice on the subject of quality assurance when
needed and appropriate;
h) To receive and evaluate the reports and recommendations of the Chief Inspector;
i) Submit regularly to the Board a list of erring memberCPA practitioners including the
findings and actions taken by the QARD. The Committee shall likewise regularly
submit to the Board policies that it adopted and implementing issues that it settled for
the oversight review of the Board. It shall also report to the Board any form of
interference by the APO on its policy making or on QARD’s operation;
j) To issue through the Board, annual reports for the benefit of the general public.

The selection of the CPA-Practitioners to be reviewed for the year is a responsibility of


the Chief Inspector of the QARD.
52. Which of the following is an incorrect statement concerning the term of the office of the
Executive Committee members?
A. The term of office of the Chairman and members shall be co-terminus with their
respective term in the BOA or PICPA unless replaced earlier by the Board or PICPA.
B. The term of office of the Chairman and members in no case shall be more than 3 years.
C. The Chairman and members may continue to be in active practice of public accountancy
during their term, subject to certain restriction imposed by the Board.
D. The Chairman of the Executive Committee shall be from the Board’s appointees.

Section 6 provides that the Chairman and members of the Executive Committee should not
be in active practice of public accountancy during his term in the Executive Committee.

53. The Head of the QARD who will be appointed by the Executive
Committee is the
A. Chief Inspector
B. Chief of Administration
C. Chief Auditor
D. Chief Quality Assurance Reviewer

Under Section 7, the QARD’s personnel shall be composed of:


1) Head (the Chief Inspector)
2) Assistants to the Chief Inspector
3) Chief of Administration
4) Assistants to the Chief of Administration
5) Staff auditors
6) Such other employees that may be necessary to carry out effectively the functions of the
QARD.

54. Which of the following is an incorrect qualification of the Chief Inspector of the QARD?
A. Must have at least 5 years of experience in the practice of public accountancy.
B. Must be of good character.
C. Must not have been found guilty of violating any professional, ethical and regulatory
auditing standards.
D. Must have reached the level of senior manager or partner of an accounting firm that has
among its clients, publicly listed entities.

Section 9 provides that the Chief Inspector of the QARD should have at least 10 years of
experience in the practice of public accountancy.

55. Who is responsible for the non-technical aspect of the QAR program? A. Chief Inspector
B. Chief of Administration
C. Assistant to the Chief Inspector
D. Chairman of the Executive Committee

56. The QAR program covers all CPAs in


A. Public Accounting
B. Government
C. Commerce and Industry
D. Education/Academe

Section 15 provides that the QAR program covers all CPAs in public accounting, whether as
an individual practitioner, a firm or a partnership.

57. Which of the following statements concerning enrollment in QAR program is/are
correct?
I. Enrollment in the QAR program under the prescribed category is a pre-requisite for
accreditation or renewal of accreditation as a CPA in public practice by the Board of
Accountancy.
II. The CPA practitioner’s registration category should be stated in the BOA certificate of
accreditation.
A. I only
B. II only
C. Both I and II
D. Neither I nor II
58. CPAs in public practice are required to register with the QAR program under the
prescribed category. Those whose clients are public-interest entities (listed and not-listed
but with public accountability) should register under
A. Category A B. Category B C.
Category C
D. Category D

Section 19 provides that CPA practitioners should be registered in accordance with the
following categories

• Category A – registration for CPA practitioners handling clients that use the full IFRS
or their Philippine equivalents. These would cover CPA practitioners auditing public-
interest entities (listed and notlisted but with public accountability).

• Category B – registration for CPA practitioners handling clients that use the IFRS or
their Philippine equivalents for Small and Medium-sized enterprises.

59. Public-interest entities include publicly-listed entities and not listed entities but with
public accountability. The following are examples of not-listed public-interest entities,
except
A. Entities which have sold a class of their securities pursuant to a registration under
Section12 of the Securities Regulation Code.
B. Entities with assets of at least P50 million and having 200 or more holders each holding
at least 100 shares of a class of its equity securities as of the first day of the issuer’s fiscal
year.
C. Entities which are in the process of filing their financial statements for the purpose of
issuing any class of instruments in a public market.
D. Entities not in the process of filing their financial statements for the purpose of issuing
any class of instruments in a public market.

Under Section 19, audit clients are classified as:

I. Public-interest Entities (Entities with Public Accountability) Using Full IFRS or their
Philippine
Equivalents

1. Publicly-listed entities
a. Entities which have issued a class of securities listed for trading on an Exchange

2. Other Public-interest Entities but not Listed


a. Entities which have a sold a class of their securities pursuant to a registration
under Section 12 of the Securities Regulation Code.
b. Entities with assets of at least P50 million and having 200 or more holders each
holding at least 100 shares of class of its equity securities as of the first day of the
issuer’s fiscal year.
c. Entities which are in the process of filing their financial statements for the
purpose of issuing any class of instruments in a public market.
d. Entities that hold assets in a fiduciary capacity for a board group of outsiders such
as a bank (all types of banks), an investment house, a finance company, an
insurance company, a securities broker/dealer, a mutual fund and pre-need
company or entities with secondary license.
e. Public utility entities.

f. Entities which are economically significant. These are entities whose total assets
exceed P350 million or whose total liabilities exceed P250 million. Total assets
and total liabilities are based on the entity’s annual financial statements and on
consolidated totals, if the entity presents consolidated financial statements. An
entity that is a subsidiary of a parent that is considered to have public
accountability is similarly considered to have public accountability.

II. Entities Using the IFRS or their Philippine


Equivalent for SMEs
a. Entities with total assets of between P3 million and P350 million or total
liabilities of between P3 million and P250 million.
b. Entities not required to file financial statements under Part II or Rule 68;
c. Entities not in the process of filing their financial statements for the purpose of
issuing any class of instruments in a public market;
d. Entities which are not holders of secondary license issued by a regulatory agency,
such as a bank (all types of banks), an investment house, a finance company, an
insurance company, a securities broker/dealer, a mutual fund and a pre-need
company; and
e. Entities which are not public utility companies.

60. There are rules to observe in connection with the registration of CPA practitioners with
the QAR program. Which is incorrect?
A. A CPA practitioner will register in only one category division.
B. Change in registration category is not allowed.
C. Change in registration sub-category due to the change in the number of clients should be
done upon renewal of registration within the first two months of each calendar year.
D. All applications for registration must include information as to the number of clients
classified as to publicinterest entities using full IFRS and small and mediumsized entities
using IFRS for SMEs.

According to the registration riles in Section 19, the change in registration from Category A
to Category B or vice versa should be done within the year of change.
RULES AND REGULATIONS FOR THE ACCREDITATION OF ACCOUNTING TEACHERS

BOA Resolution No.88, Series of 2008

61. What is the effectivity date of the BOA Resolution No.88 (Series of 2008) prescribing the
rules and regulations for the accreditation of accounting teachers?
A. June 4, 2008
B. June 20, 2008
C. December 31, 2008
D. June 30,2008

62. Which of the following requirements for the accreditation of accounting teachers shall
not apply to those CPAs already engaged in teaching as of the effectivity date of the
Rules and Regulations?

I. Possession of relevant Master’s degree.


II. Completion of 12 units of relevant education subjects from CHED recognized
schools.
III. A total of 3 years meaningful experience in actual accounting work either in Public
Practice, Commerce & Industry or Government sector.
IV. Proof that the CPA has undergone Continuing Professional Education (CPE).

A. I and II only
B. II and III only
C. I and IV only
D. II only

63. The following statements relate to the requirement to complete 12 units of relevant
education subjects for the accreditation of accounting teachers. Which is incorrect?
A. For purposes of compliance, the 12 units may be earned from the undergraduate
education program or from a graduate degree program of any Higher Education
Institution (HEI) duly recognized by CHED.
B. The 12 units may be earned from in-service or in-house training on relevant education
subjects offered by schools or training centers.
C. The 12 units may be a combination of in-service trainings and units earned in an
undergraduate or graduate education
programs.
D. CPAs who have passed the Teachers Board Exams and are licensed Professional Teachers
should earn the 12 units through in-service or in-house trainings.
64. Which of the following shall be considered proof of compliance with the CPE
requirement for accreditation of accounting teachers?

I. Certification of CPE units from accredited CPE providers issued by the PICPA.
II. Certificate of Attendance or other proofs of meaningful participation in other CPE
programs as approved by the PRC/CPE Council upon recommendation of the PICPA
of the individual CPA of a minimum of sixty (60) credits units earned for the past
immediate 3 years.

A. I only
B. II only
C. Either I or II
D. Neither I nor II

65. What is the required minimum number of CPE credit units that an accounting teacher
shall earn in each year prior to renewal of accreditation?
A. 15 units B. 20 units
C. 10 units
D. No minimum number of credit units required

66. The Certificate of Accreditation issued by the PRC to an accounting teacher shall
A. Be valid for 2 years and renewable every 2 years.
B. Be valid initially for 3 years renewable annually.
C. Remain in full force and effect unless revoked, cancelled or withdrawn.
D. Be valid for 3 years and renewable every 3 years.

67. According to the Transitory Provisions of the BOA Resolution No.88, any tenured/full
time/full load faculty member who does not meet the accreditation requirements as of the
effectivity date of the Rules and Regulations may be issued a provisional Accreditation.
Choose the correct statement.
I. The Provisional Accreditation will be valid for a period not exceeding 3 years unless
earlier withdrawn, revoked, or cancelled for cause by the BOA.
II. The Provisional Accreditation may be issued only once and is not renewable.
III. The transitory provision shall also apply to returning teachers who have not been
teaching for the last 5 years.

A. I only
B. I and II only
C. II and III only
D. I, II, and III
B. DIFFERENT GOVERNMENT AGENCIES THAT INFLUENCE THE PRACTICE OF
ACCOUNTANCY

SECURITIES AND EXCHANGE COMMISSION (SEC)

68. Which of the following is responsible for the registration of corporations and partnerships, as
well as monitoring of compliance with the Corporation Code, Civil Code provisions on
partnerships, Foreign Investments Act, and other related
laws?
A. Bangko Sentral ng Pilipinas (BSP)
B. Securities and Exchange Commission (SEC)
C. Bureau of Internal Revenue (BIR)
D. Philippine Stock Exchange (PSE)

69. Which of the following laws/regulations govern the registration of corporations and
partnerships with the SEC?
A. Civil Code
B. Corporation Code of the Philippines
C. Securities Regulation Code
D. A,B, and C

70. Which of the following entities need not be registered with the SEC? A. Sole proprietorships
B. General and limited partnerships
C. Stock corporations
D. Non-stock corporations

71. The SEC is composed of a chairperson and four (4) commissioners appointed by the
President of the Philippines for a term of A. 3 years B. 6 years C. 7 years
D. 8 years

72. The following statements relate to the qualifications of the SEC commissioners. Which is
incorrect?
A. The commissioners must be natural-born citizen of the Philippines.
B. The commissioners must be of good moral character, of unquestionable integrity, of
known probity and patriotism, and with recognized competence in social and economic
disciplines.
C. The majority of the commissioners, including the Chairperson, shall be the members of
the Philippine Bar.
D. The Chairperson and the commissioners must be at least thirty-five (35) years of age.

The Chairperson must be at least forty (40) years of age and the commissioners must be at
least thirty-five (35) years of age. (Subsection 4.2, Securities Regulation Code)
73. Subsection 8.1 of the Securities Regulation Code (SRC) provides that securities shall not be
sold or offered for sale or distribution within Philippines without a
A. Registration Statement
B. Register of Securities
C. Certificate of Accreditation
D. Certificate of Registration

Securities shall not be sold or offered for sale or distribution within the Philippines without a
Registration Statement duly filed with and approved by the Securities and Exchange
Commission.

74. The following securities are exempted from the registration requirement under Subsection
8.1 of the Securities Regulation Code (SRC), except A. A bank’s own shares of stock.
B. Any security issued or guaranteed by the Government of the Philippines.
C. Any security issued by the Government of any country with which the Philippines
maintains diplomatic relations on the basis of reciprocity.
D. Any security or its derivatives, the sale or transfer of which by law, is under supervision
and regulation of the Office of the Insurance Commission, Housing and Land Use
Regulatory Board, or the Bureau of Internal Revenue.

Under Subsection 9.1 of the SRC, exempted from the registration requirement is any security
issued by a bank except its own shares of stock.

75. The following transactions are exempted from the registration requirement under Subsection
8.1 of the Securities Regulation Code (SRC), except
A. The distribution by a corporation, actively engaged in the business authorized by its
articles of incorporation, of securities to its stockholders as a stock dividend.
B. The sale of securities by an issuer to more than twenty (20) persons in the Philippines
during any 12-month period.
C. The issuance of bonds and notes secured by mortgage upon real estate or tangible
personal property, where the entire mortgage together with the bonds or notes secured
thereby are sold to a single purchaser at a single sale.
D. Broker’s transactions, executed upon customer’s orders, on any registered Exchange or
other trading market.

Under Subsection 10.1 of the SRC, exempted from the registration requirement I the sale of
securities by an issuer to fewer than twenty (20) persons in the Philippines during any 12-month
period.

76. Rule 68 of the Securities Regulation Code (SRC) prescribes the requirements applicable to
the form and content of financial statements to be filed by stock corporations except
those whose paid-up capital is less than
A. P500,000 B. P 50,000 C. P250,000
D. P400,000

SRC Rule 68, as amended, prescribes the form and content of financial statements required to
be filed with the Securities and Exchange Commission (SEC) by corporations which meet
the following thresholds:
a) Stock corporations with paid-up capital stock of P50,000 or more;
b) Non-stock corporations with total assets of P500,000 or more, or with gross annual
receipts of P100,000 or more; c) Branch offices of stock foreign corporations with
assigned capital in the equivalent amount of P1,000,000 or more;
d) Branch offices of non-stock corporations with total assets in the equivalent amount of
P1,000,000 or more;
e) Regional operating headquarters of foreign corporations with total revenues in the
equivalent amount of P1,000,000 or more.

77. The audited financial statements to be filed with the SEC shall be accompanied by a A.
Management Report
B. Registration Statement
C. Statement of Management’s Responsibility for Financial Statements
D. Statement of the Board of Director’s Responsibility for Financial Statements

The SEC requires management of all operations covered by SRC Rule 68, as amended, to
acknowledge their responsibility for their financial statements. Hence, the financial
statements to be filed with the SEC shall be accompanied by a Statement of Management’s
Responsibility (SMR) for Financial Statements.

78. The Statement of Management’s Responsibility to accompany the financial statements to be


filed with the SEC shall be signed by the

I. Chairman of the Board of Directors


II. Chief Executive Officer
III. Chief Financial officer

A. II and III only


B. I only
C. I and II only
D. I, II, and III

SRC Rule 68, as amended, prescribe the wording of the wording of the SMR that shall be
attached to the financial statements.
Part I, Section B (ii) of the Rule states that the SMR for Financial Statements that shall be
attached to the financial statements shall read as follows:

STATEMENT OF MANAGEMENT’S RESPONSIBILITY


FOR FINANCIAL STATEMENTS

The management of (name of reporting company) is responsible for the preparation and fair
presentation of the financial statements for the year(s) ended (date), in accordance with the
prescribed financial reporting framework indicated therein. This responsibility includes
designing and implementing internal controls relevant to the preparation and fair
presentation of financial statements that are free from material misstatement, whether due to
fraud or error, selecting and applying appropriate accounting policies, and making
accounting estimates that are reasonable in the circumstances.

The Board of Directors or Trustees reviews and approves the financial statements and
submits the same to the stockholders or members.

(name of auditing firm), the independent auditors, appointed by the stockholders has
examined the financial statements of the company in accordance with Philippine Standards
on Auditing, and in its report to the stockholders or members, has expressed its opinion on
the fairness of presentation upon completion of such examination.

Signature_______________
Printed Name of the Chairman of the
Board________________

Signature_______________
Printed Name of the Chief Executive
Officer______________

Signature_______________
Printed Name of the Chief Financial
Officer______________

The SMR of the entities covered under Part II of the SRC Rule 68, as amended, shall be
signed under oath.

79. In addition to the requirements prescribed under Part I of the Rule 68, as amended, Part II
of the Rule prescribes special requirements on the financial statements required to be
filed with the SEC by
I. An issuer which has sold a class of its securities pursuant to a registration under
Section 12 of the SRC.
II. An issuer with a class of securities listed for trading on an Exchange.
III. An issuer with assets of at least P50,000,000 and has 200 or more holders, each
holding at least 100 shares of a class of its equity securities as of the first day of the
issuer’s fiscal year.
A. I only
B. II only
C. II and III only
D. I,II and III

80. The financial statements to be filed with the SEC shall be presented in comparative form.
The audited balance sheets or statements of financial position shall be as of the end of
each of the ___ most recently completed fiscal years.
A. 2 B. 3 C. 4
D. 5

81. SRC Rule 68, as amended, requires a regulated entity to report to the SEC its action on a
report of its independent auditor pertaining to any material findings involving fraud or
error within _____ business days from the date the report is submitted by the independent
auditor.
A. 2 B. 3 C. 4
D. 5

A regulated entity shall report to the SEC its action on a report of its independent auditor
pertaining to the following:
1) Any material findings involving fraud or error;
2) Losses or potential losses the aggregate of which amounts to at least 10% of the
consolidated total assets of the company;
3) Any finding to the effect that the consolidated total assets of the company, on a going
concern basis, are no longer adequate to cover the total claims of creditors;
4) Material internal control weaknesses which may lead to financial reporting problems.

Rule 68, as amended, prescribed the form of report on the audit findings to be submitted by
the regulated entity to the SEC within 5 business days from the date the report is submitted
by the independent auditor.

82. If a regulated entity fails to submit the required report if its action concerning any of the
audit findings specified in Rule 68, as amended, the independent auditor shall, within
____ business days from the submission of his findings to the entity, file a report to the
SEC.
A. 10 B. 15 C. 20
D. 30

83. According to Part II of Rule 68, as amended, the Rule is considered violated if the
auditor’s opinion on the financial statements of listed entities or other issuers of securities
to the public is other than unqualified because of
I. Deviation(s) from the required financial reporting framework.
II. A scope limitation imposed by the company.
A. I only
B. II only
C. Either I or II
D. Neither I nor II

BANGKO SENTRAL NG PILIPINAS (BSP)

84. The primary objective of this government agency is to maintain price stability conductive
to a balanced and sustainable economic growth. It also aims to promote and preserve
monetary stability and the convertibility of the
peso.
A. Bureau of Internal Revenue (BIR)
B. Securities and Exchange Commission (SEC)
C. Philippine Deposit and Insurance Corporation (PDIC)
D. Bangko Sentral ng Pilipinas (BSP)

85. Which of the following is not a function of the Bangko Sentral ng Pilipinas (BSP)
A. Recommend measures to improve the efficiency and effectiveness of government
operations.
B. Supervise banks and exercise regulatory powers over nonbank institutions performing
quasi-bank functions.
C. Determine the exchange rate policy of the Philippines.
D. Extend discounts, loans, and receivables to banking institutions for liquidity purposes.

To recommend measures to improve the efficiency and effectiveness of government


operations is one of the principal duties of the Commission on Audit (COA).

86. The powers and functions of the BSP shall be exercised by


the
A. Board of Directors
B. Monetary Board
C. Board of Trustees
D. BSP Governor

87. The BSP Monetary Board is composed of seven (7) members appointed by the President
of the Philippines for a term of
A. 6 years B. 5 years C. 7 years
D. 3 years
Section 6 of the New Central Bank Act (RA 7653) states that the powers and functions of the
BSP shall be exercised by the BSP Monetary Board composed of seven (7) members
appointed by the President of the Philippines for a term of six (6) years.

88. The seven (7) members of the Monetary Board are


A. The BSP governor, a member of the cabinet to be designated by the President of the
Philippines, and five (5) members from the private sector.
B. The BSP governor and six (6) members from the private sector.
C. A cabinet member and six (6) members from the private sector.
D. The BSP governor, 2 members of the cabinet to be designated by the President of the
Philippines, and four (4) members from the private sector.

89. The internal auditor of a universal or commercial bank

I. Should be a CPA.
II. Must have at least five (5) years of experience in the regular audit (internal or
external) of a universal or commercial bank as auditor-in-charge, senior auditor, or
audit manager.

A. I only
B. II only
C. Neither I nor II
D. Both I and II

The internal auditor of a universal or commercial bank must be a CPA and must have at least
five (5) years of experience in the regular audit (internal or external) of a universal or
commercial bank as auditor-in-charge, senior, auditor, or audit manager. He/she must be
competent to examine all areas in which the institution operates. (BSP Circular No. 499,
Series of 2005)

90. In its Circular No. 425, Series of 2000, the BSP requires that only external auditors
accredited by the BSP shall be engaged by banks for regular audit or special
engagements. Which of the following is not an accreditation requirement for external
auditors?
A. No external auditor may be engaged by a bank if he/she has or was committed to acquire
any direct or material indirect financial interest in the bank, or if his/her independence is
considered impaired under the circumstances specified in the Code of Professional Ethics
for CPAs.
B. The external auditor and the members of the audit team do not have/shall not have
outstanding loans or any credit accommodations (including credit card obligations) with
any bank to be audited / being audited at the time of signing the engagement and during
the engagement.
C. The external auditor should have at least five (5) years track record in conducting
external audit.
D. The external auditor and members of the audit team adhere to the highest standards of
professional conduct, including integrity and objectivity.

The requirement that the external auditor and the members of the audit team do not have /
shall not have outstanding loans or may credit accommodations with any bank to be audited /
being audited does not include credit card obligations.

The other accreditation requirements are as follows:

a. The external auditor must have at least twenty (20) existing corporate clients with
resources of P50 million each; and
b. The external auditor must have at least one (1) existing client commercial bank in the
regular audit or in lieu thereof, the external auditor and the auditor who will head the
team must have at least five (5) years of experience in the bank audit.

91. One of the documents that should be submitted by an auditor who is applying for BSP
accreditation is a Certification from the Professional Regulation Commission (PRC) that
the external auditor and the members of the audit team have no derogatory information,
previous conviction or any pending investigation. However, in the event that the
certification cannot be obtained because of the pendency of a case, the BSP
may dispense with this requirement if the case
A. Involves purely legal question
B. Does not, in any way, negate the auditor’s adherence to the highest standards of
professional conduct and degrade his/her integrity and objectivity.
C. Either A or B
D. Neither A nor B

Section 3(i) of BSP Circular No. 245 dated May 25, 2000, was amended by Circular No.318
(Series of 2002) which states that in the event the PRC Certification cannot be obtained
because of the pendency of a case, the BSP may dispense with this requirement upon
determination by the Monetary Board that the case involves purely legal question, or does
not, in any way, negate the auditor’s adherence to the highest standards of professional
conduct and degrade his/her integrity and objectivity.

92. Which of the following cases that may be discovered by a Bangko Sentral ng Pilipinas
(BSP) accredited external auditor during his/her audit fieldwork must be reported to the
BSP?
I. Any material finding discovered during the period of audit involving fraud or
dishonesty (except cases that were resolved during the audit period).
II. Adjustments or potential losses, the aggregate of which amounts to at least 1% of the
capital funds of the bank.
III. Any finding to the effect that the total bank assets, on a going concern basis, are no
longer adequate to cover the total claims of creditors.

A. I only
B. I and II only
C. II and III only
D. I, II, and III

A BSP accredited external auditor must report to the BSP any material finding which will
reduce the capital funds of the bank by at least 1%, including those cases that were resolved
during the period of audit.
(Section 4 of BSP Circular No.425, Series of 2000)

93. All banks are required to prepare the Financial Reporting Package (FRP). The FRP,
composed of the balance sheet, income statement, and supporting schedules, shall be
prepared on a solo and consolidated basis. Solo basis shall refer to the
combined financial statements of the A. Head office and
subsidiaries.
B. Head office and branches/other offices.
C. Parent bank, head office, and subsidiaries.
D. Parent bank and subsidiaries.

Solo basis refers to the combined financial statements of the head office and branches/other
offices.

Consolidated basis refers to the combined financial statements of parent bank and
subsidiaries.

94. Within how many banking days after the end of the reference quarter should banks
submit their consolidated FRP to the BSP?
A. 10 B. 20
C. 15
D. 30

The solo and consolidated FRP shall be prepared on a quarterly basis, except for the solo
balance sheet which shall be prepared on a monthly basis.

The consolidated FRP shall be submitted within 30 banking days after the end of the
reference quarter.

The solo FRP shall be submitted within 15 banking days after the end of the reference
quarter. The solo balance sheet shall be submitted within 15 banking days after the end of the
reference month.
95. Within how many banking days after the end of the reference month or quarter should
banks submit their solo balance sheet and solo income statement to the Bangko Sentral ng
Pilipinas?
A. 10 B. 20
C. 15
D. 30

COMMISSION ON AUDIT (COA)

96. This Constitutional Commission has the power, authority, and duty to examine, audit, and
settle all accounts pertaining
to the revenue and receipts of, and expenditures or uses of funds and property, owned or held
in trust by, or pertaining to, the government, or any of its subdivisions, agencies, or
instrumentalities, including government-owned or controlled corporations and recommend
measures to improve the efficiency and effectiveness of government operations.
A. Securities and Exchange Commission
B. Insurance Commission
C. Commission on Audit
D. Bureau of Internal Revenue

97. The COA is composed of


A. A chairman and two (2) commissioners.
B. A chairman and three (3) commissioners.
C. A chairman and four (4) commissioners.
D. A chairman and five (5) commissioners.

The COA is composed of a chairman and two (2) commissioners.


Together they are called the “Commission Proper”

98. The COA Chairman and the two (2) commissioners shall be appointed by the President of
the Philippines with consent of
the Commission on Appointments for a term of
A. 5 years B. 6 years C. 7 years
D. 9 years

99. Which of the following is not one of the principal duties of the COA?
A. Keep the general accounts of the government and preserve the vouchers and supporting
papers pertaining thereto.
B. Maintain price stability conducive to a balanced and sustainable economic growth.
C. Promulgate accounting and auditing rules and regulations including those for the
prevention and disallowance of irregular, unnecessary, excessive, extravagant or
unconscionable expenditures, or uses of government funds and properties.
D. Submit to the President of the Philippines and Congress, within the time fixed by law, an
annual report covering the financial condition and operation of the government.

To maintain price stability conducive to a balanced and sustainable economic growth is the
primary objective of the Bangko Sentral ng Pilipinas.

100. The COA conducts a comprehensive audit that includes


A. Financial and compliance audits
B. Compliance and performance audits.
C. Financial and financial-related audits.
D. Financial, compliance, and performance audits.
KEY ANSWERS

1. C 36. D 71. C
2. D 37. A 72. D
3. D 38. A 73. A
4. B 39. D 74. A
5. A 40. B 75. B
6. A 41. B 76. B
7. A 42. B 77. C
8. C 43. B 78. D
9. C 44. C 79. D
10. B 45. A 80. A
11. D 46. A 81. D
12. B 47. C 82. D
13. A 48. C 83. C
14. D 49. A 84. D
15. D 50. A 85. A
16. B 51. D 86. B
17. B 52. C 87. A
18. A 53. A 88. A
19. A 54. A 89. D
20. A 55. B 90. B
21. A 56. A 91. C
22. C 57. C 92. C
23. D 58. A 93. B
24. D 59. D 94. D
25. B 60. B 95. C
26. B 61. B 96. C
27. B 62. B 97. A
28. D 63. D 98. C
29. D 64. C 99. B
30. C 65. A 100. D
31. A 66. D
32. B 67. D
33. B 68. B
34. A 69. D
35. B 70. A
Chapter 3
The CPA’s Professional Responsibilities

1. The Code of Ethics for Professional Accountants in the


Philippines consists of three parts. Part A

A. Applies to professional accountants in public practice.


B. Establishes the fundamental principles for professional accountants.
C. Applies to professional accountants in business.
D. Provides a conceptual framework for the application of fundamental principles and
illustrates how the framework is to be applied in specific situations.
The Code of Ethics for Professional Accountants in the Philippines consists
of three parts. Part A establishes the fundamental principles for professional
accountants and provides a conceptual framework for their application. Parts B
and C illustrate how the conceptual framework is to be applied in specific
situations. Part B applies to professional accountants in public practice. Part C
applies to professional accountants in business.

2. Which part of the Code of Ethics applies to professional accountants in public practice?

A. Part A Part C
C .
B. Part B Part D
D .

3. Which of the following fundamental ethical principles requires a professional accountant to be


straightforward and honest in all professional and business relationships?

A. Objectivity
B. Professional behavior
C. Professional competence and due care
D. Integrity
Part A of the Code establishes the following fundamental ethical principles:

1. Professional Behavior
- A professional accountant should comply with relevant laws and
regulations and should avoid any action that discredits the profession.
2. Integrity
- A professional accountant should be straightforward and honest in all
professional and business relationships.

3. Confidentiality
- A professional accountant should respect the confidentiality of
information acquired as a result of professional and business relationships.
Such information should not be disclosed to third parties without proper and
specific authority unless there is a legal or professional right or duty to
disclose. Also, it should not be used for the personal advantage of the
professional accountant or third parties.

4. Objectivity
- A professional accountant should not allow bias, conflict of interest or
undue influence of others to override professional or business judgments.

5. Professional Competence and Due Care


- A professional accountant has a continuing duty to maintain
professional knowledge and skill at the level required to ensure that a client
or employer receives competent professional service based on current
developments in practice, legislation and techniques. When rendering
professional services, a professional accountant should act diligently and in
accordance with applicable technical and professional standards.

4. The following statements relate to the fundamental principles of professional


ethics:

A B C
D
Integrity implies fair dealing
And truthfulness. True False True
False

The principle of objectivity imposes an


obligation on all professional
accountants to maintain professional
knowledge and skill at the
level required. True
False True False

The principles of professional behavior


requires all pro- fessional
accountants to act diligently and in
accordance with applicable
technical and professional
standards when rendering
professional

Services. False
False True True

5. Competence as a certifies public accountant includes all the


following except

A. Having the technical qualifications to perform an engagement.


B. Possessing the ability to supervise and to evaluate the quality of staff work.
C. Warranting the infallibility of the work performed.
D. Consulting others if additional technical information is needed.

6. Which of the following fundamental ethical principles prohibits association


of professional accountants with reports, returns, communications or other
information that is believed to contain a materially false or misleading
statement?

A. Integrity
B. Objectivity
C. Professional competence and due care
D. COnfidentiality
Under the principle of integrity, professional accountants are required to be
straightforward and honest in professional and business relationships.
A professional accountant shall not knowingly be associated with reports,
returns, communications or other information where the professional accountant
believes that the information:

1. Contains a materially false or misleading statement;


2. Contains statements or information furnished recklessly; or
3. Omits or obscures required information where such omission or obscurity would
be misleading.
There will be no violation of the above provision if a modified report is
issued in respect of a matter described in the foregoing paragraph.

7. The principle of professional competence and due care imposes which of the
following obligations on professional
accountants?

A. To maintain professional knowledge and skill at the level required to ensure that
clients or employers receive competent professional service.
B. To refrain from disclosing confidential information obtained as a result of
professional and business relationships without proper and specific authority unless there
is a legal or professional right or duty to disclose.
C. To comply with relevant laws and regulations and avoid any situation that may
bring discredit to the profession.
D. Not to compromise professional or business judgment because of bias, conflict of
interest or undue influence of others.

8. According to the Code of Ethics, professional competence may be divided


into two phases: attainment of professional competence and maintenance of
professional competence. The attainment of professional competence
requires the following, except

A. A high standard of general education


B. Specific education, training and examination in professionally relevant
subjects.
C. Whether prescribed or not, a period of work experience.
D. A continuing awareness and an understanding of relevant technical professional
and business developments.
According to the Code, “The maintenance of professional competence
requires a continuing awareness and an understanding of relevant technical
professional and business developments. Continuing professional developments
enables a professional accountant to develop and maintain the capabilities to
perform competently within the professional environment.”

Answers A, B and C are the requirements to attain professional competence.

9. The Code of Ethics provides a Conceptual Framework for applying the


fundamental ethical principles. This framework
requires a professional accountant to

I. Identify threats to compliance with the fundamental principles.


II. Evaluate the significance of the identified threats.
III. Apply safeguards to eliminate the threats or reduce them to an acceptable level.

A. I and II only B. I and III only


C. II and III only
D. I, II, and II

The Code of Ethics provides a conceptual framework to identify,


evaluate, and respond to threats to compliance with the fundamental ethical
principles.

10. Which of the following threats to compliance with the fundamental principles
may occur as a result of the financial or other interests of a professional
accountant or of an immediate or close family member?
A. Self-interest
B. Self-review
C. Advocacy
D. Familiarity
The Code of Ethics identifies the following threats to compliance with the fundamental
principles:

1. Self-interest threat
- The threat that a financial or other interest will inappropriately
influence the professional accountant’s judgment or behavior.
2. Self-review threat
- the threat that a professional accountant will not appropriately
evaluate the results of a previous judgment made or service performed by the
professional accountant, or by another individual within the professional
accountant’s firm or employing organization, on which the accountant will
rely when forming a judgment as part of providing current service.

3. Advocacy threat
- The threat that a professional accountant will promote client’s or
employer’s position to the point that the professional accountant’s objectivity
is compromised.

4. Familiarity threat
- The threat that due to a long or close relationship with a client or
employer, a professional accountant will be too sympathetic to their interests
or too accepting of their work.

5. Intimidation threat
- The threat that a professional accountant will be deterred from acting
objectively because of actual or perceived pressures, including attempts to
exercise undue influence over the professional accountant.

11. Which of the following may be considered by a professional accountant to


eliminate or reduce identified threats to an acceptable level?

I. Safeguards created by the profession, legislation or regulation.


II. Safeguards in the work environment.
III. Resign from the client or the employer.
IV.Decline or discontinue the professional engagement.

A. I and II only
B. III and IV only
C. I and IV only
D. II and III only
The Code states that when a professional accountant identifies threats to
compliance with the fundamental principles and, based on an evaluation of those
threats, determines that they are not an acceptable level, the professional
accountant shall determine whether appropriate safeguards are available and can
be applied to eliminate the threats or reduce them to an acceptable level.

The Code states further that in making determination, the professional


accountant shall exercise professional judgment and take into account whether a
reasonable and informed third party, weighing all the specific facts and
circumstances available to the professional accountant at the time, would be
likely to conclude that the threats would be eliminated or reduced to an
acceptable level by the application of the safeguards, such that compliance with
the fundamental principles is not compromised.

Safeguards are categorized into:

1. Safeguards created by profession, legislation and


regulations; and
2. Safeguards in the work experience.

If appropriate safeguards are not available or cannot be applied, a professional


accountant shall:

1. Decline or discontinue the specific professional service involved; or


2. When necessary, resign from the engagement (if in public practice) or the
employing organization (if in business).

12. The Code of Ethics allows an auditor to perform which of the following
services for an audit client that is not a public interest entity?

A. Performance of bookkeeping services for the client.


B. Authorization of transactions for the client.
C. Preparation of client source documents.
D. Preparation and posting of journal entries without the client’s approval.

The Code states that a CPA may render services related to the preparation of
accounting records and financial statements to an audit client that is not a public
interest entity where the services are of a routine or mechanical nature, provided
that any self-review threat created is reduced to an acceptable level.

The following are examples of such services:


 Providing payroll services based on client-prepared data.
 Recording transactions for which the client has determined or approved
the appropriate account classification.
 Posting transactions coded by the client to the general ledger.
 Posting client-approved entries to the trial balance; and
 Preparing financial statements based on information in the trial balance.

13. A CPA provides audit services to a large company. Almost eighty-five


percent of a CPA’s revenues come from this client. Which statement is most
likely to be true?

A. Appearance of independence may be lacking.


B. The CPA firm does not have the competence to perform the audit.
C. The situation is satisfactory if the auditor exercises due skeptical negative
assurance care in the audit.
D. The auditor should provide an “other matter paragraph” to his/her audit report
adequately disclosing this information and then it may issue an unmodified opinion.

14. Safeguards created by the profession, legislation or regulation include the


following, except

A. Continuing professional development requirements.


B. Professional standards.
C. Firm-wide and engagement specific safeguards.
D. Educational, training and experience requirements for entry into the profession.

Firm-wide and specific safeguards are safeguards in the work environment.

According to the Code of Ethics, safeguards created by the profession, legislation or


regulation include:

1. Educational, training and experience requirements for entry into the profession.
2. Continuing professional development requirements.
3. Corporate governance regulations.
4. Professional standards.
5. Professional or regulatory monitoring and disciplinary procedures.
6. External review by a legally empowered third party of the reports, returns,
communications or information produced by a professional accountant.

15. Which of the following circumstances may create self-interest threat for a
professional accountant in public practice?

A. A member of the assurance team having a direct financial interest in the assurance
client.
B. Performing a service for an assurance client that directly affects the
subject matter information of the assurance engagement.
C. Being threatened with litigation by the client.
D. Acting as an advocate on behalf of an audit client in litigation or disputes
with third parties.

The Code of Ethics gives the following examples of circumstances that may
create self-interest threats for a professional accountant in public practice:

1. A member of the assurance team having a direct financial interest in the


assurance client.
Financial interest is defined in the Code as “an interest in an equity or
other security, debenture, loan or other debt instrument of an entity, including
rights and obligations to acquire such an interest and derivatives directly related
to such interest.”

2. A firm having undue dependence on total fees from a client.

3. A member of the assurance team having a significant close business


relationship with an assurance client.

4. A firm being concerned about the possibility of losing significant client.

5. A member of the audit team entering into employment negotiations with the audit
client.
6. A firm entering into a contingent fee arrangement relating to an assurance
engagement.

7. A professional accountant discovering a significant error when evaluating the


results of previous professional service performed by a member of the professional
accountant’s firm.

As defined in the Code, a contingent fee is “a fee calculated on a


predetermined basis relating to the outcome of a transaction or the result of the
services performed by the firm.” A fee that is established by a court or other
public authority is not a contingent fee.

In an assurance engagement, a professional accountant in public practice


expresses a conclusion designed to enhance the degree of confidence of the
intended users other than the responsible party about the outcome of the
evaluation or measurement of a subject matter against criteria.

Performing a service for an assurance client that directly affects the subject
matter information of the assurance engagement (Answer B) may create self-
review threat.

Being the threatened with litigation by the client (Answer C) may create
intimidation threat.

Acting as an advocate on behalf of an audit client in litigation or disputes


with third parties (Answer D) may create advocacy threat.

16. The following are examples of circumstances that may create familiarity
threat, except

A. The firm promoting shares in an audit client.


B. Long association of senior personnel with the assurance client.
C. A member of the engagement team having a close or immediate family member
who is a director or officer of the client.
D. A director or officer of the client or an employee in a position to exert
significant influence over the subject matter of the engagement having recently served as
the engagement partner.
In addition to the circumstances described in Answers B, C, and D, the following
may also create familiarity threat:

 A member of the engagement team having a close or immediate


family member who is an employee of the client who is in a position
to exert significant influence over the subject matter of the
engagement.

 A professional accountant accepting gifts or preferential treatment


from a client, unless the value is trivial or inconsequential.

As defined in the Code, immediate family refers to “a spouse (or


equivalent) or dependent.” Close family refers to “a parent, child or sibling
who is not an immediate family member.”

Promoting shares in an audit client may create advocacy threat.

17. The following circumstances may create intimidation threat,


except

A. A firm being threatened with dismissal from a client engagement.


B. A firm being pressured to reduce inappropriately the extent of work performed
in order to reduce fees.
C. A firm being threatened with litigation by the client.
D. A member of the assurance team being, or having recently been, a director or
officer of the client.

The following are examples of circumstances that may create self-review threat:

1. The discovery of a significant error during a reevaluation of the work of


the professional accountant in public practice.
2. A firm issuing an assurance report on the effectiveness of the operation of
financial systems after designing or implementing the systems.
3. A firm having prepared the original data used to generate records
that are the subject matter of the assurance engagement.
4. A member of the assurance team being, or having recently been, director
or officer of the client.
5. A member of the assurance team being, or having recently been,
employed by the client in a position to exert significant influences over the
subject matter of the engagement.
6. The firm performing a service for an assurance client that directly affects
the subject matter information of the assurance engagement.

18. On which of the following safeguards a professional accountant in public


practice cannot rely solely to reduce threats to an acceptable level?

A. Safeguards created by profession, legislation or regulation.


B. Firm-wide safeguards.
C. Engagement specific safeguards.
D. Safeguards within the client’s systems and procedures.

Safeguards to eliminate or reduce threats to an acceptable level are categorized into:

1. Safeguards created by the profession, legislation or regulation; and


2. Safeguards in the work environment which include firm- Wide safeguards
and engagement specific safeguards.

The Code provides that, depending on the nature of the engagement, a


professional accountant in public practice may also be able to rely on safeguards
within the client’s systems and procedures. However, a professional accountant
cannot rely solely on such safeguards to reduce threats to an acceptable level.

The Code gives the following examples of safeguards within the client’s systems
and procedures:

1. The client requires persons other than management to ratify or


approve the appointment of a firm to perform an engagement.
2. The client has competent employees with experience and seniority
to make managerial decisions.
3. The client has implemented internal procedures that ensure objective
choices in commissioning non-assurance engagements.
4. The client has a corporate governance structure that provides
appropriate oversight and communications regarding the firm’s services.

19. Which of the following is an example of engagement-specific safeguards in


the work environment?

A. Advising partners and professional staff of those assurance clients and


related entities from which they must be independent.
B. Disclosing to those charged with governance of the client the nature of services
provided the extent of fees charged.
C. A disciplinary mechanism to promote compliance with the firm’s
policies and procedures.
D. Published policies and procedures to encourage and empower staff to
communicate to senior levels within the firm any issue relating to compliance with the
fundamental principles that concerns them.

In addition to the examples in Answers A, C, and D, firmwide safeguards in the


work environment may include:

1. Leadership of the firm that stresses the importance of compliance


with the fundamental principles.

2. Leadership of the firm that establishes the expectation that members


of an assurance team will act in the public interest.

3. Policies and procedures to implement and monitor quality control of


engagements.

4. Documented policies regarding the need to identify threats to compliance


with the fundamental principles, evaluate the significance of those threats, and
apply safeguards to eliminate or reduce the threats to an acceptable level or,
when appropriate safeguards are not available or cannot be applied, terminate or
decline the relevant engagement.

5. Documented internal policies and procedures requiring compliance with


the fundamental principles.
6. Policies and procedures that will enable the identification of interests or
relationships between the firm or members of engagement teams and clients.

7. Policies and procedures to monitor and, if necessary, manage the reliance


on revenue received from a single client.

8. Using different partners and engagement teams with separate reporting


lines for the provision of nonassurance services to an assurance client.

9. Policies and procedures to prohibit individuals who are not members of


an engagement team from inappropriately influencing the outcome of the
engagement.

10. Timely communication of the firm’s policies and procedures,


including any changes to them, to all partners and professional staff, and
appropriate training and education on such policies and procedures.

11. Designating a member of senior management to be responsible for


overseeing the adequate functioning of the firm’s quality and control system.

12. Advising partners and professional staff of assurance clients


and related entities from which independence is required.

13. A disciplinary mechanism to promote compliance with policies


and procedures.
14. Published policies and procedures to encourage and empower staff
to communicate to senior levels within the firm any issue relating to compliance
with the fundamental principles that concerns them.

Engagement-specific safeguards in the work environment may include:

1. Having a professional accountant who was not involved with


the non-assurance service review the non-assurance work performed or
otherwise advise as necessary.

2. Having a professional accountant who was not a member of


the assurance team review the assurance work performed or otherwise
advise as necessary.
3. Consulting an independent third party, such as a

committee of independent directors, a professional


regulatory body or another professional accountant.

4. Discussing ethical issues with those charged with governance


of the client.

5. Disclosing to those charged with governance of the client the


nature of services provided and extent of fees charged.

6. Involving another firm to operate or re-perform part of the


engagement.

7. Rotating senior assurance team personnel.

20. If the fee quoted for a professional service is so low, it may be difficult for
the CPA to perform the engagement in accordance with applicable technical
and professional standards for that price. This situation may create a self-
interest threat to

A. Professional competence and due care


B. Objectivity
C. Integrity
D. Professional behavior

21. According to Section 240 of the Code of Ethics, fees charged for assurance
engagements should be fair reflection of the value of the work involved. In
determining professional fees, the following should be taken into account,
except

A. The time necessarily occupied by each person engaged on the work.


B. The outcome or result of a transaction or the result of the work performed.
C. The skill and knowledge required for the type of work involved.

D. The level of training and experience of the persons necessarily


engaged on the work.

A fee computed on a predetermined basis relating to the outcome or result of


a transaction or the result of the work performed is a contingent fee. Contingent
fees may create threats to compliance with the fundamental ethical principles, for
example, a self-interest threat to objectivity.

22. Which of the following is not a contingent fee?

A. A fee that is dependent upon the approval of the assurance client’s loan
application.
B. An audit fee that is based on 5% of the client’s adjusted net income
for the current year.
C. A fee that is fixed by the court or other public authority.
D. An arrangement whereby no fee will be charged unless a specified finding or
result is attained.

According to the Code of Ethics, a fee established by a court or other public


authority is not a contingent fee.

23. The Code of Ethics, requires that members of assurance teams, firms and,
when applicable, network firms be independent of assurance clients.
Independence requires

A. Independence of mind only


B. Independence in appearance only
C. Both independence of mind and independence in appearance
D. Either independence of mind or independence in appearance

24. Which of the following most completely describes how independence has
been defined by the accountancy
profession?

A. Possessing the ability to act with integrity, and exercise objectivity and
professional skepticism.
B. Accepting responsibility to act professionally and in accordance with
laws and regulations.
C. Avoiding the appearance of significant interests in the affairs of an
assurance client.
D. Performing an assurance service from the viewpoint of the public.

Independence, as stated in the Code of Ethics, requires:


Independence of mind - the state of mind that permits the expression of a
conclusion without being affected by influences that compromise professional
judgment, thereby allowing an individual to act with integrity, and exercise
objectivity and professional skepticism.

Independence in appearance - the avoidance of facts and circumstances that


are so significant that a reasonable and informed third party, would be likely to
conclude, weighing all the specific facts and circumstances, that a firm’s, or a
member of the audit team’s, integrity, objectivity or professional skepticism had
been compromised.

Answer B is incorrect because one’s acceptance of responsibility to act


professionally and in accordance with laws and regulations does not necessarily
require independence.

Answer C is incorrect because to be independent, one should be independent


both in mind and in appearance.

Answer D is incorrect because professional standards require objectivity, not the adoption of
any viewpoint.

25. Which of the following is a misunderstanding created by the use of the word
“independence?”

A. Possessing the ability to act with integrity and objectivity.


B. Independence precludes relationships that may appear to impair objectivity in rendering
assurance services.
C. A person exercising professional judgment should be free from all economic, financial and other
relationships.
D. Possessing the ability to express a conclusion without being affected by influences
that compromise professional judgment.

It is impossible for a person exercising professional judgment to be free from


all economic, financial and other relationships as every member of society has
relationships with others. The significance of economic, financial and other
relationships should be evaluated to identify threats that may exist.
26. Which of the following would not, in itself, create a network?

A. A larger structure where the entities within the structure share costs that are limited
only to those costs related to development of audit methodologies, manuals, or training
courses.
B. A larger structure that is aimed at cooperation and the entities within the structure share
common ownership, control or management.
C. A larger structure that is aimed at cooperation and the entities within the structure share
common quality control policies and procedures.
D. A larger structure that is aimed at cooperation and it is clearly aimed at profit or cost sharing
among the entities within the structure.

The Code of Ethics defines a network as a larger structure that is:


 Aimed at cooperation, and
 Clearly aimed at profit or cost sharing or shares
common ownership, control or management, common quality
control policies and procedures, common business strategy, the use
of a common brand name, or a significant part of professional
resources.

27. The Code of Ethics provides that where the larger structure is aimed at
cooperation and the entities within the structure share a significant part of
professional resources, it is considered to be a network. Professional
resources include the following, except

A. Audit methodology or audit manuals


B. Training courses and facilities
C. Brand name
D. Partners and staff
According to the Code, professional resources include:
 Common systems that enable firms to exchange information such as client
data, billing and time records;
 Partners and staff;
 Technical departments to consult on technical or industry specific issues,
transactions or events for assurance engagements;
 Audit methodology or audit manuals; and  Training courses and facilities.
28. In cases when the threat to independence is significant and no safeguards are
available to reduce it to an acceptable level, which of the following actions
should be taken?

I. Eliminating the activities or interests creating the threat.


II. Refusing to accept or continue the assurance engagement.

A. I only
B. II only
C. Neither I nor II
D. Either I or II

29. When identified threats to independence are significant and the firm decides
to accept or continue the assurance engagement, the decision should be
documented. The firm’s documentation should include
I. A description of the threats identified.
II. The safeguards applied to eliminate or reduce threats to an acceptable level.

A. I only
B. II only
C. Neither I nor II
D. Both I and II

30. Which of the following threats to independence would most likely be created
by a financial interest in an assurance client?

A. Self-interest threat
B. Self-review threat
C. Familiarity threat
D. Intimidation threat
31. A self-interest threat may be created when a member of the assurance team
knows that his close family member has a direct financial interest or a
material indirect financial interest in the assurance client. Which of the
following should be considered in evaluating the significance of the
identified threat to independence?

I. The nature of the relationship between the member of the assurance team
and the close family member.
II. The materiality of the financial interest.

A. I only
B. II only
C. Neither I nor II
D. Both I and II

32. In which of the following circumstances would a CPA be considered


independent when performing the audit of the financial statements of a new
client for the year ended December 31, 20X5?

A. The CPA resigned on January 17, 20X5 from the board of directors of the client, prior to
accepting the new audit engagement.
B. The CPA continues to hold an immaterial indirect financial interest in the client.
C. The CPA continues to serve as a trustee for the client’s pension plan and has the authority to
make investment decisions.
D. The CPA’s spouse owns an immaterial amount of ordinary shares in the client.

33. A loan, or guarantee of a loan, to the firm from an assurance client that is a
bank or similar institution, would not create a threat to independence
provided.

I. The loan, or guarantee, is made under normal lending procedures,


terms and requirements.
II. The loan is immaterial to both the firm and the assurance client.

A. I only
B. II only
C. Neither I nor II
D. Both I and II

34. A loan, or a guarantee of a loan, from an assurance client that is a bank or a


similar institution, to a member of the assurance team or his immediate
family, would not create a threat to independence provided the loan, or
guarantee, is

A. Material to the member of the assurance team or his immediate family.


B. Material to the assurance client.
C. Material to both the member of assurance team or his immediate family and the assurance
client.
D. Made under normal lending procedures, terms and requirements.

According to the Code of Ethics, a loan, or guarantee of a loan, from an


assurance client that is a bank or a similar institution, to a member of the
assurance team or his immediate family would not create a threat to
independence provided the loan, or guarantee, is made under normal lending
procedures, terms and requirements. Examples of such loans include credit card
obligations which are normally available to other credit card holders and fully
secured car loans and housing loans which are not past due.

35. Which of the following would not create a threat to independence?

A. A loan, or a guarantee of a loan, from an assurance client that is a bank or a similar


institution and the loan or guarantee is made under normal lending procedures, terms and
requirements and it is material to the assurance client or firm receiving the loan.
B. A loan, or a guarantee of a loan, from an assurance client that is a bank or a similar
institution and the loan or guarantee is material to both the firm and the assurance client.
C. A deposit made by the firm or a member of the assurance team with an assurance client that is a
bank and such deposit is held under normal commercial terms.
D. A loan, or a guarantee of a loan, from an assurance client that is a bank or a similar
institution and the loan or guarantee is not made under normal lending procedures.

The Code of Ethics states that if a firm or a member of the assurance team,
or a member of that individual’s immediate family, has deposits or a brokerage
account with an assurance client that is a bank, broker, or similar institution, a
threat to independence is not created if the deposit or account is held under
normal commercial terms.
36. A self-interest threat would be created if the firm, or a member of the
assurance team, makes a loan to an assurance client that is not a bank or
similar institution, or guarantees such an assurance client’s borrowing. The
selfinterest threat created would be so significant that no safeguard could
reduce the threat to an acceptable level unless the loan or guarantee is

A. Made under normal lending terms, procedures and requirements.


B. Immaterial to the firm or the member of the assurance team.
C. Immaterial to both the firm and the member of the assurance team and the assurance client.
D. Made under normal lending terms, procedures and requirements and the loan or guarantee is
immaterial to both the firm and the member of the assurance team and the assurance client.
37. A self-interest threat that would be created if the firm, or a member of the
assurance team, accepts a loan from, or has borrowing guaranteed by, an
assurance client that is not a bank or similar institution. The self-interest
threat created would be so significant that no safeguard could reduce the
threat to an acceptable level unless the loan or guarantee is

A. Made under normal lending terms, procedures and requirements.


B. Immaterial to the firm or the member of the assurance team.
C. Immaterial to both the firm and the member of the assurance team and the assurance client.
D. Made under normal lending terms, procedures and requirements and the loan or guarantee is
immaterial to both the firm and the member of the assurance team and the assurance client.

38. A close business relationship between a firm or a member of the assurance


team and the assurance client or its management, or between a firm, a
network firm and financial statement
audit client may create

A. Self-interest and intimidation threats


B. Self-review and familiarity threats
C. Advocacy and self-review threats
D. Self-interest and self-review threats
The Code of Ethics states that a close business relationship between a firm
and a member of the assurance team and the assurance client or its management,
or between the firm, a network firm and a financial statement audit client, will
involve a commercial or common financial interest and may create self-interest
and intimidation threats.

The Code gives the following examples of such relationships:


 Having a material financial interest in a joint venture with the assurance client or a
controlling owner, director, officer or other individual who performs senior
managerial functions for that client.  Arrangements to combine one or more
services or products of the firm with one or more services or products of the
assurance client and to market the package with reference to both parties.
 Distribution or marketing arrangements under which the firm acts as a distributor or
marketer of the assurance client’s products or services, or the assurance client acts as
the distributor or marketer of the products or services of the firm.

39. Which of the following threats to independence may be created by family


and personal relationships between a member of the assurance team and a
director, an officer, or an employee of an assurance client in a position to
exert direct and significant influence over the subject matter information of
the assurance engagement?

A. Self-interest, familiarity or intimidation threats


B. Self-review, familiarity or advocacy threats
C. Advocacy, familiarity or intimidation threats
D. Self-interest, advocacy or self-review threats

40. when an immediate family member of a member of the assurance team is a


director, an officer, or an employee of the assurance client in a position to
exert direct and significant influence over the subject matter information of
the assurance engagement, or was in such a position during the period
covered by the engagement, the threats to independence can only be reduced
to an acceptable level by

A. Where possible, structuring the responsibilities of the assurance team so that the
professional does not deal with matters that are within the responsibility of the immediate
family member.
B. Withdrawing from the assurance engagement.
C. Removing the individual from the assurance team.
D. Discussing the issue with those charged with governance, such as the audit committee.

According to the Code of Ethics, the only safeguard that can reduce the
threats to independence is by removing the individual from the assurance team. If
this safeguard is not used, the only course of action is to withdraw from the
assurance engagement.
41. When a close family member of a member of the assurance team is a
director, an officer, or an employee of the assurance client in a position to
exert direct and significant influence over the subject matter information of
the assurance engagement, threats to independence may be created. If the
threats are other than clearly insignificant, which of the following safeguards
can be applied to reduce the threats to an acceptable level?

I. Removing the individual from the assurance team


II. Where possible, structuring the responsibility of the assurance team so that the
professional does not deal with matters that are within the responsibility of the close
family member.

A. I only
B. II only
C. Either I or II
D. Neither I nor II

42. Which of the following threats to independence is created when a member of


the assurance team participates in the assurance engagement while knowing,
or having reason to believe, that he is to, or may, join the assurance client
sometime in the future?

A. Intimidation threat
B. Self-interest threat
C. Self-review threat
D. Familiarity threat
According to the Code of Ethics, firm policies and procedures shall require
members of an assurance team to notify the firm when entering employment
negotiations with the client. The significance of the threat shall be evaluated and
safeguards applied when necessary to eliminate the threat or reduce it to an
acceptable level. Examples of such safeguards include:

1. Removing the individual from the assurance team; or


2. A review of any significant judgments made by that individual while on the
team.

43. Using the same engagement partner or the same individual for the
engagement quality control review on a financial statement audit over a
prolonged period may create a
A. Self-review threat
B. Intimidation threat
C. Familiarity threat
D. Self-interest threat

44. CPAs may provide bookkeeping services to their nonpublic-interest audit


clients, but there are a number of conditions that must be satisfied if the
auditor is to maintain independence. Which of the following conditions is
not necessary?

A. The CPA must not assume a management role or function.


B. The client must hire an external CPA to approve all of the journal entries prepared by the
auditor.
C. The auditor must comply with GAAS when auditing work prepared by his/her firm.
D. The client must accept responsibility for the financial statements.

45. Several months after an unmodified audit report was issued, the auditor
discovers the financial statements were materially misstated. The client’s
CEO agrees that there are misstatements, but refuses to correct them. She
claims that “confidentiality” prohibits the CPA from informing anyone.
Is the CEO correct?
A. Yes. The auditor must maintain confidentiality.
B. No. But because the audit report has been issued it is too late.
C. Yes. But to be ethically correct, the auditor should violate the confidentiality rule and disclose
the error.
D. No. The auditor has an obligation to issue a revised audit report, even if the CEO will
not correct the financial statements.

46. Which of the following would not generally create a threat to independence?

A. The purchase of goods and services from an assurance client by the firm (or from a financial
statement audit client by a network firm) or a member of the assurance team provided that the
transaction is in the normal course of business and on an arm’s length basis.
B. A partner or employee of the firm or a network firm serves as Company Secretary for a financial
statement audit client.
C. Determining which recommendations of the firm should be implemented.
D. Reporting, in a management role, to those charged with governance.

Serving as the Company Secretary for a financial statement audit client may
create self-review and advocacy threats (Answer B).
Determining which recommendations of the firm should be implemented (Answer C)
and reporting in a management role, to those charged with governance (Answer D) would
generally create self-interest and advocacy threats.

47. The following activities may create self-interest or selfreview threats, except

A. Preparing source documents or originating data evidencing the occurrence of a


transaction.
B. Supervising assurance client employees in the performance of their normal recurring activities.
C. Having custody of an assurance client’s assets.
D. Using the same senior personnel on an assurance engagement over a long period of time.

Using the same engagement partner on an assurance engagement over a long


period of time may create a familiarity threat. In evaluating the significance of
the threat, the following factors are to be considered:
 The length of time that the individual has been a member of the assurance team;
 The role of the individual on the assurance team;  The structure of the firm; and
 The nature of the assurance engagement.

48. The following forms of assistance to a financial statement audit client do not
generally threaten the firm’s independence, except

A. Analyzing and accumulating information for regulatory reporting.


B. Assisting in resolving account reconciliation problems.
C. Authorizing or approving transactions.
D. Assisting in the preparation of consolidated financial statements.

The provision to audit clients of technical assistance and advice on


accounting principles is an appropriate means to promote the fair presentation of
the financial statements and does not generally threaten the firm’s independence.
The following services are considered to be a normal part of the audit process,
and, under normal circumstances, do not threaten the firm’s independence:
 Assisting an audit client in resolving account reconciliation problems.
 Analyzing and accumulating information for regulatory reporting.
 Assisting in the preparation of consolidated financial statements.
 Drafting disclosure items.
 Proposing adjusting journal entries.
 Providing assistance and advice in preparation of local statutory accounts of
subsidiary entities.

49. As defined in the Code, “a valuation comprises the making of assumptions


with regard to future developments, the application of certain methodologies
and techniques, and the combination of both in order to compute a certain
value, or range of values, for an asset, a liability or for a business as a
whole.” which of the following threats may be created when a firm or a
network firm performs valuation for an audit client that is to be incorporated
in the client’s financial statements?

A. Advocacy threat
B. Familiarity threat
C. Self-review threat
D. Intimidation threat

50. A firm provides valuation services to an audit client. The service involves
valuation of matters material to the financial statements and involves a
significant degree of subjectivity. Which of the following safeguards should
be applied to eliminate the self-review threat created, or reduce it to an
acceptable level?

A. Confirming with the audit client their understanding of the underlying assumptions of the
valuation and the methodology to be used and obtaining approval for their use.
B. Obtaining the audit client’s acknowledgement of responsibility for the results of the work
performed by the firm.
C. Making arrangements so that personnel providing such services do not participate in the
audit engagement.
D. The self-review threat created could not be reduced to an acceptable level by the application of
any safeguard.
The Code states, “If the valuation services involves the valuation of matters
material to the financial statements and the valuation involves a significant
degree of subjectivity, the self-review threat could not be reduced to an
acceptable level by the application of any safeguard. Accordingly, such valuation
services should not be provided or, alternatively, the only course of action could
be to withdraw from the audit engagement.”

If valuation services (either separately or in the aggregate) are not material to


the financial statements, or do not involve a significant degree of subjectivity, the
selfreview threat created could not be reduced to an acceptable level by the
application of the safeguards described in answers A, B, and C.

51. The following statements relate to the provision of taxation, internal audit or
IT Systems services to audit clients.
Which is false?

A. Tax return preparation services may create a self-review threat.


B. A self-review threat may be created when a firm, or network firm, provides internal audit
services to an audit client.
C. The provision of services by a firm or network firm to an audit client that involve the
design and implementation of financial information technology systems that are used to
generate information forming part of a client’s financial statements may create a self-
review threat.
D. The provision of services in connection with the assessment, design, and implementation of
internal accounting controls and risk management controls does not create a threat to independence
provided that firm or network firm personnel do not perform management functions.

52. Litigation support services include the following activities, except

A. Acting as an expert witness.


B. Providing assistance to an audit client’s internal legal department.
C. Calculating estimated damages or amounts that might become receivable or payable
as the result of litigation or other legal dispute.
D. Assistance with document management and retrieval in relation to a dispute or
litigation.

The provision of assistance to an audit client’s internal legal department is a


legal service, not a litigation support service.

53. What threat to independence is created when the litigation support services
provided to an audit client include the estimation of the possible outcome
and thereby affects the amounts or disclosure to be reflected in the financial
statements?

A. Self-review threat
B. Advocacy threat
C. Intimidation threat
D. Familiarity threat
54. According to the Code, legal services encompasses a wide and diversified
range of areas including both corporate and commercial services to clients -
such as contract support; litigation, mergers, and acquisition advice and
support, and the provision of assistance to a client’s internal legal
department. The provision of legal services by a firm, or
network firm, to an audit client ay create

A. Self-interest threat
B. Self-review and advocacy threat
C. Advocacy and intimidation threat
D. Familiarity and intimidation threat

55. The following statements refer to the provision of legal services to an audit
client. Which is incorrect?

A. The provision of legal services to an audit client involving matters that would not be expected to
have a material effect on the financial statements may create a self-review threat.
B. Legal services to support an audit client in the execution of a transaction (e.g., contract support)
may create an s elf-review threat.
C. Acting for an audit client in the resolution of a dispute or litigation in such circumstances when
the amounts involved are material in relation to the financial statements of the audit client would create
advocacy and self-review threats that are so significant no safeguards could reduce the threats to an
acceptable level.
D. The appointment of a partner or an employee of the firm or network firm as General Counsel for
legal affairs to an audit client would create self-review and advocacy threats that are so significant no
safeguards could reduce the threats to an acceptable level.

According to the Code, the provision of legal services to an audit client


involving matters that would not be expected to have a material effect on the
financial statements is not considered to create an unacceptable threat to
independence.

56. The recruitment of senior management for an audit client may create the
following current or future threats to independence, except

A. Self-interest threat
B. Familiarity threat
C. Intimidation threat
D. Self-review threat

57. The provision of corporate finance services, advice or assistance to an audit


client may create

A. Self-interest threat
B. Self-interest and intimidation threats
C. Advocacy and self-review threats
D. Advocacy and intimidation threats

58. When the total fees generated by an assurance client represent a large portion
of a firm’s total fees, the dependence on that client or client group and
concern about the possibility of losing the client may create a/an

A. Self-interest threat
B. Self-review threat
C. Intimidation threat
D. Advocacy threat

59. What threat to independence may be created when the fees generated by the
assurance client represent a large portion of the revenue of an individual of
the firm?

A. Self-review threat
B. Familiarity threat
C. Self-interest threat
D. Advocacy threat

60. What threat to independence may create when fees due from an assurance
client for professional services remain unpaid for a long time, especially if a
significant part is not paid before the issue of the assurance report for the
following year?

A. Advocacy threat
B. Self-interest threat
C. Intimidation threat
D. Self-review threat

61. These are fees calculated on a predetermined basis relating to the outcome or
result of a transaction or the result of the work performed.

A. Contingent fees
B. Fixed fees
C. Predetermined fees
D. Commissions

62. What threats to independence are created when a contingent fee is charged
by a firm in respect of an assurance engagement?

A. Self-review and intimidation threats


B. Self-interest and advocacy threats
C. Familiarity and intimidation threats
D. Self-interest and self-review threats

63. Accepting gifts or hospitality (unless inconsequential or


trivial) may create

A. Self-interest and familiarity threats


B. Advocacy and intimidation threats
C. Familiarity and self-review threats
D. Self-interest and self-review threats

64. Which of the following threats to independence may be created when


litigation takes place, or appears literally, between the firm or a member of
the assurance team and the assurance engagement?

A. Self-interest or advocacy threat


B. Advocacy or intimidation threat
C. Self-interest or intimidation threat
D. Familiarity or self-review threat

65. After evaluating the significance of the threat created by an actual or


threatened litigation, the following safeguards should be applied to reduce
the threat to an acceptable level, except

A. Disclosing to the audit committee, or others charged with governance, the extent and nature of
the litigation.
B. If the litigation involves a member of the assurance team, removing that individual from the
assurance team.
C. Involving an additional professional accountant in the firm who was not a member of the
assurance team to review the work or otherwise advice as necessary.
D. Withdraw from, or refuse to accept, the assurance engagement.

66. Which of the following is not a factor to consider in determining the


professional fee of a professional accountant in public practice?

A. The skill and knowledge required for the type of professional services involved.
B. The result of the assurance work.
C. The level of training and experience of the persons necessarily engaged in
performing the professional services.
D. The time necessarily occupied by each person engaged in performing the professional services.

An assurance engagement should not be performed for a fee that is


contingent on the result of the assurance work or on items that are the subject
matter of the assurance engagement.

In addition to factors mentioned in answers A, C and D, the professional


accountant should also take into account the degree of responsibility the
performance of the professional services entails.

67. Janus De Belen, CPA, was offered the engagement to audit the financial
statements of ABC Company for the year ended December 31, 2015. Janus
had served as a director of ABC Company until December 31, 2013, and his
spouse currently owns 1,000 of 200,000 outstanding shares of ABC
Company. Janus disassociated form ABC Company prior to being offered the
engagement. Moreover, the engagement does not cover any period that
includes Janus’ association or employment with ABC
Company. Under the Code of Ethics, Janus should

A. Decline the engagement because of his spouse’s stock ownership.


B. Accept the engagement.
C. Decline the engagement because he had served as director.
D. Accept the engagement because his spouse’s stock ownership is an indirect financial interest.

68. Under the Code of Ethics,

A. An immediate family member of a professional accountant, whether or not in public practice,


may not accept a gift from a client.
B. A close relative of a professional accountant not in public practice may not accept a gift from a
client.
C. A professional accountant in public practice may accept an inconsequential gift from a client.
D. A professional accountant, whether or not in public practice, may not accept a gift from a
client.

69. A client company has not paid its 20X5 audit fees. According to the Code of
Professional Ethics, in order for the auditor to be considered independent
with respect to the 20X6 audit, the 20X5 audit fees must be paid before the:

A. 20X5 report is issued


B. 20X6 fieldwork is started C. 20X6 report is issued

D. 20X7 fieldwork is started

70. As defined in the Code of Ethics, ______________ is the communication to


the public of information as to the services or skills provided by professional
accountants in public practice with a view to procuring professional
business.

A. Advertising
B. Publicity
C. Solicitation
D. Marketing professional services
71. As defined in the Code of Ethics, ______________ is the communication to
the public of facts about a professional accountant which are not designed
for the deliberate promotion of that professional accountant.

A. Advertising
B. Publicity
C. Solicitation
D. Marketing professional services

72. The following statements relate to the provisions of the Code of Ethics that
deal with the professional accountant’s marketing of professional services.
Which is false?

A. When a professional accountant in public practice solicits new work through advertising or
other forms of marketing, a self-interest threat to compliance with the principle of professional
behavior may be created.
B. The professional accountant should be honest and truthful when marketing professional
services.
C. Advertising and publicity are generally unacceptable.
D. When marketing professional services, the professional accountant should not make
exaggerated claims for services offered, qualifications possessed or experience gained.

73. The Board of Accountancy Resolution No. 126, Series of 2008 (Adoption of
the Rules and Regulations on Advertising for the Philippine Accountancy
Profession) states that, generally, advertising and publicity in any medium
are acceptable, provided:

a) It has as its objective the notification to the public or such sectors of the public
as are concerned, of matters of fact (e.g., name, address, contact numbers,
services offered) in a manner that is not false, misleading or deceptive;
b) It is in good taste;
c) It is professionally dignified; and
d) It avoids frequent repetition of, and any undue prominence being given to, the
name of the firm or professional accountant in public practice.
B. Other Professional Responsibilities

74. A firm should establish and maintain a system of quality control to


provide it with reasonable assurance that:
I. The firm and its personnel comply with professional standards
and applicable legal and regulatory requirements.
II. Reports issued by the firm or engagement partners are
appropriate in the circumstances.
A. I only B. II
only
C. Both I and II D. Neither I
nor II
75. The firm's system of quality control should include policies
and procedures that address each of the following elements, except
A. Monitoring
B. Control environment
C. Relevant ethical requirements
D. Human resources
According to PSQC 1, the firm should establish and maintain a system of quality control
that includes policies and procedures that address each of the following elements:

a) Leadership responsibilities for quality within the firm.


b) Relevant ethical requirements,
c) Acceptance and continuance of client relationships and specific engagements.
d) Human resources.
e) Engagement performance.
f) Monitoring.
Control environment is a component of an entity's internal control system.

76. Which of the following are elements of a CPA firm's quality control that
should be considered in establishing its quality control policies and
procedures?
Ethical Human Engagement
Requirements Resources Performance
No Yes No
Yes No No
c. Yes Yes Yes
No No Yes
77. Which of the following is an element of a CPA firm's quality control
system that should be considered in establishing its quality control
policies and procedures?
A. Considering audit risk and materiality.
B. Managing human resources.
C. Using statistical sampling techniques.
D. Complying with laws and regulations.

78. Which of the following quality control elements is most closely


associated with the requirement to promote a culture of quality?
A. Monitoring
B. Leadership responsibilities for quality within the firm
C. Engagement performance
D. Human resources

79. The statement, "Quality control policies and procedures should be


relevant, adequate, effective, and complied with." is most closely
associated with what quality control element?
A. Engagement performance
B. Leadership responsibilities for quality within the firm
C. Monitoring
D. Relevant ethical requirements
80. This quality control element requires a firm to establish policies and
procedures to provide it with reasonable assurance that engagements are
performed in accordance with professional standards and regulatory and
legal requirements, and that the firm or the engagement partner issue
reports that are appropriate in the circumstances.
A. Ethical requirements
B. Engagement performance
C. Monitoring
D. Human resources
81. In pursuing a firm's quality control objectives, a firm should adopt
policies and procedures to enable it to identify and evaluate
circumstances and relationships that create threats to independence, and
to take appropriate action to eliminate those threats or reduce them to an
acceptable level by applying safeguards, or, if considered appropriate, to
withdraw from the engagement. Which quality control element would
this be most likely to satisfy?
A. Ethical requirements
B. Monitoring
C. Human resources
D. Leadership responsibilities for quality within the firm
82. The primary purpose of establishing quality control policies and
procedures for deciding whether to accept a new client is to
A. Anticipate before performing any fieldwork whether an unmodified opinion can be
expressed.
B. Enable the CPA firm to attest to the reliability of the client.
C. Satisfy the CPA firm's duty to the public concerning the acceptance of new clients.
D. Minimize the likelihood of association with clients whose management lacks
integrity.
According to PSQC l, a firm should establish policies and procedures for the acceptance and
continuance of client relationships and specific engagements, designed to provide it with
reasonable assurance that it will only undertake or continue relationships and engagements
where the firm:

a. Has considered the integrity of the client and does not have
information that would lead it to conclude that the client lacks
integrity;
b. Is competent to perform the engagement and has the
capabilities, time and resources to do so; and
c. Can comply with ethical requirements.
83. As defined in PSQC 1, is a process comprising an ongoing
consideration and evaluation of the firm's system of quality control,
including a periodic inspection of a selection of completed engagements,
designed to provide the firm with reasonable assurance that its system of
quality control is operating effectively.
A. Monitoring
B. Inspection
C. Engagement quality control review
D. Supervision
84. The firm shall obtain written confirmation of compliance with its
policies and procedures on independence from all firm personnel
required to be independent by relevant ethical requirements
A. At least annually
B. At least monthly
C. At least semi-annually
D. At the completion of each engagement

85. Which element of a system of quality control is addressed by the


establishment of policies and procedures designed to provide the firm
with reasonable assurance that it has sufficient personnel with the
competence, capabilities, and commitment to ethical principles?
A. Monitoring
B. Leadership responsibilities for quality within the firm
C. Human resources
D. Engagement performance
86. The firm shall establish policies and procedures designed to provide it
with reasonable assurance that the firm and its personnel comply with
relevant ethical requirements. The Code of Ethics for Professional
Accountants in the Philippines establishes the fundamental principles of
professional ethics which include the following, except
A. Integrity
B. Objectivity
C. Relevance
D. Professional behavior
The Code of Ethics for Professional Accountants in the Philippines establishes
the fundamental principles of professional ethics, which include:

a. Integrity;
b. Objectivity;
c. Professional competence and due care:
d. Confidentiality; and
e. Professional behavior.
87. The audit work performed by each assistant should be reviewed by
personnel of at least equal competence to determine whether it was
adequately performed and to evaluate whether the
A. Firm's system of quality control has been maintained at a high level.
B. Work performed and the results obtained have been adequately documented.
C. Audit procedures performed are approved in the professional standards.
D. Audit procedures performed are in accordance with Philippine Standards on
Auditing (PSAs).
The work performed by each assistant should be reviewed to consider whether:

a) The work has been performed in accordance with the audit program;
b) The work performed and the results obtained have been adequately documented;
c) All significant audit matters have been resolved or are reflected in audit conclusions;
d) The objectives of the audit procedures have been achieved; and
e) The conclusions expressed are consistent with the results of the work performed and
support the audit opinion.
88. The nature, timing, and extent of an audit firm's quality control policies
and procedures depend on
The Nature Appropriate
The CPA of the CPA Cost-Benefit
Firm's Size Firm's Practice Considerations
A. Yes Yes No
B. Yes Yes Yes
C. No No No
D. Yes No Yes
The nature, timing, and extent of an audit firm's quality control policies
and procedures depend on a number of factors such as the size and nature of
its practice, its geographic dispersion, its organization, and appropriate
cost/benefit considerations.
89. An audit firm should implement quality control policies and procedures
designed to ensure that all audits are conducted in accordance with PSAs
or relevant national standards or practices. These policies and procedures
should be
implemented
A. At the audit firm level only.
B. On individual audits only.
C. Either at the audit firm level or on individual audits.
D. Both at the audit firm level and on individual audits.
90. For audits of financial statements of listed entities, the engagement
partner should not issue the auditor's report
until the completion of the
A. Engagement Quality Control Review
B. Management Review
C. Engagement Team Review
D. Engagement Partner Review
Engagement quality control review, as defined in the Standard, is "a
process designed to provide an objective evaluation, on or before the date of
the report, of the significant judgments the engagement team made and the
conclusions it reached in formulating the report." The Standard provides
further that and engagement quality control review is for audits of financial
statements of listed entities, and those other engagements, if any, for which
the firm has determined an engagement quality control review is required.

91. The following statements relate to the engagement partner's


responsibility to conduct timely reviews of the audit documentation to be
satisfied that sufficient appropriate evidence has been obtained to
support the conclusions reached and for the auditor's report to be issued.
Which is false?
A. The engagement partner's review of the audit documentation allows
significant matters to be resolved on a timely basis to his/her
satisfaction before the auditor's report is issued.
B. The engagement partner should review all audit documentation.
C. The engagement partner should document the extent and timing of
the reviews.
D. The reviews cover critical areas of judgment, especially those relating
to difficult or contentious matters identified during the course of the
engagement, significant risks, and other areas the engagement partner
considers important.
The engagement partner is not required to review all audit documentation.
92. The engagement partner should be satisfied that appropriate procedures
regarding the acceptance and continuance of client relationships and
specific audit engagements have been followed, and that conclusions
reached in this regard are appropriate and have been documented.
Acceptance and continuance of client relationships and specific audit
engagements include considering:
I. The integrity of the principal owners, key management, and
those charged with governance of the entity.
II. Whether the engagement team is competent to perform the audit
engagement and has the necessary time and resources.
III. Whether the firm and the engagement team can comply with
ethical requirements.
A. I only
B. I and II only
C. II and III only D. I, II, and III
93. The engagement partner should take responsibility for the direction,
supervision, and performance of the audit engagement in compliance
with professional standards and regulatory and legal requirements, and
for the auditor's report that is issued to be appropriate in the
circumstances.
Supervision includes the following except,
A. Tracking the progress of the audit engagement.
B. Addressing significant issues arising during the audit engagement,
considering their significance, and modifying the planned approach
appropriately.
C. Informing the members of the engagement team of their responsibilities.
D. Identifying matters for consultation or consideration by more experienced
engagement team members during the audit engagement.
The engagement partner directs the audit engagement by informing the
members of the engagement team of their responsibilities; the nature of the
entity's business; riskrelated issues; problems that may arise; and the detailed
approach to the performance of the engagement.
In addition to those activities described in answers A, B, and D,
supervision also includes considering the capabilities and competence of
individual members of the engagement team, whether they have significant
time to carry out their work, whether they understand their instructions, and
whether the work is being carried out in accordance with the planned
approach to the audit engagement.

94. Who should take responsibility for the overall quality on each audit
engagement?
A. Engagement quality control reviewer
B. Engagement partner
C. Engagement team
D. CPA firm
PSA 220 provides that the engagement partner should take responsibility
for the overall quality on each audit engagement to which that partner is
assigned. As defined in this Standard, the engagement partner is "the partner
or the person in the firm who is responsible for the audit engagement and its
performance, and for the auditor's report that is issued on behalf of the firm,
and who, where required, has the appropriate authority from a professional,
legal or regulatory body,"

95. The implementation of quality control procedures that are applicable to


the individual audit engagement is the responsibility

A. CPA firm
B. Engagement quality control reviewer
C. Engagement team
D. Expert contracted by the firm in connection with the audit engagement

PSA 220 states that the engagement team should implement quality
control procedures that are applicable to the individual audit
engagement. The engagement team is composed of all personnel
(partners and staff) performing an audit engagement, including any
experts contracted by the firm in connection with that audit
engagement.

A CPA firm has an obligation to establish a system of quality control


designed to provide it with reasonable assurance that the firm and its
personnel comply with professional standards and regulatory and legal
requirements, and that the auditors' reports issued by the firm or
engagement partners are appropriate in the circumstances. (PSQC l)

THE AUDITOR'S RESPONSIBILITIES RELATING TO FRAUD IN AN


AUDIT OF FINANCIAL STATEMENTS
96. Misstatements in the financial statements can arise from fraud or error.
The distinguishing factor between fraud and error is whether the
underlying action that results in the
misstatement of the financial statements is
A. Simple or complex
B. Intentional or unintentional
C. Voluntary or involuntary
D. Planned or unplanned
The term "error" refers to an unintentional misstatement in
financial statements. The term "fraud" refers to an intentional act
by one or more individuals among management, those charged
with governance, employees, or third parties, involving the use of
deception to obtain an unjust or illegal advantage.

97. "Error" includes


A. Engaging in complex transactions that are structured to
misrepresent the financial position or financial performance
of the entity.
B. Concealing, or not disclosing, facts that could affect the
amounts recorded in the financial statements.
C. An incorrect accounting estimate arising from oversight or
misinterpretation of facts.
D. Intentional misapplication of accounting policies relating to
amounts, classification, manner of presentation, or disclosure.

The term "error" refers to an unintentional misstatement in


financial statements, including the omission of an amount or a
disclosure, such as the following:
• A mistake in gathering or processing data from which financial
statements are prepared.
• An incorrect accounting estimate arising from oversight or
misinterpretation of facts.
• A mistake in the application of accounting principles relating to
measurement, recognition, classification, presentation, or disclosure.

98. Fraud involving one or more members of management or those charged


with governance is referred to as
A. Management fraud
B. Employee fraud
C. Fraudulent financial reporting
D. Misappropriation of assets
99. The auditor is concernned with fraud that causes a material misstatement
in the financial statements. There are two types of intentional
misstatements that are relevant to the auditor: misstatements resulting
from fraudulent financial reporting and misstatements resulting from
A. Management fraud
B. Employee fraud
C. Misappropriation of assets
D. Collusion within the entity or with third parties

100. Fraudulent financial reporting involves intentional misstatements


including omissions of amounts or disclosures in financial statements to
deceive financial statement users. It may be accomplished in a number of
ways, including
A. Embezzling receipts.
B. Stealing physical assets or intellectual property.
C. Using an entities assets for personal use.
D. Manipulation, falsification, or alteration of accounting records or supporting
documentation from which the financial statements are prepared

Fraudulent financial reporting may be accomplished by the following:


• Manipulation, falsification (including forgery), or
alteration of accounting records or supporting
documentation from which the financial statements are
prepared.

• Misrepresentation in, or intentional omission from, the


financial statements of events, transactions, or other
siB1ificant information.

• Intentional misapplication of accounting principles


relating to amounts, classification, manner of
presentation, or disclosure.

The answers in A, B, and C involve misappropriation of assets.

101. Which of the following conditions are generally present when


misstatements due to fraud occur? I. Incentive or pressure
II. Perceived opportunity
III. Rationalization
A. I and II only
B. II and only
C. I and III only
D. I, II, and III
Fraud involves the incentive or pressure to commit fraud, a
perceived opportunity to do so, and some rationalization of the act.
102. Individuals may have an incentive or be under pressure to commit fraud,
or circumstances may provide an opportunity. Also, certain individuals
may have an attitude, character,
or set of values that allow them to rationalize fraud. The auditor's concern
about the risk of material misstatement is least likely to be increased if
management
A. Is interested in inappropriate means of minimizing reported earnings for tax
purposes.
B. Commits to unduly aggressive forecasts,
C. Operating and financing decisions are made by numerous individuals.
D. Has an excessive interest in increasing the entity’s share price through the
application of unduly aggressive accounting practices.
103. Three conditions are generally present when fraud occurs. Which of the
following is not one of them?
A. Attitude or rationalization about the act of fraud.
B. Opportunity to commit fraud.
C. Professional skepticism about the likelihood of fraud.
D. Incentive or pressure to commit fraud.

104. Which of the following is a required audit planning procedure


concerning potential fraud?
A. Consider whether estimates prepared and recorded by management
could indicate a biased reporting.
B. Consider the nature of journal entries, particularly those made near the
end of the reporting period.
C. Document the results of procedures used to address the risk of fraud.
D. Conduct discussions among the members of the audit team regarding
the risks of material misstatement due to fraud or error.

105. Which of the following is a false statement concerning fraud?


A. Fraud generally involves incentive or pressure to commit fraud, a
perceived opportunity to do so, and some rationalization of the
act.
B. Two types of misstatements relevant to the auditor include
material misstatements arising from fraudulent financial reporting
and material misstatements arising from misappropriation of
assets.
C. Fraud involves actions of management but excludes the actions of
employees or third parties.
D. An audit rarely involves the authentication of document ; thus,
fraud may go undetected by the auditor.

106. Why is computer fraud often much more difficult to detect than other
types of fraud?
A. Perpetrators can commit a fraud and leave little or no evidence.
B. Perpetrators usually only steal very small amounts of money at a time,
thus requiring a long period of time to have elapsed before they are
discovered.
C. Most computer criminals are older and are considered to be more
cunning when committing such a fraud.
D. Most perpetrators invest their illegal income rather than spend it, thus
concealing key evidence.

107. A classification of fraud where the perpetrator causes a company to pay


too much for ordered goods, or to pay for
goods never ordered is called
A. Payroll fraud
B. Disbursement fraud
C. Cash receipts fraud
D. Inventory fraud

108. In payroll fraud, funds can be stolen by


I. Paying a fictitious or ghost employee.
Il. Increasing pay rates without permission.
Ill. Keeping a real but terminated employee on the payroll.

A. I and II only
B. I and III only
C. II and III only
D. I, II, and III
109. Stealing a master list of customers and selling it to a competitor is an
example of what classification of fraud?
A. Output theft
B. Data theft
C. Disbursement fraud
D. Cash receipt fraud

110. The primary responsibility for the prevention and detection of fraud rests
with
A. Those charged with governance of the entity.
B. Management of the entity.
C. Both those charged with governance of the entity and management. D. The auditor.

111. The following are examples of misappropriation of assets, except

A. The treasurer diverts customer payments to his personal due,


concealing his actions by debiting an expense account, thus
overstating expenses.
B. An employee steals inventory and the "shrinkage" is recorded in
cost of goods sold.
C. An employee steals small tools from the company and neglects to
return them; the cost is reported as a miscellaneous operating
expense.
D. Company management changes inventory count tags and
overstates ending inventory, while understating cost of goods
sold.

Changing inventory count tags resulting in misstatement of inventory


and cost of goods sold is an example of fraudulent financial reporting.

112. Which of the following statements best describes an auditor's


responsibility regarding misstatements?
A. An auditor should obtain reasonable assurance that the financial
statements taken as a whole are free from material misstatement,
whether caused by fraud or error.
B. An auditor should obtain absolute assurance that material
misstatements in the financial statements will be detected.
C. An auditor is responsible to detect material errors but has no
responsibility to detect material fraud that is concealed through
employee collusion or management override of internal control.
D. An auditor's failure to detect a material misstatement resulting
from fraud is an indication of noncompliance with the
requirements of the Philippine Standards on Auditing (PSAs).
PSA 240 states, "An audit conducted in accordance with PSAs is
designed to provide reasonable assurance that the financial statements
taken as a whole are free from material misstatement, whether caused
by fraud or error."

113. When obtaining an understanding of the entity and its environment,


including its internal control, the auditor may identify events or
conditions that indicate an incentive or pressure to commit fraud or
provide an opportunity to commit fraud. Such events or conditions are
referred to as
A. Fraud conditions
B. Fraud risk factors
C. Fraudulent activities
D. Fraud environment

114. The following are examples of fraud risk factors relating to


misstatements arising from misappropriation of assets, except
A. Recurring negative cash flows from operating activities while reporting
earnings and earnings growth.
B. Inadequate physical safeguards over cash, investments, inventory, or fixed
assets.
C. Inadequate segregation of duties or independent checks.
D. Adverse relationship the entity and employees with access to cash or
other assets susceptible to theft created by recent changes made to
employee compensation or benefit plans.

Recurring negative cash flows while reporting earnings and earnings


growth is a fraud risk factor relating to

fraudulent financial reporting.

115. Opportunities to misappropriate assets increase when there are


A. Known or anticipated future employee layoffs.
B. Promotions, compensation, or other rewards inconsistent with expectations.
C. Recent or anticipated changes to employee compensation or benefit plans.
D. Inventory items that are small in size, of high value, or in high demand.
Fraud risk factors are classified according to the three conditions generally present
when fraud exists: (1) incentives/pressures, (2) opportunities, and (3)
attitudes/rationalizations.

Opportunities to misappropriate assets increase when there are:


• Inventory items that are small in size, of high value, or in high demand.
• Large amounts of cash on hand or processed.
• Easily convertible assets, such as bearer bonds, computer chips, or
diamonds.
• Fixed assets which are small in size, marketable, or lacking observable
identification of ownership.

Answers A, B, and C are incorrect because they may create adverse


relationships between the entity and employees that may motivate them to
misappropriate the entity's assets.

116. Which of the following conditions or events may create


incentives/pressures to commit fraud?
A. Inadequate system of authorization and approval of transactions.
B. Lack of mandatory vacations for employees performing key
control functions.
C. Excessive pressure on management or operating personnel to meet
financial targets established by those charged with governance,
including sales or profitability incentive goals.
D. Inadequate access controls over automated records.

Answers A, B, and D are incorrect because they create opportunities to commit fraud.

117. Because of the risk of material misstatement, an audit of financial


statements in accordance with PSAs should be planned and performed
with an attitude of A. Impartial conservatism
B. Objective judgment
C. Independent integrity
D. Professional skepticism

As required by PSA 200, the auditor plans and performs the audit with
an attitude of professional skepticism recognizing that circumstances
may exist that cause the financial statements to be materially misstated.
Professional skepticism includes a questioning -mind and critical
assessment of audit evidence.

118. Which of the following statements describes why an audit that is


properly planned and performed in accordance with PSAs may not detect
a material misstatement resulting from fraud?
A. Fraud may involve carefully laid out plans of concealment.
B. The auditor did not consider audit risk factors for accounts
having pervasive effects on the financial statements.
C. An audit is designed to provide reasonable assurance of
detecting misstatements arising from errors, but there is no
similar responsibility concerning material misstatements
resulting from fraud.
D. The risk of the auditor not detecting a material misstatement
resulting from employee fraud is greater than for management
fraud.
PSA 240 states, "The risk of not detecting a material misstatement
resulting from fraud is higher than the risk of not detecting a material
misstatement resulting from error because fraud may involve sophisticated
and carefully organized schemes to conceal it, such as forgery, deliberate
failure to record transactions, or intentional misrepresentations being made
to the auditor. Such attempts at concealment may be even more difficult to
detect when accompanied by collusion. Collusion may cause the auditor to
believe that evidence is persuasive when it is, in fact, false."
119. When planning the audit, the auditor should make inquiries of
management. Such inquiries should address the following, except
A. Management's assessment of the risk that the financial statements
may be misstated due to fraud.
B. Management's process for identifying and responding to the risks of
fraud in the entity.
C. Management's consideration of how an element of unpredictability
will be incorporated into the nature, timing, and extent of the audit
procedures to be performed.
D. Management's communication, if any, to those charged with
governance regarding its processes for identifying and responding to
the risks of fraud in the entity.

The consideration of how an element of unpredictability will be


incorporated into the nature, timing, and extent of the audit procedures
to be performed is a matter to be discussed by the auditor with other
members of the audit team.

120. Which of the following circumstances most likely would cause an


auditor to consider whether material misstatements exist in an entity's
financial statements?
A. Those charged with governance exercise oversight of
management's processes for identifying and responding to the
risks of fraud in the entity and the internal control that
management has established to mitigate these risks.
B. Significant, unusual, or highly complex transactions, especially
those close to an entity's financial year-end that pose difficult
"substance over form" questions.
C. Operating profits making the threat of bankruptcy, foreclosure, or
hostile takeover remote. Low vulnerability to changes in
technology, product obsolescence, or interest rates.
Significant, unusual, or highly complex year-end transactions may
provide opportunities to engage in fraudulent financial reporting.

121. Which of the following characteristics most likely would heighten an


auditor's concern about the risk of material misstatement arising from
fraudulent financial reporting?
A. Excessive interest by management in increasing stock price or
earnings trend through aggressive accounting practices.
B. Effective accounting and internal control systems.
C. Low turnover of senior management, legal counsel, or those charged
with governance.
D. Management is dominated by a single person or a small group with
compensating controls such as effective oversight by those charged
with governance.
122. When the auditor encounters circumstances that may indicate that there
is a material misstatement in the financial statements resulting from
fraud or error, the auditor should perform procedures to determine
whether the financial statements are materially misstated. The nature,
timing, and extent of the procedures to be performed depend on the
auditor's judgment as to the
A B C D
Type of fraud or error indicated No Yes Yes No
Likelihood of occurrence Yes No Yes No Likelihood that a
particular type of fraud or error could have a material effect on
the financial statements Yes No Yes No

123. When the auditor identifies a misstatement in the financial statements,


the auditor should consider whether such a misstatement may be
indicative of fraud and if there is such an indication, the auditor should
A. Consider the implications of the misstatement in relation to other aspects of the
audit.
B. Withdraw from the
C. Communicate information to regulatory and enforcement authorities.
D. Report the matter to the person or persons who made the audit appointment.

PSA 240 requires the auditor to consider the implications of the


misstatement in relation to other aspects of the audit, particularly the
reliability of management representations.

124. PSA 230 (Audit Documentation) requires the auditor to document


matters which are important in providing evidence to support the audit
opinion, and states that working papers include the auditor's reasoning
all significant matters which require the auditor's judgment, together
with the auditor's conclusion thereon. Which of the following should be
documented by the auditor?
A. Fraud risk factors identified as being present during auditor's risk
assessment
B. Auditor's to fraud risk factors
C. Both fraud risk factors identified as being present during the auditor's risk
assessment and the auditor's response to any such factors.
D. The standard does require documentation of the identified fraud risk factors
and the auditor's responses to them,
PSA 240 states that because of the importance of fraud risk factors
in the assessment of the inherent or control risk of material misstatement,
the auditor documents fraud risk factors identified and the response
considered appropriate by the auditor.
125. Because of the nature of fraud and the difficulties encountered by
auditors in detecting material misstatements in the financial statements
resulting from fraud, the auditor should obtain written representations
from management. The following should be confirmed by management
in its written representations, except
A. It is not responsible for the implementation and operations of
internal control that is designed to prevent and detect fraud.
B. It has disclosed to the auditor its knowledge of any allegations
of fraud or suspected fraud, affecting the entity's financial
statements communicated by employees, former employees,
analysts, regulators, or others.
C. It has disclosed to the auditor its knowledge of fraud or
suspected fraud affecting the entity.
D. It has disclosed to the auditor the results of its assessment of
the risk that the financial statements may be materially
misstated as a result of fraud.

The standard states that in addition to acknowledging its responsibility


for the financial statements, it is important that, irrespective of the size
of the entity, management acknowledges its responsibility for internal
control designed and implemented to prevent and detect fraud.

126. Which of the following statements concerning the auditor's


responsibility to detect conditions relating to financial stress of
employees or adverse relationships between a company and its
employees is correct?
A. The auditor is required to plan the audit to detect these conditions
whenever they may result in misstatements.
B. The auditor is required to plan the audit to detect these conditions
on all audits.
C. These conditions relate to fraudulent financial reporting, and an
auditor is required to plan the audit to detect these conditions when
the client is exposed to a risk of misappropriation of assets
D. The auditor is not required to plan the audit to discover these
conditions, but should consider them if he/she becomes aware of
them during the audit. The financial stress of employees or adverse
relationships between a company and its employees are conditions
that indicate an incentive or pressure to commit fraud. PSA 240
states that using the auditor's knowledge of the business, the auditor
may identify events or conditions that provide an opportunity, a
motive, or a means to commit fraud, or indicate that fraud may
already have occurred. Such events or conditions are referred to as
"fraud risk factors."

In designing the substantive procedures, the auditor should address


the fraud risk factors that he/she has identified as being present. The
standard, therefore, does not specifically require the auditor to plan
the audit to discover fraud risk factors, but should consider them if
he/she becomes aware of them during the audit.

127. The following statements relate to communication of misstatements


resulting from fraud to management and to those charged with
governance. Which is false?
A. The auditor need not bring to the attention of those charged
with governance any material weaknesses in internal control
related to the prevention and detection of fraud.
B. If the auditor has identified a fraud, whether or not it re suits in
a material misstatement in the financial statements, the auditor
should communicate these matters to the appropriate level of
management on a timely basis, and consider the need to report
such matters to those charged with governance.
C. If the auditor has obtained evidence that indicates that fraud may
exist (even if the potential effect on the financial statements
would not be material), the auditor should communicate these
matters to the appropriate level of management on a timely
basis, and consider the need to report such matters to those
charged with governance.
D. The auditor's communication with those charged with
governance may be made orally or in writing. The standard
states that the auditor should make those charged with
governance and management aware, as soon as practicable, and
at the appropriate level of responsibility, of material weaknesses
in the design and implementation of internal control to prevent
and detect fraud which may have come to the auditor's attention.

CONSIDERATION OF LAWS AND REGULATIONS IN AN AUDIT OF FlNANCIAL


STATEMENTS

128. As used in PSA 250, this term refers to acts of omission or commission
by the entity being audited, either intentional or unintentional, which are
contrary to prevailing laws or regulations.
A. Noncompliance
B. Illegal acts
C. Deplorable acts D. Unforgivable acts
129. According to PSA 250, the term "noncompliance" as used in the standard
refers to acts of omission or commission by the entity being audited,
either intentional or unintentional, which are contrary to the prevailing
laws or regulations. Such acts do not include
A. Transactions entered into by the entity.
B. Transactions entered into in the name of the entity.
C. Transactions entered into on the entity's behalf by its
management or employees.
D. Personal misconduct (unrelated to the entity's business activities)
by the entity's management or employees.
130. The responsibility for the prevention and detection of noncompliance
rests with
A. The auditor
B. Management
C. The auditor's lawyer
D. The client's lawyer
PSA 250 states that it is management's responsibility to ensure that the
entity's operations are conducted in accordance with laws and
regulations.

131. Which of the following statements best describes why the auditor's
examination cannot reasonably be expected to bring all acts of
noncompliance with existing laws and regulations by the client to the
auditor's attention?
A. Acts of noncompliance by clients often relates to accounting,
aspects rather than operating aspects.
B. Noncompliance may involve conduct designed to conceal it, such
as collusion, forgery, deliberate failure to record transactions,
senior management override of controls, or intentional
misrepresentations being made to the auditor.
C. Noncompliance may be perpetrated by the only person in the
client's organization with access to both assets and the accounting
records.
D. The client's internal control may be so strong that the auditor
performs only minimal substantive testing.

132. PSA 250 states that in order to plan the audit, the auditor should obtain a
general understanding of the legal and regulatory framework applicable
to the entity and the industry and how the entity is complying with that
framework. To obtain this understanding, the following procedures
would ordinarily be considered by the auditor, except

A. Use the existing understanding of the entity's industry, regulatory,


and other external factors.
B. Inquire of management concerning the entity's policies and
procedures regarding compliance with laws and regulations.
C. Inquire of management to the laws and regulations that may be
expected to have a fundamental effect on the operations of the entity.
D. Inspect correspondence with relevant licensing or regulatory
authorities.
Inspecting correspondence with the relevant licensing or regulatory
authorities is a procedure to identify instances of noncompliance with
laws and regulations.

133. When the auditor becomes aware of information concerning a possible


instance of noncompliance, the auditor should obtain an understanding
of A B
C D
The nature of the act No Yes Yes No
The circumstances in which it has

occurred No Yes No Yes


Sufficient other information to evaluate the
possible effect on
financial statements Yes Yes No No

134. Which of the following statements is incorrect concerning reporting of


noncompliance?
A. The auditor, as soon as practicable, either communicate with those
charged with governance, or obtain evidence that they are
appropriately informed, regarding noncompliance that comes to
the auditor's attention.
B. If the auditor suspects that members of senior management,
including members of the board of directors, are involved in
noncompliance, the auditor should report the matter to the next
higher level of authority at the entity, if it exists, such as an audit
committee or a supervisory board.
C. The auditor should, as soon as practicable, communicate with
those charged with governance regarding noncompliance,
including matters that are clearly inconsequential or trivial.
D. If in the auditor’s judgement, the noncompliance is believed to be
intentional and material, the auditor should communicate the
finding without delay.
135. If the auditor concludes that the noncompliance has a material effect on
the financial statements, and has not been properly reflected in the
financial statements, the auditor should express
A. A qualified or an adverse opinion
B. A qualified opinion or a disclaimer of opinion
C. A disclaimer of opinion
D. A qualified opinion
136. If the auditor is precluded by the entity from obtaining sufficient
appropriate audit evidence to evaluate whether noncompliance that may
be material to the financial statements, has, or is likely to have, occurred,
the auditor should express
A. A qualified opinion or an adverse opinion
B. A qualified opinion or a disclaimer of opinion
C. An adverse opinion
D. An adverse opinion or a disclaimer of opinion
137. Under which of the circumstances below would the auditor conclude that
withdrawal from the engagement is necessary?
A. The auditor concludes that the noncompliance has a material effect
on the financial statements and has not been properly reflected in
the financial statements.
B. The auditor is precluded by the entity from obtaining sufficient
appropriate audit evidence to evaluate whether noncompliance that
may be material to the financial statements, has, or is likely to
have, occurred.
C. The auditor is unable to determine whether noncompliance has
occurred because of limitations imposed by the circumstances
rather than by the entity.
D. The entity does not take the remedial action that the auditor
considers necessary in the circumstances.
According to PSA 250, the auditor may consider that withdrawal from
the engagement is necessary when the entity does not take the remedial
action that the auditor considers necessary in the circumstances, even
when the noncompliance is not material to the financial statements.

COMMUNICATION WITH THOSE CHARGED WITH GOVERNANCE


138. Under PSA 260, this term is used to describe the role of persons
entrusted with the supervision, control, and direction of an entity.
A. Oversight
B. Governance
C. Direction
D. Control
PSA 260 states that for the purposes of this PSA, "governance" is the
term used to describe the role of persons entrusted with the supervision,
control, and direction of an entity. It states further that those charged
with governance ordinarily are accountable for ensuring that the entity
achieves its objectives with regard to reliability of financial reporting,
effectiveness and efficiency of operations, compliance with applicable
laws, and reporting to interested parties.
139. According to PSA 260, those matters that arise from the audit of
financial statements and, in the opinion of the auditor, are both important
and relevant to those charged with governance in overseeing the
financial reporting and disclosure process are catted
A. Audit matters of governance interest
B. Significant audit matters
C. Auditor's findings
D. Material misstatements in the financial statements
140. Which the following statements relating to communication of audit
matters of governance interest is incorrect?
A. Audit matters of governance interest include only those matters that
have come to the attention of the auditor as a result of the
performance of the audit.
B. In an audit in accordance with PSAs, the auditor should design audit
procedures for the specific purpose of identifying matters of
governance interest.
C. The auditor should identify relevant persons who are charged with
governance and with whom audit matters of governance interest are
to be communicated.
D. The auditor's communications with those charged with governance
may be made orally or in writing.
PSA 260 states that audit matters of governance interest include only those
matters that have come to the attention of the auditor as a result of the
performance of the audit. The auditor is not required, in an audit in
accordance with PSAs, to design audit procedures for the specific purpose
of identifying matters of governance interest.

141. Audit matters of governance interest to be communicated to those


charged with governance ordinarily include

I. Audit adjustments, whether or not recorded by the entity that have,


or could have, a material effect on its financial statements.
II. Expected modifications to the auditor's report.
III. Material uncertainties related to events and conditions that may cast
significant doubt on the entity's ability to continue as a going
concern.
A. I only
B. II and III only

C. l and IIII only


D. I, II, and III

In addition to those described in answers A, B, and C, audit matters of governance


interest include the
following:
o The general approach and overall scope of the audit, including any
expected limitations thereon, or any additional requirements.
o The selection of, or changes in, significant accounting policies and
practices that have, or could have, a material effect on the entity's
financial statements.
o The potential effect on the financial statements of any material risks
and exposures, such as pending litigation, that are required to be
disclosed in the financial statements.
o Disagreements with management about matters that, individually or
in aggregate, could be significant to the entity's financial statements
or the auditor's report. These communications include consideration
of whether the matter has, or has not, been resolved and the
significance of the matter.
o Other matters warranting attention by those charged with
governance, such as material weaknesses in internal control,
questions regarding management integrity, and fraud involving
management.
o Any other matters agreed upon in the terms of the audit
engagement.
142. PSA 260 requires the auditor to determine the relevant persons who are
charged with governance and with whom audit matters of governance
interest are communicated, for corporations covered by the SEC Code of
Corporate Governance, which of the following is primarily responsible
for corporate governance?
A. President
B. Controller
C. Board of Directors
D. Management
The Code of Corporate Governance promulgated by the
Securities and Exchange Commission (SEC) states, "The Board of
Directors is primarily responsible for the governance of the
corporation."

143. The auditor shall communicate with those charged with governance
his/her responsibilities in relation to the audit of the entity's financial
statements, including that

I. The auditor is responsible for forming and expressing an opinion


on the financial statements. Il. The audit of the financial statements
does not relieve management or those charged with governance of
their responsibilities.

A. I only
B. II only
C. Neither I nor II
D. Both I and II

144. Which of the following matters will an auditor most likely communicate
to those charged with governance?
A. The level of responsibility assumed by management for the
preparation of the financial statements.
B. The effects of significant accounting policies adopted by management
in emerging areas for which there is no authoritative guidance.
C. A list of negative trends that may lead to working capital deficiencies
and adverse financial ratios.
D. Difficulties encountered in achieving a satisfactory response rate from
the entity's customers in confirming accounts receivables.

145. Which of the following matters is an auditor required to communicate to


those charged with governance?
I. Disagreements with management about matters significant to the entity's
financial statements that have been satisfactorily resolved.
II. Material weaknesses in internal control.
A. I only Both
B. II only
C. I and II
D. Neither I nor II

TRUE OR FALSE

1. Familiarity threat is a threat that a CPA will promote a client’s or employer’s


position to the point that his/her objectivity is compromised.

2. Safeguards fall into two broad categories: safeguards in the work environment
and firm-wide safeguards.

3. The principle of professional behavior imposes an obligation on all CPAs to


comply with relevant laws and regulations and avoid any action that may
discredit the profession.
4. When identified threats are not an acceptable level, a CPA may rely solely on
safeguards that his/her client has implemented.

5. A primary purpose for establishing a code of professional ethics is to demonstrate


acceptance of responsibility to the interests of those served by the profession.

6. Safeguards implemented by the firm, including policies and procedures to


implement professional and regulatory requirements, may be applied to mitigate
or eliminate threats to independence and other ethical principles.

7. The auditor must be independent of the audit client unless the lack of
independence does not influence his or her professional judgment.

8. With regard to detecting fraud, auditing standards require auditors to issue an


unmodified opinion only when the auditor is satisfied that no instances of fraud
have occurred.

9. Due professional care requires auditors to plan and perform their duties with the
skill and care that is commonly expected of accounting professionals.

10. Fraud is either an intentional or unintentional misstatement of the financial


statements, depending on the materiality and consistency.

11. A factor that relates to opportunities to commit fraudulent financial reporting is


lack of controls relating to the calculation and approval of accounting estimates.

12. When determining whether independence is impaired because of an ownership


interest in a client company, materiality will affect ownership only for direct
ownership.

13. A direct financial interest violates independence when close relatives such as a
brother, sister, or in-laws are employed by the client.

14. If an entity asks a CPA to perform a review engagement, and the CPA has an
immaterial direct financial interest in the entity, the CPA is not independent and,
therefore, may not issue a review report.
KEY ANSWERS 41. C 83. A 125. A
42. B 84. C 126. D
1. B 43. C 85. C 127. A 2. B 44. B 86. B 128. A 3. D
45. D 87. B 129. D 4. B 46. A 88. D 130. B 5. C 47. D
89. A 131. B 6. A 48. C 90. B 132. D 7. A 49. C 91. D
133. B 8. D 50. D 92. C 134. C 9. D 51. A 93. B 135. A
10. A 52. B 94. C 136. B 11. A 53. A 95. B 137. D 12. A 54.
B 96. C 138. B 13. A 55. A 97. A 139. A 14. C 56. D 98. C
140. B 15. A 57. C 99. D 141. D
16. A 58. A 100. D 142. C 17. D 59. C 101. C 143. D 18. D 60.
B 102. C 144. B
19. B 61. A 103. D 145. C
20. A 62. B 104. C
21. B 63. A 105. A
22. C 64. C 106. A TRUE OR FALSE
23. C 65. D 107. B
24. A 66. B 108. D
25. C 67. A 109. B 1. False
26. A 68. C 110. C 2. False
27. C 69. C 111. D 3. True
28. D 70. A 112. A 4. False
29. D 71. B 113. B 5. True
30. A 72. C 114. A 6. True
31. D 73. C 115. D 7. False
32. B 74. B 116. C 8. False
33. D 75. C 117. D 9. True
34. D 76. B 118. A 10. False
35. C 77. B 119. C 11. True
36. C 78. C 120. B 12. False
37. C 79. B 121. A 13. False
38. A 80. A 122. C 14. True
39. A 81. D 123. A
40. C 82. A 124. C
CHAPTER 4
The Financial Statement Audit: Client Acceptance and Planning
1. In assessing whether to accept a client for an audit engagement, an auditor
should consider the
I. Client’s business risk
II. Auditor’s business risk
A. I only
B. II only
C. Both I and II
D. Neither I nor II

2. Which of the following factors most likely would cause an auditor to decline a
new audit engagement?

A. Conducting that the entity’s engagement probably lacks integrity.


B.An inability to perform preliminary analytical procedures before assessing control
risk.
C.An inadequate understanding of the entity’s internal control. D. The close proximity
to the end of the entity’s reporting period.

PSQC 1 requires a firm (including sole practitioner) to establish policies and


procedures for the acceptance and continuance of client relationships and specific
engagements, designed to provide reasonable assurance that it will only accept
engagements if it:
1.) Is competent to perform the engagement;
2.) Can comply with relevant ethical requirements; and
3.) Has considered the integrity of the client, and does not have information to
conclude that the entity’s management lacks integrity.
Answer B is incorrect because the auditor performs analytical procedures after
accepting engagement.
Answer C is incorrect because the auditor obtains an understanding of the entity’s
internal control system subsequent to the acceptance of the engagement.
Answer D is incorrect because an auditor may accept an engagement near or after
the end of the reporting period.

3. Before accepting an engagement to audit a new client, an auditor is required to


A. Obtain a copy of the client’s financial statements.
B. Prepare a memorandum setting forth the staffing requirements and
documenting the preliminary audit plan.
C. Make of the predecessor auditor after obtaining the consent of the prospective
client.
D. Discuss the management representation letter with the client’s audit
committee.

PSA 300 (Planning an Audit of Financial Statements) states that the auditor shall
undertake the following activities prior to an initial audit:
1.) Performing procedures required by PSA 220 (Quality Control for an Audit of
Financial Statements) regarding the acceptance of the client relationship and the
specific audit engagement; and
2.) Communicating with he predecessor auditor, where there has been a change of
auditors, in compliance with relevant ethical requirements.
Answer A is incorrect because the entity’s annual financial statements will be
prepared after the end of its reporting period and the auditor’s acceptance of the
engagement will likely be prior to that time.
Answers B and D are incorrect because the procedures described will be
performed only after the acceptance of engagement.

4. Which of the following conditions most likely would pose the greatest risk in
accepting a new audit engagement?
A. There will be a client-imposed scope limitation.
B. The client’s financial reporting system has been in place for 10 years.
C. The firm will have to hire an expert in one audit area.
D. Staff will need to be rescheduled to cover this new client.

According to PSA 210 (Agreeing the Terms of Audit Engagements), the auditor
shall not accept the engagement if management or those charged with governance
impose the limitation on the scope of the auditor’s work in the terms of a proposed
audit engagement such that the auditor believes the limitation will result in the
auditor disclaiming an opinion on the financial statements.
5. Which of the following circumstances would permit an independent auditor to
accept an engagement after the end of the reporting period?
A. Expectation of the operating effectiveness of controls.
B. Issuance of a disclaimer opinion as a result of inability to conduct certain tests
required by PSAs due to the timing of acceptance of the engagement.
C. Remedy the limitations resulting from accepting the engagement after the end
of the reporting period, such as those relating to the existence of physical
inventory.
D. Receipt of an assertion from the predecessor auditor that the entity will be
able to continue as a going concern.
Prior to accepting apropos audit engagement subsequent to the end of the entity’s
reporting period, the auditor should determine whether the circumstances permit
an audit with accordance with PSAs and expression of an unmodified opinion.
Otherwise, the auditor should discuss with the prospective client the possibility of
rendering a qualified opinion or a disclaimer of opinion. However in some cases,
the auditor may remedy the audit limitations uch as by observing another physical
count of inventories.
Answer A is incorrect because sufficient appropriate evidence regarding the
operating effectiveness of the entity internal control during the year should be
available after the year-end.
Answer B is incorrect because a disclaimer of opinion is appropriate only if the
auditor cannot resolve an issue by performing alternative procedures.
Answer D is incorrect because there is no need to obtain representations regarding
the prospective client’s ability to continue as going concern from the predecessor
auditor.

6. In an audit based on Philippine Standards of Auditing (PSAs), a successor


auditor would normally become satisfied with opening balances by
A. Performing analytic review procedures.
B. Reviewing the predecessor’s working paper.
C. Auditing the previous year’s working papers.
D. Interviewing client personnel.

7. A predecessor withdrew from the engagement after discovering that the client’s
financial statements are materially misstated that it would not revise. If asked by
the successor auditor about the termination of the engagement, the predecessor
should
A. Suggest that the successor auditor should obtain the client’s consent to discuss
the reasons.
B. Indicate that there was a misunderstanding.
C. State that the audit revealed material misstatement that the client would not
revise.
D. Suggest that the successor auditor ask the client.

8. Which of the following is not correct regarding the communications between


successor/incoming and predecessor/previous auditors?

A. The burden of initiating the communication rests with predecessor auditor.


B.The burden of initiating the communication rests with successor auditor.
C.The predecessor auditor may choose to a limited response to a successor auditor.
D. The predecessor auditor must received his/her former client’s permission prior to
disclosing client information to the auditor.
9. The auditor may accept or continue an audit engagement only when the basis
upon which it is to be performed has been agreed, through
I. Establishing whether the preconditions for audit are present.
II. Confirming that there is a common understanding between the auditor and
management and, where appropriate those charge with governance of the terms of
the audit engagement.
A. I only
B. II only
C. Both I and II
D. Neither I nor II

As defined in PSA 210 (Agreeing the Terms of Audit Engagements),


“preconditions for an audit” refers to:
a.) The use of management of an acceptable financial reporting
framework in the preparation of the financial statements; and
b.) The agreement of management and, where appropriate those charged
with governance to the premise of which audit is conducted.
The auditor establishes whether the preconditions for audit are present by:

1.) Determining whether the financial reporting framework to be applied in


preparation of financial statements is acceptable; and
2.) Obtaining management’s agreement that it acknowledges and understands
its responsibilities that are fundamental to the conduct of an audit n
accordance with PSAs.

10. An audit is conducted on the premise that management and, where


appropriate, those charged with governance have acknowledge and understand
that they have responsibilities that are fundamental to the conduct of an audit in
accordance with PSAs. Which of the following is not one of those
responsibilities?
A. The preparation of financial statements with accordance with relevant
pronouncements issued by the AASC.
B.The establishment and maintenance of an adequate internal control system that is
necessary to enable the preparation of financial statements that is free from
material misstatement, whether due to fraud or error.
C.To provide the auditor with access to all information that is relevant to the
preparation of the financial statements such as records, documentation and other
matters.
D. To provide the auditor with unrestricted access to persons within the entity from
which the auditor determines it necessary to obtain audit evidence.
Management is responsible for the preparation of the financial statement in
accordance with the applicable financial reporting framework, including where
relevant their fair presentation.
11. The auditor shall agree the terms of the audit engagement with management or
those charged with governance, as appropriate. The agreed terms shall be
recorded in a/an
A. Engagement letter
B. Letter of audit inquiry
C. Management representation letter
D. Confirmation letter

The standard states that it is in the interest of both the entity and the auditor that
the auditor sends an audit engagement letter before the commencement of the
audit to help avoid misunderstandings with respect to the audit. The engagement
letter shall include:

a.) The objective and scope of the audit of the financial statement;
b.) The responsibilities of the auditor;
c.) The responsibilities of the management;
d.) Identification of applicable financial reporting framework for the
preparation of financial statements; and
e.) Reference to the expected form and content of any reports to be
issued by the auditor and a statement that there may be
circumstances in which a report may differ from its expected form
and content.

12. The following matters are generally included in the auditor engagement letter,
except
A. The factors to be considered in determining the overall materiality.
B. The fact that because of the test nature and other inherent limitations of an
audit, together with the inherent limitations of internal control, there is an
unavoidable risk that even some material misstatements may remain
undiscovered.
C. The scope of the audit
D. Management’s responsibility for the financial statements.

13. The following are usually included in an auditor’s engagement letter, except
A. List of audit procedures to be used in inventory observation.
B. The financial statements are the responsibility of the company’s management.
C. A reference to PFRS.
D. A reference to PSAs.
14. Which of the following statements would least likely appear an auditor’s
engagement letter?

A. Our audit will be made with the objective of our expressing an opinion on the
financial statements.
B. We remind you that the responsibility for the preparation of financial
statements including adequate disclosure is that of the management of the
entity.
C. After performing our preliminary analytical procedures, we will discuss with
you the other procedures we consider necessary to complete the engagement.
D. Our fees, which will be billed as work progresses, are based on the time
required by individuals assigned to the engagement plus out-of-the-pocket
expenses.

15. An auditor is required to establish an understanding with a client regarding the


services to be performed for each
engagement. This understanding generally includes

A. The auditor’s responsibility to apply the concept of materiality in planning and


performing the audit.
B.Management’s responsibility for providing the auditor with an assessment of the
risk of material misstatement due to fraud. C. The auditor’s responsibility for
ensuring that those charged with governance are aware of any significant
deficiencies in internal control that may come to auditor’s attention.
D. Management’s responsibility for errors and the illegal activities of employees
that may cause material misstatement.

An auditor’s engagement letter shall include, among the other things, the auditor’s
responsibility to communicate to those charged with governance of the client
significant internal control deficiencies that may be discovered during the audit.
Answer A is incorrect because determining materiality to planning and performing
an audit is a responsibility that the auditor is not required to share with the client.
Answer B is incorrect because the auditor assesses the risk of material
misstatement.
Answer D is incorrect because management is not responsible for the errors and
illegal activities of employees.
16. An auditor’s engagement letter most likely will include
A. A request for permission to contact the client’s lawyer for assistance in identifying
litigation, claims and assessments.
B.A reminder that management is responsible for illegal acts committed by
employees.
C.The auditor preliminary assessment of the risk factors relating to misstatements
arising from fraudulent financial reporting.
D. Management’s acknowledgement of its responsibility for such internal control as
it determines is necessary to enable the preparation of financial statements that are
free from material misstatements.
Among the matters addressed in an engagement letter is management’s
responsibility for:
a.) The preparation and fair presentation of the financial statements in
accordance with Philippine Financial Reporting Standards.
b.) Such internal control as it determines is necessary to enable the
preparation of financial statements that are free from material
misstatements, whether due to fraud or error.
Answer A is incorrect because management is responsible for adopting policies
and procedures to identify, evaluate and account for litigation, claims and
assessment.
Answer B is incorrect because management is responsible for ensuring that the
entity’s operation are conducted accordance with laws and regulations. However,
it is not responsible for the illegal acts of employees that are unrelated to the
entity’s business activity.
Answer C in incorrect because an auditor submit an engagement before the
commencement of an audit. Prior to performing procedures, the auditor does not
assess the risk factors relating to misstatements that may arise from fraudulent
financial reporting.
17. The auditor of a parent entity is also the auditor of its component. Which of
the following factors may influence the auditor’s decision whether to send a
separate engagement letter to entity’s component?
A. Whether a separate auditor’s report is to be issued on the component.
B. The component’s management does not accept its responsibilities that are
fundamental to the conduct of an audit.
C. The financial reporting framework use by the component is unacceptable.
D. The preconditions for an audit of the component financial statement are not
present.

The following factors may influence the auditor’s decision whether to send a
separate audit engagement letter to the component:

1.) Who appoints he component auditor;


2.) Whether a separate auditor’s report is to be issued on the component;
3.) Legal requirements in relation to audit appointments;
4.) Degree of ownership by parent; and
5.) Degree of independence of the component management from the parent
entity.
18. On recurring audits, the auditor may decide not to send a new engagement letter
to each period. Which of the following factors may make it appropriate to send a
new engagement
letter?
A B C D

Any revised or special terms of the audit


Yes Yes
engagement
A recent change of senior
Yes
management
A change in legal or regulatory Ye
Yes Yes Yes
requirements s
A significant change in nature or size of the Ye
Yes Yes Yes
entity’s business s

According to the standard, the following factors may make it appropriate to revise
the terms of the audit engagement or to remind the entity of existing terms:

• Any indication that the entity misunderstands the objective and scope of
the audit.  Any revised or special terms of the audit  A recent change in
senior management.
• A significant change of ownership.
• A significant change in nature or size of the entity business.
• A change on legal and regulatory requirements.
• A change in the financial reporting framework adopted in the preparation
of the financial statements.

19. The auditor shall not agree to a request from the entity to change the terms of the
audit engagement or to change the audit engagement to an engagement that
conveys a lower level of assurance when there is no reasonable justification for
doing so. Which of the following may be considered reasonable justifications for
the change in audit engagement?
I. A change in circumstance affecting the need for the service.
II. A misunderstanding as to the nature of an audit as originally requested.
III. A restriction on the scope of the engagement, whether imposed by
management or caused by other circumstances.

A. I and II only
B. I and III only
C. II and III only
D. I, II and III
PSA 210 states that a change in circumstances that affects the entity’s
requirements or a misunderstanding concerning the nature of the service originally
requested may be considered reasonable basis for requesting a change in the audit
engagement.
A restriction on the scope of the audit would not be considered a reasonable
justification for change. For example, a change would not be considered
reasonable if the auditor is unable to obtain sufficient appropriate audit evidence
regarding receivable and the client requests that the audit engagement be changed
into a review engagement to avoid a qualified opinion or a disclaimer of opinion.

20. Before the completion of the audit engagement, an auditor is requested to


change the engagement to one that provides a lower level of assurance. If the
auditor concludes that there is a reasonable justification for the change of
engagement, the report to be issued would
A. Be that appropriate for the revised terms of engagement.
B. Include reference to the original engagement.
C. Include reference to any procedures that may have been performed in the
original engagement.
D. Not include reference to any procedures that may have been performed,
particularly when the new engagement is to undertake agreed-upon
procedures.

PSA 210 states that if the auditor concludes that there is a reasonable justification
to change the engagement and if the audit work performed complies with the
PSAs applicable to the change engagement, the report issued would be that
appropriate for the revised terms of engagement. In order avoid confusing the
reader, the report would not include the reference to:
a.) The original engagement; or
b.) Any procedures that may have been performed in the original
engagement, except where the engagement is changed to an
engagement to undertake agreed-upon procedures. And thus,
reference to the procedures performed is normal part of the report.

21. If the auditor is unable to agree to a change of the engagement and is not
permitted to continue the original engagement, the auditor should
A. Insist on continuing the original engagement.
B. Express a qualified opinion.
C. Express an adverse opinion.
D. Withdraw from the engagement.

Under PSA 210, if the auditor is unable to agree to a change of the engagement
and is not permitted to continue the original engagement, the auditor should
withdraw and considered whether there is any obligation, either contractual or
otherwise, to report to other parties, such as the board of directors or shareholders,
the circumstances necessitating the withdrawal.

22. Planning an audit involves


I. Establishing the overall audit strategy for the engagement.
II. Developing an audit plan.

A. I only
B. II only
C. Both I and II
D. Neither I nor II

23. Which of the following activities should be performed by the auditor at the
beginning f the current audit engagement?
I. Perform procedures regarding th continuance of the client relationship and
the specific audit engagement.
II. Evaluate compliance with relevant ethical requirements, including
independence.
III. Establish an understanding of the terms of the engagement.

A. I and II only
B. II and III only
C. I and III only
D. I, II and III

24. Initial audit planning involves the following matters, except


A. Identify the client’s reason for the engagement.
B. Schedule engagement staff and auditor’s experts.
C. Develop an overall audit strategy.
D. Bequest that blank balances be confirmed.
25. Adequate planning helps to ensure that
A B C D

Appropriate attention is devoted to


N
No Yes Yes
o
important areas of the audit
Potential problems are identified and N
Yes No
Yes resolved on a timely basis o
The audit engagement is properly N
Yes Yes No o
organized and managed

The benefit of adequate planning includes the following:

• Helping the auditor to devote appropriate attention to important areas of


the audit.
• Helping the auditor to identify and resolve potential problems on a timely
basis.
• Helping the auditor properly organize and manage the audit engagement
so that it is performed in an effective and efficient manner.
• Assisting in the selection of engagement team members with appropriate
levels of capabilities and competence to respond to anticipated risk, and
the proper assignment of work to them.
• Facilitating the direction and supervision of engagement team members
and the review of their work.
• Assisting, where applicable, in coordination of work done by auditors of
components and experts.

26. Which of the following statements concerning audit planning is incorrect?


A. Planning is a discrete phase of an audit.
B. Planning is a continual and iterative process.
C. In recurring audit, planning often begins shortly after (or in connection with)
the completion of the previous audit and continuous until the completion of
the current audit engagement. D. In planning audit, the auditor considers the
timing of certain planning activities and audit procedures that are to be
completed prior to the performance of further audit procedures.

The standard states, “Planning is not a discrete phase of an audit but rather a
continual and iterative process that begins shortly after (or in connection with) the
completion of the previous audit and continuous until the completion of the
current audit engagement. Planning, however, includes consideration of a timing
of certain activities and audit procedures that need to be completed prior to
performance of further audit procedures.”
27. In performing an audit of financial statements, the auditor should obtain a
sufficient knowledge of a client’s business and industry to
A. Develop an attitude of professional skepticism concerning managements’ financial
statements assertions.
B.Make constructive suggestions concerning improvements to the client’s internal
control.
C.Evaluate whether the aggregation of known misstatements causes the financial
statements taken as a whole to be materially misstated.
D. Understand the events and transactions that may have an effect on the client’s
financial statements.

The auditor is required to have or obtain a sufficient knowledge of the client’s


business and industry to identify and understand the events, transactions, and
practices that, in the auditor’s judgment, may have a significant effect on the
financial statements.

28. Which of the following is least likely procedure to be performed in planning


financial statement audit?
A. Selecting a sample of sales invoices for comparison with shipping documents.
B. Coordinating the assistance of entity personnel in data preparation.
C. Reading the current year’s interim financial statements.
D. Discussing matters that may affect the audit with firm personnel responsible
for non-audit services to the entity.

Selecting a sample of sales invoices for comparison with shipping documents is


substantive test procedure. Substantive tests are performed to accumulate
sufficient appropriate audit evidence, not to plan the audit.

Answers B, C, and D are all planning procedures.

29. The establishment of an overall audit strategy involves I. Determining the


characteristics of the engagement that define its scope.
II.Ascertaining the reporting objectives of the engagement to plan the timing of the
audit and the nature of the communications required.
III. Considering the important factors that will determine the focus of the engagement
team’s efforts.

A. I and II only
B. II and III only
C. I and III only
D. I, II and III
PSA 300 (Planning an Audit of Financial Statements) requires the auditor to
establish an overall audit strategy that sets the scope, timing and direction of the
audit, and that guides the development of the audit plan.
In establishing the overall audit strategy, the auditor is required to

a.) Identify the characteristics of the engagement that define its scope.
b.) Ascertain the reporting objectives of the engagement to plan the timing of
the audit and the nature of the communication required.
c.) Consider the factors that, in the auditor’s professional judgment, are
significant in directing the engagement team’s effort.
d.) Consider the results of preliminary engagement activities and, where
applicable, whether knowledge gain
on other engagement performed by the engagement partner for the entity
is relevant, and
e.) Ascertain the nature, timing and extent of resources necessary to perform
the engagement.

30. Which of the following should be included in the audit plan?


I. The nature, timing and extent of planned risk assessment procedures.
II.The nature, timing and extent of planned further audit procedures at the
assertion level.

A. I only
B. II only
C. Both I and II
D. Neither I nor II
The audit plan shall include a description of

a.) The nature, timing and extent of planned risk assessment procedures as
determined under PSA 315 (Identifying and Assessing the Risks of Material
Misstatement through Understanding the entity and its Environment).
b.) The nature, timing and extent of planned further audit procedures at the
assertion level, as determined under PSA 330 (The Auditor’s Responses to
Assessed Risks).

31. Which of the following matters would an auditor least likely consider when
setting the direction of the audit?
A. The selection of the engagement team and the assignment of the audit work to
the team members.
B The engagement budget which includes consideration of the appropriate amount
of time to allot for areas where there may be higher risk for material misstatement.
C. The availability of client personnel and data.
D. The manner in which the auditor emphasizes to engagement team members the
need to maintain a questioning mind and to exercise professional skepticism in the
gathering and evaluation of audit evidence.

The availability of client personnel and data is a matter to consider when


establishing the scope of the audit.
32. Which of the following matters would an auditor most likely consider when
establishing the scope of the audit?
A. The expected audit coverage, including the number and location of the entity’s
components to be included.
B.The entity’s timetable for the reporting, such as at interim and final stages.
C.The discussion with the entity’s management concerning the expected
communications on the status of the audit work throughout the engagement and
the expected deliverables resulting from the audit procedures.
D. Audit areas where there is a higher risk of material misstatement.

Answers B and C are matters to consider when determining the reporting


objectives of the engagement, the timing of the audit, and the nature of
communication required.
Answer D is a matter to consider when setting the directions of the audit.

33. In the planning stage of an audit engagement, the auditor is required to


perform audit procedures to obtain an understanding of the entity and its
environment, including its internal control. These procedures are called
A. Risk assessment procedures
B. Substantive tests
C. Tests of controls
D. Dual-purpose tests
The audit procedures to obtain an understanding of the entity and its environment,
including its internal control are referred to as “risk assessment procedures”
because some of the information obtained through these procedures may be used
as audit evidence to support assessments of the risks of material misstatement.

34. In planning the audit engagement, the auditor should consider each of the
following, except
A. The type of opinion that is likely to be expressed.
B. The entity’s accounting principles and procedures.
C. Matters relating to the entity’s business and industry in which it operates.
D. Materiality level and audit risk.

The nature of the report expected to be issued (e.g. a report on consolidated


financial statements) should be considered in planning an audit engagement. The
type of opinion to be rendered is determined after the performance of audit
procedures.
35. Audit programs are modified to suit the circumstances of particular
engagements. A complete audit program usually should be developed
A. When the engagement letter is prepared.
B. After obtaining an understanding of the control environment and control
activities component of the entity’s internal control.
C. After the auditor has obtained an understanding of the entity and its
environment, including its internal control and assessed the risk of material
misstatement. D. Prior to beginning the actual audit work.

Only after obtaining knowledge of the entity and its environment, including its
internal control and assessed the risk of material misstatement can be the auditor
determine the nature, timing, and extent of substantive tests of financial statement
assertion.
36. In designing written audit programs, an auditor should establish specific audit
objectives that can relate primarily to the
A. Selected audit techniques.
B. Cost-benefit of gathering audit evidence.
C. Timing of audit procedures.
D. Financial statement assertions.

The audit procedures included in the audit program should enable the auditor to
gather sufficient appropriate audit evidence about management presentations
embodied in the entity’s financial statements. Such management representations
are called management assertions. Hence, in designing written audit programs, the
auditor develops specific audit objectives in light of those management assertions.
37. An audit program should be designed for each individual and should
incorporate steps and procedure to
A. Detect and eliminate fraud of any type.
B. Gather sufficient amount of management information available.
C. Provide assurances that the objectives of the audit are satisfied.
D. Insure that only material items are audited.
A written audit program sets forth, in reasonable detail, the specific audit
procedures that in the auditor’s judgment are necessary to satisfy the specific audit
engagement.
38. Which of the following is an aspect of scheduling and controlling of the audit
engagement?
A. Including in the engagement letter an estimate of the minimum and maximum
audit fee.
B.Writing a conclusion in individual working papers indicating how the results of the
audit will affect the auditor’s report.
C.Performing audit work only after the entity’s books have been closed for the period
under audit.
D. Including in the audit program a column for budgeted and actual time.

The audit program/plan serves as set of instructions to assistants involved in the


audit and as a means to control and record the proper execution of the work.
By including a column for estimated and actual time for each audit step, the audit
program can be a useful tool for controlling and scheduling audit work.

39. In connection with the planning phase of an audit engagement, which of the
following statements is always correct?
A. Final staffing decisions must be made prior to completion of planning stage.
B.Observation of inventory count should be performed at yearend.
C.A portion of the audit of a continuing audit client can be performed at interim dates.
D. An engagement should not be accepted after the client’s financial year-end.

40. The auditor shall undertake which of the following activities prior to starting
an initial audit?
I. Performing procedures required by PSA 220 (Quality Control for an audit of
Financial Statements) regarding the acceptance of the client relationship and the
specific audit engagement.
II.Communicating with the predecessor auditor, where there has been a change of
auditors, in compliance with relevant ethical requirements.

A. I only
B. II only
C. Either I or II
D. Neither I nor II
41. Before accepting an audit engagement, a proposed (successor/incoming)
auditor should make inquiries of the previous (predecessor) auditor regarding the
previous auditor’s
A. Evaluation of all matters of continuing accounting significance.
B. Understanding as to the reasons for the change of auditors.
C. Awareness of the consistency in the application of PAS/PFRS between
periods.
D. Opinion on any subsequent events occurring since the previous auditor’s
report was issued.

The standard requires the proposed auditor to communicate directly with the
previous auditor before accepting the engagement. The proposed auditor should
initiate the communication, although both must obtain client permission to
communicate. The proposed auditor should inquire about reasons for the change
in auditors, disagreement with management concerning accounting principles and
auditing procedures, and facts bearing on the integrity of the entity’s management.
42. The auditor is required to determine three different levels of materiality: (1)
materiality for the financial statements as a whole, (2) performance materiality,
and (3)
A. Overall materiality
B. Planning materiality C. General materiality
D. Specific materiality

The auditor is required to determine three different levels of materiality. These are
1. Materiality for the financial statements as a whole ( also called overall
materiality, general materiality, or tolerable misstatement)
2. Performance materiality (also called planning materiality or scoping
materiality)
3. Materiality applied to specific classes of transaction, account balances or
disclosures (also called specific materiality or individual materiality)

43. What materiality level would be considered by the auditor to determine


whether the proposed adjustments are significant or not?
A. Overall materiality
B. Scoping materiality
C. Specific materiality
D. Performance materiality

AASC Bulletin, Series 001 of 2010 states, “Materiality for the financial
statements as a whole (herein referred to as the “overall materiality”) is the
materiality determined at the overall financial statement level. This materiality
level helps the auditor determine whether the proposed audit adjustments are
significant or not,. I the audit adjustments exceed this level, the auditor may need
to adjust the financial statements.”
44. What materiality level is used by the auditor in determining which line items
in the financial statements are to be tested?
A. Overall materiality
B. Performance materiality
C. Specific materiality
D. Individual materiality

Performance materiality, calculated as a certain percentage of overall materiality,


is used in scoping of financial statement line items to be tested by the auditor. This
will ensure that significant accounts in the financial statements are covered by
audit testing.
45. _____________________ is the amount set by the auditor for particular
classes of transactions, account balances or disclosures for which misstatements,
well though lower than overall materiality could reasonably be expected to
influence be expected to influence the economic decisions of users of the financial
statements.
A. Performance materiality
B. Planning materiality
C. Specific materiality
D. General materiality

46. Which of the required materiality levels is calculated by multiplying a certain


percentage by the appropriate benchmark, which is either an element or
component of an entity’s financial statements?

A. Overall materiality
B. Planning materiality C. Scoping materiality
D. Specific materiality

47. Which of the following factors are normally considered by the auditor in
determining the appropriate benchmark for the purpose of calculating overall
materiality?
I. Components of the entity’s financial statements
II. Laws and regulations
II. Nature of the entity

A. I and II only
B. I and III only
C. II and III only
D. I, II and III
The following factors are normally considered in choosing the appropriate
benchmark:

• Components of the entity’s financial statements


• Focus of the users of the financial statements
• Nature of the entity
• Ownership structure of the entity
• Volatility of the benchmark identified
• Laws and regulation (e.g SEC)

48. Which of the following statements concerning materiality is not correct?


A. When establishing the overall audit strategy, the auditor shall determine
materiality for the financial statements as a whole.
B.If, in the specific circumstances of the entity, there is one or more particular classes
of transactions, account balances or disclosures for which misstatements of lesser
amounts than materiality for the financial statements as a whole could reasonable
expected to influence the economic decisions of users taken on the basis of the
financial statements, the auditor shall also determine the materiality level or levels
to those particular classes of transactions, account balances or disclosures.
C.Determining materiality involves the exercise of professional judgment.
D. The materiality level for the financial statements as a whole determined in the
planning stage of the audit should not be affected by changes in the circumstances
of the engagement.

PSA 320 (Materiality in Planning and Performing an Audit) states that materiality
for the financial statements as a whole (and if applicable, the materiality level or
levels for particular classes of transactions, account balances or disclosures) may
need to be revised as a result of:
• A change in circumstances that occurred during the audit (e.g. a decision
to dispose of a major part of the entity’s business);
• New information; or
• A change in the auditor understands of the entity and its operations as a
result of performing further audit procedures.

49. Which of the following would an auditor most likely use in determining a
preliminary judgment about materiality?
A. The content of the management representation letter.
B. The anticipated sample size of the planned substantive tests.
C. The entity’s annualized interim financial statements.
D. The results of the internal control questionnaire. The measurement of a
preliminary materiality level usually
relates to an annual figure (e.g. net income)
50. An auditor shall consider materiality when
I. Determining the nature, timing and extent of audit procedures.
II. Evaluating the effect of misstatements.

A. I only
B. II only
C. Both I and II
D. Neither I nor II
The auditor considers materiality when determining the nature, timing and extent
of audit procedures and evaluating the effect of misstatements.

51. It is an appraisal activity established within the entity. Its functions include,
among other things, examining, evaluating and monitoring the adequacy and
effectiveness of the accounting and internal control systems.
A. External auditing
B. Internal auditing
C. Governmental auditing
D. Internal auditing

52. Which is not a similarity between external and internal auditors?

A. Both consider materiality and risk in their work.


B. Both use similar methodologies in performing their work.
C. Both must be competent.
D. Both must be independent of the company.

53. The external auditor should obtain a sufficient


understanding of the internal audit function because

A. The understanding of the internal audit function is an important substantive test to


be performed by the external auditor.
B.The audit programs, working papers, and reports of internal auditors may often be
used as a substitute for the work of the external auditor’s staff.
C.The procedures performed by the internal audit staff may eliminate the external
auditor’s need for considering internal control.
D. The work performed by the internal auditors may be a factor in determining the
nature, timing and extent of the external auditor’s procedures.

The standard requires the external auditor to obtain a sufficient understanding of


internal audit activities to identify and assess the risks of material misstatement in
the financial statements and to design and perform further audit procedures. Thus,
an internal audit function is one of the many factors that influence the nature,
timing and extent of the external auditor’s procedures.
54. Internal auditing can affect the scope of the external auditor’s audit of
financial statements by
A. Decreasing the external auditor’s need to perform detailed tests.
B.Eliminating the need to observe the physical inventory taking. C. Allowing the
external auditor to limit his/her audit to the performance of substantive test
procedures.
D. Limiting direct testing by the external auditor to management assertions not
directly tested by internal auditing.

55. Which of the following is incorrect statement concerning the relationship of


the internal auditor and the scope of the external audit of an entity’s financial
statements?

A. The external auditor is not required to give consideration to the internal audit
function beyond obtaining a sufficient understanding to identify and assess the
risks of material misstatement of the financial statements and to design and
perform further audit procedures.
B.The internal auditors may determine the extent to which audit procedures should be
employed by the external auditor.
C.Under certain circumstances, the internal auditors may assist the external auditor in
performing substantive tests and tests controls.
D. The nature, timing and extent of the external auditor’s substantive tests may be
affected by the work of the internal auditors.
The standard categorically states that the external auditor has sole responsibility
for the audit opinion expressed, and the responsibility is not reduced by any use
made of internal auditing. All judgments relating to the audit of an entity’s
financial statements are those of the external auditor.
56. In determining whether the work of the internal auditors is likely to be
adequate for purposes of the audit, the external auditor shall evaluate the internal
auditor’s
A. Efficiency and experience
B. Independence and review skills
C. Training and supervisory skills
D. Competence and objectivity
The external auditor shall evaluate

• The objectivity of the internal audit function


• The technical competence of the internal auditors;
• Whether the work of the internal auditors is likely to carried out with due
professional care; and
• Whether there is likely to be effective communication between the
internal auditors and external auditor.

57. In assessing the technical competence of an internal auditor an external auditor


most likely would obtain information about the
A. Quality of working paper documentation, reports, and recommendations.
B. Organizational level to which the internal auditor reports.
C. Influence of management on the internal auditor’s duties.
D. Entity’s commitment to integrity and ethical values.

In assessing the competence of an internal auditor, the external auditor should


consider the following factors, such as the following:
• Educational level and professional experience.
• Professional certification and continuing education.
• Audit policies, programs and procedures.
• Supervision and review of the internal auditor’s activities.
• Departmental practices regarding assignments.
• Quality of working papers documentation, reports, and recommendations.
• Evaluation of the internal auditor’s performance.
Answers B, C and D are incorrect because they all relate to objectivity rather than
technical competence.

58. Which of the following is s false statement about the use of the internal
auditor’s work by the external auditor?
A. The PSAs do not allow the external auditor to use to work of the internal auditor.
B.PSAs do not allow the external auditor to substitute the work of the internal auditor
for the work of the external auditor in critical judgments.
C.The PSAs state that, when specific work of the internal auditor is to be used, it
should be evaluated and tested.
D. PSAs states that, when considering whether to use the work of the internal auditor,
the external auditor should consider the internal auditor’s competence and
objectivity.
59. The coordination between internal and external auditors

A. Eliminates duplication of audit efforts


B. Includes the exchange of audit reports and management letters. C. Prevents
external auditors from having access to the programs used by the internal
auditors.
D. Prohibits the internal auditor from using the same audit techniques as external
auditors and vise versa.
Coordination of internal and external audit efforts helps to ensure adequate audit
coverage and to minimize duplication of audit efforts. Coordination involves:
• Meetings at appropriate intervals during the period discuss matters of
mutual interest.
• Access to each other’s audit programs and working papers.  Exchange of
audit reports and management letters.

60. Which of the following are included in the activities of the internal audit
function?
I. Monitoring of internal control.
II. Examination of financial and operating information.
III. Review of operating activities.

A. I and II only
B. I and III only
C. II and III only
D. I, II and III
The activities of the internal audit function may include one or more of the
following:

• Monitoring of internal control.


• Examination of financial and operating information.
• Review of operating activities.
• Review of compliance with laws and regulation.
• Risk management
• Governance

61. ______________ is an individual or organization possessing expertise in a


field other than accounting or auditing, whose work in that field is used by the
auditor to assist the auditor in obtaining sufficient appropriate audit evidence.
A. Auditor’s expert
B. Management’s expert
C. Expert
D. Specialist

62. ________________ is an individual or organization possessing expertise in a


field other than accounting or auditing, whose work in that field is used by the
entity to assist the entity in preparing financial statements.
A. Auditor’s expert
B. Management’s expert
C. Expert
D. Specialist

63. When planning to use the work of an expert, the auditor should evaluate the
experts
I. Professional competence
II. Objectivity

A. I only
B. II only
C. Both I and II
D. Neither I nor II

64. Which of the following statements is correct concerning the auditor’s use of
the work of an expert?
A. The auditor is required to perform substantive test procedures to verify the
expert’s assumptions and findings.
B.The auditor should obtain understanding of the methods and assumptions used by
the expert.
C.The entity should not have an understanding of the nature of the work to be
performed by the expert.
D. The expert should not have an understanding of the nature of the auditor’s
corroborative use of the expert’s findings.

The expert is responsible for the appropriateness and reasonableness of


assumptions and methods used. Because the auditor does not have the same
expertise, he/she is not in the position to challenge the expert’s methods and
assumptions. However, the standard states that the auditor will need to obtain an
understanding of the methods and assumptions used and consider whether they
are appropriate and reasonable, based on his/her knowledge of the entity’s
business and the results of other procedures.
Answer A is incorrect because, if the expert’s findings corroborative the financial
statement assertions being considered, the auditor may reasonably conclude that
sufficient appropriate evidence has been obtained.
Answers C and D are incorrect because the auditor, the entity and the expert
should have an understanding about the nature, scope, and objective of the work
to be performed.

65. Which of the following is not an expert upon whose work an auditor may
rely?
A. An actuary.
B. An individual with expertise in complex modeling for the purpose of valuing
financial instruments. C. An expert in taxation law.
D. An individual with expertise in applying methods of accounting for deferred
income tax.

66. If the results of the expert’s work do not provide sufficient appropriate audit
evidence or are not consistent with other audit evidence, the auditor should

A. Report the matter to the appropriate regulatory agency of the government.


B. Resolve the matter.
C. Withdraw from the engagement.
D. Express an unmodified opinion with reference to the work of the expert.

If the results of the expert’s work do not provide sufficient appropriate audit
evidence or are not consistent with other audit evidence, the auditor should
resolve the matter. The auditor may need to discuss the matter with the entity and
the expert, apply additional audit procedures, possibly engage another expert, or
modify the auditor’s report.

67. When issuing an unmodified auditor’s report, the auditor


A. May refer to the work of an expert.
B. Should refer to the work of an expert to indicate a division of responsibility.
C. Should include in the auditor’s report the identity of the expert and the extent
of the expert’s involvement. D. Should not refer to the expert’s work.

The standard prohibits the auditor to refer to the expert’s work when issuing an
unmodified auditor’s report because such a reference might be misunderstood to
be a qualification of the auditor’s opinion or a division of responsibility, neither of
which is intended.
68. In using the work of an expert, an auditor referred to the expert’s finding in the
auditor’s report. This is an appropriate practice if the
A. Auditor, as a result of the expert’s work, decides to indicate a division of
responsibility with the expert.
B.Expert is aware that his/her work will be used to evaluate the assertions in the
financial statements.
C.Auditor, as a result of the expert’s work, issues a report that an unmodified opinion.
D. Auditor, as a result of the expert’s work, adds an emphasisof-matter paragraph I
his/her unmodified auditor’s report.

An auditor shall not refer to the expert’s work in an auditor’s report containing an
unmodified opinion. However, if as a result of the expert’s work, the auditor
decides to express a modified opinion, it may be appropriate—in explaining the
nature of the modification—to refer to or describe the expert’s work

69. As used in PSA 600, Special Considerations – Audits of


Group, Financial Statements (Including the Work of Component Auditor) ,
______________ is an entity or business activity for which group or component
management prepares financial information that should be included in the group
financial statements.
A. Component
B. Group
C. Significant component
D. Group management

70. As used in PSA 600, financial statements that include the financial information
of more than one component are called

A. Component financial statements


B. Group financial statements
C. Consolidated financial statements
D. Common financial statements

71. The __________________ is the partner or other person in the firm who is
responsible for the group audit management and its performance, and for the
auditor’s report on the group financial statements that is issued on behalf of the
firm.

A. Engagement partner
B. Component engagement partner
C. Principal auditor
D. Group engagement partner

72. The group engagement team shall obtain an understanding that is sufficient to

I. Confirm or revise its initial identification of components that are likely to be


significant.
II.Assess the risks of material misstatement of the group financial statements, whether
due to fraud or error.
A. I only
B. II only
C. Both I and II
D. Neither I nor II

73. If the group engagement team plans to request a component, auditor to


perform work on the financial information of a component, the group engagement
team shall obtain an understanding of

I. Whether the component auditor understands and will comply with the ethical
requirements that are relevant to the group audit and, in particular, is
independent.
II. The component auditor’s professional competence.
A. I only
B. II only
C. Both I and II
D. Neither I nor II

74. Which of the following statements concerning the group audits is incorrect?

A. The group engagement team has the responsibility to establish an overall group
audit strategy and audit plan.
B.The group engagement team shall determine the materiality for the group financial
statements as a whole when establishing the overall group audit strategy.
C.The component engagement partner shall review the overall group audit strategy
and audit plan.
D. The group engagement team shall agree on the terms of the group audit
engagement in accordance with PSA 210.

75. An auditor who, at the request of the group engagement team, performs work
on financial information related to a component for the group audit is a

A. Group auditor
B. Component auditor
C. Component engagement team D. Group engagement team
KEY ANSWERS

1. C 26. A 51. B
2. A 27. D 52. D
3. C 28. A 53. D
4. A 29. D 54. A
5. C 30. C 55. B
6. B 32. C 56. D
7. A 32. A 57. A
8. A 33. A 58. A
9. C 34. A 59. B
10. A 35. C 60. D
11. A 36. D 61. A
12. A 37. C 62. B
13. A 38. D 63. C
14. C 39. C 64. B
15. C 40. D 65. D
16. D 41. B 66. B
17. A 42. D 67. D
18. A 43. A 68. C
19. A 44. B 69. A
20. A 45. C 70. B
21. D 46. A 71. D
22. C 47. D 72. C
23. D 48. D 73. C
24. D 49. C 74. C
25. B 50. C 75. B
CHAPTER 5
RISK ASSESSMENTS AND INTERNAL COTROL

1. PSA 135 (Identifying and Assessing the Risks of Material Misstatement


through Understanding the Entity and its Environment) requires the auditor to
perform risk assessment procedures at
A. The financial statement level only.
B. The assertion level only.
C. The financial statement level and the assertion level for classes of
transactions, account balances and disclosures. D. Either the financial
statement or assertion level.

Risk assessment procedures are performed to obtain an understanding of the


entity and its environment, including the entity’s internal control, to identify
and assess the risks of material misstatement, whether due to fraud or error, at
the financial statement and assertion levels.

2. The auditor’s risk assessment procedures should always include the


following, except
A. Inquiries of management and of others within the entity.
B. Analytical procedures.
C. Observation and inspection.
D. Substantive test procedures and tests of controls.

The auditor’s risk assessment procedures shall include:


1. Inquiries of management and of others within the entity who, in the auditor’s
judgment, may have information that Is likely to assist in identifying risks
of material mis- Statement due to fraud or error.

2. Analytical procedures.

3. Observation and inspection.

Substantive test procedures are audit procedures designed to detect material


misstatements at the assertion level. Tests of controls are audit procedures
designed to evaluate the effectiveness of controls in preventing, or detecting
and correcting, material misstatements at the assertion level.

3. The auditor’s risk assessment procedures


A. By themselves, do not provide sufficient appropriate audit evidence on which
to base the audit opinion.
B. Should not consider information obtained from the auditor’s previous
experience with the entity.
C. Are designed to detect material misstatements at the assertion level for
classes of transactions, account balances and disclosures.
D. Are designed to test the effectiveness of the entity’s controls.

4. The auditor should obtain an understanding of the entity’s objectives and


strategies, and those business risks that may result in risks of material
misstatement. Which of the following statements concerning the entity’s
business risk is incorrect?
A. Business risk is broader than the risk of material misstatement of the financial
statements, though it includes the latter.
B. An understanding of the business risks facing the entity increases the
likelihood of identifying risks of material misstatement.
C. The auditor has a responsibility to identify or assess all business risks.
D. Business risk may arise from the development of new products or services
that may fail.

As defined in PSA 315, business risk is “a risk resulting from significant


conditions, events, circumstances, actions or inactions that could adversely
affect an entity’s ability to achieve its objectives and execute its strategies, or
form the setting of inappropriate objectives and strategies.”

The auditor does not have a responsibility to identify or assess all business risks
facing the entity because not all business risks give rise to risks of material
misstatement.

5. The risk that the auditor may give an inappropriate opinion when the financial
statements are materially misstated is called
A. Detection risk
B. Business risk
C. Audit risk
D. Inherent risk

6. Audit risk has three components: inherent risk, control risk, and detection
risk. Which is correct?
A. Detection risk is a function of the efficiency of an auditing procedure.
B. Cash is more susceptible to theft than an inventory of coal because it has a
greater inherent risk.
C. The risk that material misstatements will not be prevented or detected on a
timely basis by internal control can be reduced to zero by effective controls.
D. The existing levels of inherent risk, control risk, and detection risk can be
changed at the discretion of the auditor.
The three components of audit risk are:

1. “Inherent risk” is the susceptibility of an account balance or class of


transactions to misstatement that could be material, individually or when
aggregated with misstatements in other balances or classes, assuming that
there were no related internal controls.

2. “Control risk” is the risk that a misstatement that could occur in an account
balance or class of transactions that could be material, individually or when
aggregated with other misstatements in other balances or classes, will not be
prevented, or detected and corrected, on a timely basis, by the accounting and
internal control systems.

3. “Detection risk” is the risk that an auditor’s substantive procedures will not
detect a misstatement that exists in an account balance or class of transactions
that could be material, individually or when aggregated with misstatements in
other balances or classes.

Cash has a greater inherent risk than an inventory of coal.

Answer A is incorrect because detection risk is a function of the effectiveness


of an auditing procedure, not its efficiency.

Answer C is incorrect because even the most effective internal control has
inherent limitations. Thus, control risk cannot be reduced to zero.

Answer D is incorrect because inherent risk and control risk exist


independently of the audit and, therefore, cannot be changed at the discretion
of the auditor.

The acceptable level of detection risk is a function of the desired level of


overall audit risk and the assessed levels of inherent risk and control risk.
Hence, detection risk can be changed at the discretion of the auditor.
However, it should be emphasized that the auditor’s preliminary assessments
of inherent risk and control risk may change as the audit work continues.

7. The risk that an auditor’s substantive procedures will lead to the conclusion
that a material misstatement does not exist in an account balance or
transaction class when, in fact, such misstatement does exist is
A. Control risk
B. Inherent risk
C. Audit risk
D. Detection risk

8. The existence of audit risk is recognized by the statement in the auditor’s


report that the
A. Financial statements are presented fairly, in all material respects, in
accordance with Philippine Financial Reporting Standards.
B. Audit includes examining, on a test basis, evidence supporting the
amounts and disclosures in the financial statements.
C. Auditor obtains reasonable assurance about whether the financial
statements are free of material misstatement.
D. Auditor is responsible for expressing an opinion on the financial
statements, which are management’s responsibility.

An audit conducted in accordance with PSAs provides only reasonable, not


absolute, assurance that the financial statements are free of material
misstatement, whether caused by error or fraud.

9. Which of the following audit risk components may be assessed I quantitative


terms?
Inherent Risk Control Risk Detection Risk
A. Yes No Yes B. Yes
Yes Yes
C. No No No
D. No No Yes

Audit risk and its components may be assessed in quantitative or non-quantitative


terms.

10. Some account balances, such as those for retirement benefits and finance
leases, are the results of complex calculations. The susceptibility to material
misstatements in these types is referred to as
A. Audit risk
B. Detection risk
C. Inherent risk
D. Control risk

Inherent risk, which is the susceptibility of an assertion to material misstatement


in the absence of related controls, exists independently of the audit. Some
assertions and related account balances and classes of transactions have
greater level of inherent risk than others. For example, account balances
resulting from complex calculations such as those for retirement benefits and
finance leases have a higher risk of misstatement.
11. There is an inverse relationship that exists between the acceptable level of
detection risk and the
A. Risk of failing to discover material misstatements.
B. Assurance provided by substantive tests.
C. Preliminary judgments about materiality levels.
D. Risk of misapplying audit procedures.
An auditor assesses control risk by considering internal control. There exists an
inverse relationship between control risk and detection risk – that is, the
greater (lower) the assessed level of control risk, the lower (greater) the
acceptable level of detection risk.

The acceptable level of detection risk affects substantive testing. As the


acceptable level of detection risk decreases, the auditor changes the nature,
timing, and extent of substantive tests to increase the assurance they provide.
Therefore, there is an inverse relationship between the acceptable level of
detection risk and substantive testing.

12. Which of the following statements concerning audit risk and its components
is incorrect?
A. Regardless of the assessed levels of inherent and control risks, the auditor
should always perform some substantive procedures for material account
balances and classes of transactions.
B. The higher the assessment of inherent and control risks, the more evidence
the auditor should obtain from the performance of substantive procedures.
C. The assessed level of inherent risk need not be considered in determining
the nature, timing, and extent of substantive procedures required to reduce
audit risk to an acceptably low level.
D. After obtaining an understanding of the accounting and internal control
systems, the auditor should make a preliminary assessment of control risk,
at the assertion level, for each material account balance or class of
transactions.

According to the standard, the auditor should consider the assessed levels of
inherent and control risks in determining the nature, timing, and extent of
substantive procedures required to reduce audit risk to an acceptable level.

13. Because the concepts of audit risk and materiality are interrelated, they must
be considered together by the auditor. Which of the following statements is
correct?
A. The phrase in the auditor’s report “present fairly, in all material respects, in
accordance with Philippine Financial Reporting Standards” indicates the
auditor’s belief that the financial statements taken as a whole are not
materially misstated.
B. If misstatements are not individually material, but are material when
aggregated with other misstatements, the concept of materiality does not
apply.
C. Audit risk is the risk that an auditor may unknowingly modify his/her
opinion when, in fact, the financial statements are fairly presented.
D. Only material errors cause financial statements to be materially misstated.
The opinion paragraph of the auditor’s report explicitly refers to materiality.
By stating that the financial statements are presented fairly, in all material
respects, in accordance with an applicable financial reporting framework, the
auditor is of the opinion that the financial statements are not materially
misstated.

Answer B is incorrect because the concept of materiality recognizes that


some misstatements, either individually or when aggregated with other
misstatements, are important for the fair presentation of financial statements.

Answer C is incorrect because audit risk is the probability that an auditor may
give an inappropriate opinion when the financial statements are materially
misstated.

Answer D is incorrect because material misstatements can result from errors


or fraud.

14. The statements below describe the interrelationship of audit risk


components. Which is false?
A. There is an inverse relationship between detection risk and the combined
level of inherent and control risks.
B. When inherent and control risks are high, the acceptable level of
detection risk needs to be low to reduce audit risk to an acceptably low
level.
C. When inherent and control risks are low, an auditor can accept a higher
detection risk and still reduce audit risk to an acceptably low level.
D. The assessed level of inherent and control risks can be sufficiently low to
eliminate the need for the auditor to perform any substantive procedures.

The standard states that regardless of the assessed levels of inherent and
control risks, the auditor should perform some substantive procedures for
material account balances and classes of transactions.
15. Based on audit evidence gathered and evaluated, an auditor decides to
increase the assessed level of control risk from that originally planned. To
achieve an overall audit risk level that is substantially the same as the
planned audit
risk level, the auditor would A. Increase
materiality levels.
B. Decrease detection risk.
C. Decrease substantive testing.
D. Increase inherent risk.

16. A basic premise underlying analytical procedures is that


A. Plausible relationships among data may reasonably be expected to exist
and continue in the absence of known conditions to the contrary.
B. These procedures cannot replace tests of details of transactions and
balances.
C. Statistical tests of financial information may lead to the detection of
material misstatements in the financial statements.
D. The study of financial ratios is an acceptable alternative to the
investigation of unusual fluctuations.

“Analytical procedures” as defined in the standard means evaluations of


financial information made by a study of plausible relationships among both
financial and nonfinancial data.

A basic premise underlying the application of analytical procedures is that


plausible relationships among data may reasonably be expected to exist in the
absence of known conditions to the contrary.

Answer B is incorrect because for some assertions, the application of


analytical procedures alone may provide the auditor with the level of
assurance he/she desires.

Answer C is incorrect because the use of analytical procedures (e.g., simple


comparisons) does not necessarily require statistical testing.

Answer D is incorrect because analytical procedures, such as ratio analysis,


enable an auditor to identify significant differences that should be evaluated
and investigated.
17. For all audits of financial statements conducted in accordance with PSAs, the
use of analytical procedures is required to some extent
A B C D
As a risk assessment procedure
in the planning phase Yes Yes Yes N
o
As a substantive test procedure Yes No No Ye
In the overall review of the s
financial statements Yes Yes No Ye
s

Analytical procedures are used for the following purposes:

1. As risk assessment procedures to obtain an understanding of the entity


and its environment.

2. As substantive test procedures when their application is, based on the


auditor’s judgment, more effective and efficient than tests of details in
reducing the risk of material misstatement.
3. As an overall review of the financial statements at the completion
stage of the audit engagement.
18. Which of the following statements concerning analytical procedures is true?
A. Analytical procedures are more efficient, but not more effective, than
tests of details of transactions.
B. Analytical procedures used as risk assessment procedures use data
aggregated at a high level.
C. Analytical procedures can replace tests of controls in gathering audit
evidence to support the assessed level of control risk.
D. Analytical procedures usually involve comparisons of ratios developed
from recorded amounts with ratios developed by management.

Analytical procedures applied as risk assessment procedures focus on


enhancing the understanding of the entity’s business, identifying unusual
transactions and events, identifying areas that may represent specific audit
risks, and most often use data aggregated at a high level.

Answer A is incorrect because for many assertions, tests of details are still
more effective or efficient in providing the level of assurance desired by the
auditor.

Answer C is incorrect because analytical procedures can be used as


substantive tests, not as tests of controls.

Answer D is incorrect because analytical procedures involve comparisons of


recorded amounts or ratios developed from recorded amounts with
expectations developed by the auditor – not with assertions developed by
management.
19. Analytical procedures used in planning an audit should focus
on
A. Enhancing the auditor’s understanding of the client’s business.
B. Evaluating the adequacy of evidence gathered concerning unusual
balances.
C. Identifying significant deficiencies in internal control.
D. Testing individual account balances that depend on accounting estimates.

20. Analytical procedures performed in the


A. Planning stage of the audit should assist in assessing the validity of the
conclusions reached.
B. Planning stage of the audit should address the risk of material
misstatement of revenue due to fraudulent financial reporting.
C. Final review stage of the audit should achieve audit objectives related to
specific assertions in the financial statements.
D. Final review stage of the audit should focus on identifying specific risks.
21. An auditor obtains an understanding of the nature of the entity to
A. Make constructive suggestions concerning improvements in the client’s
internal control.
B. Understand the account balances, transactions, and disclosures in the
financial statements.
C. Develop an attitude of professional skepticism concerning management’s
financial statement assertions.
D. Evaluate whether the aggregation of known misstatements causes the
financial statements taken as a whole to be materially misstated.

22. It is the process designed, implemented and maintained by those charged


with governance, management, and other personnel to provide reasonable
assurance about the achievement of the entity’s objectives.
A. Internal auditing
B. Internal control
C. Business strategy
D. Accounting process

PSA 315 states that internal control is designed and implemented to achieve the
entity’s objectives with regard to:
• Reliability of financial reporting;
• Effectiveness and efficiency of operations; and
• Compliance with applicable laws and regulations.

23. Which of the following statements concerning internal control is incorrect?


A. Internal control is effected by people.
B. Internal control is expected to provide absolute assurance.
C. Internal control is a process.
D. Internal control is geared to achieving objectives in the overlapping
categories of financial reporting, compliance and operations.

24. The inherent limitations in an entity’s internal control system include


I. Collusion among individuals II. Breakdowns in
established controls.
III. Mistakes in judgment.

A. I and III only C. II and III only


B. I and II only D. I, II and III

25. The procedures performed to obtain an understanding of the internal control


system provide an auditor with
A. Sufficient appropriate evidence to use in forming an overall opinion on
the entity’s financial statements.
B. Enough understanding to design procedures to gather sufficient
appropriate audit evidence.
C. Enough understanding to express an opinion on the effectiveness of the
entity’s internal control system.
D. Audit evidence to use in forming an overall opinion on the entity.

26. Some inherent limitations in an entity’s internal control system include


I. The possibility of management overriding the internal control
in place for illegitimate purposes.
II. Collusion among employees to conceal fraud.
III. Mistakes in judgment as the result of inadequate information
or time pressures.
A. I and II only C. II and III only
B. I and III only D. I, II and III

27. When considering an entity’s internal control system, an auditor is not


required to
A. Search for significant deficiencies in the operation of the internal control
system.
B. understand the components of the entity’s internal control system.
C. Determine whether relevant controls have been placed in operation.
D. Perform procedures to understand the design of the internal control
system policies.

28. The following are components of internal control, except


A. Control activities
B. The entity’s risk assessment process
C. Control environment
D. Business risk

The five components of internal control are:


1. The control environment.
2. The entity’s risk assessment process.
3. The information system, including the related business processes
relevant to financial reporting, and communication.
4. Control activities.
5. Monitoring of controls.

29. An entity’s internal control system contains manual elements and often
contains automated elements. Manual elements in internal control may be
less reliable than automated elements because
A. Manual control elements can be more easily bypassed, ignored, or
overridden and they are also more prone to simple errors and mistakes.
B. Manual control elements facilitate the additional analysis of information.
C. Consistency of application of manual control elements can always be
assumed.
D. Manual control elements include reliance on systems or programs that
are inaccurately processing data, processing inaccurate data, or both.

30. IT benefits an entity’s internal control by enabling an entity to


A. Reduce the risk that controls will be circumvented.
B. Process large, unusual or non-recurring transactions.
C. Rely on systems or programs that are inaccurately processing data.
D. Allowing unauthorized changes to data in master files.

The use of IT allows an entity to:


• Consistently apply predefined business rules and perform complex
calculations in processing large volumes of transactions and data;
• Enhance the timeliness, availability, and accuracy of information;
• Facilitate the additional analysis of information;
• Enhance the ability to monitor the performance of the entity’s
activities and its policies and procedures;
• Reduce the risk that control will be circumvented; and
• Enhance the ability to achieve effective segregation of duties by
implementing security controls in applications, databases, and
operating systems.

31. When obtaining an understanding of controls that are relevant to the audit,
the auditor is required to
A. Evaluate the design of those controls.
B. Determine whether those controls have been implemented.
C. Evaluate the design of those controls and determine whether they have
been implemented.
D. Evaluate the design of those controls and determine whether they have
been implemented by performing tests of controls.

PSA 315 states that when obtaining an understanding of controls that are
relevant to the audit, the auditor shall evaluate the design of those controls
and determine whether they have been implemented.

Evaluating the design of a control involves considering whether the control,


individually or in combination with other controls, is capable of effectively
preventing, or detecting and correcting, material misstatements.
Implementation of a control means that the control exists and that the entity is
using it.
Risk assessment procedures to obtain an understanding of controls relevant to
the audit include of the following:

• Inquiring of entity personnel.


• Observing the application of specific controls.
• Inspecting documents and reports.
• Tracing transactions through the information system to financial
reporting.

As defined in the standard, tests of controls are audit procedures designed


to evaluate the operating effectiveness of controls in preventing, or
detecting and correcting, material misstatements at the assertion level.
32. This internal control component is the foundation for all other components.
It sets the tone of the organization, provides discipline and structure, and
influences the control consciousness of employees.
A. Control activities
B. Monitoring of controls
C. Control environment
D. The entity’s risk assessment process

PSA 315 states that the control environment includes the governance and
management functions and the attitudes, awareness, and actions of those
charged with governance and management concerning the entity’s internal
control and its importance in the entity. The control environment sets the tone
of the organization, influencing the control consciousness of its people.

33. Which of the following are considered control environment


elements?
Commitment Detection Organizational To
Competence Risk Structure
A. No Yes No
B. Yes Yes Yes C. Yes No Yes
D. No No Yes

The control environment components of internal control encompasses the


following elements:

a. Communication and enforcement of integrity and ethical values.


b. Commitment to competence.
c. Participations by those charged with governance.
d. Management’s philosophy and operating style.
e. Organizational structure.
f. Assignment of authority and responsibility.
g. Human resource policies and practices.

Detection risk is a component of audit risk.

34. Which of the following elements are included in an entity’s control


environment?
Integrity and Assignment of Ethical Values
Authority and Responsibility
A. Yes Yes
B. Yes No
C. No Yes
D. No No

35. Which of the following components of an entity’s internal control includes


development and use of training policies that communicate prospective roles
and responsibilities to employees?
A. Monitoring of controls
B. Control activities
C. Control environment
D. Information and communication

The control environment component of internal control includes human resource


policies and practices relative to recruitment, orientation, training, evaluating,
counseling, promoting, compensating, and remedial actions.

36. Which of the following statements concerning the relevance of various types
of controls to a financial statement audit is correct?
A. All controls are ordinarily relevant to a financial statement audit.
B. Controls over safeguarding of assets and liabilities are of primary importance,
while controls over the reliability of financial reporting may also be relevant.
C. Controls over the reliability of financial reporting are ordinarily most directly
relevant to a financial statement audit, but other controls may also be a
relevant.
D. An auditor may ordinarily ignore a consideration of controls when a
substantive audit approach is taken.

Controls that are relevant to a financial statement audit pertain to the entity’s
objective of preparing financial statements for external purpose and the
management of risk that may give rise to a material misstatement in those
financial statements.
37. Which of the following internal control components relates to an entity’s
process for identifying and responding to business risks?
A. Control activities
B. Information and communication
C. Risk assessment
D. Monitoring of controls

PSA 315 states that the entity’s risk assessment process forms the basis for how
management determines the risks to be managed.

For financial reporting purposes, an entity’s risk assessment process includes how
management identifies risks relevant to the preparation of financial
statements, estimates their significance, assesses the likelihood of their
occurrence, and decides upon actions to manage them.
38. The following are subcomponents of the control environment, except
A. Management’s philosophy and operating style.
B. Adequate separation of duties.
C. Organizational structure.
D. Commitment to competence.

39. The purpose of an entity’s accounting information and communication system


is to
Record and Monitor
process Initiate transactions transaction transactions
A. No No No
B. Yes Yes Yes
C. Yes No No
D. No Yes Yes

40. Which of the following statements concerning physical controls is incorrect?


A. Access to computer hardware should not be limited to authorized personnel.
B. Physical controls limit access to assets and important records.
C. In systems with online entry of data, many users may have access through
remote input devices.
D. Access often extends beyond the entity’s employees to customers and
suppliers through remote terminals.

41. Inherent risk is ______ related to detection risk and ______ related to the
amount of audit evidence.
A. directly, inversely
B. directly, directly
C. inversely, inversely
D. inversely, directly

42. An auditor’s flowchart of an entity’s information system is a diagrammatic


representation that depicts the auditor’s
A. Documentation of the study and evaluation of the internal control system.
B. Understanding of the system.
C. Understanding of the types of fraud that are probable, given the present
system.
D. A program for tests of controls.

43. Which of the following statements about auditor documentation of the


client’s internal controls is correct? A. Documentation must include
flowcharts.
B. Documentation must include procedural write-ups.
C. Documentation is desirable but not necessary.
D. No one particular form of documentation is necessary.

44. Which of the following are the two key issues that an auditor considers when
obtaining an understanding of a client’s internal controls?
A. The effectiveness and efficiency of the controls.
B. The implementation and efficiency of the controls.
C. The design and utilization of the controls.
D. The frequency and effectiveness of the controls.

45. The auditor documents his or her understanding of the internal control
system to substantiate
A. The fairness of presentation of the financial statements.
B. Adherence to the requirements of management.
C. Compliance with PSAs.
D. Conformity of the accounting records with the applicable financial reporting
framework.

46. Which of the following is not an advantage of an IT system used as part of


the entity’s internal control system?
A. A reduced amount of information is required to be reviewed manually.
B. Detection of potential problems only if the computer is programmed to do so.
C. The delivery, timeliness and accuracy of information are maintained as the
volume of transactions increases.
D. Potentially fewer errors as individual calculation discrepancies are reduced.
47. The best way for an auditor to obtain evidence about segregation of duties is
to
A. Observe personnel performing their duties.
B. Reperform the task.
C. Inspect documents and records to ensure an independent check has been
performed.
D. Discuss the performance of duties within the company with the internal
auditor.

48. An auditor must assess control risk as high when he has A. Decided not to
test the internal control system.
B. Decided on a high level of detection risk.
C. Assessed inherent risk to be high.
D. Found no significant deficiencies in the internal control system.

49. Which of the following describes “narrative memoranda?”


A. A schematic diagram that uses standardized symbols, interconnecting flow
lines and annotations to portray the steps involved in processing information
through the information system.
B. Series of questions about accounting and control policies and procedures that
an auditor considers necessary to prevent material misstatements in the
financial statements.
C. A form of documentation that summarizes the auditor’s overall understanding
of the information system or specific control policies or procedures.
D. A written report on the effectiveness of the entity’s internal control system.

50. ______________ are groups of related business activities such as the


acquisition of merchandise and payment of
vendors.
A. Transactions
B. Transaction cycles
C. Economic cycles
D. Business events

51. The following are transaction cycles, except


A. General ledger and reporting
B. Human resources
C. Expenditure/Disbursement
D. Revenue/Receipt
52. The collection of job time tickets or time sheets is part of which of the
following transaction cycles?
A. Revenue/Receipt
B. Human resources
C. Production
D. Expenditure/Disbursement

53. Which of the following best explains why many modern accounting software
packages offer separate transaction cycle modules?
A. A properly designed Accounting Information System does not use the concept
of separate business transaction cycles to process transactions.
B. The nature of a given transaction cycle is the same irrespective of the type of
organization.
C. Most businesses do not need the revenue cycle module as part of their
Accounting Information System.
D. Every organization does not need to implement all of the available transaction
cycle modules.

54. One advantage of pre-numbering source documents is that it helps to verify


that
A. All cash has been collected.
B. No inventory has been misplaced.
C. All transactions have been recorded since the numerical sequence serves as a
control.
D. Documents have been used in order.

Use the following flowchart symbols to answer the questions below:


55. Which symbol would be used to represent a computer process?
A. #1 C. #4
B. #2 D. #12

56. Which symbol would be used to represent a decision?


A. #9 C. #8
B. #11 D. #5

57. Which symbol would be used to represent an invoice sent to a customer?


A. #2 C. #1
B. #5 D. #9

58. Which symbol would be used to represent a general ledger?


A. #2 C. #3
B. #1 D. #4
59. Which symbol would be used to represent a manual process?
A. #4 C. #9
B. #5 D. #10

60. Which symbol would be used to represent a connection to another part of the
flowchart on the same page?
A. #4 C. #10
B. #9 D. #11

61. Which symbol would be used to represent a connection to another part of the
flowchart on a different page?
A. #4 C. #10
B. #9 D. #11

62. Which symbol would be used to represent a file of paper documents?


A. #6 C. #8
B. #7 D. #10

63. Which symbol would be used to represent the display of a report on a


computer screen?
A. #1 C. #3
B. #2 D. #10

64. Which symbol would be used to represent a beginning, an ending, or a


connection to another procedure?
A. #8 C. #11
B. #10 D. #12

65. Duplicate checking of calculations is an example of a corrective control.

Adherence to appropriate procedures to resubmit rejected transactions is an


example of a detective control.

A. True; False C. False; True


B. True; True D. False; False
66. General IT controls are policies and procedures that relate to many
applications and support the effective functioning of application controls.
General IT controls commonly include
controls over the following, except
A. Data center and network operations
B. Manual follow-up of exception reports
C. Access security
D. Program change

Manual follow-up of exception reports is an example of application controls.


These controls relate to procedures used to initiate, record, process and report
transactions or other financial data.

67. Application controls are


A. Manual or automated procedures that typically operate at a business process
level and apply to the processing of transactions by individual applications.
B. Policies and procedures that relate to many applications.
C. Controls that maintain the integrity of information and security of data such
as controls over system software acquisition, change and maintenance.
D. Controls that relate to many applications and support the effective functioning
of general controls.

68. Under PSA 315, monitoring of controls is an internal control component that
involves a process of assessing the quality of internal control performance
over time. It involves assessing the design and operation of controls on a
timely basis and taking necessary corrective actions. Monitoring of controls
is accomplished through ongoing monitoring activities, separate evaluations,
or a combination of the two. An entity’s ongoing monitoring activities often
include
A. Periodic reporting by the entity’s internal auditors about the functioning of
internal control. B. Reviewing the purchasing function.
C. Periodic audits by the audit committee.
D. The audit of the annual financial statements.

According to the standard, ongoing monitoring activities are built into the
normal recurring activities of an entity and include regular management
supervisory activities, such as reviewing the purchasing function.

Answer A is incorrect because regularly providing information about the


functioning of internal control is monitoring of controls by the entity’s
internal auditors through separate evaluations.
Answer C is incorrect because the audit committee (a subcommittee of the
board of directors that is composed of board members who are not offices of
the entity) does not ordinarily perform periodic audits.

Answer D is incorrect because the annual audit of financial statements is not


a monitoring activity.

69. Control activities relate to the following, except


A. Segregation of duties
B. Performance reviews
C. An internal audit function
D. Authorization

Control activities are the policies and procedures that help ensure that
management directives are carried out. They are intended to ensure that
necessary actions are taken to address risks that threaten the achievements of
the entity’s objectives. Control activities have various objectives and are
applied at various organizational and functional levels. Specific control
activities include those that relate to:
• Authorization
• Performance reviews
• Information processing
• Physical controls
• Segregation of duties

An internal audit function is part of the monitoring component of internal


control.
70. The following are examples of monitoring activities, except
A. Information Technology (IT) management regularly generates reports
for unusual transactions or volumes of transactions and follows up
with investigation as to causes.
B. Management has asked internal auditing to perform regular audits of
the controls over cash processing.
C. Management regularly compares divisional performance with budgets
for the division.
D. IT management regularly reconciles batch control totals for items
processed with batch controls for items submitted.

Reconciling batch control totals being a processing control is not part of the
internal control’s monitoring component.

71. Which of the following is a reason to establish internal


control?
A. To provide reasonable assurance that the entity’s objectives are achieved.
B. To safeguard the resources of the organization.
C. To encourage compliance with the organizational objectives.
D. To ensure the accuracy, reliability, and timeliness of information.

According to PSA 315, “Internal control is the process designed,


implemented and maintained by those charged with governance,
management, and other personnel to provide reasonable assurance about the
achievement of the entity’s objectives with regard to reliability of financial
reporting, effectiveness and efficiency of operations, and compliance with
applicable laws and regulations.”

Answers B, C, and D are incorrect because they are subsumed under the
overall purpose of providing reasonable assurance that the entity’s objectives
are achieved.

72. An internal control system that is working effectively A. Eliminates risk and
potential loss to the entity.
B. Cannot be circumvented by management.
C. Reduces the need for management to review exception reports on a day-
to-day basis.
D. Is unaffected by changing circumstances and conditions encountered by
the entity.

An effective internal control system reduces the need for management to


spend tie reviewing exception reports on a dayto-day basis. An entity’s
internal control system, if working effectively, should prevent as well as
detect and correct exceptions.

Answer A is incorrect because some risks are unavoidable and others are too
costly to eliminate.

Answer B is incorrect because a basic limitation of internal control is the


potential for management override.

Answer D is incorrect because changes in circumstances and conditions may


require modification of internal control.

73. Because of inherent limitations, internal control cannot be designed to


provide reasonable assurance regarding the achievement of objectives
concerning A. Effectiveness and efficiency of operations.
B. Elimination of all fraud.
C. Reliability of financial reporting.
D. Compliance with applicable laws and regulations.

74. When considering an entity’s system of internal control, one of the auditor’s
major concerns is to ascertain whether internal control is designed to provide
reasonable assurance
that
A. Financial statements are fairly presented.
B. The accounting manager reviews all accounting transactions.
C. Profit margins are maximized, and operational efficiency is optimized.
D. Corporate morale problems are addressed immediately and effectively.

Controls that are relevant to an audit pertain to the entity’s objective of


preparing financial statements for external purposes that are fairly presented
in accordance with an applicable financial reporting framework.

75. When an entity’s internal control is functioning effectively, management can


expect various benefits. The benefit least likely to occur is
A. Some assurance of compliance with laws and regulations.
B. Availability of reliable data for decision-making purposes.
C. Reduced cost of an external audit.
D. Elimination of employee fraud.
Because of inherent limitations of any system of internal control, even the
most effective internal control cannot guarantee the elimination of employee
fraud.

76. Internal controls should be designed to provide reasonable assurance that


A. Management’s planning, organizing, and directing processes are properly
evaluated.
B. Management’s plans have not been circumvented by employee collusion.
C. Material errors or fraud will be prevented, or detected and corrected,
within a timely period by employees in the course of performing their
assigned duties.
D. The internal auditing department’s guidance and oversight of
management’s performance is accomplished economically and
efficiently.

Internal controls should be designed to prevent, or detect and correct,


material errors or fraud within a timely period by employees in the normal
course of their assigned duties.

Answer A is incorrect because it is the entity’s internal auditing department


that evaluates management processes.
Answer B is incorrect because collusion is an inherent limitation of internal
control.

Answer D is incorrect because those charged with governance are responsible


for the guidance and oversight of management.

77. Which of the following is a correct statement about internal


control?
A. The cost-benefit relationship is a primary criterion that should be
considered in designing internal control.
B. The auditor can eliminate substantive tests on significant account
balances and classes of transactions for an entity that has exceptionally
strong internal control.
C. The internal auditor has the responsibility to establish and maintain
internal control.
D. Properly maintained internal control reasonably ensures that collusion
among employees cannot occur.

The cost-benefit relationship is a primary criterion in designing internal


control – that is, the cost of a control should not exceed its benefits. Because
it is impossible to precisely measure the costs and benefits of internal control,
both quantitative and qualitative estimates and judgments are used by
management to evaluate the relationship.
78. When obtaining knowledge about an entity’s internal control, it is important
for the auditor to consider the competence of its employees, because their
competence bears directly
and importantly upon the
A. Cost-benefit relationship of internal control.
B. Comparison of recorded accountability with assets.
C. Achievement of the objectives of internal control.
D. Timing of substantive tests to be performed.

An important element of the control environment component of internal control is


human resource policies and practices. Human resource policies and practices
concern hiring, orientation, training, evaluating, counseling, compensating,
promoting, and remedial actions. The objectives of internal control cannot be
achieved without sufficient competent personnel who will operate the system.

Answer A is incorrect because the cost-benefit relationship is a basic concept of


internal control.
Answer B is incorrect because although comparison of recorded accountability
with assets is an important characteristic of internal control, it is not directly
related to the competence of employees.

Answer D is incorrect because the timing of substantive tests to be performed


depends on a number of factors such as the assessed level of risk of material
misstatement.

79. Control activities are the policies and procedures that help ensure that
management directives are carried out. These include activities relating to
authorization, performance reviews, information processing, physical
controls, and segregation of duties. There is proper segregation of duties
when an individual who
A. Records a transaction does not compare the accounting record of the
asset with the asset itself. B. Authorizes a transaction records it.
C. Authorizes a transaction maintains custody of the asset that resulted from
the transaction.
D. Maintains custody of an asset has access to the accounting records for the
asset.

A proper segregation of duties requires that one person should not be


responsible for all phases of a transaction: authorization, recording, and
custodianship of the related assets. Separate individuals should perform these
incompatible duties to reduce the opportunities for any person to be in a
position to both perpetrate and conceal errors or fraud in the normal course of
his/her duties.

80. Proper segregation of duties reduces the opportunities for persons to be in


positions to both
A. Establish internal control and authorize transactions.
B. Perpetrate and conceal errors or fraud.
C. Record cash receipts and cash disbursements.
D. Record transactions and prepare financial statements.

81. Control activities include those that relate to physical controls over access to
and use of assets and records. A departure from the purpose of physical
controls is that
A. The mail room clerk compiles a list of the checks received in the
incoming mail.
B. Access to safe-deposit box requires two officers.
C. Only warehouse personnel and production supervisors have access to
raw materials storeroom.
D. Only sales personnel use sales department vehicles.

82. Which of the following most likely would not be considered an inherent
limitation of internal control?
A. Management override
B. Incompatible functions
C. Mistakes in judgment
D. Collusion among employees

The performance of incompatible functions is not an inherent limitation of


internal control but a failure to segregate functional responsibilities properly.

Answers A, C, and D are inherent limitations of internal control.

83. A basic concept of internal control is the concept of reasonable assurance,


which recognizes that
A. Establishing and maintaining internal control is an important
responsibility of management.
B. Internal control may be ineffective due to collusion among employees.
C. The cost of an entity’s internal control should not exceed the benefits
expected to be derived.
D. Adequate safeguards over access to assets and records should permit an
entity to maintain proper accountability.
84. An auditor should obtain sufficient knowledge of an entity’s information
system, including the related business processes relevant to financial
reporting, to understand the
A. Policies used to detect the concealment of fraud.
B. Process used to prepare significant accounting estimates.
C. Safeguards used to limit access to computer facilities.
D. Procedures used to assure proper authorizations of transactions.

The auditor should obtain an understanding of the information s ystem relevant to


financial reporting to understand:

a) The classes of transactions which are significant to th e financial


statements.
b) The procedures – within both IT (Information Technology) and manual
systems – by which those transactions are init iated, recorded, processed,
and reported in the financial statements.
c) The accounting records and supporting documents for those transactions.
d) How the information system captures events and conditions , other than
transactions, that are significant to the fi nancial statements.
e) The financial reporting process used to prepare the entit y’s financial
statements, including significant accountin g estimates and disclosures.

Answers A, C, and D are incorrect because they describe activities outside the
accounting system relevant to financial reporting.

85. The primary objective of procedures performed to obtain an understanding


of internal control is to provide an auditor
with
A. Knowledge necessary to plan the audit.
B. A basis for modifying tests of controls.
C. Information necessary to prepare flowcharts.
D. Evidence to use in reducing detection risk.

Understanding the entity’s internal control system is a matter that should be


considered by an auditor in developing overall strategy.

Answer B is incorrect because the auditor’s understanding is of internal


control is the basis for determining the nature, timing, and extent of both
substantive tests and tests of controls.

Answer C is incorrect because flowcharts are but one form of internal


control documentation.

Answer D is incorrect because substantive tests are formed to gather


evidence in order to reduce detection risk to an acceptable level.

86. In a financial statement audit, the auditor is required to obtain an


understanding of the entity’s internal control to assess the risk of material
misstatement in the financial statements. The result of the understanding

A. Bear no relationship to the nature, timing, and extent of further audit


procedures to be performed.
B. Must be reported to the stockholders and the SEC.
C. Are not reported to client management.
D. May be used as the basis for withdrawing from an audit engagement.

PSA 315 states that the auditor’s understanding of internal control may raise
doubts about the auditability of an entity’s financial statements.

Concerns about the integrity of the entity’s management may be so serious


as to cause the auditor to conclude that the risk of management
representation in the financial statements is such that an audit cannot be
conducted.

Also, concerns about the condition and reliability of an entity’s records may cause
the auditor to conclude that it is unlikely that sufficient appropriate audit
evidence will be available to support an unmodified opinion on the financial
statements. In such circumstances, the auditor considers a qualification or
disclaimer of opinion, but in some cases, the auditor’s only recourse may be
to withdraw from the engagement.

87. In obtaining an understanding of internal control relevant to the audit, an


auditor is required to obtain knowledge about the
A. Effectiveness of controls that have been implemented.
B. Consistency in which controls are currently being applied.
C. Design of the controls pertaining to internal control components.
D. Controls related to each class of transactions and account balance.

According to PSA 315, obtaining an understanding of internal control


involves evaluating the design of a control and determining whether it has
been implemented.

Evaluating the design of a control means determining whether the control


individually or in combination with other controls capable of effectively
preventing, or detecting and correcting, material misstatements.

Implementation of a control means that the control exists and that the entity
is using it.

Answers A and B are incorrect because in some cases, the auditor may
determine that performing only substantive procedure is appropriate for
specific assertions and, therefore, may choose not to obtain information
about the operating effectiveness of controls and their consistency of
application.

Answer D is incorrect because the standard does not explicitly require


obtaining knowledge about controls related to each transaction class and
account balance.

88. The auditor uses the understanding of internal control to

I. Identify types of potential misstatements.


II. Consider factors that affect the risks of material misstatement.
III. Design the nature, timing, and extent of further audit procedures.
A. I and II only
B. I and III only
C. II and III only
D. I, II, and III

89. In obtaining an understanding of internal control relevant to the audit, the


auditor may trace several transactions through the system, including how the
transaction interface with any service organizations whose services are part
of the entity’s information system. The primary objective of this procedure is
to
A. Evaluate the design of the internal control and determine whether it has
been implemented.
B. Determine the effectiveness of internal control.
C. Detect fraud.
D. Replace substantive tests.

90. Information about segregation of duties ordinarily is best obtained by


A. Performing tests of transactions that corroborate management’s financial
statement assertions.
B. Developing audit objectives that reduce control risk.
C. Observing employees as they apply specific controls.
D. Obtaining a flowchart of activities performed by entity personnel.

According to PSA 315, risk assessment procedures to obtain audit evidence about
the design and implementation of relevant controls may include:

• Inquire entity personnel.


• Observing the application of the controls.  Inspecting documents and
reporting.
• Tracing transactions through the information system relevant to financial
reporting

91. In obtaining an entity’s internal control in a financial statement audit, an


auditor is not required to
A. Determine whether the controls have been implemented.
B. Perform procedures to evaluate the design of controls.
C. Document the understanding of the entity’s internal controls
components.
D. Search for significant deficiencies in the operation of internal control.

The auditor should obtain a sufficient understanding of internal control to


assess the risks of material misstatement. The understanding includes
knowledge about the design of relevant controls and whether they have been
implemented, i.e., whether they have been placed in operation. Though the
auditor may become aware of material weaknesses of internal control,
he/she is not required to search for such internal control weaknesses or
deficiencies. A financial statement audit is not designed to determine the
adequacy of internal control for management purposes.

92. In a financial statement audit, the auditor is required to perform tests of


controls when

I. The auditor's risk assessment includes an expectation of the operating


effectiveness of controls.
II. When substantive procedures alone do not provide sufficient
appropriate audit evidence at the assertion level.

A. I only
B. II only
C. Either I or II
D. Neither I nor II

Under PSA 330 (The Auditor' Responses to Assessed Risks), the auditor
should perform tests of controls when his/her assessment of risks of material
misstatements at the assertion level includes an expectation that controls are
operating effectively or when substantive procedures alone do not provide
sufficient appropriate evidence to reduce the risk of material misstatement at
the assertion level.

93. Which of the following determines the extent of the auditor's tests of
control? A. Auditor's knowledge
B. Auditor's initial/planned assessment control of risk
C. Resources available to the auditor
D. Management's desire to help the auditor

94. After obtaining an understanding of internal control relevant to the audit, the
auditor may attempt to assess control risk at below the maximum level. In
turn, the auditor will (1) identify specific controls that are likely to prevent
or detect material misstatements in the relevant financial statement
assertions and (2) perform tests of controls. The purpose of the tests of
controls is to A. Evaluate inherent risk.
B. Assure that the auditor has a sufficient understanding of internal control.
C. Evaluate the effectiveness of such controls.
D. Provide recommendations to management to improve internal control.
The auditor performs tests of controls to obtain assurance about the
operating effectiveness of controls.

95. Tests of controls are concerned primarily with each of the following
questions, except
A. By whom were the controls applied?
B. Were the necessary controls consistently performed?
C. How were the controls applied?
D. Why were the controls applied?

Testing the operating effectiveness of controls includes obtaining audit


evidence about (1) how controls were applied at relevant times during the period
under audit, (2) the consistency with which they were applied, and (3) by whom
or by what means they were applied. Determining why controls were applied
relates more to obtaining an understanding of internal control than testing the
effectiveness of controls.
96. In an audit of financial statements in accordance with PSAs, an auditor is
required to
A. Identify specific controls relevant to management’s financial statement
assertions.
B. Determine whether controls are suitably designed to prevent or detect
material misstatement.
C. Perform tests, of controls to evaluate the operating effectiveness of
controls
D. Document the auditor's understanding of the entity's internal control.

The auditor is required to document in the audit working papers the


understanding obtained of the entity's internal control and the assessment of
control risk. When control risk is assessed at less than the maximum, the
auditor should also document the basis for the conclusion.

Answers A, B, and C are incorrect because these procedures are to be


performed only when the auditor wishes to assess control risk at less than
the maximum.

97. In performing a walk-through test, the


A. Auditor traces one or several transactions within each major class through
the transaction trail to confirm his
or her understanding of the information system and control procedures.
B. Auditor performs a physical, review of the entity's property, plant and
equipment to confirm the physical location of major assets.
C. Senior auditor "walks-through" the processing of the entity's transactions
with other members of the audit team focusing on the control process
which aim to prevent misstatements in the financial statements.
D. Auditor traces one or several transactions within each major class through
the transaction trail to confirm the cut-off period.

98. In evaluating the effectiveness of the design of an entity's internal control


system in order to make a preliminary assessment of control risk for an
assertion the auditor has to

I. Identify potential misstatements that could occur in the entity's


assertion.
II. Identify the necessary controls that would be likely to prevent or detect
the material misstatements.
III. Evaluate the evidence and makes the assessments.

A. I and II only
B. I and III only
C. II and III only
D. I, II, and III

99. The ultimate purpose of assessing control risk is to contribute the auditor's
evaluation of the risk that
A. Material misstatements may exist in the financial statements.
B. Specified controls requiring segregation of duties may be circumvented
by collusion.
C. Entity's control may be overridden by management.
D. Tests of controls may fail to identify procedures relevant to assertions.

The ultimate purpose of assessing control risk at the assertion level for each
material account balance or class of transactions is to contribute to the
auditor's evaluation of the risk that material misstatements exist in the
financial statements.

100. An auditor may decide to assess control risk at a high level for some or all
assertions because the auditor believes
A. The entity's internal control system is not effective.
B. More emphasis on tests of controls than substantive tests is warranted.
C. Sufficient appropriate audit evidence to support the assertions is likely to
believe.
D. The entity's internal control components are interrelated.
The auditor ordinarily assesses control risk at a high level for some or all
assertions when:
a) the entity's internal control system is not effective; or
b) evaluating the operating effectiveness of the entity's
control would not be efficient.

101. An auditor assesses control risk in terms of A. Types of potential fraud.


B. Financial statement assertions.
C. Specific control activities.
D. Control environment elements.

The assessment of control risk is the process of evaluating the effectiveness


of an entity's internal control in preventing or detecting and correcting
material misstatements. Control risk is assessed in terms of financial
statements assertions.

102. The preliminary assessment of control risk for a financial statement assertion
should be high unless the auditor
I. Is able to identify controls relevant to the assertion which are likely to
prevent, or detect and correct, material misstatements.
II. Plans to perform tests of controls to support the assessment.

A. I only
B. II only
C. Either I or II
D. Neither I nor II

103. In performing tests of the operating effectiveness of an entity's controls, an


auditor selects from a variety of techniques, including
A. Reperformance and observation.
B. Inquiry and analytical procedures.
C. Comparison and confirmation. D. Inspection and verification.

Tests of controls ordinarily include inquiry, observation, inspection, and


reperformance of a policy or procedure that pertains to an assertion.

Answer B, C, and D are incorrect because they include procedures which are
more closely associated with substantive testing — namely, analytical
procedures, comparison, confirmation, and verification.
104. An auditor intends to perform tests of control on a client's cash
disbursements procedures. If the control procedures leave no audit trail of
documentary evidence, the auditor
most likely will test the procedures by A. Inquiry and
analytical procedures.
B. Inquiry and observation.
C. Analytical procedures and confirmation.
D. Confirmation and observation.

According to PSA 330, documentation of operation of controls may not exist


for some factors in the control environment, such as assignment of authority
and responsibility, or for some types of control activities, such as those
performed by a computer. In such cases, audit evidence about operating
effectiveness may be obtained through inquiry in combination with other
procedures like observations and the use of computer-assisted audit
techniques (CAATs).

105. Which of the following procedures concerning accounts receivable would an


auditor most likely perform to obtain audit evidence in support of an
assessed level of control risk that is less than high?

A. Comparing an entity's bad debt expense to actual bad debt losses.


B. Inspecting an entity's analysis of accounts receivable for unusual
balances.
C. Observing an entity's staff prepare the schedule of past due accounts
receivable.
D. Sending confirmation requests to an entity's principal customers to
verify the existence of accounts receivable.
An auditor who wishes to assess control risk at less than high should
perform tests of operating effectiveness of relevant controls. Tests of
controls include inquiry, observation, inspection, and reperformance of a
control by the auditor.

106. The following statements relate to an auditor's assessment of control risk.


Which is true?
A. The lower the assessed level of control risk, the less assurance the audit
evidence must provide about operating effectiveness of controls.
B. The basis for an auditor's conclusion about the assessed level of control
risk need not be documented unless control risk is assessed at the
maximum level.
C. Assessing control risk and obtaining an understanding of an entity's
internal control may be performed concurrently.
D. When assessing control risk, an auditor should not consider evidence
obtained in prior audits about the operation of controls.
For efficiency reasons, an auditor may plan to perform tests of controls when
evaluating the design of controls and obtaining evidence of their
implementation.

107. The following statements relate to the use of audit evidence when testing the
operating effectiveness of relevant controls. Which is false?
A. An auditor who obtains sufficient appropriate audit evidence about the
operating effectiveness of controls during the interim period should no
longer obtain additional evidence of operating effectiveness for the
remaining period.
B. An auditor may plan to use audit evidence about the operating
effectiveness of controls obtained in the prior audits.
C. If an auditor plans to rely on controls that have changed since they were
last tested, the auditor should test the operating effectiveness of such
controls in the current audit.
D. Audit evidence pertaining only to a point in time may be sufficient for the
auditor's purpose, for example, when testing controls over an entity's
physical count of inventories at year-end.

PSA 330 states that when the auditor obtains audit evidence about the
operating effectiveness of controls during an interim period, the auditor
should determine what additional audit evidence should be obtained for the
remaining period. In addition, the auditor is required to obtain audit
evidence about the nature and extent of any significant changes in internal
control, including changes in the information system, processes, and
personnel that occur subsequent to the interim period.

Answers B and C are correct statements. PSA 330 states that if the auditor
plans to use audit evidence about the operating effectiveness of controls
obtained in prior audits, the auditor should obtain audit evidence about
whether changes in those specific controls have occurred subsequent to the
prior audit. In this regard, the auditor may perform inquiry in combination
with observation or inspection. If the auditor plans to rely on controls that
have changed since they were last tested, the auditor is required to test the
operating effectiveness of such controls in the current audit.

Answer D is a correct statement. According to PSA 330, the timing of tests


of controls depends on the auditor's objective and determines the period of
reliance on those controls.

Tests of controls at a particular time provide evidence of operating


effectiveness at that time, while tests controls throughout a period provide
evidence of operating effectiveness for that period.
Audit evidence of operating effectiveness at a point in time
may be sufficient for the auditor's purpose, such as tests of controls over
physical inventory at year-end.

108. According to PSA 330, an auditor who plans to rely on controls that have
not changed since they were last tested should test the operating
effectiveness of such controls at least once in every A. Second audit B.
Third audit
C. Fourth audit
D. Fifth audit

An auditor may plan to rely on controls that have not changed since they
were last tested. The length of time period between retesting such controls is
a matter of professional judgment but cannot exceed two years. Therefore,
the operating effectiveness of such controls should be tested at least once in
every third audit.

109. Which of the following procedures are performed by an auditor who wants
to detect material misstatements at the assertion level? A. Compliance tests
B. Tests of controls
C. Substantive tests
D. Dual-purpose tests

Substantive tests are procedures performed in order to detect material


misstatements at the assertion level, including tests of details of account
balances, classes of transactions, and disclosures, and substantive analytical
procedures.

110. After gaining an understanding of internal control and assessing the risks of
material misstatement, an auditor decided to perform tests of controls. The
auditor most likely decided that
A. Additional evidence to support a further reduction in control risk is not
available.
B. It is not possible or practicable to reduce the risks of material
misstatements at the assertion level to an acceptably low level with audit
evidence obtained only from substantive test procedures.
C. There were many internal control weaknesses that could allow
misstatements to enter the accounting system.
D. An increase in the assessed level of control risk is justified for certain
financial statement assertions.

An auditor shall perform tests of controls when:


1. The auditor's risk assessment includes an expectation of the
operating effectiveness of controls; or
2. Substantive procedures alone do not provide sufficient appropriate
audit evidence at the assertion level.

Answer A is incorrect because tests of controls are performed to obtained


sufficient appropriate audit evidence to support a further reduction in control
risk.

Answer C is incorrect because the auditor should not perform tests of


controls when, as a result of his/her assessment, there is no expectation of
the operating effectiveness control, i.e., if there are many internal control
weaknesses.

Answer D is incorrect because tests of controls are designed to decrease the


auditor's preliminary assessment of control risk, not increase it.

111. In conducting an audit in accordance with PSAs, the auditor is required to


identify and assess the risk of material misstatement at financial statement
level, and at the assertion level for classes of transactions, account balances,
and disclosures. Some of these risks, in the auditor's judgment, require
special audit consideration, such as those involve fraud or complex
transactions. Such risks are called A. Business risk
B. Audit risk
C. Significant risk
D. Material risk

112. As a result of obtaining an understanding of the entity's internal control


system, the auditor may become aware of material weaknesses in the design
or implementation of internal control. The auditor is required to report this
mater to
A. Those charged with governance or management
B. Chief executive office
C. Securities and Exchange Commission
D. Board of Accountancy

PSA 315 states, "The auditor shall communicate material weaknesses in


internal control identified during the audit on a timely basis to management
at an appropriate level of responsibility, and, as required by PSA 260,
(Communication with Those Charged with Governance), with those charged
with governance (unless all of those charged with governance are involved
in managing the entity.)"
113. The auditor's final assessment of control risk for financial statement
assertions is based on
A. Evaluating the evidence obtained from the entity's management.
B. Evaluating the evidence obtained from third parties.
C. Evaluating the evidence obtained from an understanding of relevant
internal control system components and related tests of controls.
D. Evaluating the evidence obtained from an understanding of relevant
internal control system components.

REVENUE/RECEIPT CYCLE

114. Which of the following is not a source document for the revenue cycle? A.
Sales order
B. Receiving report C.
Credit memo
D. Delivery receipt

115. Which of the following activities is not part of the revenue cycle?
A. Sales order entry
B. Receiving
C. Billing
D. Shipping

116. In the revenue cycle, a customer places an order for a certain product. What
step should be taken before the order is checked for inventory availability?
A. A packing list should be generated for the warehouse.
B. The customer's credit should be checked or a sale on account.
C. The sales order should created and written to a file.
D. The shipping department should be notified of an order in process.

117. ________________ is the maximum allowable account balance for a given


customer. A. Credit limit
B. Reorder point
C. Credit checkpoint
D. Backorder point

118. An essential part of the revenue cycle is filling customer orders and
shipping goods to customer. Automating warehouse systems cut costs,
improves efficiency, and enables more customer-responsive shipments.
Which of the following elements is required for an automated perpetual
inventory system? A. Forklifts
B. Conveyor belts
C. Bar-code scanners
D. VAN

119. The basic document created in the billing process is the


A. Bill of lading
B. Purchase order
C. Sales invoice
D. Packing list
120. To ensure proper segregation of duties, who should make decisions
concerning issuance of credit memos?
A. Warehouse manager
B. Credit manager
C. Cashier
D. Accounts receivable supervisor

121. Which of the following is not an objective of the revenue/receipt cycle?


A. Received goods are counted and inspected or quality.
B. Custody over assets resulting from the revenue/receipt is properly
maintained.
C. Transactions related to the revenue/receipt cycle are properly authorized,
executed, and recorded.
D. Approved sales orders are shipped on a timely basis and in accordance
with customer specifications.

The revenue/receipt cycle encompasses the activities pertaining to the sale


of goods or services and the collection of cash for the goods or services
provided. Counting and inspecting received goods for quality is an objective
of the expenditure/disbursement cycle and is only indirectly related to the
revenue/receipt cycle.
122. The following are directly involved in revenue/receipt cycle, except?
A. Treasurer and controller
B. Receiving department clerk
C. Billing clerk
D. Sales manager and the credit manager

The receiving department clerk counts and inspects the goods received from
suppliers and prepares receiving reports which serve as partial authorization
for invoice payment. The activity relates more directly to
expenditure/disbursement cycle.

Answer A is incorrect because the treasurer has custody of cash receipts


from customers and the controller maintains records such as those relating to
sales and cash receipts.
Answer C is incorrect because the billing clerk is responsible for the billing
process, including the preparation of the sales invoices.

Answer D is incorrect because the sales manager executes sales transactions


and the credit manager is responsible for authorizing sales.

123. Which of the following controls most likely would provide reasonable
assurance that all credit sales transactions of an entity are recorded?
A. The accounting department supervisor controls the mailing of monthly
statements to customers and investigates any differences reported by
customers.
B. The accounting department supervisor independently reconciles, on a
monthly basis, the account receivable subsidiary ledger to the accounts
receivable control account.
C. The billing department supervisor matches prenumbered shipping
documents with entries in the sales journal.
D. The billing department supervisor sends copies of approved sales orders
to the credit department for comparison to authorized credit limits and
current customer account balances.

Comparing prenumbered shipping documents with entries in the sales


journal will detect unrecorded sales transactions.

Answer A is incorrect because unrecorded credit sales transactions result to


understatement of receivables. Customers are unlikely to report
understatement of their accounts.

Answer B is incorrect because the reconciliation will not detect unrecorded


sales transactions.

Answer D is incorrect because credit approval does not provide assurance


that all sales transactions have been recorded.

124. For effective internal control, employees maintaining the accounts


receivable subsidiary ledger should not also approve
A. Cash disbursements
B. Write-offs of customer accounts
C. Granting of credit customers
D. Employee overtime wages

Segregating the functions of authorization, record-keeping, and custody of


assets provides reasonable assurance that perpetration and concealment of
fraud or error by the same employee will be minimized. Therefore, the
authorization function of approving write-offs of receivables should not be
done by the same employee who performs a record-keeping function of
maintaining the accounts receivable subsidiary ledger.

Answer A and D are incorrect because authorizing cash disbursements and


employee overtime wages are not related to receivables.

Answer C is incorrect because although granting of credit authorizes a debit


to receivable, combining this function with maintaining the accounts
receivable subsidiary ledger is less likely to result in fraud.

125. A sound internal control procedure should require that defective


merchandise returned by the customers be presented initially to the
A. Receiving clerk
B. Accounts receivable supervisor
C. Billing clerk
D. Shipping department supervisor

Sound internal control procedures dictate that all receipts of merchandise


should be initially handled by the receiving department personnel.
Receiving reports must be prepared for all goods received.

126. An employee misappropriates cash receipts from sales on account. Which


of the following acts would conceal this defalcation and be least likely to be
detected by an auditor?
A. Understating the cash receipts journal.
B. Overstating the accounts receivable subsidiary ledger.
C. Overstating the accounts receivable control account.
D. Understating the sales journal.

The most effective way of concealing theft of cash receipt is by not


recording sales on account. The procedure that will be applied in the
accounting records will not detect the defalcation because the accounts
remain balanced, yet incomplete. In addition, customers are unlikely to
report the irregularity to the auditor because unrecorded sales will not be
billed and unrecorded receivables will not be confirmed.

Questions 127 through 130 are based on the following information:


In connection with your audit of the financial statements of ABC Company, you
gathered the following information about its sales procedures:

Customer orders are received by the sales-order department. A clerk computes the
peso amount of the order and sends it to the credit department for approval. Credit
approval is stamped on the order and returned to sales order department. A sales
invoice is prepared, and the sales order is filled in the customer order file.
The customer copy of the sales invoice is sent to the billing department and held
in the pending file awaiting notification that the order has been shipped.

The shipping copy of the sales invoice is routed to warehouse and the shipping
department as authority for the respective departments to release and ship the
goods.

Shipping department personnel pack the order and prepare a threecopy bill of
lading:
Original copy - mailed to the customer
Second copy - sent with the shipment
Third copy - filed in sequence in the bill of lading file

The shipping copy of the sales invoice is sent to the billing department.

The billing clerk matches the received shipping copy of the sales invoice with the
customer copy from the pending file. Both copies of the invoice are priced,
extended, and footed. The customer copy is then mailed to the customer, and the
shipping copy is sent to the accounts receivable clerk.

The accounts receivable clerk enters the sales invoice data in the sales journal,
posts the customer's account in the accounts receivable subsidiary ledger, and files
the shipping copy in the sales invoice file. The sales invoices are numbered and
filed in sequence.

127. To obtain evidence, concerning the proper credit approval of sales, the
auditor would select a sample of transaction document from the population
represented by the
A. Bill of lading file
B. Accounting receivable subsidiary ledger
C. Sales invoice file
D. Customer order file

As described, the entity's procedure is to send the customer order to the


credit department for approval. Approval is stamped on the customer order
which is then returned to the sales department and filed. Thus, the only
documentation for credit approvals is the customer order file.
128. In determining whether the internal control operated effectively to
minimize errors of failure to post sales invoices to the accounts receivable
subsidiary ledger, the auditor should select a sample of transactions from
the population represented by the
A. Sales invoice file
B. Accounts receivable subsidiary ledger
C. Customer order file
D. Bill of lading file

A sample of sales invoices should be traced to the accounts receivable


subsidiary ledger. Sales invoices without corresponding entries in the
accounts receivable subsidiary ledger represent transaction not posted.

129. To effectively determine whether the entity's control activities minimize


errors of failure to invoice goods that have been shipped, the auditor should
select a sample of transactions from the population represented by the
A. Sales invoice file
B. Bill of lading
C. Accounts receivable subsidiary ledger
D. Customer order file

To determine if goods that have been shipped were not invoiced, an effective
audit procedure is to match a sample from the bill of lading file to sales
invoices.
130. To obtain evidence that uncollected items in customer's accounts
represented valid trade receivables, the auditor should select a sample of
items from the population represented by the A. Bill of lading
B. Customer order file
C. Accounts receivable subsidiary ledger
D. Sales invoice file

Confirmation directly with customers of a sample of open accounts from the


accounts receivable subsidiary ledger would provide an auditor with
evidence that the receivables are valid assets.

131. Which of the following most likely would be the result of ineffective
controls in the revenue/receipt cycle?
A. Omission of shipping documents could go undetected, causing an
understatement of inventories.
B. Irregularities in recording transactions in the subsidiary accounts could
result in a delay in goods shipped.
C. Final authorization of credit memos by sales department personnel
could permit an employee defalcation scheme.
D. Fictitious transactions could be recorded causing an understatement of
revenues and an overstatement of receivables.

Final approval of credit memos in the sales department may allow sales
department personnel to commit irregularities.
Answer A is incorrect because if shipping documents are omitted, shipped
goods may not be credited to inventory, causing the account to be
overstated, not understated.

Answer B is incorrect because posting in the subsidiary accounts takes place


only after shipment of goods.

Answer D is incorrect because recording fictitious sales would overstate


revenues.

132. At which point in an ordinary sales transaction of a wholesaling business is


a lack of specific authorization of least concern to the auditor in performing
risk assessment procedure?
A. Granting of credit
B. Determination of discounts
C. Selling of goods for cash
D. Shipment of goods

Selling of goods for cash would likely be covered by a general


authorization.

Answers A, B, and D are incorrect because granting of credit, determination


of discounts, and shipment of goods may require specific authorization.

133. EFG Company uses its sales invoice for posting perpetual inventory
records. Inadequate internal control over the invoicing function allows good
to be shipped but not invoiced. The inadequate controls could cause what
type of misstatement in each of the following accounts?

Revenues Receivables Inventories


A. Understatement Understatement Understatement B. Overstatement
Overstatement Understatement
C. Understatement Understatement Overstatement
D. Overstatement Overstatement Overstatement

If goods are shipped but are not invoiced, there will be no documentation for
sales, thus understanding revenues and receivables. Moreover, inventory
will not be credited thereby overstating the account.

134. Which of the following controls is often lacking in a retail cash sales
environment? A. Segregation of functions
B. Competent personnel
C. Monitoring
D. Access to assets limited to authorized personnel.

In a retail cash sales environment, the sales clerk ordinarily authorizes and
records the transactions and has custody of assets. But the apparent lack of
proper segregation o functions is compensated by controls such as close
supervision of the cash registers that limits access to assets, and an effective
internal recording function that maintains control over cash receipts.

135. An auditor's risk assessment procedures disclosed that the accounts


receivable clerk approves credit memos and has access to cash. Which of
the following controls would be most effective in offsetting these
weaknesses?
A. The controller reconciles accounts to the amount shown in the ledger.
B. The owner reviews credit memos after they are recorded.
C. The controller receives the monthly bank statement directly and
reconciles the cash accounts.
D. The owner reviews errors in billings to customers and postings to the
subsidiary ledger.

The clerk is holding incompatible duties and thus, is an position to both


perpetrate and conceal a fraud in the normal course of his/her duties.

The clerk has custody of cash, authorizes credit memos, and performs the
record-keeping function for accounts receivable. Hence, the clerk can easily
misappropriate cash received from customers and conceal such defalcation
by recording fictitious sales returns.

In a small business, it may not be practicable to institute formal control


activities. In this situation, the active involvement of the owner may
compensate for the absence of some control activities such as proper
segregation of duties. The owner may consider reviewing credit memos for
authenticity.

Answer A is incorrect because there will be no discrepancy between the


subsidiary ledger and the control account even if improper credits to
accounts receivable are posted.

Answer C is incorrect because reconciliation of cash accounts will not


reveal misappropriation of cash collected from customers concealed by
recording improper credit memos.
Answer D is incorrect because the absence of errors is billings and postings
to the subsidiary ledger would not preclude the clerk from defalcating cash
collected from customers.
136. The following controls are ordinarily found in an entity's revenue/receipt
cycle. Which controls most likely would be effective in offsetting the
tendency of sales personnel to maximize sales volume at the expense of high
bad debt writeoffs?
A. Employees responsible for authorizing sales and bad debt write-offs are
denied access to cash.
B. Subsidiary account receivable ledgers are reconciled to the control
account by an employee independent of the authorization of credit.
C. Employees involved in the credit-granting function are separated from the
sales function.
D. Shipping documents and sales invoices are matched by an employee who
does not authorize write-offs of customers' accounts.

The sales department's function should be confined to the generation of sales


and provision of services to customers. The credit approval process should
be handled by the credit department.

The problem is the incompatibility of the sales and the credit—granting


functions. The controls described in answers A, B, and D do not address the
issue.
137. Which of the following control activities in an entity's revenue/receipt cycle
would provide reasonable assurance that all billed sales are correctly posts to
the accounts receivable ledger?
A. Each shipment of goods on credit is supposed by a prenumbered sales
invoice.
B. The accounts receivable subsidiary ledger is reconciled daily to the
accounts receivable control account in the general ledger.
C. Daily sales summaries are compared to daily postings to the accounts
receivable ledger.
D. Each sales invoice is supported by a prenumbered shipping document.

Daily sales summaries include information about billed sales. Reconciliation


of these summaries to the accounts receivables ledger would provide
reasonable assurance that all billed sales were posted.

Answer A is not incorrect because matching shipment with sales invoices


would provide reasonable assurance that goods shipped were billed.

Answer B is incorrect because reconciling the subsidiary ledger with the


control account would only prove that the aggregate amount debited to the
subsidiary ledger agrees with the amount debited to the control account in
the general ledger.
Answer D is incorrect because if sales invoices are supported by shipping
documents, it would assure that billed sales were actually shipped.

138. Which of the following tests of controls would most likely be performed by
an auditor to obtain evidence about management's assertion concerning the
completeness of sakes transactions?
A. Inquiries about the entity's credit granting policies and whether credit
checks are consistently applied.
B. Inspect the entity's reports of prenumbered shipping documents that
have not been recorded in the sales journal.
C. Compare prices on prenumbered sales invoices to the entity's
authorized price list.
D. Verify that extensions and footings on sales invoices and monthly
statements of customers' accounts have been checked.

Inspecting shipping that has not been recorded in the sales journal will
possibly disclose items that have been sold as evidence by the shipping documents
but were not recorded as sales.
139. An online sales order processing system most likely would have an
advantage over a batch sales order processing system by
A. Enabling shipment of customer orders to be initiated as the orders are
received.
B. Maintaining more accurate records of accounts receivables and
inventory.
C. Maintaining backup copies of the database.
D. Detecting errors in the data entry process by the use of edit checks.

In an online system, terminals installed in different locations are directly


connected to the computer network that makes it possible to process
transactions as they are entered. Hence, shipments of customer orders can be
initiated instantaneously as they are received. On the other hand, batch
processing involves processing of transactions in batches on a delayed basis.

140. When an office supply company is unable to fill an order completely, it


marks the out-of-stock items as back ordered on the customer's order file
that forms and enters these items in a back order file that can be viewed or
printed by management. Customers are becoming dissatisfied with the
company because of its failure to keep track of and ship out-of-stock items
as soon are they are available. Which of the following is the best approach to
ensure prompt delivery of out-of-stock items?
A. Increase inventory levels to minimize the occurrence of stock outs.
B. Match the back order file to goods received daily.
C. Reconcile the sum of filled and back orders with the total of orders of
all orders placed daily.
D. Implement electronic data interchange (EDI) with supply vendors to
decrease the time to replenish inventory.

The inability to keep track of and ship out-of-stock items as soon as they
become available may be addressed by matching the back order file to
goods received daily. Once identified, appropriate action on unfilled orders
could then be made.

Answers A, C, and D, are incorrect because the approaches described do not


ensure prompt delivery of items that are out of stocks.

EXPEDITURES/DISBURSEMENT CYCLE

141. Which of the following is a source document that would be found in the
expenditure cycle?
A. Journal voucher
B. Purchase order
C. Time card
D. Delivery report
142. ____________ is a standing order to purchase specified items at a
designated price, from a particular supplier for a set period of time.
A. Set order
B. Blanket purchase order
C. Purchasing order
D. Commodity order

143. Which of the following is probably the most effective control or the
prevention of kickbacks to purchasing agents? A. Review a vendor
performance.
B. A corporate policy to prohibit purchasing agents from accepting
kickbacks.
C. Purchasing from approved vendors.
D. Good supervision in the purchasing area.

144. The easiest way to prevent the acceptance of unordered goods is to


A. Always require that a valid purchase order exists before goods can be
accepted at the time of delivery. B. Order only from approved vendors.
C. Have an appropriate conflict of interest policy in place.
D. Require receiving department personnel to call the vendor before
accepting any goods.
145. The auditor's primary objective in obtaining an understanding of the client's
control over the purchasing function is to
A. Investigate the recording of unusual transactions regarding raw
materials.
B. Determine the reliability of financial reporting by the purchasing
function.
C. Observe the annual physical count.
D. Ascertain that raw materials paid for are on hands.

Controls are the relevant of to the audit are those that relate to the reliability
of financial reporting.

Answer A is incorrect because investigation of the recording of unusual


transactions is an audit procedure, not an audit objective.

Answer C is incorrect because observation of the annual physical count of


inventories relates more to the production cycle of the entity.

Answer D is incorrect because materials paid for need not be on hand —


materials may have been sold or used in production.

146. In a well-designed internal control system, employees in the same


department approves purchase orders and also
A. Negotiate terms with vendors
B. Authorize requisitions of goods.
C. Inspect and count goods upon receipt
D. Reconcile the open invoice file,

In a well-designed internal control system, the entity's purchasing


department approves purchase orders and negotiates terms with vendors as
part of the authorization process.

Answers B, C, and D are incorrect because authorizing the requisition of


goods, inspecting and counting goods received, and reconciling the open
invoice file are functions incompatible with the authorization performed by
the purchasing department.

147. Effective controls relevant to purchasing of raw materials should usually


include all of the following, except
A. Determining the need for the raw materials prior to preparing the
purchase orders
B. Systematic reporting of product changes that will affect the raw
materials.
C. Obtaining financial approval prior to making a commitment.
D. Obtaining third-party written quality and quantity reports prior to
payment for the raw materials.

Third-party written quality and quantity reports are not ordinarily sought
before making the payment. The entity receiving department personnel
usually have sufficient knowledge about the purchased goods. Only in
exceptional cases when an outside may be considered.

Answer A is incorrect because determining the need for the raw materials
before preparing the purchase order will ensure that only needed materials
will be ordered.

Answer B is incorrect because reporting of product changes affecting raw


materials ensures that obsolete materials will not be ordered.

Answer C is incorrect because obtaining financial approval before making a


commitment is important to ensure that sufficient funds will be available on
payment date.

148. As part of its purchasing system, entity's receiving department receives


copies of purchase orders for use in identifying and recording goods
received. The purchase orders list the name of the vendor and the quantities
of material ordered. A possible error that this system could allow is
A. Payment for unauthorized purchases
B. Payment to unauthorized vendors
C. Overpayment of partial deliveries
D. Delay in recording purchases.
If the quantities of the materials ordered are include in the purchase order
copy that is sent to the receiving department, receiving department
personnel may be tempted to just copy the quantities ordered from the
purchase order when they prepare receiving reports. Hence, a possible error
that this system could allow is overpayment for partial deliveries.

From the standpoint of a good internal control, a "blind copy" of the


purchase order-that is, no indication of the quantities ordered-should be sent
to the receiving department. That will ensure that its personnel will conduct
an actual count and inspection of goods received.
149. Which of the following controls is not usually performed in the accounts
payable department?
A. Indicating on the vouchers the affected asset and expense to be debited.
B. Approving vouchers for payment by having authorized employee sign
the vouchers.
C. Accounting for unused prenumbered purchase orders and receiving
reports.
D. Matching the vendor's invoice with the related purchase requisition,
purchase order, and receiving report.

The accounts payable department processes vendor invoices for payment.


Employees in this department should not perform functions related to
purchasing and receiving goods.

Accounting for unused prenumbered purchase orders and receiving reports


should be performed by employees in the purchasing and receiving
departments, respectively.

Answers A, B, and D are incorrect because the responsibilities described are


all functions of accounts payable department.

150. Which of the following procedures would best discourage the resubmission
of vendor invoices after they have been paid?
A. The mailing of payments directly to payees by accounts payable
department personnel.
B. A requirement for double endorsement of checks.
C. The cancellation of vouchers by accounting personnel.
D. The cancellation of vouchers by treasurer personnel.

Canceling the voucher package — voucher and supporting documents —


upon payment prevents duplicate payment of a voucher. This should be done
in the treasurer's office to ensure that the document will not be recycled for
duplicate payments.

Answer A is incorrect because mailing payments directly to payees does not


prevent recycling of invoices by unscrupulous individuals.

Answer B is incorrect because a single endorsement is not a control


weakness provided that proper supporting documents are required before the
check is signed and the signatory does not have incompatible duties.

Answer A is incorrect because the voucher package should not be canceled


before payment.

151. A university does not have a centralized receiving function for departmental
purchases of books, supplies, and equipment. Which of the following
controls would most effectively prevent payment for goods not received, if
performed prior to invoice payment?
A. Vendor invoices should be approved by a departmental supervisor other than
the employee ordering the goods.
B. Invoices for a specified amount should be approved by the vice president of
finance.
C. Names and addresses on vendor invoices should be compared to a list of
department-authorized vendors.
D. Vendor invoices should be matched with department purchase orders.

The departmental supervisors would most likely be aware f all goods


received by their departments. Thus, approval of vendor invoices by
departmental supervisors would most effectively prevent payment for goods
not received.

Answer B is incorrect because the vice president of finance does not receive
incoming goods.

Answer C and D is incorrect because comparing invoices with lists of


authorized vendors and matching invoices with purchase orders do not
provide evidence that goods were actually received.

152. To minimize the risk that purchasing agents will use their positions for
personal gain, an entity should
A. Direct the purchasing department to maintain records on purchase prices
paid, to be reviewed every 6 months.
B. Request internal auditors to send confirmation requests to selected
vendors.
C. Require competitive building.
D. Specify that all items purchased must pass quality control tests.
The purchase of goods through competitive bidding will reduce costs and
the possibility that purchasing agents will have side agreements with vendors.
The controls described in answers A,B and D are detective, not preventive
controls.
153. The following are appropriate questions on an internal control
questionnaire concerning purchase transactions, except
A. All are goods received in a centralized receiving department counted,
inspected, and compared with purchase orders on receipt?
B. Are intact cash receipts deposited daily in the bank?
C. Are pre-numbered purchase orders and receiving reports used and
accounted for?
D. Are an approved purchase requisition and a signed purchase order
required for each purchase?
The question about the daily deposit of intact cash receipts relates to the
revenue/receipt cycle, not the expenditure/ disbursement cycle.
154. The following questions ordinarily appear in an internal control
questionnaire on cash disbursements, except
A. Are pre-listings made of all cash receipts?
B. Is each check supported by an approved voucher?
C. Are imprinted and pre-numbered checks used and is a check provision
device used in printing the check amount? D. Are all disbursements except
for petty cash made by check?
Pre-listing of cash receipts is part of the revenue/receipt cycle.
155. In a well-designed internal control, the same employee may be permitted
to
A. Prepare receiving reports and also approve purchase orders.
B. Approve vouchers for payment and also have access to unused
purchased orders.
C. Mail signed checks and also cancel supporting documents D. Mail signed
checks and also prepare bank reconciliations.
In a typical cash disbursement system, approved check vouchers and
supporting documents are forwarded to the cash disbursements department.
This department, having a custodial function, is responsible for signing checks,
cancelling supporting documents, and mailing signed checks.
Answer A is incorrect because approving purchase orders is a function of
the purchasing department. The receiving department, responsible for the
preparation of receiving reports, should not know the quantity ordered.
Answer B is incorrect because vouchers are approved in the accounts
payable department, and only the purchasing department should be allowed
access to unused purchase orders.
Answer D is incorrect because the bank reconciliation should be performed
by an employee who does not have a custodial responsibility.
156. Which of the following is of least concern to an auditor in assessing the
risks of material misstatement?
A. Signed checks are distributed by the controller to approved payees.
B. Checks are signed by one person.
C. Cash receipts are not deposited intact daily.
D. Treasurer does not verify the names and addresses of check payees.
Answer A is incorrect because the controller, who performs a record-
keeping function, should not have access to signed checks.
Answer C is incorrect because daily deposit of intact cash receipts should
be required to minimize defalcation.
Answer D is incorrect because the treasurer should sign checks only after
verification of supporting documentation has been performed.
157. Under which of the following circumstances would an auditor be most
likely to intensify an audit of a P100,000 petty cash fund?
A. Petty cash vouchers are not pre-numbered.
B. The custodian endorses reimbursement checks.
C. Reimbursement occurs twice each week.
D. The custodian occasionally uses the petty cash fund to cash employee
checks.
Frequent reimbursement of petty cash fund (for example, twice each week)
suggests that the fund is not functioning as intended. In this case, the auditor
may need to intensify the audit of the petty cash fund.
Answer A and D are incorrect because although failure to pre- number
vouchers and occasional use of the fund to cash employee checks are not
preferable practices, they do not necessarily lead to commission of errors or
fraud.
Answer B is incorrect because the custodian’s endorsement is required to
cash a replenishment check.
158. In ABC Company’s accounting system, the quantities counted by the
receiving department and entered at a terminal are transmitted to the
computer, which immediately transmits the amounts back to the terminal for
display to enable the operator to
A. Verify that the amount was entered accurately.
B. Establish the validity of the account number.
C. Verify the authorization of the disbursement.
D. Prevent the overpayment of the account.
The display of the amounts entered is called closed-loop verification-an
effective control to verify the accuracy of data input.
159. What document is prepared to authorize the removal of the necessary
quantity of raw materials from storeroom to factory?
A. Production order
B. Materials requisition
C. Movie ticket
D. Purchase invoice
160. Which of the following is an essential control procedure to ensure the
accuracy of the recorded inventory quantities?
A. Calculating unit costs and valuing obsolete or damaged inventory items
in accordance with inventory policy. B. Testing inventory extensions.
C. Performing a gross profit test.
D. Establishing a cutoff for goods received and shipped.
Establishing a cutoff for goods received and shipped would ensure that
only goods owned by the entity are included in inventory.
Answers A, B, and C are incorrect because the procedures described relate
more directly to inventory valuation rather than inventory quantities.
161. The following questions are appropriate for an internal control
questionnaire concerning inventory except
A. Are goods stored in locked storage areas?
B. Is access to the storage area limited to authorized personnel?
C. Are disbursement vouchers approved before payment?
D. Are there independent, periodic comparisons of inventory records with
goods on hand?
Vouchers for all disbursements (not only for inventory purchases) must be
approved before payment. Therefore, a question on voucher approval would
be more appropriately included in the expenditure/disbursement cycle
questionnaire.
162. Effective internal controls over inventories are designed and
implemented for the following reasons, except
A. Inventories typically represent a large component of an entity’s current
assets.
B. Inventories are the most liquid asset.
C. Inventories directly affect the financial performance of an entity.
D. Inventories typically represent a large portion of an entity’s total assets.
Cash, not inventories, is the most liquid asset and has the greatest inherent
risk.
Answers A and D are incorrect because inventories typically represent a
material component of an entity’s current assets and total assets.
Answer C is incorrect because once sold, inventories become cost of goods
sold and is a determinant of an entity’s financial performance for a given
period.
163. ABC Manufacturing Corporation mass produces ten different products.
The company’s controller is interested in strengthening internal control over
the accounting for materials used in production. He/she would most likely
design and implement
A. An economic order quantity (EOQ) system.
B. A perpetual inventory system.
C. A separation of duties among production personnel.
D. A job-order cost accounting system.
The entity can easily keep track of materials usage by maintaining
perpetual inventory records.
Answer A is incorrect because EOQ system will ensure cost- effective
recording of materials but will not strengthen the control over accounting for
material usage.
Answer C is incorrect because segregation of duties among production
personnel does not strengthen control over accounting for materials used in
production unless the authorization, custodial, and record-keeping functions
are likewise separated.
Answer D is incorrect because a process-cost system, not a job-order cost
system, is appropriate for goods that are mass produced.
164. Your client, a merchandising concern, has annual sales of
P150,000,000 and a 40& gross profit rate. Tests revealed that 2% of the
peso amount of purchases do not get into inventory because of breakage and
inventory pilferage by employees. The company estimates that these losses
could be reduced to 0.5 of purchases by designing and implementing certain
controls costing approximately P1,750,000. Should the contols be
designed and implemented?
A. Yes, regardless of cost-benefit considerations, because the situation
involves employee theft.
B. Yes, because the ideal system of internal control is the most expensive
one.
C. No, because the cost of designing and implementing the
added controls exceeds the projected savings.
D. Yes, because the expected benefits to be derived exceed the cost of the
added controls.
A basic concept of internal control is the concept of reasonable assurance,
which recognizes that the cost of internal control should not exceed the
benefits expected to be derived.
The additional controls should not be considered because the cost is
P1,750,000, but the estimated saving is only P1,350,000 {(2%-0.5%)x(P150
million sales x 60% cost of sales ratio)}
165. The objectives of internal control for a production cycle are to provide
assurance that transactions are properly executed and recorded, and that
A. Production orders are pre-numbered and signed by a supervisor.
B. Custody of work-in-process and of finished goods is properly maintained.
C. Independent internal verification of activity reports is established.
D. Transfers to finished goods are documented by a completed production
report and a quality control report.
A primary objective of internal control in the production cycle is to
safeguard inventories from misuse and theft. The inventories should be in the
custody of a storekeeper, and inventory movements should be properly
documented and recorded to establish accountability.
Answers A, C and D are incorrect because the use of pre- numbered
production orders signed by a supervisor, independent internal verification of
activity reports, and documenting inventory transfers are control activities-an
internal control component, not control objectives.
166. Which of the following controls most likely would be implemented to
achieve the production cycle control objective of maintaining accurate
inventory records.
A. Periodic inventory counts are used to adjust the perpetual inventory
records.
B. A just-in-time inventory ordering system keeps inventory levels to a
desired minimum.
C. Perpetual inventory records are periodically compared with the net
realizable value of individual inventory items.
D. Purchase requisitions, receiving reports, purchase orders, and vendor
invoices are independently matched before payment is approved.
A well-designed internal control system should require comparison of the
recorded accountability for assets with existing assets at reasonable intervals.
In the production cycle, periodic inventory counts should be reconciled to
the perpetual inventory records. This control will provide reasonable
assurance about the accuracy of inventory records.
Answer B is incorrect because just-in-time ordering a system provides
assurance that desired inventory levels are maintained but does not ensure that
accurate inventory records are maintained.
Answer C is incorrect because periodic comparison of perpetual inventory
records with net realizable value relates more to inventory valuation rather
than the accuracy of inventory records.
Answer D is incorrect because matching of purchase documents provides
assurance that payments are made only for goods authorized and received but
does not ensure that the accuracy of perpetual inventory records.
167. Which of the following questions would an auditor most likely include
in the production cycle internal control questionnaire?
A. Are details of individual disbursements for raw materials compared to
the total for posting to the general ledger?
B. Are vendor invoices for raw materials approved before payment?
C. Are all issuances of raw materials to production based on approved
requisition forms?
D. Are signed checks for the purchase of raw materials sent directly to
intended payees after signing, without being returned to the person who
authorized the invoice processing?
Answers A, B, and D are incorrect because comparing individual
disbursements for raw materials to totals, approving vendor invoices before
payment, and sending checks directly to payees after signing pertain more
directly to the expenditure/disbursement cycle.
168. Which of the following is the most likely procedure an auditor would
perform in obtaining an understanding of a manufacturing entity’s internal
control for inventory balances?
A. Perform test counts of inventory when observing the entity’s physical
count.
B. Perform analytical procedures designed to identify significant cost
variances.
C. Analyze the liquidity and turnover ratio of the inventory.
D. Review the entity’s description of inventory policies and procedures.
The auditor will review the entity’s description of inventory policies and
procedures to evaluate the design of controls and determine whether they have
been implemented.
169. A properly designed internal control should require that defective
merchandise returned by customers be presented initially to the
A. Receiving clerk
B. Purchasing clerk
C. Billing clerk
D. Inventory control clerk
All incoming goods, including defective merchandise returned by
customers, should be received by the receiving clerk.
INVESTING CYCLE
170. The following controls are appropriate for property, plant, and
equipment (PPE) except
A. Written policies for capitalization and expenditure and review of
application of depreciation methods. B. Disposal of fully depreciated PPE
items.
C. Proper authority for acquisition and retirement of
PPE items.
D. Detailed PPE records and physical controls over PPE items.
Fully depreciated PPE items need not be disposed of because they may still
be useful in the business. These assets should remain on the books until
disposal.
Answers A, C, and D are incorrect because written policies for
capitalization and expenditure and review of depreciation methods, proper
authority for acquisition and retirement, and detailed PPE records and physical
controls are proper controls over PPE.
171. The question that an auditor would least likely include on an internal
control questionnaire concerning the initiation and execution of equipment
transactions is
A. Are requests for purchases of equipment reviewed for consideration of
soliciting competitive bids?
B. Are requests for major repairs approved at a higher level than the
department initiating the request? C. Are procedures in place to monitor
and properly restrict access to equipment?
D. Are pre-numbered purchase orders used for equipment and periodically
accounted for?
The issue is the initiation and execution of equipment transactions.
Restricting access to equipment to authorized personnel only does not address
the issue because it deals more with proper custody of assets.
Answers A, B and D are incorrect because competitive bidding, approval of
major repairs, and use of pre-numbered purchase orders are related to the issue
concerning initiation and execution of equipment transactions.
172. Which of the following controls would most likely detect equipment
acquisitions that are misclassified as maintenance expense?
A. Segregation of duties of employees in the accounts payable department.
B. Authorization by the board of directors of significant equipment
acquisitions.
C. Independent verification of invoices for disbursements recorded as
equipment acquisitions.
D. Investigation of variances within a formal budgeting system.
If equipment acquisitions are misclassified as maintenance expense, an
entity’s formal budgeting system that includes estimates of maintenance
expense will report a significant variance. The misclassification may be
detected by investigating the variance.
Answer A is incorrect because the accounts payable department that
processes payment transactions is unlikely to question the classification of
expenditures that are based on proper documentation.
Answer B is incorrect because verification of invoices from the population
of recorded equipment acquisitions will not disclose items misclassified as
maintenance expense.
Answer C is incorrect because verification of invoices from the population
of recorded equipment acquisitions will not disclose items misclassified as
maintenance expense.
173. Which of the following control activities is most likely to prevent the
improper disposition of equipment?
A. A periodic analysis of the scrap sales and the repairs and maintenance
accounts.
B. Periodic comparison of removal work orders with authorizing
documentation.
C. The use of serial numbers to identify equipment that could be sold.
D. A separation of duties between those authorized to dispose of
equipment and those authorized to approve removal work orders.
There should be proper segregation of duties to reduce the opportunity for
an individual to both perpetrate and conceal errors of fraud.
The functions of authorization, recording, and asset custody should be
separated. Accordingly, the authorization to dispose of equipment by
approving removal work orders (authorization) and the disposal of equipment
(asset custody) should not be assigned to the same person.
Answers A, B and C are incorrect because the control activities described
are detective, not preventive, in nature.
174. Which of the following control activities most likely would justify a
reduced level of control risk concerning property, plant, and equipment
(PPE) acquisitions?
A. Periodic physical inspection of PPE by the internal audit staff.
B. Approval of periodic depreciation entries by a supervisor independent
of the accounting department. C. The review of pre-numbered purchase orders
to detect unrecorded trade-ins.
D. Comparison of current-year PPE account balances with prior-year
figures.
A periodic physical inspection by an objective and competent internal audit
staff is the best way to verify the existence of PPE. This will reduce the
possibility of recording fictitious PPE acquisitions and other fraudulent
acts. The implementation of this control activity would justify a lower
assessed level of control risk.
Answer B is incorrect because depreciation is computed based on recorded
amounts of PPE. Depreciation will be misstated if the basis used in calculating
it is also misstated.
Answer C is incorrect because reviewing purchase orders to detect
unrecorded trade-ins is not as effective as directly inspecting the assets.
Answer D is incorrect because comparing recorded PPE account
balances may not be effective for detecting non- existent PPE.
175. An internal control objective concerning property, plant, and equipment
(PPE) acquisitions is that they be recorded at the correct amounts and in
the proper period, and properly classified. In which of the following
conditions would an auditor most likely assess a high level of risk of
material misstatement?
A. All material acquisitions of PPE are required to be approved by the
board of directors.
B. Most additions are self-constructed by the entity. C. Recently acquired
loans include covenants that preclude further plant acquisitions for 5 years.
D. Gross PPE increased 30% during the current period.
If an entity has on-going in-house construction projects, labor and overhead
costs should be allocated between its inventories and in-house construction
projects. Moreover, capitalization of borrowing costs would involve complex
calculations. Therefore, the inherent risk of misstatement for self-constructed
assets is high.
Answer A is incorrect because the required approval of the entity’s board of
directors for all material PPE additions relates to authorization, not recording.
Answer C is incorrect because the loan covenant that precludes further
plant additions for 5 years is most likely to decrease risk.
Answer D is incorrect because the significant increase in PPE does not
necessarily increase the risk of misstatement. For example, the significant
increase may be due to material acquisitions of PPE from outside vendors. In
this case, the cost of PPE can easily be determined by just referring to
amounts on vendor invoices.
176. Why is property, plant, and equipment (PPE) typically considered to be
one of the accounts least susceptible to fraud?
A. Internal control on this account is inherently effective.
B. The depreciated values are always smaller than cost.
C. The inherent risk of PPE is usually low.
D. For most companies, the recorded amounts of PPE are immaterial.
PPE is one of the accounts that is least susceptible to misstatement in the
absence of related controls-that is,
its inherent risk is low. This is because of the infrequency of
transactions in the account and the relative ease in verifying its existence.
Answer A is incorrect because inherent risk, not control risk, pertains to the
susceptibility of an account to misstatement.
Answer B is incorrect because not all PPE items are depreciable, for
example, land.
Answer D is incorrect because although transactions affecting PPE are
infrequent, the amounts involved are usually large.
177. Which of the following misstatements or questionable practices may be
uncovered if an auditor tours an entity’s production facility?
A. Insurance coverage on the facility has lapsed.
B. Overhead has been over applied.
C. Depreciation expense on fully depreciated machinery has been
recognized.
D. Necessary facility maintenance has not been performed.
The auditor’s tour of an entity’s production facility would involve direct
observation of the asset condition. Hence, it is likely that the auditor will
discover that necessary facility maintenance has not been performed during the
year.
Answer A is incorrect because inspection of insurance contracts, not a tour
of the plant facility, will detect lapsed insurance coverage.
Answer B is incorrect because comparison of the actual overhead incurred
with applied overhead will determine if overhead has been over applied.
Answer C is incorrect because the auditor should examine depreciation
records to determine if depreciation has been provided on fully depreciated
machinery.
178. Which of the following controls would an entity most likely use in
safeguarding against the loss of trading securities?
A. The independent auditor traces all purchases and sales of trading
securities through the subsidiary ledgers to general ledger.
B. An independent trust company that has no direct contact with the
employees who have record-keeping responsibilities has possession of the
securities.
C. The internal auditor inspects the trading securities
in the entity’s safe each year on the balance sheet date.
D. A designated member of each board of directors controls the securities
in a bank safe-deposit box.
Engaging an independent trust company for the custody of an entity’s
trading securities may be considered the best way to safeguard such securities
against loss. Such an institution normally has very strict controls over assets
under it’s custody, including access to its vaults.
Answer A is incorrect because an independent auditor’s tracing of trading
securities transactions is an audit test, not a control.
Answer C is incorrect because the internal auditor’s inspection of trading
securities in the entity’s safe each year on the balance sheet date does not
assure that no securities have been removed between inspections dates.
Answer D is incorrect because a better control is to require the presence of
two authorized persons to access a safe deposit box.
179. Which of the following controls would a company most likely use to
safeguard marketable securities when such securities are not in the custody of
an independent trust agent?
A. The chairman of the board of directors verifies the marketable
securities, which are kept in a bank safe- deposit box, each year on the balance
sheet date.
B. The internal auditor and the controller independently trace all
purchases and sales of marketable securities from the subsidiary ledgers to the
general ledger.
C. Two company officials have joint control of marketable securities,
which are kept in a bank safe-deposit box. D. The investment committee of the
board of directors periodically reviews the instrument decisions
delegated to the treasurer.
A physical control to safeguard an entity’s assets such as marketable
securities is to keep them in a bank safe- deposit box requiring two
signatures to gain access.
Answer A is incorrect because verification of marketable securities once
a year is unlikely to provide adequate control.
Answer B is incorrect because tracing of marketable securities
transactions to the accounting records assures proper recording but does not
physically safeguard the asset.
Answer D is incorrect because periodic review of the investment
decisions delegated to the treasurer does not physically safeguard the asset.
180. Which of the following internal control activities would an entity most
likely implement to assist in satisfying the completeness assertion related to
noncurrent investments?
A. The internal auditor compares the securities in the bank safe-deposit
box with recorded investments.
B. Senior management verifies that securities in the bank safe-deposit
box are registered in the entity’s name.
C. The controller compares the current market prices of recorded
investments with the brokers’ advices on file.
D. The treasurer vouches the acquisition of securities by comparing
brokers’ advices with canceled checks.
Comparison of securities in bank safe-deposit box with the accounting
records assures that the investment balance is complete, that is, all investment
securities have been reflected in the account.
Answer B is incorrect because senior management’s verification that
securities are registered in the entity’s name relates to the right assertion.
Answer C is incorrect because comparing market prices with brokers’
advices pertains to the valuation assertion.
Answer D is incorrect because vouching securities purchased by comparing
brokers’ advices with canceled checks relates to the rights assertion.
181. Which of the following is not a proper control over investment
securities?
A. Separation of custodial and treasury functions.
B. Employing an independent trust agent.
C. Proper authorization of transactions.
D. Storage in a safe-deposit box.
An entity’s treasury function should include custody of cash and
securities.
Answers B, C, and D are incorrect because employing an independent
trust agent, proper authorization of transactions, and storage of investment
securities in a safe-deposit box are proper controls over investment
securities.
182. The following controls are designed to protect investment securities, except
A. Investment securities should be properly controlled physically in order
to prevent unauthorized usage.
B. Custody over investment securities should be limited to personnel having
record-keeping responsibility over the securities.
C. Securities should be registered in the entity’s name. D. Access to securities
should be vested in two individuals.
As with other assets, the custody of investment securities should be
delegated to individuals who do not have recording responsibility over
securities.

FINANCING CYCLE
183. Which of the following questions is most likely to be included by an
auditor on an internal control questionnaire for notes payable?
A. Are direct borrowings on notes payable authorized by the board of
directors?
B. Are assets that collateralize notes payable critically needed for the
entity’s continued existence?
C. Are two or more authorized signatures required on checks that repay
notes payable?
D. Are the proceeds from notes payable used for the purchase of noncurrent
assets?
Most companies are ordinarily require that direct borrowings on notes
payable be authorized by the board of directors. Accordingly, an auditor should
verify if such control has been properly implemented.
184. The audit program for long-term debt should include steps that require
the
A. Verification of the existence of the bondholders.
B. Examination of any bond trust indenture.
C. Inspection of the accounts payable master file.
D. Investigation of credits to the bond interest income account.
185. During the year under audit, a company has completed a private
placement of a substantial amount of bonds. Which of the following is
the most important step in the auditor’s program for the audit of bonds
payable?
A. Confirming the amount issued with the bond trustee. B. Tracing the cash
received from the issue to the accounting records.
C.Examining the bond records maintained by the transfer agent.
D. Recomputing the annual interest cost and the effective yield.
186. Several years ago, ABC, Inc., secured a conventional real estate
mortgage loan. Which of the following audit procedures would be least
likely to be performed by an auditor auditing the mortgage balance?
A. Examine the current year’s cancelled checks.
B. Review the mortgage amortization schedule.
C. Inspect public records of lien balances.
D. Recompute mortgage interest expense.
187. During an audit of publicly held company, the auditor should obtain
written confirmation regarding debenture transactions from the
A. Debenture holders
B. Client’s authority
C. Internal auditors
D. Trustee
188. An audit program for the audit of retained earnings account should
include a step that requires verification of
A. Market value used to charge retained earnings to account for a 2-for-1
stock split.
B. Approval of the adjustment to the beginning balance as a result of a
write-down of an account receivable. C. Authorization for both cash and
stock dividends.
D. Gain or loss resulting from disposition of treasury shares.
189. Where no independent stock transfer agents are employed and the
corporation issues its own stocks and maintains stock records, cancelled
stock certificates should
A. Be defaced to prevent reissuance and attached to their corresponding
stubs.
B. Not be defaced, but segregated from other stock certificates and retained
in a cancelled certificates file.
C. Be destroyed to prevent fraudulent reissuance.
D. Be defaced and sent to the Secretary of the Department of Finance.

HUMAN RESOURCES CYCLE


190. A potential threat to the payroll processing activity is theft or fraudulent
distribution of payroll checks. One control that can be implemented to
help prevent paychecks being issued to “ghost” employees is
A. Use of payroll clearing account.
B. Paychecks should be physically distributed by someone who does not
authorize or record payroll.
C. Periodic reconciliation of the payroll bank account.
D. The cashier should sign all payroll checks.
191. Effective controls over the payroll function may include
A. Custody of rate authorization records by the supervisor of the personnel
department.
B. Preparation of payroll transaction journal entries by an employee who
reports to the supervisor of the personnel department.
C. Verification of agreement of job time tickets with employee clock card
hours by a payroll department employee.
D. Reconciliation of totals on job time tickets with job reports by
employees responsible for those specific jobs.
Comparison of the job time tickets, which show the total time spent on jobs,
and time cards will provide an independent check of the accuracy of time
indicated on time cards.
Answer A is incorrect because the pay rates used in the calculation of
payroll should be authorized by the personnel department.
Answer B is incorrect because the payroll department, not the personnel
department, prepares the payroll transaction journal entries. This is to segregate
the authorization and record-keeping functions.
Answer D is incorrect because an independent party, not the employees
involved, should reconcile totals on job time tickets with job reports.
192. Employees of a manufacturing entity are often required to use time cards
and job time tickets. Which of the following statements concerning the use of
these documents is incorrect?
A. Time reported on job time tickets should be reconciled to time cards.
B. Payroll should be calculated based on job time tickets.
C. Each employee should have only one time card.
D. An employees may have one or many job time tickets in a day.
Time cards are the official records of time worked by employees and
should be the basis for payroll preparation. 193. Organizational independence in
the processing of payroll can be achieved by segregating the functions of
authorization, record-keeping, and custody of assets. Which one of the
following functional separations is not required for internal control purposes?
A. Separation of payroll preparation and paycheck distribution.
B. Separation of personnel function from payroll preparation.
C. Separation of timekeeping from payroll preparation.
D. Separation of payroll preparation and maintenance of year-to-date
records.
194. Which of the following controls is most effective in providing reasonable
assurance that salary, wage, and benefit expenses are incurred only for worked
performed?
A. The payroll register is used as the source document for posting
employees benefit costs to the general ledger.
B. All time cards and reports are reviewed and approved in writing by
immediate line supervisors who do not have responsibilities for paycheck
distribution.
C. Actual payroll amounts are regularly compared against budgeted
amounts by management, with all material budget variances being
investigated.
D. The accuracy of extensions of hours worked and pay rates is rechecked
by an independent party, and pay rate and other key payroll information is
changed only upon the receipt of a written authorization from the
personnel department.
An appropriate control is the review and approval of time cards and reports
by line supervisors because they have direct knowledge about whether work
has been performed. And because they have no responsibilities for paycheck
distribution, they are not in a position to misappropriate paychecks.
Answer A is incorrect because using the payroll register as the source
document for posting to the general ledger controls recording of employee
benefit costs, not the propriety of the reported time worked.
Answer C is incorrect because investigating material variances between
actual payroll and budgeted amounts may probably not reveal specific
improprieties. It is, therefore, less effective than the review of time cards
and reports by line supervisors/
Answer D is incorrect because although an arithmetic check of payroll
calculations and personnel authorization of pay rate changes are effective
controls over payroll processing, they do not assure that employee benefit
costs are incurred only for work performed.
195. The purpose of segregating the duties of hiring personnel and
distributing payroll checks is to separate the
A. Authorization of transactions from the custody of related assets.
B. Operational responsibility from the record-keeping responsibility.
C. Human resource function from the controllership function.
D. Administrative controls from internal accounting controls.
A well-designed internal control provides for proper segregation of
authorization, recording, and asset custody functions.
An entity’s personnel department provides authorization for hiring, pay
rates, and deductions; the production department provides authorization for
hours worked. In turn, the payroll department calculates and records the
payroll based on these authorizations. Based on these calculations, the
treasurer signs and distributes paychecks to employees-an asset custody
function.
196. Which of the following departments most likely approved changes in
pay rates and deductions from employee salaries?
A. Payroll
B. Personnel
C. Controller
D. Treasurer
An entity’s personnel department provides authorization for hiring,
termination, deductions, and changes in pay rates.
197. Which of the following situations represents an internal control
weakness in the payroll department?
A. The timekeeping function is independent of the payroll department.
B. Payroll records are periodically reconciled with tax reports.
C. Paychecks are distributed by the employees’ immediate supervisor.
D. Payroll department personnel are rotated in their duties.
A sound internal control dictates that there should be proper segregation of
functional responsibilities to minimize the risk that errors or fraud will be
committed and concealed by the same person.
The functions of authorization, recording, and asset custody should be
separated. If the employees’ immediate supervisor also distributes paychecks,
paychecks of fictitious employees can be diverted if that supervisor also
has access to personnel records.
198. Which of the following personnel department procedures reduces the
risk of payroll fraud and represents an appropriate responsibility for the
department>
A. Authorizing the addition or deletion of employees from the payroll.
B. Authorizing overtime hours.
C. Collection and retention of unclaimed paychecks.
D. Distributing paychecks.
The personnel department has the responsibility of authorizing employee
transactions such as hiring, terminating, and changes in pay rates and
deductions.
199. Each Saturday afternoon, paychecks are distributed by the production
department’s shift supervisor. The company’s production department is so
large and the turnover of factory workers is so great that the supervisor does
not know many of the workers. Unclaimed paychecks are return claim
them at some later time. The payroll clerk routinely continues the payroll
record for workers one week after their departure from the company and
ultimately diverts the unclaimed paychecks. Which of the following controls
would most likely prevent this misappropriation?
A. Require the treasurer’s office to prepare checks only on the basis of
supporting documentation from both the timekeeper and payroll
accounting.
B. Periodically rotate the shift supervisor.
C. Require the timekeeper to compute weekly pay of each factory worker
and to make distribution of the checks received from the treasurer’s
office.
D. Require the shift supervisor to know all the workers by name.
The payroll accounting should calculate and record the payroll based on
authorized pay rates and deductions and each employee’s timecard.
Paychecks should be prepared based on this documentation. The payroll
department, having a record-keeping responsibility, should not have custody
of unclaimed paychecks.
Answer B is incorrect because the identity of the shift supervisor is not
an element of the payroll clerk’s fraud
Answer C is incorrect because the calculation of payroll distribution of
paychecks are incompatible functions and should not be performed by the
timekeeper.
Answer D is incorrect because it would be very difficult to implement
the control of knowing all factory workers by name.
200. Proper internal control over the cash payroll function mandates which of
the following?
A. A separate checking account for payroll should be maintained.
B. Each employee should be asked to sign a receipt.
C. The payroll clerk should fill the envelopes with cash and a computation
of the net pay.
D. Unclaimed pay envelopes should be retained by the paymaster.
In a cash payroll system, employees must be required to sign a receipt
that will serve as documentation for the payment.
Answer A is incorrect because a separate checking account need not be
maintained.
Answer C is incorrect because the payroll clerk who

has a recording responsibility should not be assigned to fill the


envelopes with cash.
Answer D is incorrect because unclaimed cash payroll should be
deposited in the bank for proper custody.
TRUE OR FALSE
1. As the risk of material misstatement increases, detection risk should
increase.
2. If planned detection risk is reduced, the amount of evidence the auditor
accumulates will decrease.
3. Inherent risk and control risk are inversely related to each other.
4. The risk of material misstatement refers to the combination of inherent
risk and control risk.
5. In a financial statement audit, inherent risk is evaluated to help an auditor
assess the susceptibility of a financial statement assertion to a material
misstatement assuming there are no related controls.
6. Inherent risk is inversely related to the amount of audit evidence
whereas detection risk is directly related to the amount of audit evidence
required.
7. Inherent risk is directly related to the amount of evidence whereas
detection risk is inversely related to the amount audit evidence required.
8. Inherent risk and control risk are assessed by the audit and function
independently of the financial statement audit.
9. As the acceptable level of detection risk increases, an auditor may
change the timing of the tests on controls by performing them throughout
the year rather than at one time.
10. As the acceptable level of detection rick increases, an auditor may
change the timing of substantive tests by performing them at an interim
date rather than year end.
11. The risk that an auditor will conclude, based on substantive tests, that a
material misstatement does not exist in account balance, when, in fact,
such misstatement does exist is referred to as detection risk.
12. When an auditor increases the assessed level of risk of material
misstatement because certain control procedures were determined to be
ineffective, the auditor would most likely increase the level of detection
risk.
13. The risk of material includes non-sampling risk.
14. As the acceptable level of detection risk decreases, an auditor may
change the nature of substantive procedures from less effective to more
effective procedures.
15. As the acceptable level of detection risk decreases, the assurance directly
provided from substantive procedures should increase.
16. Materiality judgements are made in light of surrounding circumstances
and necessarily involve both quantitative and qualitative judgements.
17. The documentation of an auditor’s understanding of internal controls
must include flowcharts.
18. To obtain evidential matter about control risk, an auditor selects tests
from a variety techniques including confirmation.
19. Reports on service organizations typically provide reasonable assurance
that their financial statements are free of material misstatements.
20. Where computer is processing is used in significant accounting
applications, internal control activities may be defined by classifying
control activities into two types: general and application.
21. In evaluating internal control, the auditor is basically concerned that the
system provides reasonable assurance that operational efficiency has
been achieved in accordance with management plans.
22. The maintenance of the system of internal control is an important
responsibility of the internal auditor.
23. Because of the cost/benefit relationship, tests of controls may be applied
on a test basis in some circumstances.
24. The concept of reasonable assurance in the context of an entity’s internal
controls recognizes that auditors may fail to detect material
misstatements.
25. An effective control environment guarantees that all controls are
followed as prescribed.
26. The risk assessment component of internal controls refers to the
auditor’s assessment of control risk.
27. Assessing control risk at below the maximum would involve identifying
specific internal controls relevant to specific assertions.
28. In the audit of financial statements, an auditor’s primary consideration
regarding an internal control policy or procedure is whether the policy or
procedure affects management’s financial statement assertions.
29. As part of gaining an initial understanding of internal control, an auditor
is required to obtain knowledge about the operating effectiveness of the
internal control.
30. Assessing control risk below maximum involves concluding that
controls are ineffective.
31. After the auditor has prepared a flowchart of the internal controls
surrounding sales and evaluated the design of the system, the auditor
would perform tests of controls on all control activities documented in
the flowchart.
32. Observation is a procedure that would most likely be used by an auditor in
performing tests of control activities
that involve segregation of functions and that leave no transaction trail.
33. Tracing bills of lading to sales invoices provides evidence that billed
sales were shipped.
34. Tracing copies of sales invoices to shipping documents will provide
evidence that all shipments to customers were
billed.
35. The accounts payable department receives the purchase order form to
ensure that the goods had been received by the party requesting the
goods.
36. The authority to accept incoming goods in receiving should be based on
approved purchase order.
37. For effective internal control purposes, the vouchers payable department
generally should stamp, perforate, or otherwise cancel supporting
documentation after payment is mailed.
38. Vendors’ statements and vendors’ invoices are both relatively reliable
evidence because they originate from a third party.
39. Credit memos are normally issued to adjust the customers balance to the
amount owed to the company.
40. The document that the accounting staff will use as the primary basis for
recording sales transactions and updating the customer’s accounts
receivable subsidiary ledger is the sales invoice.

KEY ANSWERS

1. C 21. B 41. D 61. D 81. C

2. D 22. B 42. B 62. C 82. B


3. A 23. B 43. D 63. C 83. C

4. C 24. D 44. C 64. D 84. B


5. C 25. B 45. C 65. D 85. A

6. B 26. D 46. B 66. D 86. A

7. D 27. A 47. A 67. B 87. C

8. C 28. D 48. A 68. B 88. D

9. B 29. A 49. A 69. C 89. A

10. C 30. A 50. B 70. D 90. C

11. B 31. C 51. A 71. A 91. D

12. C 32. C 52. B 72. A 92. C

13. A 33. C 53. D 73. B 93. B

14. D 34. A 54. C 74. A 94. C

15. B 35. C 55. D 75. D 95. D

16. A 36. C 56. A 76. C 96. D

17. B 37. C 57. C 77. A 97. A

18. B 38. B 58. A 78. C 98. D

19. A 39. B 59. B 79. A 99. A

20. B 40. A 60. C 80. B 100. A

101. B 121. A 141. B 161. C 181. A

102. C 122. B 142. B 162. B 182. B

103. A 123. C 143. B 163. B 183. A


104. B 124. B 144. A 164. C 184. B

105. C 125. D 145. B 165. B 185. A

106. C 126. D 146. A 166. A 186. C

107. A 127. D 147. D 167. C 187. D


108. B 128. A 148. C 168. D 188. C

109. C 129. B 149. C 169. A 189. A

110. B 130. C 150. D 170. B 190. B

111. C 131. C 151. A 171. C 191. C 112. A 132. C 152. C


172. D 192. B

113. C 133. C 153. B 173. D 193. D

114. B 134. A 154. A 174. A 194. B

115. B 135. B 155. C 175. B 195. A

116. B 136. C 156. B 176. C 196. B

117. A 137. C 157. C 177. D 197. C

118. C 138. B 158. A 178. B 198. A

119. C 139. A 159. B 179. C 199. A

120. B 140. B 160. D 180. A 200. B

TRUE OR
FALSE
1. True 9. False 17. False 25. False 33. False

2. False 10. True 18. False 26. False 34. False

3. False 11. True 19. False 27. True 35. False

4. True 12. False 20. True 28. True 36. True


5. True 13. False 21. False 29. False 37. False

6. False 14. True 22. False 30. False 38. True

7. True 15. True 23. True 31. False 39. True

8. True 16. True 24. False 32. True 40. True CHAPTER 6
AUDITING IN A COMPUTER INFORMATION SYSTEMS (CIS) OR
INFORMATION
TECHNOLOGY (IT) ENVIRONMENT

1. IT has several significant effects on an entity. Which of the following would be


important from an auditing perspective?

I. The potential for material misstatement. II. The visibility of


information.
III. Changes in the organizational structure.

A. I and II only C. II and III only


B. I and III only D. I, II, and III

2. The use of a computer changes the processing, storage, and communication of


financial information. A CIS environment may affect the following, except

A. The accounting and internal control systems of the entity.


B. The overall objective and scope of an audit.
C. The auditor’s design and performance of tests of control and substantive
procedures to satisfy the audit objectives.
D. The specific procedures to obtain knowledge of the entity’s accounting and
internal control systems.

A CIS environment does not affect the overall objective and scope of an audit.

3. The following are benefits of using IT-based controls, except

A. Ability to process large volume of transactions.


B. Over-reliance on computer-generated reports.
C. Ability to replace manual controls with computer-based controls.
D. Reduction in misstatements due to consistent processing of transactions.

4. Which of the following statements concerning the Internet is incorrect?

A. The Internet is a shared public network that enables communication with other
entities and individuals around the world.
B.The Internet is a private network that only allows access to authorized persons or
entities.
C.The Internet is interoperable, which means that any computer connected to the
Internet can communication.
D. The Internet is a worldwide network that allows entities to engage in e-
commerce/e-business activities.
5. In planning the portions of the audit which may be affected by the client’s CIS
environment, the auditor should obtain an understanding of the significance and
complexity of the CIS activities and the availability of data for use in the audit.
The following relate to the complexity of CIS activities except when

A. Transactions are exchanged electronically with other organizations (for example,


in electronic data interchange systems [EDI])
B.Complicated computations of financial information are performed by the computer
and/or material transactions or entries are generated automatically without
independent validation.
C.Material financial statement assertions are affected by the computer processing.
D. The volume of transactions is such that users would find it difficult to identify and
correct errors in processing.

The materiality of the financial statement assertions affected by the CIS relates to
the significance, not the complexity of computer processing.

6. The auditor shall consider the entity’s CIS environment in designing audit
procedures to reduce risk to an acceptably low level. Which of the following
statements is incorrect?

A. The auditor’s specific audit objectives do not change whether financial


information is processed manually by computer.
B.The methods of applying audit procedures to gather audit evidence are not
influenced by the methods of computer processing.
C.The auditor may use either manual audit procedures, computerassisted audit
techniques (CAATs), or a combination of both to obtain sufficient appropriate
audit evidence.
D. In some CIS environments, it may be difficult or impossible for the auditor to
obtain certain data for inspection, inquiry, or confirmation without the aid of a
computer.

The methods of applying audit procedures to gather audit evidence may be


influenced by the methods of computer processing.

7. Regardless of the nature of an entity’s information system, the auditor must


consider internal control. In a CIS environment, the auditor must, at a minimum,
have
A. A background in programming procedures.
B. An expertise in computer systems analysis.
C. A sufficient knowledge of the computer’s operating system.
D. A sufficient knowledge of the computer information system.
The auditor should have a sufficient knowledge of the CIS to plan, direct,
supervise, and review the work performed.

Answers A and B are incorrect because an auditor need not have expertise in
programming and computer systems analysis. If specialized CIS skills are needed
in the audit, the auditor may seek the assistance of an auditor’s expert.

Answer C is incorrect because the auditor should have sufficient knowledge of the
entire CIS not only of the computer’s operating systems.

8. Who is ultimately responsible for the design and implementation of cost-


effective controls in a CIS environment?

A. The internal audit manager


B. The entity’s management
C. The CIS manager
D. The control group in the CIS environment

An entity’s management is ultimately responsible or designing and implementing


systems that will provide reasonable assurance that the entity’s objectives will be
achieved.

9. Are the following risks greater in CIS than in manual systems?

A B C D
Erroneous data conversion Yes Yes Yes Yes
Erroneous source document
preparation Yes Yes Yes No
Repetition of errors No No Yes Yes
Concentration of data Yes No Yes Yes

The preparation of source documents wither precedes or is not done at all in a


computer information systems. Thus, the risk of erroneous source document
preparation in a CIS environment may be equal to or less than the equivalent risk
in a manual system.

In a CIS environment, the computer converts data to machinereadable form prior


to processing of transactions. This will increase that risk of input error. In
addition, the computer’s ability to uniformly process like transactions with the
same processing instructions will ordinarily result in all transactions being
processed incorrectly if there are programming errors (or other systematic errors
in hardware aor software). Also, the concentration of data stored on magnetic disk
increases the risk of loss of valuable financial information from damage or theft.
10. Which of the following is not a hardware element in an IT environment?
A. Scanners
B. CD-ROM drive
C. Application programs
D. Modems

An IT environment consists of hardware and software components. Computer


hardware consists of the computer and all other physical equipment. The software
component consists of computer programs that are either purchased from a
software vendor or developed in-house by the entity.

Application software – a type of computer software – performs desired processing


tasks such as payroll processing.

Answers A, B, and D are incorrect because optical scanners, CDROM drive, and
modems are elements of computer hardware.

11. Which of the following computer hardware elements is not associated with
data input?

A. Touch screen
B. Printer
C. Mouse
D. Optical scanner

A printer is an output device that procedures a hard copy of computer processing


results.

Answers A, C, and D are incorrect because a touch screen, a mouse, and an optical
scanner can be used for data input.

12. A hardware element that takes the computer’s digital information and
transforms it into signals that can be sent, over ordinary telephone lines is a/an

A. Intelligent terminal
B. Point-of-sale terminal
C. Terminal emulator
D. Modem

A modem converts data in digital form into analog or wave form (the process is
called modulation) so that data can be sent to remote locations through the
telephone system.
The modem at the receiving end of the transmission path converts the analog or
wave form back to the digital form (the process is called demodulation) used by
the terminal or CPU.
13. Uninterruptible power supplies are used in computer
facilities to minimize the risk of

A. Crashing disk drive read-write heads


B. Dropping bits in data transmission
C. Failing to control concurrent access to data
D. Losing data stored in main memory

An uninterruptible power source such a s a generator or battery backup used in a


computer facility will reduce the likelihood of losing data stored in the computer’s
main memory in the event of an electrical failure such as a power outage or
voltage fluctuation.

14. In a computer system, the parts of the operating system program and language
translator program are stored in the

A. Read only memory (ROM).


B. Random access memory (RAM).
C. Magnetic tape drive.
D. Magnetic disk drive.

ROM consists of semiconductor chips that can be read from (but not written to)
and are used as permanent storage of the operating system and language
translator.

Answers B, C, and D are incorrect because RAM and magnetic tape and disk
drives are temporary storage devices.

15. A characteristic that distinguishes computer processing from manual


processing is

A. The potential for systematic error is ordinarily greater in manual processing than
in computerized processing.
B.Errors or fraud in computer processing will be detected soon after their
occurrences.
C.Most computer systems are designed so that transaction trails useful for audit
purposes do not exist.
D. Computer processing virtually eliminates the occurrence of computational errors
normally associated with manual processing.
Computational or clerical errors are virtually eliminated in computer processing
because of the computer’s capability to uniformly process like transactions with
the same processing instructions.
Answer A is incorrect because the risk of systematic or programming error is
greater in computer processing than is manual processing.
The computer’s ability to subject like transactions to uniform processing will
result in all transactions being processed incorrectly if there are errors embedded
in the program logic.

Answer B is incorrect because errors or fraud in computer processing may remain


undetected for long periods of time, or worse, may never be detected at all.

The potential for observing errors or fraud is reduced in computer processing


because of decreased human involvement in handling transactions processed by
CIS.

Answer c is incorrect because CIS are designed to include transaction trails.


However, some transaction trails in computer processing may exist for only a
short period of time or only in computer-readable form.

16. An affordable yet powerful self-contained general purpose computer which


consists typically of a central processing unit (CPU), monitor, keyboard, disk
drives, printer cables, and modems is a /an

A. Personal computer
B. Mainframe
C. On-line computer
D. Terminal

17. A CIS where two or more personal computers are linked together through the
use of special software and communication lines and allows the sharing of
application software, data files, and computer peripherals, such as printers and
optical scanners is a/an

A. Local area network (LAN)


B. On-line system
C. Batch processing system
D. Wide area network (WAN)

Each personal computer linked to a LAN is called a workstation that can access
data, software, and other resources through a file server – a linked PC that
manages the network.

A LaN is usually confined to a small geographic location such as a building or two


or more adjacent buildings.

Two or more LANs can be linked together to form a wide area network (WAN).
18. A file server in a local area network (LAN) is
A. A workstation that is dedicated to a single user on the LAN. B. A computer that
stores programs and data files for users of the LAN.
C. The cabling that physically interconnects the nodes of the LAN.
D. A device that connects the LAN to other networks.

Common resources such as programs and data shared by LAN nodes are stored
and managed by special-purposed computers called file servers.

Answer A is incorrect because a workstation or node in a LAN is called a client.

Answer C is incorrect because the cabling that physically interconnects the nodes
of the LAN is communications link.

Answer D is incorrect because bridges and gateways are used to link networks
together. Bridges connect LANs of the same type while gateways connect LANs
of different types.

19. Audit team members can use the same database and programs when their PCs
share a hard disk and printer in a LAN. Which of the following communication
devices enables a PC to connect to a LAN?

A. A network interface card (NIC) that plugs into the motherboard.


B. a fax modem that send signals through telephone lines.
C. An internal modem that plugs into the motherboard.
D. An external modem with a cable connection to a serial port.

A workstation’s physical connection to the LAN is achieved through a network


interface card (NIC) which plugs into one of the expansion slots on the PC.

Answers B, C, and D are incorrect because modems connect PCs to ordinary


telephone lines.

20. A computer information system that allows individual users to develop and
execute application programs, enter and process data, and generate reports in a
decentralized manner is called
a/an

A. Online system
B. Batch processing system
C. End-user computing
D. Networking

In end-user computing, management empowers individual users to develop and


execute application programs, enter and process data, and generate computer
processing results. This system is an example of decentralized processing and
usually involves the use of PCs.

21. Which the following statements most likely represents a disadvantage for an
entity that maintains data files on personal computers (PCs) rather than manually
prepared files?

A. It is usually more difficult to compare recorded accountability with the physical


count of assets.
B.Random error associated with processing similar transactions in different ways is
usually greater.
C.Attention is focused on the accuracy of the programming process rather than errors
in individual transactions.
D. It is usually easier for unauthorized persons to access and alter the files.

In a PC environment, unauthorized individuals can easily gain access to and


change data files without visible evidence.

Answer A is incorrect because an advantage of CIS is the computer’s ability to


process like transactions in the same way.
Answer C is incorrect because focusing on the accuracy of the programming
process is an advantage of CIS.

22. The following are risks specific to IT environments, except

A. Reduced segregation of duties.


B. Loss of data due to insufficient backup.
C. Increased human involvement.
D. Reliance on the functioning capabilities of hardware and software.

23. Most personal computers have both a CD-ROM drive and a hard disk drive.
The major difference between the two types of storage is that a hard disk.

A. Is suitable for an online system, whereas a CD-ROM is not.


B. Provides an automatic audit trail, whereas a CD-ROM does not.
C. Has a much larger storage capacity than a CD-ROM.
D. Is a direct-access storage medium, whereas a CD-ROM is a sequential-access
storage medium.

24. What type of online computer system is characterized by data that are
assembled from more than one location and records that are updated immediately?
A. Online, batch processing system
B. Online, real-time processing system
C. Online, inquiry system
D. Online, downloading/uploading system
In an online processing system, individual transactions are entered through
workstations or terminals that are connected to the mainframe.

A type of online system is online, real-processing system that involves immediate


validation and processing of data input to update related computer files that allows
users to receive the output soon enough to affect a current decision to be made.

Answer A is incorrect because in an online, batch processing system, individual


transactions are entered through remote terminals, subjected to certain validation
routines and added to a transaction file containing other transactions entered
during the period.

The transaction file is to be subjected to further validation checks and then used in
updating the relevant master file in the subsequent processing cycle.

Answer is incorrect because in an online, inquiry system, users are restricted to


making inquiries of master files (for example, inquiry of a customer account
balance).

Answer D is incorrect because online, uploading/downloading system involves


the transfer of data between the mainframe and workstations.

25. Misstatements in a batch computer system caused by incorrect programs or


data may not be detected immediately because

A. The processing of transactions in a batch system is not uniform.


B.There are time delays in processing transactions in a batch system.
C.The identification of errors in input data typically is not part of the program.
D. Errors in some transactions may cause rejection of other transactions in the batch.

In a batch processing system, similar transactions are processed in groups or


batches periodically – for example, daily, weekly, or even monthly. Hence, errors
in a given batch may be detected only after the lapse of consideration time from
the initiation of the transactions.
Answer A is incorrect because like transactions are processed uniformly in a batch
system.

Answer C is incorrect because data validation routines may be embedded in the


computer program.

Answer D is incorrect because although similar transactions are processed


together in batches, individual transactions are not dependent upon one another.

26. Which of the following features is least likely to be found in an online, real-
time processing system?
A. Turnaround documents B.
User manuals
C. Preformatted screens
D. Automatic error correction

A turnaround document is a source document generated by the computer system


as output and then later used as input for subsequent processing. Turnaround
documents are least likely to be found in an online, real-time processing system
because it normally does not use source documents.

Answer B is incorrect because user manuals provide explanations on the proper


use of the system, making them an important component of the real-time system.

Answer C is incorrect because users usually interact with the mainframe through
preformatted screens of remotes terminals.

Answer D is incorrect because automatic error correction is a principal advantage


of real-time systems – that is, errors are immediately detected and corrected.

27. Which of the following is usually not a factor to consider in designing and
implementing an online, real-time system?

A. Priority allocation
B. Queues
C. Interrupts
D. Hardware diagnostics

Computers are designed to include hardware diagnostic routines that allow


identification of hardware problems such as a party check to determine if the
integrity of the bot structure of each character has been destroyed during the
internal transaction of data within the system.
Hardware diagnostic routines are applicable to all systems, not only to online,
real-time systems.

Answers A and B are incorrect because priority allocation and queues are
important factors in real-time systems. Both of them relate to deciding which jobs
should be given priority in processing.

Answer C is incorrect because interrupts allow high priority jobs to get immediate
action. In a multiprogramming environment, work on one program is interrupted
so the CPU may attend to another.

28. Workstations or terminals are an integral component of online computer


systems. Which of the following statements concerning workstations is incorrect?
A. Workstations may be located either locally or at remote sites. B. Both local and
remote workstations require the use of telecommunications to link them to the
main computer.
C.Local workstations are connected directly to the main computer through cables.
D. Workstations may be used by different users, for different purposes, in different
locations, all at the same time.

Only remote workstations require the use of telecommunications to link them to


the main computer. Local workstations are linked through cables.

29. Online computer systems use workstations or terminals that are located either
locally or at remote sites. There are two types of workstations: general purpose
terminals and special purpose terminals. General purpose terminals include the
following, except

A. Basic keyboard and monitor


B. Point of sale devices
C. intelligent terminal
D. Personal computers

General purpose terminals include:

• Basic keyboard and monitor – used for entering data without any
validation checks; the monitor displays data from the computer system.
• Intelligent terminal – performs the functions of the basic keyboard and
monitor with the additional functions of validating data within the
terminal, maintaining transaction logs, and performing other local
processing.
• Personal computers – perform all the functions of an intelligent terminal
with the additional local processing and storage capabilities.

Special purpose terminals include:

• Point of sale devices – used to record sales transactions as they occur and
to transmit them to the main computer such as electronic cash registers
and optical scanners.
• Automated teller machines (ATMs) – used to initiate, validate, record,
transmit, and complete various banking transactions.

30. The “test data approach”

A. Involves reprocessing actual entity data using the entity’s computer software.
B. Involves reprocessing actual entity data using the auditor’s computer software.
C. Is where dummy transactions are prepared by the auditor and processed under
the auditor’s control using the entity’s computer software.
D. Is where actual transactions are prepared by the auditor.

31. Which of the following is a primary example of source data automation?

A. A subsidiary ledger
B. A utility bill
C. Point-of-sale (POS) scanners in malls
D. A bill of lading

32. Express Padala, Inc. stated in one of its mission statements that “positive
control of each package will be maintained by utilizing . . . electronic tracking and
tracing systems.” Express Padala uses what type of IT system?

A. Batch processing which features immediate updating as to the location of


packages.
B.Real-time processing which features updating at fixed time periods.
C.Batch processing which features updating at fixed timeperiods.
D. Real-time processing which features immediate updating as to the location of
packages.

33. In a file-oriented approach to data and information, data is maintained in many


separate files. This may create problems for organizations because of
A. Multiple users.
B. Multiple transaction files.
C. Multiple master files which may contain redundant data.
D. A lack of sophisticated file maintenance software.

34. __________________ refers to the combination of the database, the Database


Management System (DBMS), and the application programs that access the
database through the DBMS.

A. Data warehouse
B. Data administrator
C. Database system
D. Database manager

35. Who is the individual responsible for the database?

A. Data coordinator
B. Database master
C. Database administrator
D. Database manager
36. Which feature of many database systems simplifies the creation of reports by
allowing users to specify the data elements desired and the format of the output?

A. Report generator
B. Report writer
C. Report printer
D. Report creator

37. Which of the following is probably the most significant effect of database
technology on accounting?

A. Quicker access to and greater use of accounting information in decision-


making.
B. Replacement of the double-entry system.
C. Change in the nature of financial reporting.
D. Elimination of traditional records such as journals and ledgers.

38. An entity should have a disaster recovery plan to ensure that data processing
capacity can be restored as smoothly and quickly as possible. The following
would typically be part of an adequate disaster recovery plan, except

A. A system upgrade due to operating system software changes.


B. Backup computer and telecommunication facilities.
C. Scheduled electronic vaulting of files.
D. Uninterruptible power systems installed for key system components.

39. Which of the following statements concerning computer program


modifications is incorrect?

A. After the amended program has received final approval, the change is
implemented by replacing the production version with the developmental version.
B.During the modification process, the developmental version of the program must be
kept separate from the production version. C. When a program change is
submitted for approval, a list of all required updates should be compiled and then
approved by management and program users.
D. Only material program changes should be thoroughly tested and documented.

40. Old and new systems operating simultaneously in all locations is a test
approach known as parallel testing.

Pilot testing involves implementing a new system in one part of the organization,
while other locations continue to use the current system.

A. True; False C. False; True


B. Both are True D. Both are False

41. A collection of data that is shared and used by a number of


different users for different purposes is a
A. Database
B. Memory C. File
D. Record

The standard defines “database” as a collection of data that is shared and used by
a number of users for different purposes.

42. Which of the following computer software is used to create, maintain, and
operate a database?
A. Application software
B. Systems software
C. Database management system (DBMS)
D. Database administrator

The DBMS is used to create, maintain, and operate a data-base. It facilitates the
physical storage of the data, maintains the interrelationships among the data, and
makes the data available to application programs.

43. Two important characteristics of a database system are


A. The database and the DBMS.
B. Data sharing and data independence
C. The DBMS and data sharing.
D. The DBMS and data independence.

The two important characteristics of a database system are data sharing and data
independence.

Data sharing can be achieved if the database contains data which are setup with
defined relationships and are organized in a manner that permits several users to
access and use the data in different application programs.

The need for data sharing creates the need for data independence from application
programs. Through the DBMS, data are recorded only once, for use by different
application programs. There will be true data independence if the structure of data
can be changed without affecting the application programs, and vice versa.

44. To protect the integrity of the database, data sharing by different users requires
organization, coordination, rules, and guidelines. The individual responsible for
managing the database resource is the
A. Programmer
B. Database administrator
C. User
D. CIS manager

The database administrator is responsible generally for the definition, structure,


security, operational control, and efficiency of databases, including the definition
of the rules by which data are accessed and stored.

45. An auditor who wishes to trace data through several application programs
should know what programs use the data, which files contain the data, and which
printed reports display the data. In a database system, the information could be
found in a
A. Decision table
B. Data dictionary
C. Database schema
D. Data encryptor

A software within the DBMS that keeps track of the location of the data in the
database is called the data dictionary.

Answer A is incorrect because a decision table is a matrix presentation of the


decision points and related actions included in a computer program.
Answer C is incorrect because the database schema describes the database
structure.

Answer D is incorrect because an encryptor encodes messages.

46. Which of the following is the greatest advantage of a database system?


A. Data redundancy can be reduced.
B. Backup and recovery procedures are minimized.
C. Multiple occurrence of data items are useful for consistency checking.
D. Conversion to a database system is inexpensive and can be accomplished
quickly.

In a database system, data redundancy is kept to a minimum because the DBMS


records the data once, for use by various application programs. Storage structures
are created that make the application programs independent of the location of the
data.
Because each item in the database has a standard definition, name, and format;
and related items are linked by a system of pointers, the application programs
need only to specify the data name, not the location.

Answer B is incorrect because backup and recovery procedures in a database


system are just as crucial as in a traditional flatfile system.
Answer C is incorrect because data redundancy-that is, multiple occurrences of
data items-is substantially reduced in a database system.

Answer D is incorrect because converting large amount of data to a database is


costly and time consuming.

47. The following statements relate to a database management system (DBMS)


application environment. Which is false? A. Data definition is independent of any
one program.
B. The physical structure of the data is independent of user needs.
C. Data are used concurrently by different users.
D. Data are shared by passing files between programs or systems.
In a database system, application programs share the data in the common database
for different purposes. Thus, there is no need to pass files between applications.

48. Which of the following is an advantage of a database management system


(DBMS)?
A. A decreased vulnerability as the DBMS has numerous security controls to
prevent disasters.
B. Each organizational unit takes responsibility and control for its own data.
C. Data independence from application programs.
D. The cost of the CIS department decreases because users are now responsible
for establishing their own data handling techniques.

An important characteristic of a database system is that applications are


independent of the database structure. This allows programs to be developed for
the user’s specific needs without concern for data retrieval problems. Moreover,
changes to the physical or logical structure of the database can be made without
the need to modify any of the application programs that use the database.

Answer A is incorrect because the DBMS is no safer than any other computer
information systems.

Answer B is incorrect because each organizational unit develops its application


programs that will use the data items in the common database.

Answer D is incorrect because data handling techniques remain to be the


responsibility of the CIS department.

49. Which of the following is usually a benefit of transmitting transactions in an


electronic data interchange (EDI) environment?
A. A reduced need to test computer controls related to sales and collections
transactions.
B.A compressed business cycle with lower year-end receivables balances.
C.No need to rely on third-party service providers to ensure security.
D. An increased opportunity to apply statistical sampling techniques to account
balances.

Because EDI transactions are transmitted and processed in real time, delays are
eliminated in receiving and processing an order, shipping goods, and receiving
payment. Thus, EDI compresses an entity’s business cycle and results in lower
year-end receivables balances.

Answer A is incorrect because the use of a complex processing system increases


the need to test computer controls.

Answer C is incorrect because an EDI system typically uses a VAN value added
network) as a third-party service provider, and reliance on VAN controls may be
critical.

Answer D is incorrect because all transactions (not just a sample) may be tested
with the aid of computer technology.

50. The internal controls over computer processing include both manual
procedures and procedures designed into computer programs (programmed
control procedures). These manual and programmed control procedures comprise
the general CIS controls and CIS application controls. The purpose of general CIS
controls is to
A. Establish specific control procedures over the accounting applications in order to
provide reasonable assurance that all transactions are authorized and recorded and
are processed completely, accurately, and on a timely basis.
B.Establish a framework of overall controls over the CIS activities and to provide a
reasonable level of assurance that the overall objectives of internal control are
achieved.
C.Provide reasonable assurance that systems are developed and maintained in an
authorized and efficient manner.
D. Provide reasonable assurance that access to data and computer programs is
restricted to authorized personnel.

The purpose of general CIS Controls is to establish a framework of overall


controls over the CIS activities and to provide a reasonable level of assurance that
the overall objectives of internal control are achieved.

General CIS controls may include:


• Organization and management controls
• Application systems development and maintenance controls.
• Computer operation controls.
• Systems software controls.
• Data entry and program controls.

Answer A is incorrect because the establishment of specific control procedures


over the accounting applications is the purpose of CIS application controls.

Answer C is incorrect because controls designed to provide reasonable assurance


that systems are developed and maintained in an authorized and efficient manner
are application systems development and maintenance controls.

Answer D is incorrect because controls designed to provide reasonable assurance


that access to data and programs are restricted to authorized personnel are data
entry and program controls.

51. CIS application controls include the following, except

A. Controls over input.


B. Controls over processing and computer data files.
C. Controls over output.
D. Controls over access to systems software and documentation.

Restricting access to systems software and documentation to authorized personnel


is a general CIS control.

CIS application controls include:

1. Controls over input – designed to provide reasonable assurance that:

• Only authorized transactions are submitted for processing.


• All authorized transactions are accurately converted into machine-readable
form.
• Incorrect transactions are rejected, corrected, and, if necessary,
resubmitted on a timely basis.

2. Controls over processing and computer data files – designed to provide reasonable
assurance that:

• All transactions are processed as authorized.


• No authorized transactions are omitted.
• No unauthorized transactions are processed.
• Processing errors are identified and corrected on a timely basis.

3. Controls over output – designed to provide reasonable assurance that:

• The results of processing are accurate.


• Output is distributed only to authorized users.

52. The auditor is required to consider how an entity’s general CIS controls affect
the CIS applications significant to the
audit. Accordingly, the auditor should

A. Review the design of the general CIS controls only.


B. Review the design of the CIS application controls only.
C. Review the design of the general CIS controls before reviewing the CIS
application controls.
D. review the design of the CIS application controls before reviewing the design
of the general CIS controls.
General CIS controls that relate to some or all applications are typically
interdependent controls in that their operation is often essential to the
effectiveness of CIS application controls. A more efficient approach is to review
the design of the general CIS controls before reviewing the CIS application
controls.

53. The two broad categories of IT controls are general controls and application
controls. General controls include controls

A. For developing, maintaining, and modifying computer programs. B. That relate


to the correction and resubmission of erroneous data.
C.Designed to provide reasonable assurance that only authorized users receive output
from processing.
D. Designed to provide reasonable assurance that all data submitted for processing
have been properly authorized.

General controls relate to all or many IT activities and often include organization
and management controls, application systems development and maintenance
controls, and data entry and program controls.

Answers B, C, and D are incorrect because controls over correction of erroneous


input data, output distribution, and authorization of input data are IT application
controls.

54. Which of the following statements concerning application controls is correct?

A. Application controls relate to the processing of individual transactions.


B.Application controls relate to the processing of individual transactions.
C.Application controls relate to various aspects of the IT function including software
and hardware acquisitions.
D. Application controls relate to various aspects of the IT function including physical
security and the processing of transactions in various cycles.
55. The significance of hardware controls is that they

A. Ensure that run-to-run totals in application systems are consistent.


B. Reduce the incidence of user input errors in online systems.
C. Ensure correct programming of operating system functions.
D. Assure that machine instructions are executed correctly.

To detect and control errors arising from the use of computer equipment, hardware
controls are built into the equipment by the manufacturer, such as parity checks,
read-after-write checks, and echo checks.

Answer A is incorrect because run-to-run totals are used to determine the


completeness of update in an online system. Separate totals are accumulated for
all transactions processed throughout a period and compared with the total of
items submitted for computer processing.

Answer B is incorrect because input controls such as the use of limit checks, self-
checking digits, and input screens can reduce the incidence of user input errors in
online systems.

Answer C is incorrect because computer programmers and/or systems analysts are


responsible for correcting program errors.
56. The following statements relate to internal control in an electronic data
interchange (EDI) environment. Which is the true?

A. In EDI systems, preventive controls are generally more important than detective
controls.
B.Control objectives for EDI systems generally are different from the objectives for
other computer information systems.
C.Internal controls that relate to the segregation of duties generally are the most
important controls in EDI systems.
D. Internal controls in EDI systems rarely permit control risk at below the maximum.

In all information system – manual and computerized – preventive controls are


more important than detective controls because typically, the benefits exceed the
costs. In an EDI environment, it may be difficult to apply detective controls once
a transaction enters the computer system.

Answer B is incorrect because the basic objectives of internal control are the same
regardless of the nature of data processing.
Answer C is incorrect because adequate segregation of incompatible functions in a
CIS environment may not be feasible.
Answer D is incorrect because control risk in an EDI system may be assessed at
below the maximum level if relevant controls exist and tests of controls provide
evidence that those controls are functioning effectively.
57. An entity has recently converted its revenue/receipt cycle from a manual
processing to an online, real-time processing system. Which is the most probable
result associated with conversion to the new computerized processing system?

A. Less segregation of traditional duties.


B. significant increase in processing time.
C. Reduction in the entity’s risk exposures.
D. Increase in processing errors.

The basic segregation of functions – authorization, recordkeeping, and asset


custody – in a manual system is not usually feasible in a computerized system
because of decreased human involvement in processing financial information.

Answer B is incorrect because processing time is decreased in a computerized


system.

Answer C is incorrect because computer processing does not necessarily reduce


the number of risk exposures.

Answer D is incorrect because processing errors will decrease as a result of the


conversion to a new computerized system.

58. The most important segregation of duties in the organization of the


information systems function is

A. Using different programming personnel to maintain utility programs from those


who maintain the application programs.
B.Having a separate information officer at the top level of the organization outside of
the accounting function.
C.Assuring that those responsible for programming the system do not have access to
data processing operations.
D. Not allowing the data librarian to assist in data processing operations.

An important general CIS control is segregation of duties. Although some


separation of duties common in a manual system may not be feasible in a CIS
environment, come functions should not be combined.

The functions of systems analysts and programmers should not be combined with
the functions of computer operators. Programmers and systems analysts may be
able to effect changes in programs, files, and controls and should therefore have
no access to computer equipment.

Computer operators should have no opportunity to modify programs and data


files, and should not have programming duties or responsibility for installing new
or modifying existing systems.
Answer A is incorrect because computer programmers handle all types of
computer software.
Answer B is incorrect because having a separate information officer at the top
level of the organization outside of the accounting function would be less critical
than separation of duties between programmers and computer operators.

Answer D is incorrect because computer librarians may assist in data processing


operations. However, because they maintain control over system and program
documentation and data files, they should not have access to computer equipment.

59. A systems analyst should have access to each of the following, except

A. Edit criteria
B. Source code
C. Password identification tables
D. User procedures

Unauthorized changes to application programs and data files can be made by the
analyst if he/she has access to password identification tables.

Answer A, B, and D are incorrect because the systems analyst needs access to edit
criteria, source code, and user procedures.
60. Which of the following would represent an internal control weakness in an IT
environment?

A. The computer librarian maintains custody of computer application programs and


files.
B.The data control group is solely responsible for distributing computer-generated
reports.
C.Computer operators have access to operator instructions and have the authority to
modify application programs.
D. Computer programmers write and modify programs designed by systems analysts.

Computer operators should have access to operator instructions so they can


perform their duties. However, they should not have the authority to modify
application programs.

Answer A is incorrect because the computer librarian is responsible for


maintaining custody and recordkeeping for computer application programs and
data files.

Answer B is incorrect because an appropriate function of the data control group is


distribution of computer output and othe reports.
Answer D is incorrect because computer programmers are responsible for writing
and revising programs designed by systems analysts.
61. The manager of computer operations prepares a weekly schedule of planned
computer processing and sends a copy to the computer librarian. The control
objective of this procedure serves is to

A. Authorize the release of data files to computer operators.


B. Specify the distribution of computer results.
C. Specify file retention and disaster recovery policies.
D. Keep improper and unauthorized transactions from entering the computer
facility.

A computer librarian has in his/her custody data files, programs, and


documentation, all of which are his/her accountability. The weekly schedule of
planned computer processing provides authorization for release of files to
computer operators and a consequent transfer of accountability.

Answers B and D are incorrect because the data control group keeps unauthorized
and improper transactions from entering the computer facility and specifies the
distribution of computer results.

Answer C is incorrect because file retention and disaster recovery policies are
specified in the entity’s backup and recovery plan.

62. One of the major problems in a CIS environment is that incompatible duties
may be performed by the same individual. One compensating control is the use of

A. Computer-generated hash totals


B. A computer log
C. A self-checking digit system
D. Echo checks

Computer and software y=usage is recorded in a computer (console) log,


including operator intervention during computer processing. A compensating
control for the lack of adequate segregation of duties is by proper monitoring of
the computer log. For example, a computer log may include a list of operator
intervention during computer processing.

Answer A is incorrect because hash totals are control totals calculated using
nonfinancial data (for example, the sum of sales order numbers) to keep track of
the records in a batch.

Answer C is incorrect because a self-checking digit system is an input control to


detect data coding errors. It involves adding a control digit to a code (for example,
a bank account number) when it is originally designed to allow the code’s
integrity to be established during subsequent processing.
Answer D is incorrect because echo check is a hardware control that involves the
receiver of the message returning the message to the sender to determine if the
correct message was received.
63. In the organization of the information systems function, the most important
separation of duties is

A. Using different programming personnel to maintain utility programs from those


who maintain the application programs.
B.Assuring that those responsible for programming the system do not have access to
data processing operations.
C.Not allowing the data librarian to assist in data processing operations.
D. Having a separate information officer at the top level of the organization outside
of the accounting function.

64. An entity has recently converted its purchasing cycle from a manual process to
an online computer system. Which of the following is a probable result associated
with conversion to the new IT system?

A. Traditional duties are less separated.


B. Increased processing time.
C. Reduction in the entity’s risk exposure.
D. Increased processing errors.

65. An entity should plan the physical location of its computer facility. Which of
the following is the primary consideration for selecting a computer site?

A. It should be in the basement or on the ground floor.


B. It should maximize the visibility of the computer.
C. It should minimize the distance that data control personnel must travel to
deliver data and reports and be easily accessible by a majority of company
personnel. D. It should provide security.

The computer and other peripheral pieces of hardware should be protected from
disasters such as fire, flood, sabotage, and theft. Thus, the primary consideration
for selecting a computer site should be the security of the computer facility.

Answer A is incorrect because the basement or the ground floor is not always a
secured place. For example, installing a computer facility on the ground floor or
in the basement of an old office building in Malabon City could be disastrous
because of frequent flooding.

Answer B is incorrect because maximizing the visibility of the computer would be


an invitation to burglars and other computer criminals.
Answer C is incorrect because a majority of entity personnel need not have an
easy access to computer site since only authorized personnel should be allowed in
the computer facility.
66. Which of the following statements regarding security concerns for notebook
computers is false?

A. The primary methods of control usually involve application controls.


B.Centralized control over the selection and acquisition of hardware and software is a
major concern.
C.Some conventional controls such as segregation of duties may not be feasible.
D. As their use becomes more sophisticated, the degree of concern regarding
physical increases.

General controls apply to all CIS activities. Given the nature of notebook
computers, general controls to prevent theft of equipment and data and restrict
access to the use of equipment and data must be the primary concerns.

67. The following are a database administrator’s


responsibilities, except

A. Develop application programs to access the database.


B. Design the content and organization of the database.
C. Protect the database and its software.
D. Monitor and improve the efficiency of the database.

Systems analysts and programmers, not a database administrator, have the


responsibility of developing application programs to access the database.

Answe4r B, C, and D are incorrect because designing the content and organization
of the database; protecting the database and its software; and monitoring and
improving the efficiency of the database are appropriate responsibilities of a
database administrator.

68. Which of the following groups should have the operational responsibility for
the accuracy and completeness of computerbased information?

A. External auditors
B. Internal auditors
C. Users
D. Top management

Users are in the best position to review the accuracy and completeness of
computer output in relation to the input provided. Thus, the operational
responsibility for the accuracy and completeness of computer-based information
should be placed on users.
Answer A is incorrect because the primary purpose of external auditing is the
expression of an opinion on an entity’s financial statements.

Answer B is incorrect because internal auditing is an independent appraisal


activity within an organization. Therefore, internal auditors should not have
operational responsibility.

Answer D is incorrect because top management is responsible for the overall


control of the CIS.

69. An inexperienced computer operator mounted an incorrect version of the


accounts receivable master file on a tape drive during processing. Consequently,
the entire processing run had to be repeated at a prohibitive cost. Which of the
following software controls would be most effective in preventing this type of
operator error from affecting the processing of files?

A. File header and label check B. Data


transmission check
C. Memory isolation protection
D. Unauthorized access protection

An effective control to reduce risk of mounting an incorrect version of a master


file is the use of external, header, and trailer labels. An external label is a human-
readable label written on a gummed paper to be attached to the file. A header label
is a machine-readable label at the beginning of a file that identifies it. A trailer
label is also a machine-readable label at the end of a file containing control totals
and record counts.

Answer B is incorrect because only the accuracy of the communication is verified


by a data transmission check.

Answer C is incorrect because memory isolation protection (also called boundary


protection) ensures that while multiple jobs are running simultaneously, the
memory partition allocated to each job is not changed.

Answer D is incorrect because access controls (for example the use of personal
identification codes such as passwords and PINs) ensure that unauthorized access
to programs and files is prevented.

70. Which of the following is the best method to prevent unauthorized alteration
of online records?

A. Computer sequence checks B.


Computer matching
C. Database access controls
D. Key verification

Unauthorized access to online records can be prevented by establishing and


implementing access controls to ensure that only authorized personnel have access
to the company’s database.

73. Which of the following controls would most likely provide protection against

unauthorized changes in production programs? A. Restricting programmer access

to the computer room

B. Requiring two operators to be present during equipment operation.

C. Limiting program access solely to operators.

D. Implementing management review of daily run logs.

The risk unauthorized changes will be reduced if systems analysts, programmer,


and others are denied access to the resident programs. However, computer
operators should have access to the production programs in order to run the
programs.

Answers A and B are incorrect because unauthorized changes to programs can be


made by programmers at terminals regardless of whether they are denied access to
the computer room and regardless of whether two operators are present during
equipment operation.
Answer D is incorrect because management review of computer (console) logs,
not run logs, would be an effective control.

74. Which of the following would most likely indicate that a computer virus is
present?

A. Numerous copyright violations due to unauthorized use of purchased


software.
B. Unexplained losses of or changes of data.

C. Frequent power surges that harm computer equipment.

D. Inadequate backup, recovery, and contingency plans.

A virus is a program that attaches itself to legitimate program to penetrate the


operating system and cause destruction to the operating system, application
programs, and data files. For example, a virus can simply copy itself a number of
times within the memory to destroy resident programs and data.

Answers A, C, and D are incorrect because copyright violations, frequent power


surges, and inadequate backup, recovery, and contingency plans are not indicators
of a computer virus.

75. Which of the following operating procedures would most likely increase an
entity’s exposure to computer viruses?

A. Downloading public-domain software from electronic bulletin boards.

B. Installing original copies of purchased software on hard disk drives.

C. Frequent back up of files.


D. Encryption of data files.

Personal computers are major source of virus penetration. Downloading public-


domain software carries a risk that virusinfected data may enter the system.

Answer B is incorrect because original copies of purchased software should be


virus- free.
Answers C and D are incorrect because viruses are spread through distribution of
infected files, not through encryption or frequent backup of files.

76. An entity installed antivirus software on all its personal computers. The
software was designed to prevent initial infections, stop replication attempts,
detect infections after their occurrence, mark affected system components, and
remove viruses from infected components. The major risk in relying on antivirus
software is that it may A. Consume too many system resources.

B. Interfere with system operations.

C. Not detect certain viruses.

D. Make software installation too complex.

Antiviral programs (also called vaccines) are used to examine application and
operating system programs for the presence of viruses and remove them from the
affected programs. However, a vaccine works only on known viruses and there is
no guarantee that it will work if virus has been mutated.

Answers A and B are incorrect because antiviral software can be set to execute at
startup so as not to consume too many system resources.

Answer D is incorrect because installation of antiviral software is not an overly


complex process.

77. The accountant who prepared a spreadsheet model for workload forecasting
left the company, and his successor was unable to understand how to use the
spreadsheet. The best control to permit new employees to understand internally
developed programs is A. Adequate backups are made for spreadsheet models.

B. Use of end-user computing resources is monitored.


C. End- user computing efforts are consistent with strategic plans.

D. Documentation standards exist and are followed.

Because of inadequate program documentation, the accountant’s successor could


not use the spreadsheet model. New employees will be able to understand
internally developed programs if documentation standards exist and are being
followed.

Answer A is incorrect because the accountant’s successor could not use the
spreadsheet model due to inadequate documentation, not inadequate backups.

Answer B is incorrect because monitoring means controlling the use of resources.

Answer C is incorrect because ensuring consistency with strategic plans relates to


the system’s effectiveness.

78. What is the appropriate term for the process of monitoring, evaluating, and
modifying a system?

A. Feasibility study

B. Maintenance

C. Implementation

D. Analysis

Systems maintenance means keeping a new system that has been designed and
implemented current wit user needs. This basically involves revising the systems
and application programs to meet new user needs and to correct designed errors.
The responsibility for systems maintenance is assumed by systems analysts and
programmers.
Answer A is incorrect because a feasibility study is made to determine the
technical, legal, operational, and schedule (i.e., the company’s ability to
implement the project within an acceptable time) feasibility of a proposed system.

Answer C in incorrect because system implementation involves data conversion;


coding and testing applications; purchased and installation of equipment; training
of employees; system documentation; and installation of the new system.

Answer D is incorrect because systems analysis involves a survey of the current


system, an analysis of the user’s needs, and gathering and evaluation of facts.

79. Program documentation is a control designed primarily to provide reasonable


assurance that

A. Programs are kept up to date and perform as intended.

B. No one uses the computer hardware for personal reasons.


C. Programs are free of syntax and logic errors.

D. Programmers have access to operational materials.

Program documentation provides detailed information about each application


program including the source program, file formats and record layouts, program
flowcharts, written authorizations for all program changes, and operating
transactions. For a computer system to operate efficiently, adequate and up-to-date
program documentation is necessary.

Answer B is incorrect because program documentation cannot ensure security of


computer hardware.

Answer C is incorrect because debugging should uncover errors in programs.

Answer D is incorrect because programmers should not have access to operational


materials such as tape library or information on disk files.
80. An entity updates its account receivable master file weekly and retains the
master files and corresponding update transactions for the most recent two-week
period. The purpose of this periodic retention of master files and transaction data
is to

A. Validate groups of update transactions for each version.

B. Permit reconstruction of the master file if needed.

C. Verify run-to-run control totals for receivables.

D. Match internal labels to avoid writing on the wrong volume.

The grandparent-parent-child approach (also called grandfatherfather-son


approach) is used in sequential file batch systems. This backup technique begins
when the current master file (the parent) is processed against a transaction file to
create a new updated master file (the child). When a new batch of transactions is
processed, the child becomes the parent (the current master file), and the parent
(the original master file) becomes the grandparent or backup file.

As described, the grandparent-parent-child backup technique involves the creation


and retention of three generations of master files enable reconstruction of
destroyed or corrupted master file.

The systems designer is responsible for determining the number of backup files
needed for each application. The designer should consider the degree of file
activity and the financial relevance of the system in making such decision.

Answer A and D are incorrect because validation routines and internal labels may
prevent data from being destroyed but do not allow recovery of lost or destroyed
data.
Answer C is incorrect because verification of run-to-run totals ensures
completeness of processing, not data recovery.

81. An entity’s contingency plans for computer information systems should


include appropriate back up arrangements. Which of the following arrangements
would be considered too vendordependent when vital operations require almost
immediate availability of computer resources?

A. A “cold site” arrangement


B. A “hot site” arrangement

C. A “cold and hot” arrangement

D. Using excess capacity at another data center within the entity.

A “cold site” is a backup facility that has all the needed computer equipment. This
backup arrangement is too vendordependent because it relies on the vendor’s
timely delivery of the needed computer equipment.

Answer B is incorrect because a “hot site” backup facility has all the needed
resources in place, including the computer equipment, and is therefore not vendor-
dependent.

Answer C is incorrect because a “cold and hot site” backup facility has a “hot
site” component that is fully configured and available for immediate use while the
“cold site” is being configured, making it not too vendor-dependent.

Answer D is incorrect because having excess capacity at another data center


within an entity means that there are available resources that can be used.

82. Which of the following is primary objective of security software?


A. To detect the presence of computer viruses.

B. To monitor the segregation of functional responsibilities within applications.

C. To prevent installation of unauthorized utility software.

D. To control access to information system resources.

The primary objective of security software is to keep unauthorized intruders from


accessing information system resources and data files.

Answer A is incorrect because antiviral software, not security software, detects the
presence of computer viruses.

Answer B is incorrect because security software can be used to establish, not


monitor, separation of duties.

Answer C is incorrect because security software can be used to control the use of
utility software not to prevent installation of unauthorized utility software.

83. All administrative and professional staff in an entity’s legal department


prepare documents on terminals connected to a host LAN file server. Which of the
following is the best control over unauthorized access to sensitive documents in
the system? A. Required entry passwords for access to the system.

B. Required entry passwords for access to individual documents.

C. Physical security for all disks containing document files.

D. Periodic server backup and storage in a secure area.

Effective access controls normally required different passwords to access the


system, to read certain data files, and other information system resources. The
control over unauthorized access to sensitive documents is required password
entry for access to individual documents.

Answer A is incorrect because required entry of passwords for access to systems


allows all departmental personnel to gain access to all documents in the system.

Answer C is incorrect because LAN may not use floppy disks.

Answer D is incorrect because although periodic server backup and storage in a


secure area is a good security/ backup control procedure, it would not prevent
intruders from accessing sensitive documents online.

84. An internal control has just concluded a physical security audit of data center
which is primarily engaged in top-secret defense contract work. The auditor has
recommended biometric authentication for workers entering the building. The
recommendation might include devices that verify all of the following, except A.
Fingerprints

B. Password patterns

C. Speech patterns

D. Retina patterns

The use of biometrics devices is considered the ultimate in user authentication


procedures. These devices are used to establish an individual’s identity by
measuring various personal characteristics, fingerprints, voiceprints, retina prints,
or signature characteristics.

85. Which of the following best describes the process called authentication?
A. The system verifies the identity of the user

B. The user identifies himself/herself to the system

C. The user indicates to the system that the transaction was processed correctly

D. The system verifies the user is entitled to enter the transactions request.

Authentication is the process of verifying the identity of the user. Biometric


devices are used to authenticate an individual’s identity using physiological or
behavioral traits such as retina patterns, fingerprints, and speech patterns.

Answer B is incorrect because when the user identifies himself/herself to the


system, it does not necessarily mean that the system verifies his/her identity.

Answer C is incorrect because this is an application control that relates to the


accuracy of processing transactions.

Answer D is incorrect because authentication does not necessarily include


determining the function of a user whose identity has been verified.

86. Which of the following assurances is not provided by an application control?

A. Review and approval procedures for new systems are set by policy and
adhered to

B. Authorized transactions are completely processed once and only once

C. Transaction data are complete and accurate

D. Processing results are received by the intended users Answers B, C and D are
incorrect because these are the objectives of application controls.
87. Data processing activities may be classified in terms of three stages or

processes: input, processing, and output. Which of the following activities is not

normally associated with the input stages? A. Recording B. Batching

C. Reporting

D. Verifying

Reporting is normally associated with the output stage. Output is the result of
computer processing, for example, a hard copy printout of a report, magnetic files,
or invoices.

Answers A, B and D are incorrect because recording, batching, and verifying are
normally associated with the input stage.

88. Which of the following is the purpose of input controls?

A. To ensure the authorization of access to data files

B. To ensure the completeness, accuracy, and validity of updating

C. To ensure the completeness, accuracy, and validity of input

D. To ensure the authorization of access to program files.

Input controls are designed to provide reasonable assurance that the data received
for computer processing are complete, accurate and valid.

Answers A and D are incorrect because assuring the authorization of access to


date and program files is the objective of access control.

Answer B is incorrect because ensuring the completeness, accuracy and validity


of updating is the objective of processing controls.
89. If a control total were to be computed on each of the following data items,
which would best be identified as a hash total for a payroll IT application?

A. Employee numbers

B. Total debit and credit amounts

C. Gross wages earned by employees

D. Total hour worked

90. An entity uses the account code 699 for depreciation expense. However, one
of the company data input clerks often codes depreciation expense as 996. The
highest account code in the company’s system is 700. What programmed control
procedure would detect this error?

A. Pre-data input check

B. Sequence check

C. Valid-code test

D. Valid-character test

91. Which of the following provides the most valuable information for detecting
unauthorized input from a terminal?

A. User error report


B. Transaction log

C. Error file

D. Console log printout

A transaction log is a permanent record of all completely validated transactions


received for the computer processing. Subsequent comparison of the transaction
log with authorized transactions such as authorized source documents will detect
unauthorized input from a terminal.

Answer A is incorrect because a user error report only lists input that fails the
validation tests.

Answer C is incorrect because an error file is used to store and correct records
detected during validation.

Answer D is incorrect because a console log is a record of computer and software


usage. It does not record individual transactions transmitted from the terminal.

92. Many customers, managers, employees, and suppliers have blamed the
computer for making errors. In reality, computers make very few mechanical
errors. Which of the following is the most likely source of errors in fully
operational computer-based system?

A. Systems analysis and programming

B. Operator error

C. Processing

D. Input

It is garbage-in, garbage-out in computer processing--- erroneous input results in


erroneous output.

Answer A is incorrect because proper design and implementation of computer


programs would eliminate most syntax and logic errors or bugs.

Answer B is incorrect because operator (run) manuals which describe how to run
the system, decrease the chance of operator error.
Answer C is incorrect because, once a program has been thoroughly tested (for
example, by creating hypothetical master files and transaction files to be
processed by the program being tested), the processing of appropriate data does
not result in errors.

93. Data conversion is the transcription of transaction data from source documents
to magnetic tape or disk suitable for computer processing. Which of the following
data conversion methods is most difficult to audit?

A. Keying data to disk for online processing

B. Keying data to disk for batch processing

C. Reading source data using optical character recognition

D. Keying data to source documents for magnetic ink character recognition

Data conversion in online systems is difficult to audit because there is usually no


visible audit trail. Transactions are transmitted directly from terminal and hard
copy source documents are often lacking.

Answer B is incorrect because keying data to disk for batch processing creates
records that can be readily tested.

Answer C is incorrect because hard copy source documents are retained in optical
character recognition. Moreover, this method reduces the risks of conversion
error.

Answer D is incorrect because magnetic ink character recognition provides hard


copy source documents that can be used for audit purposes.

94. Which of the following best describes the online data processing control called
preformatting?
A. The display of a document with blanks for data items to be entered by the terminal
operator.

B.a program initiated prior to regular input to discover errors in data before entry so
that the errors can be corrected.

C.A series of request for required input data that requires an acceptable response to
each request before a subsequent request is made.

D. A check to determine if all data items for a transaction have been entered by the
terminal operator.

A preformatted screen approach may be used in online systems to avoid data entry
error. Under this approach, blanks for specified data items will be displayed on the
monitor. This is most appropriate when data entry is from a source document.
Moreover, the screen format may even be in the form of a transaction document.

Answer B is in correct because an edit/validation routine is a program initiated


prior to regular input to discover errors before entry so that errors can be
corrected.
Answer C is incorrect because the dialogue approach is another screen prompting
method that is most appropriate for data received orally, e.g., by phone.

Answer D is incorrect because a check to determine if all data items for a


transaction have been entered by the terminal operator is called completeness
check.

95. When erroneous data are detected by the computer program controls, such
data may be excluded from processing and printed on an error report. Who should
review and follow up this error report?

A. System analyst
B. Data control group

C. Computer operator

D. Computer programmer

Many entities have a data control group (independent of the computer processing
operation) that acts as liaison between the end user and data processing.

The data control group is responsible for receiving from users, transaction
documents for processing; and controlling the distribution of computer output
such as documents and reports. It is responsible for the following up error reports
to ensure that the erroneous records are corrected by the users and reprocessed by
the computer center.

Answers A, C, and D are incorrect because systems analysts, computer operators,


and computer programmers are not independent of computer operations.

96. If a payroll system continues to pay employees who have been terminated,
control weakness most likely exist because

A. Input file label checking routines built into the program were ignored by the
operator.

B.Programmed controls such limit checks should have been built into the system.

C.Procedures were not implemented to verify and control the receipt by the computer
processing department of all transactions prior to processing.

D. There were inadequate manual controls maintained outside the computer system.

In a payroll system, the authorization to pay employees should come from the
personnel department, which is external to the computer processing department.
Hence, inadequate controls maintained outside the computer system are likely to
allow the payments to terminated employees to continue without being detected.

Answers A, B, and C are incorrect because the use of input file labels, limit
checks, and batch total will not detect unauthorized transactions.

97. In the accounting system of ABC Company, the amounts of cash


disbursements entered at a computer terminal are transmitted to the computer,
which immediately transmits the amounts back to the terminal for display on the
terminal screen. This display enables the operator to

A. Establish the validity of the account number

B. Prevent the overpayment of the account


C. Verify the accuracy of the amount entered

D. Verify the authorization of the disbursement

Displaying the amounts entered on the terminal screen allows the terminal
operator to visually verify the accuracy of the amounts entered.

98. Which of the following input validation checks is least likely to be appropriate
in an online, real-time system?

A. Sign check

B. Sequence check

C. Reasonableness check

D. Redundant data check


The sequence check control is appropriate only in systems that uses sequential
master files. This control determines if the records are in the proper order by
comparing the sequence of records are in the batch with the previous record.
Because records are not processed sequentially in an online, real-time system, this
control is not likely to be appropriate.

Answers A, C, and D are appropriate in an online, real-time system.

A sign check tests data to determine if they have appropriate arithmetic sign.

A reasonableness check determines if an amount falls within predefined limits.


For example, the number of hours worked in a single day should be neither less
than zero nor more than 12.
A redundancy check assures that an application processes each record only once.

99. A receiving clerk keyed in a shipment from a remote terminal and


inadvertently omitted the purchase order number. Which of the following controls
would most likely detect this error?

A. Completeness check

B. Compatibility check

C. Sequence check

D. Reasonableness test

A completeness test identifies missing data within a single transaction record (for
example, missing purchase order number on the shipping document) or records
within a batch of transaction data.

Answer B is incorrect because a compatibility check (also called field test)


determines whether a field contains proper characters.
Answer C is incorrect because a sequence check determines if records have been
properly sorted.

Answer D is incorrect because a reasonableness test determines if the value is


within predetermined limits.

100. A wholesaler of automotive parts has a computerized billing system. Because


of a clerical error while entering information from the sales order, one of its
customer was billed for only three of the five items ordered and received. Which
of the following controls could have prevented or promptly detected this clerical
error?
A. Periodic comparison of total accounts receivable per accounts receivable master
file with total accounts receivable per accounts receivable control account.

B.A completeness check that does not allow a sales invoice to be processed if key
fields are blank.

C.Prenumbered shipping documents together with a procedure for follow up anytime


there is not a one-to-one relationship between shipping documents and sales
invoices.

D. Matching line control counts produced by the computer with predetermined line
control counts.

A line control count could have prevented or promptly detected the clerical error.
This control technique involves a count of individual line items on a document.
Missing lines can be detected by simply comparing these counts with
predetermined line control counts for each document.

Answer A is incorrect because the three-item sales invoice would be the basis for
updating both the accounts receivable master file and control account. Hence, no
discrepancy would be disclosed by the comparison.
Answer B is incorrect because a completeness check would not detect the billing
error because other sales invoices may properly contain three or fewer lines.

Answer C is incorrect because although the sales invoice has missing lines, it
exist and can be matched with the shipping document.

101. Which of the following computerized control procedures would most likely
provide reasonable assurance that data uploaded from personal computers to a
mainframe are complete and that no additional data are added?

A. Field-level edit controls that test each field for alphanumerical integrity.

B.Self-checking digits to ensure that only authorized part numbers are added to the
database.

C.Batch control details, including financial totals and hash totals.

D. Passwords that effectively limit access to only those authorized to upload the data
to the mainframe.

Batch totals which consist of record counts, financial or control totals, and hash
totals can be used to ensure the accuracy and completeness of data uploaded from
personal computers to mainframe. After the uploading process, these totals are
reconciled with predetermined totals to test if the data have been completely
transferred.

A record count (also called item count) is the total number of records in a batch.

A financial or control total is the peso value of a financial field, for example, the
total sales invoice amounts.

A hash total is the total of a unique nonfinancial field, for example, the total of
purchased order numbers in a batch.
Answers A, B, and D are incorrect because they do not provide assurance about
the completeness of data upload.

102. An entity’s labor distribution report requires extensive corrections each


month because of labor hours charged to inactive jobs. Which of the following
data processing input controls appears to be missing?

A. Validity check

B. Limit check

C. Missing data check

D. Control total

Validity checks compare actual values in a field (for example, a transaction code)
against acceptable (valid) values in the master file. If the value in the field does
not match one of the acceptable values. The record is considered to be in error. If
the computer checks first for validity of the jobs, labor hours would not be
erroneously assigned to inactive jobs.

Answer B is incorrect because a limit check determines if the value in the field
exceeds a predetermined limit.

Answer C is incorrect because missing data checks are used to determine if a field
contains blank spaces. The computer considers a record in error if blanks are
detected where data values are expected.

Answer D is incorrect because control totals are used to reconcile computer input
with processing result.
103. If, in reviewing an application system, it is noted that the batch controls are
not used, which of the following statements by the user of the system is
acceptable as a compensating control?

A. “The volume of transactions prohibits batching.”

B. “We do a 100% physical review of the input document to the output


document.”
C. “We do a 100% key verification of all data input.”

D. “The supervisor must approve all inputs.”

A 100% physical review of the input document to the output document will
provide evidence that all records are completely and accurately processed. Thus,
this procedure will compensate for the lack of batch control totals.

Answer A is incorrect because the use of batch control totals is most appropriate
in managing high volumes of transaction data.

Answer C is incorrect because a 100% key verification does not assure that all
records submitted for processing were keypunched.

Answer D is incorrect because the supervisor’s approval of all inputs does not

assure that all approved inputs were processed.

104. A mail-order retailer of low-cost novelty items is receiving an increasing


number of complaints from customers about the wrong merchandise being
shipped. The order code for items has the format WWXXYYZZ, which has the
following meaning:

WW- major category

XX- minor category

YY- identifies the item


ZZ- identifies the catalog

In many cases, the wrong merchandise was sent because adjacent characters in the
order code had been transposed. The most effective control to prevent this
erroneous input is to

A. Use a master file reference for all codes to verify the existence of items.
B.Separate the parts of the order code with hyphens to make the characters easier to
read.

C.Add check digits to the order codes and verify them for each order.

D. Require customers to specify the name of each item they order.

Transposition errors can corrupt data codes and cause serious data processing
problems if they go undetected.

An effective control to detect data coding errors is by adding a check digit (or
digits) to a data code.

The check digits is the result of the mathematical calculation done based on the
original data code (the simplest form is to add all the digits in the code). During
the input process, the system recalculates the check digits for each input and
compares the result with the check digit attached to the data code entered.

Answer A is incorrect because order codes containing transposed characters may


match other items in the file. Thus, the use of a master file reference code would
not detect erroneous order codes.

Answer B is incorrect because the use of hyphens would make the order code
easier to read, but would not detect order codes with transposed characters.

Answer D is incorrect because requiring customers to specify the name for each
item they order would generally not allow detection of erroneous codes.
105. Which of the following is the major purpose of the auditor’s study and
evaluation of the company’s computer processing operations?
A. Ensure the exercise of due professional care

B. Evaluate the reliability and integrity of financial information

C. Become familiar with the company’s means of identifying, measuring,


classifying, and reporting information

D. Evaluate the competence of computer processing operating personnel

The auditor studies and evaluates information systems primarily to ascertain


whether financial data are accurate, reliable, timely, and complete.

Answer A is incorrect because auditors are required to exercise due care in all
audits.

Answer B is incorrect because becoming familiar with the company’s information


system is a means to achieve the auditor’s principal objective.

Answer D is incorrect because evaluating the competence of computer processing


operating personnel is not the auditor’s primary purpose of evaluating the
company’s information system.

106. When the auditor chooses to use only the non-IT segment of a client’s control
to asses control risk, it is referred to as auditing around the computer. Which one
of the following conditions need not be present to apply this audit approach?

A. The output must be listed in sufficient detail to enable the auditor to trace
individual transactions.

B. The source documents must be filed in a manner that makes it possible to


locate them.
C. The source documents must be available in a non-machine language.
D. Computer programs must be available in English.

107. The following statements relate to the auditor’s assessment of control risk in
an entity’s computer environment. Which is correct?

A. The auditor usually can ignore the computer system if he/she can obtain an
understanding of the controls outside the computer information system.

B.If the general controls are ineffective, the auditor ordinarily can assess control risk
at a low level if the application controls are effective.

C.The auditor’s objectives with respect to the assessment of control risk are the same
as in a manual system.

D. The auditor must obtain an understanding of the internal control and test controls
in computer environments.

The overall objective and scope of an audit does not change in a CIS environment.
Regardless of the information system used by the entity--- manual or
computerized, the auditor is required to obtain an understanding of internal
control and assess control risk to plan the audit.

Answer A is incorrect because, when an entity’s computer information system is


significant (i.e., it has a material effect on financial statement assertions), the
auditors is required to obtain an understanding of the CIS environment and
determine whether it may influence the assessment of inherent and control risk.
Answer B is incorrect because, if general controls are ineffective, the auditor is
unlikely to assess control risk at a low level, regardless of whether application
controls have been designed and implemented for each significant accounting
application.
Answer D is incorrect because tests of controls should be performed only when
the auditor’s risk assessment includes an expectation of the operating
effectiveness of controls (i.e., control risk is assessed at below the maximum), or
when substantive procedures alone do not provide sufficient appropriate audit
evidence at the assertion level.

108. Computer programs and data that the auditor may us as part of the audit
procedures to process data of audit significance contained in an entity’s
information system are called

A. CAATs

B. DOOGs

C. BIIKs

D. BIIRDs

Computer-assisted audit techniques (CAATs) are computer programs and data that
the auditor may use in performing various audit procedures, including the
following:

• Tests of details of transactions and balances


• Analytical review procedures
• Tests of general and application controls
• Sampling programs to extract data for audit testing
• Re-performance of calculations performed by the entity’s accounting
system

Answer B, C, and D are incorrect because DOOGs, BIIKs, and BIIRDs are not
used in information technology (IT).
109. One common type of CAAT is the use of audit software to process data audit
significance from the entity’s information system. An audit software that has
widespread popularity because it is easy to use and requires little computer
background on the part of the auditor; it can be used on both mainframe and PC
systems; it allows the auditor to perform hi/her tests independent of the entity’s
computer processing personnel; and it can be used to audit the data in most file
formats and structures is called a A. Customized program

B. Purpose-written program

C. Utility program

D. Package or Generalized audit software (GAS)

The easy-to-use and flexibility features of generalized audit software (GAS) make
it very popular to auditors in the audit information technology (IT) environments.
This audit software is designed to perform common audit tasks or standardized
data processing functions, such as the following:

• Reading data files


• Selecting and analyzing information
• Summarizing and totaling files
• Performing or verifying calculations
• Creating data files
• Providing totals of unusual items
• Reporting in an auditor-specified format

Answer A and B are incorrect because customized or purposewritten programs are


designed to perform audit tasks in specific circumstances. These programs are
used when an entity’s computer information system is so unique or complex that
any GAS is deemed unsuitable.
Answer C is incorrect because utility programs are part of the operating system
and security software packages that are provided by computer manufacturers and
software vendors. This software performs routine data processing functions, such
as sorting, copying, creating, merging, erasing, and printing files. It is not
generally designed for audit purposes and may not contain audit features, such as
record counts or control totals.

110. Customized or purpose-written programs perform audit tasks in specific


circumstances where package audits software is deemed unsuitable usually
because system constraints make it difficult or impossible to use. A purpose-
written program may be developed by

A B C D
The auditor no yes yes no

The entity being audited yes yes no no

An outside programmer
hired by the auditor yes yes no no

111. These computer programs are enhanced productivity tools that are typically
part of a sophisticated operating systems environment, for example, data retrieval
software or code comparison software

A. Purpose-written programs
B. System management programs C. Utility programs

D. Generalized audit software

112. Embedded audit routines are sometimes built into an entity’s computer
information system to provide data for later use by the auditor. One technique
involves embedding audit software modules within an application system to
provide continuous monitoring of entity’s transactions. These audit modules are
used to create logs that collect transaction information for subsequent review by
the auditor. These logs are called

A. System control audit review files (SCARFs)

B. Console logs

C. Computer logs

D. IT logs

113. When an accounting application is processed by computed, an auditor cannot


verify the reliable operation of programmed controls by

A. Periodically submitting auditor-prepared test data to same computer process and


evaluating the results.

B.Constructing a processing system for accounting applications and processing actual


data from throughout the period through both the client’s program and the
auditor’s program.

C.Manually comparing detail transaction files used by an edit program with the
program’s generated error listings to determine that errors were properly identified
by the edit program.

D. Manually re-performing, as of a moment in time, the processing of input data and


comparing the stimulated results with the actual results.

The effectiveness of programmed controls may not be tested if auditing around


the computer (also called the black box approach) is to be applied. This involves
manual comparison of the input data with the computer output.

Because programmed controls are built into the computer program, the auditor
should instead apply the white box approach. This means that the auditor should
have an in depth understanding of how the programmed control function and
should consider using CAATs in testing their effectiveness.
Answer A is incorrect because the use of the test data approach is an effective
method of evaluating the reliability of programmed control procedures.

Answer B is incorrect because parallel simulation is also an effective method of


evaluating the reliability of programmed controls.

Answer C is incorrect because manually comparing the output of an auditor’s edit


program with the error listing generated by the client’s program would provide
evidence about the reliability of programmed controls.

114. Auditing through the computer must be used when A. Generalized

audit software is not available.

B. Processing is primarily online and updating is real-time.

C. Input transactions are batched and system logic is straightforward.

D. Processing primarily consists of sorting the input data and updating the master
files sequentially.

Auditing through the computer involves an in-depth understanding of the


computer program’s logic. This approach is appropriate when a complex and
significant application is involved and evidence external to the computer system is
unlikely to be available--for example, in an online, real-time system.

Answer A is incorrect because, in deciding on what audit approach is appropriate


—auditing through or around the computer, the auditor determines whether
evidence external to the computer is available, not whether generalized audit
software is available.

Answer C is incorrect because, in a simple batch system, auditing around the


computer (the black box approach) is appropriate because evidence external to the
computer, such as printouts and source documents, can be directly examined by
the auditor.

Answer D is incorrect because, when processing is simple (for example, when


files are stored and processed sequentially), evidence outside the computer is
likely to be available.

115. When an auditor tests a computer information system, which of the following
is true of the test data approach?

A. Test data are processed by the client’s computer programs under the auditor’s
control.

B. Several transactions of each type must be tested.

C. Test data must consist of all possible valid and invalid conditions.

D. The program tested is different from the program used throughout the year by
the entity.

Under the test data approach, the auditor processes a specially prepared set of
input data containing possible valid and invalid conditions using the client’s
application program.
The result of each test are compared with predetermined results, based on the
auditor’s understanding of the programmed controls. This approach will allow the
auditor to make an objective evaluation of the program logic and the effectiveness
of programmed controls.

Answer B is incorrect because only one of each transaction type needs to be tested
and evaluated.

Answer C is incorrect because the auditor tests only those controls that are
relevant to the financial statement audit.
Answer D is incorrect because, if the program to be used for testing is different
from the program used throughout the year by the client, no assurance can be
obtained about the effectiveness of programmed controls.

116. An auditor who is testing IT controls in a payroll system would most likely
use test data that contain conditions such as

A. Payroll checks with unauthorized signatures

B. Deductions not authorized by employees

C. Time tickets with invalid job numbers

D. Overtime not approved by supervisors

117. Auditors have learned that increased computerization has created more
opportunities for computer fraud but has also led to the development of computer
audits techniques to detect frauds. A type of fraud that has occurred in the banking
industry is programming fraud in which the programmer designs a program to
calculate daily interest on savings accounts to four decimal points. The
programmer then truncates the last two digits adds it to his account balance.
Which of the following CAATs would be most effective in detecting this type of
fraud?
A. Generalized audit software that select account balances for confirmation with
the depositor

B. Snapshot

C. Parallel simulation

D. SCARF (Systems Control and Audit Review File)

In parallel simulation, the auditor uses a specially prepared computer program that
simulates key features or processes of the application program to be tested.
Program logic and controls are evaluated by comparing the results of processing
actual data using the simulation program with the results of processing the same
actual data using the client’s application program.

Parallel simulation is the most effective CAAT application because the amounts
credited to the depositors’ accounts can be compared with the amounts calculated
by the auditor’s simulation program.

Answer A is incorrect because confirmation of a depositor’s account balance may


fail to detect errors involving a very insignificant amount (i.e., less than one
centavo daily).

Answer B and D are incorrect because SCARFs and snapshots will not detect the
computer fraud prescribed.

118. To obtain evidence that online access controls are properly functioning, an
auditor is most likely to

A. Vouch a random sample of processed transactions to assure proper authorization.

B.Create checkpoints at periodic intervals after live data processing to test for
unauthorized use of the system.
C.Enter invalid identification numbers or passwords to ascertain whether the system
rejects them.

D. Examine the transaction log to discover whether any transactions were lost or
entered twice because of a system malfunction.

The auditor can directly test whether online access controls are properly
functioning by attempting to gain access to the system by using invalid
identification numbers or passwords.
Answer A is incorrect because unauthorized transactions may be entered by any
intruder who know valid identification numbers or passwords.

Answer B is incorrect because, in a batch computer systems, checkpoints are used


as a recovery procedure.

Answer D is incorrect because examining the transaction log to discover whether


any transactions were lost or duplicated would not determine if any online access
controls are functioning effectively.

119. Which of the following CAATs allows fictitious and real transactions to be
processed together without the knowledge of client operating personnel?

A. Data entry monitor

B. Integrated test facility (ITF)

C. Parallel simulation

D. Input control matrix

The integrated test facility (ITF) approach enables the auditor to test a computer
program’s logic and controls during its normal operation. Under this approach,
fictitious records for dummy units (for example, a division, a department, or a
dummy entity) are integrated with legitimate records in the database.

During the normal computer processing, test transactions are merged with actual
transactions and processed against the dummy records in the master database.

Because computer application with ITF can be tested without intervention of


operating personnel, ITF enhances audit efficiency and increases reliability of the
audit evidence.

Answers A and D are incorrect because data entry monitor and input control
matrix are not used by the auditor in testing an entity’s computer information
system.
Answers C is incorrect because, in parallel simulation, real (not fictitious)
transactions are reprocessed.

120. In auditing an online perpetual inventory system, an auditor selected certain


file-updating transactions for detailed testing. The audit technique that will
provide a computer trail of all relevant processing steps applied to a specific
transaction is called

A. Snapshot

B. Simulation

C. Tagging and tracing

D. Code comparison

Tagging and tracing involves selection of specific transactions to be tagged (by


attaching an indicator at input) and traced through critical control points in the
computer information system.
The computer trail can be printed or stored in a computer file for the auditor’s
evaluation.

Answers A, B, and D are incorrect because snapshot, simulation, and code


comparison do not provide a trail of all relevant processing steps.

TRUE OR FALSE

1. A hash total is a numeric value computed to provide assurance that the


original value has not been altered in construction or transmission.

2. General controls include data validation controls.


3. A limit or reasonableness test is a test to ensure that a numerical value does
not exceed some predetermined value.
4. The control environment component of internal controls includes access to
computer programs.

5. As opposed to a manual control, an automated control should function


consistently in the absence of program changes.
6. The display monitor is a software component of a computer system.

7. The systems analyst should not be allowed access to programs listings of


application programs.

8. The posting of a transaction, as it occurs, to several files, without


intermediate printouts is a characteristic of a batch processed computer
system.

9. Controls which are built in by the manufacturer to detect equipment failure


are called input controls.
10. Echo checks, data encryption, and parity checks are data transmission
controls.

11. When applying the test data approach, auditors use auditor- controlled
software to do the same operations that the client’s software does, using the
same data files.
12. A problem for a CPA associated with advanced IT systems is that the audit
trail is sometimes generated only in machine readable form.

13. Controls which are designed to assure that the information processed by the
computer is authorized, complete, and accurate are called input controls.
14. A system in which the end use is responsible for the development and
execution of the computer application that he or she uses is called
decentralized computing.

15. In an IT-intensive environment, most processing controls are programmed


controls.

16. An example of an access control is a check digit.

17. Output controls are designed to assure that the data generated by the
computer are used appropriately by management.

18. An internal control deficiency occurs when computer personnel originate


changes in customer master files.

19. Auditing through the computer is generally used processing is primarily on


line and updating is real- time.
20. General control have a pervasive effect on the operating effectiveness of
application controls.

21. Random errors are more likely in a batch system than in an online system.

22. Auditing by testing the input and output of a computer system instead of the
computer program itself will detect all program errors, regardless of the
nature of the output.

23. In an IT system, automated equipment controls or hardware controls are


designed to detect and control errors arising from the use of equipment.
24. Logging in to the company’s information system via password is an
application control.

25. Controls that relate to a specific use of the IT system, such as the processing
of sales or cash receipts, are called general controls.

KEY ANSWERS

1. D 25. B 49. B 73. C 97. C


2. B 26. A 50. B 74. B 98. B
3. B 27. D 51. D 75. A 99. A
4. B 28. B 52. C 76. C 100. D
5. C 29. B 53. A 77. D 101. C
6. B 30. C 54. B 78. B 102. A
7. D 31. C 55. D 79. A 103. B
8. B 32. D 56. A 80. B 104. C
9. D 33. C 57. A 81. A 105. B
10. C 34. C 58. C 82. D 106. D
11. B 35. C 59. C 83. B 107. C
12. D 36. B 60. C 84. B 108. A
13. D 37. A 61. A 85. A 109. D
14. A 38. A 62. B 86. A 110. B
15. D 39. D 63. B 87. C 111. B
16. A 40. B 64. A 88. C 112. A
17. A 41. A 65. D 89. A 113. D
18. B 42. C 66. A 90. C 114. B
19. A 43. B 67. A 91. B 115. A
20. C 44. B 68. C 92. D 116. C
21. D 45. B 69. A 93. A 117. C
22. C 46. A 70. C 94. A 118. C
23. C 47. D 71. D 95. B 119. B
24. B 48. C 72. A 96. D 120. C

TRUE OR FALSE

1. FALSE 6. FALSE 11. FALSE 16. FALSE 21. FALSE


2. FALSE 7. FALSE 12. TRUE 17. FALSE 22. FALSE
3. TRUE 8. FALSE 13. TRUE 18. TRUE 23. TRUE
4. FALSE 9. FALSE 14. FALSE 19. TRUE 24. FALSE
5. TRUE 10. TRUE 15. TRUE 20. TRUE 25. FALSE Chapter 7
Audit Objectives and Procedures, Evidence, and Documentation

1. All the information used by the auditor in arriving at the conclusions on which
the audit opinion is based is called a. Audit information
b. Audit evidence
c. Accounting records
d. Corroborating information
According to PSA 500 (Audit Evidence), audit evidence refers to information
used by the auditor in arriving at the conclusions on which the auditor's opinion is
based. It includes both information contained in the accounting records underlying
the financial statements and other information.

2. An entity's accounting records generally include the record of initial entries


and supporting records including e) Confirmations from third parties
f) Information obtained by the auditor from such audit procedures as inquiry,
observation, and inspections.
g) Worksheets and spreadsheets supporting cost allocations
h) Other information developed by, or available to, the auditor to permit
him/her to reach conclusions through valid reasoning.
According to PSA 500, accounting records include the records of initial
accounting entries and supporting records such as the following:

f) Checks and records of electronic fund transfer (EFT)


g) Invoices
h) Contracts
i) The general and subsidiary ledgers, journal entries and other adjustments
to the financial statements that are not reflected in formal journal entries
j) Records such as worksheets and spreadsheets supporting cost allocations,
computations, reconciliations, and disclosures.
Other information that the auditor may use as audit evidence includes the
following:

A. Minutes of meetings
B. Confirmations received from third parties
C. Analyst's reports
D. Comparable data about competitors (benchmarking)
E. Controls manuals
F. Information obtained by the auditor from such audit procedures as inquiry,
observation, and inspection.
G. Other information developed by, or available to, the auditor to reach
conclusions through valid reasoning.
3. Audit evidence comprises
A. Information that supports and corroborates management assertions.

B. Any information that contradicts management’s assertions.


A. I only
B. II only
C. Neither I nor II
D. Both I and II
PSA 500 states that in some cases, the absence of information such as
management’s refusal to provide a requested representation also constitutes audit
evidence that is used by the auditor.

4. As defined in PSA 500, __________ is an individual or organization


possessing the expertise in a field other than accounting or auditing, whose
work in that field is used by the entity in preparing the financial statements.
A. Auditor’s expert
B. Management’s expert
C. Auditor’s internal expert
D. Auditor’s external expert

5. If a management’s expert’s work is used to prepare the information to be used


as audit evidence, the auditor shall
I. Evaluate the competence, capabilities, and objectivity of the
management’s expert.
II. Obtain an understanding of the work of the management’s expert.
III. Evaluate the appropriateness of the management’s expert’s work as audit
evidence for the relevant assertion.
A. I and II only
B. I and III only
C. II and III only
D. I, II, and III only

6. Which of the following statements concerning the management’s expert’s


competence, capabilities, and objectivity is correct?
A. Objectivity relates to the ability of the management’s expert to exercise the
competence in the circumstances.
B. Competence relates to the possible effects that bias, conflict of interest or
the influence of others may have on the professional or business
judgement of the management’s expert.
C. Capability relates to the nature and level of expertise of the management’s
expert.
D. The management’s expert’s competence, capabilities, and objectivity are
important factors in relation to the reliability of any information prepared
by the management’s expert.
The standard states that the competence, capabilities, and the objectivity of a
management’s expert, and any controls within the entity over the expert’s
work, are important factors in relation to the reliability of any information
produced by a management’s expert.

Competence relates to the nature and level of expertise of the management’s


expert.

Capability relates to the ability of the management’s expert to exercise the


competence in the circumstances.

Objectivity relates to the possible effect that bias, conflict of interest or the
influence of others may have on the professional or business judgement of the
management’s expert.

7. Audit evidence is information used to draw reasonable conclusions on which to


base the auditor’s opinion. Audit
evidence is obtained by performing I. Risk
assessment procedure
II. Further audit procedures

A. I only
B. II only
C. Either I or II
D. Both I and II

Risk assessment procedures includes:


1. Inquiries of management, and of others within the entity who in the author’s
judgement may have information that is likely to assist in identifying risk of
material misstatement due to fraud or error.
2. Analytical procedures.
3. Observation and inspection.

Further audit procedures compromise:


1. Test of controls when required by the PSAs or when the auditor has chosen
to do so; and
2. Substantive procedures which include tests of details and substantive
analytical procedures.

8. Which of the following statements concerning audit evidence is correct?


A. Appropriateness is the measure of the quantity of audit evidence.
B. Sufficiency is the measure of the quality of audit evidence, that is, its
relevance and reliability.
C. The quantity of audit evidence needed is affected by its quality and the
risk of misstatement.
D. The sufficiency and appropriateness of audit evidence are not interrelated
PSA 500 states,”The quantity of audit evidence needed is affected by the risk
misstatement (the higher the assessed risks, the more audit evidence is likely to be
required) and also by the quality of such audit evidence (the higher the quantity,
the less may be required).”

Answer A is incorrect because appropriateness is the measure of the quality of


audit evidence.

Answer B is incorrect because sufficiency is the measure of the quantity of audit


evidence.

Answer D is incorrect because the sufficiency and appropriateness of audit


evidence are interrelated.

9. Which of the following statements concerning audit evidence is correct?


A. An audit usually involves the authentication of documentation.
B. Agiven set of procedure may provide audit evidence that is relevant to
certain assertions, but not others.
C. Audit evidence obtained from an independent external source is
always reliable.
D. An entity’s acconting records can be sufficient audit evidence to
support the financial statements.

A given set of audit procedures may provide audit evidence that is relevant to
certain assertions, but not others. For example, inspection of records and
documents related to the collection of receivables after the period end may
provide audit evidence regarding both existence and valuation, although not
necessarily the appropriateness of period-end cutoffs.

Answer A is incorrect because an audit rarely involves the authentication of


documentation, nor is the auditor trained or expected to be an expert in such
authentication.

Answer C is incorrect because audit evidence obtained from an independent


source may not be reliable if the source is not knowledgeable.

Answer D is incorrect because accounting records alone do not provide sufficient


audit evidence in which to base an audit option.

10. Which of the following generalizations does not relate to the reliability of
audit evidence?
A. Audit evidence is more reliable when it is obtained from independent
sources outside the entity.
B. Audit evidence obtained directly by the auditor is more reliable than audit
evidence obtained indirectly or by in
C. Audit evidence that is generate internally is more reliable when the related
controls imposed by the entity are effective.
D. An auditor’s opinion, to be economically useful, is formed within a
reasonable time and based on audit evidence obtained at a reasonable cost.

Cost-benefit considers relate to the sufficiency, not the reliability, of audit


evidence.

PSA 500 gives the following generalizations about the reliability of audit
evidence:

1. Audit evidence is more reliable when it is obtained from independent


sources outside the entity.
2. Audit evidence that is generated internally is more reliable when the
related controls imposed by the entity are effective.
3. Audit evidence obtained directly by the auditor (for example, observation
of the application of a control) is more reliable than audit evidence
obtained indirectly or inference (for example, inquiry about the application
of a control).
4. Audit evidence is more reliable when it exis in documentary form,
whether paper, electronic, or other medium (for example, a
contemporaneously written record of a meeting is more reliable than a
subsequent oral representation of the matters discussed).
5. Audit evidence provided by original documents is more reliable than audit
evidence provided by photocopies or facsimiles.
11. Which of the following statements concerning audit evidence is false?

A. The auditor uses professional judgement and exercises professional


skepticism in evaluating the quantity and quality of audit evidence, and
thus its sufficiency and appropriateness, to support the audit opinion.
B. The auditor ordinarily finds it necessary to rely on audit evidence that
is persuasive rather than conclusive.
C. In forming the opinion, the auditor does not examine all the
information available because conclusions ordinarily can be reached
by using sampling approaches and other means of selecting items for
testing.
D. The difficulty and expense of obtaining audit evidence concerning an
account balance is valid basis for omitting the test

The standard states that the auditor considers the relationship between the cost of
obtaining audit evidence and the usefulness of the information obtained. However,
the matter of difficulty or expense involved is not itself a valid basis for omitting
an audit procedure for which there is no alternative.

12. In representing that the financial statements are presented fairly, in all
material respects, in accordance with the applicable financial reporting
framework, management implicitly or explicitly makes regarding the
recognition, measurement, presentation, and disclosure of the various
elements of financial statements and related disclosures.

A. Assertions
B. Allegations
C. Conclusions
D. Assurances

As defined in the standard, assertions are representations by management, explicit


or otherwise, that are embodied in the financial statements.

13. The auditor is required to use assertions for classes of transformations,


account balances, and presentation and disclosures of material
misstatement and the design and performance of further audit procedures.
Assertions about classes of transactions include occurrence, completeness,
accuracy, cutoff, and
A. Valuation and allocation
B. Rights and obligations
C. Existence
D. Classification

Assertions about classes of transactions and events for the period under audit
include:
1. Occurrence – transactions and events that have been recorded have
occurred and pertain to the entity.
2. Completeness – all transactions and events that should have been
recorded.
3. Accuracy – amounts and other data relating to recorded transactions and
event have been recorded appropriately.
4. Cutoff – transactions and events have been recorded in the correct
accounting period.
5. Classification – transactions and events have been recorded in the proper
accounts.

Answer A, B, and C are incorrect because valuation and allocating rights and
obligations and existence are assertions about account balances at the period end.

14. The following are assertions about account balances at the period end,
except

The cutoff assertion deals with whether transactions and events have been
recorded in the correct accounting period. Thus, it is an assertion about classes of
transactions and events for the period under audit

Assertions about account balance at the period end include the following:
1. Existence – assets, liabilities, and equity interests exist.
2. Rights and obligations – the entity holds or controls the rights to assets,
and liabilities are the obligations of the entity.
3. Completeness – all assets, liability, and equity interests that should have
been recorded.
4. Valuation and allocation – assets, liabilities, and equity interest are
included in the financial statements at appropriate amounts and any
resulting valuation or allocation adjustments are properly recorded.
15. The following are assertions about presentation and
disclosure.

Existence is an assertion about account balances at the period end. It deals


with whether assets, liabilities, and equity interest exist.
Assertions about presentation and disclosure include:
1. Occurrence and rights and obligations - disclosure events, transactions,
and other matters have occurred and pertain to the entity.
2. Completeness – all disclosures that should have been included in the
financial statements have been included.
3. Classification and understandability – financial information is
appropriately presented and described, and discloses are clearly
expressed.
4. Accuracy and valuation – financial and other information are disclosed
fairly and at appropriate mounts.
16. Which of the following statements concerning the auditor’s use of
assertions is correct?

A. The auditor may combine the assertions about transactions and


events with assertions about account balances.
B. In every audit engagement, the auditor should use the assertions as
described in PSA 500, i.e., the assertions should always fall into
three categories: assertions about classes of transactions and
events, account balances, and presentations and disclosure.
C. There should always be a separate assertion related to cutoff of
transactions and events.
D. The completeness assertion deals only with whether all
transactions and events that should have been recorded.

The auditor may use the assertions as described in PSA 500 or may express them
differently provided all aspects described in the standard have been covered.
Answer B is incorrect because the auditor mmay choose to combine the assertions
about transactions and events with the assertions about account balances.
Answer C is incorrect because there may not be a separate assertion related to
cutoff of transactions and events when the occurrence and completeness
assertions include appropriate consideration of recording transactions in
the correct accounting period.
Answer D is incorrect because the completeness assertions may relate to classes
of transactions and events for the period under audit, account balances at
the period end, and presentation and disclosure.

17. Which of the following statements concerning audit objectives is incorrect?

A. The auditor should resolve any substantial doubt about any of


management’s material financial statements assertions.
B. Selection of test to meet audit objectives should depend upon the
understanding of internal control.
C. There should be a one-to-one relationship between audit objectives and
procedures.
D. Audit objectives should be developed in light of management assertions
about the financial statement elements.

Audit objectives and procedures need not have a one-to-one relationship. Some
audit procedures may satisfy more than one specific audit objective. In some
cases, a combination of audit procedures may be needed to achieve a single audit
objective.

Answer A is incorrect because the auditor should resolve all substantial doubts
about any of management’s material financial statement assertions before an
opinion is rendered

Answer B is incorrect because the auditor’s understanding of internal control and


the assessed level of control risk influence the nature, timing, and extent of the
substantive test procedures.

Answer D is incorrect because, in developing specific audit objectives, the auditor


considers the assertions made by management.

18. The primary difference between an audit of the balance sheet and an audit of
the income statement is that the audit of the income statement addresses the
verification of
A. Cutoffs
B. Authorizations
C. Transactions
D. Costs

Because most income statement elements represent large volumes of transactions,


the auditor’s focus is on the propriety of the accounting for transactions and
events during the period.

Answer A is incorrect because cutoff procedures apply to both statements.

Answer B is incorrect because all transactions must be authorized.

Answer D is incorrect because the auditor is concerned with the cost reflected in
both statements.

19. Which of the following would least likely affect the appropriateness of
evidence available to an auditor?
A. The sampling method employed by the auditor to obtain a sample of such
evidence
B. The relevance of such evidence to the financial statement assertion being
verified.
C. The relationship of the preparer of such evidence to the entity being
audited.
D. The timeliness of such evidence.

Appropriateness is the measure of the quality of audit evidence, that is, its
relevance and reliability. Generally, audit evidence obtained from independent
sources outside the entity, generated under effective controls, or obtained directly
by the auditor is presumed to be the most reliable. The relevance and reliability of
audit evidence will not be affected by the sample selection method as long as the
sample is representative of the population.

Answer B is incorrect because appropriate audit evidence must be relevant and


reliable.

Answer C is incorrect because audit evidence obtained from independent source


outside the entity is generally more reliable than that obtained from the client.

Answer D is incorrect because the timeliness of evidence influences its relevance


and, therefore, its appropriateness.

20. Which of the following forms of documentary evidence would be considered


the most reliable by an auditor?
A. Internally generated
B. Pre-numbered
C. Easily duplicated
D. Authorized by a responsible official

It is generally presumed that externally generated documents are more reliable


than those prepared by the client. However, the reliability of internally generated
documents is enhanced if they are prepared under satisfactory condition of
internal control. Thus, a document that is authorized by a responsible official can
be considered credible because such authorization increases the likelihood that the
underlying transaction is authorized and valid.

Answer A and C are incorrect because internally generated and easily duplicated
documents are readily available to employees who commit fraud.

Answer B is incorrect although the use of pre-numbered documents is an effective


control, such documents may be accessible to employees who commit fraud.

21. The objective of tests of details of transactions performed as substantive tests


is to
A. Attain assurance about the reliability of the accounting system.
B. Evaluate whether management’s policies and procedures operated
effectively.
C. Detect material misstatements in the financial statements.
D. Comply with Philippine Standards on Auditing (PSAs).

Substantive tests are performed primarily to detect material misstatements at the


assertion level. These include tests of details of classes of transactions, account
balances, and disclosures and substantive analytical procedures.

Answer A and B are incorrect because test of controls are performed to test the
reliability of the accounting system and to determine whether management’s
policies and procedures operated effectively.

Answer D is incorrect because the auditor may apply a variety of audit procedures
and is not required to perform tests of details of transactions to comply with
PSAs.

22. In testing the existence assertion for an asset, an auditor ordinarily works from
the
A. Potentially unrecorded items to the financial statements.
B. Financial statements to the potentially unrecorded items.
C. Supporting evidence to the accounting records. D. Accounting records to
the supporting evidence.

Answers A, B, and C are incorrect because these procedures relate to the


completeness assertion. The completeness assertion concerns whether all assets,
liabilities, and equity interests of the entity that should have been recorded.

23. In determining whether transactions have been recorded, the direction of the
audit testing should be from the
A. General journal entries
B. Original source documents
C. General ledger balances
D. Adjusted trial balance

The completeness assertion deals with whether all transactions and events that
should have been recorded. Thus, determining whether transactions have been
recorded is testing the completeness assertion. In applying this test, the auditor
ordinarily begins with the original source documents and traces them to the
appropriate accounting records to determine whether the underlying transactions
were properly recorded.
Answers A, C, and D are incorrect because beginning with the accounting records
is unlikely to provide evidence about unrecorded transactions.

24. Which of the following audit procedures consists of looking at the process or
procedure being performed by others?
A. Observation
B. Inspection of records and documents
C. Inspection of tangible assets
D. Inquiry

As stated in PSA 500, observation consist of looking at a process or procedure


being performed by others. An example is observation of the counting of
inventories by the entity’s personnel.

Answer B is incorrect because inspection consist of examining records or


documents, whether internal or external, in paper form, electronic form, or other
media.

Answer C is incorrect because inspection of tangible assets consist of physical


examination of the assets.

Answer D is incorrect because inquiry consist of seeking information of


knowledgeable persons, both financial and nonfinancial, throughout the entity or
outside the entity.

25. Which of the following elements ultimately determines the specific auditing
procedures that are necessary in the circumstances to afford a reasonable basis
for an opinion?
A. Materiality
B. Audit risks
C. Auditor judgement
D. Reasonable assurance

The auditor is required to obtain sufficient appropriate audit evidence to afford a


reasonable basis for an opinion regarding the financial statements. The auditor
uses his/her professional judgement in determining the nature, timing, and extent
of specific audit procedures to be applied to obtain sufficient appropriate audit
evidence.

Answer A and B are incorrect because material and audit risk are considered in
exercising professional judgement.
Answer D is incorrect because the auditor exercises professional judgement in
determining whether reasonable assurance that the financial statements are free of
material misstatement has been obtained.
ANALYTIC PROCEDURES

26. The auditor should apply analytical procedures

I. As risk assessment procedures. II. As


substantive procedures.
III. In the overall review at the end of the audit.

A. I and II only
B. II and III only
C. I and III only
D. I, II, and III

The auditor should apply analytical procedures as risk assessment procedures to


obtain an understanding of the entity and its environment and in the overall
review at the end of the audit. Analytical procedure may also be applied as
substantive test procedures.

27. Analytical procedures include the consideration of comparisons of the entity’s


financial information with

I. Comparable information for prior periods. II. Anticipated


result of the entity.
III. Similar industry information.

A. I and II only
B. II and III
C. I and III
D. I, II, and II

Analytical procedures include the consideration of comparison of the entity’s


financial information with, for example:

• Comparable information of prior


• Anticipated result of the entity, such as budgets or forecast, or expectations
of the auditor, such as an estimation of depreciation.
• Similar industry information, such as a comparison of the entity’s ratio of
sales to accounts receivable with industry averages or with other entities of
comparable size in the same industry.
Moreover, analytical procedures may include consideration of relationship:

• Among elements of financial information that would be expected to


conform to a predictable pattern based on the entity’s experience, such as
gross profit ratios.
• Between financial information and relevant non-financial information,
such as payroll cost to number of employees.

28. Which of the following should be considered by the auditor when designing
and performing analytical procedures as substantive procedures?

I. Suitability of using substantive analytical procedures given the


assertions.
II. The reliability of the data, whether internal or external, from which the
expectation of recorded amount or ratios is developed.
III. Whether the expectation is sufficiently precise to identify a material
misstatement at the desired level of assurance
IV. The amount of any difference of recorded amounts from expected values
that is acceptable.

A. I, II, and III only


B. I III, and IV only
C. II, III, and IV only
D. I, II, III, and IV

29. The following statements relate to the use of analytical procedures as


substantive procedures. Which is false?
A. Substantive analytical procedures are applicable when there is only a small
volume of transactions.
B. The application of substantive analytical procedures is based on the
expectation that relationships among data exist and continue in the
absence of known conditions to the contrary.
C. The presence of relationships among data provides evidence as to the
completeness, accuracy, and occurrence of transactions captured in the
information produced by the entity’s information system.
D. Reliance on the results of substantive analytical procedures will depend on
the auditor’s assessment of the risk that the analytical procedures may
identify relationships as expected when, in fact, a material misstatement
exists.

According to PSA 520 (Analytical Procedures), substantive analytical procedures


are generally more applicable to large volumes of transactions that tend to be
predictable over time.
30. Which of the following should be considered by the auditor in determining the
suitability of substantive analytic procedures given the assertions?
I. The assessment of the risk of material misstatement.
II. Any test of details directed toward the same assertion.

A. I only
B. II only
C. Both I and II
D. Neither I nor II The standard states that the auditor considers the
understanding of the entity and its internal control, the materiality and likelihood of
misstatement of the items involved, and the nature of the assertion in determining
the suitability of substantive analytical procedures. It states further that substantive
analytical procedures may also be considered appropriate when tests of details are
performed on the same assertion.

31. Reliability of data is influenced by its source and by its nature and is
dependent on the circumstances under which it is obtained. Which of the
following should the auditor consider in determining whether data is reliable for
purposes of designing substantive analytical procedures? I. Source of the
information available.
II. Comparability of the information available.
III. Nature and relevance of the information available.
IV. Controls over the preparation of the information.

A. I, III, and IV only


B. II, III, and IV only
C. I, II, and III only
D. I, II, III, and IV
32. According to PSA 520, when analytical procedures identify significant
fluctuation or relationship that are inconsistent with other relevant information
or that deviate from predicted amounts, the auditor should investigate and
obtain adequate explanations and appropriate corroborative audit evidence.
The auditor’s investigation of unusual fluctuations and relationships ordinarily
begins with inquiries of management, followed by
I. Corroboration of management’s responses.
II. Consideration of the need to apply other audit procedures based on the
results of such inquiries, if management is unable to provide an ex
planation or if the explanation is not considered adequate.
A. I only
B. II only
C. Both I and II
D. Neither I nor II

REVENUE/RECEIPT CYCLE

33. The following are the auditor’s principal objectives in the audit of revenues,
except
A. To determine whether all cash owned by the entity at the balance sheet
date is included on the balance sheet.
B. To determine whether earned revenue has been recorded and recorded
revenue has been earned.
C. To determine whether revenues are reported in the income statement at the
appropriate amounts.
D. To determine whether revenues are properly classified, described, and
disclosed in the financial statements, including notes, in conformity with
an applicable financial reporting framework.

Determining whether all cash owned by the entity at the end of the reporting
period is included on the statement of financial position is not part of the audit of
sales and other revenues. A separate audit program is ordinarily prepared for the
audit of cash, including bank balances.

34. Auditors are often concerned with the possibility of overstatement of sales and
receivables. However, management may also have reasons for understating
these balances. Which of the following would explain understatement of sales
and
receivables?
I. To avoid paying taxes.
II. To window-dress the financial statements.
III. To meet budgets and forecasts.
A. I only
B. II only
C. I and III only D. I, II, and III

One way of avoiding tax liability is by not recording and reporting all sales and
receivables.

Answers B, C D are incorrect because management may attempt to overstate sales


and receivables to window-dress the financial statements or meet budgets and
forecast.
35. In the audit of which of the following general ledger accounts will tests of
controls be particularly appropriate?
A. Bank charges
B. Equipment
C. Bonds payable
D. Sales

Because of the large volume of sales transactions, test of controls are particularly
appropriate to restrict substantive testing, provided that the evidence obtained
supports an assessment of control risk at less than the maximum.

Answer A is incorrect because bank charges can be easily verified by examining


the monthly bank statement.

Answer B is incorrect because the small volume of equipment transactions and the
ease of verifying its physical existence and computing depreciation may make
evaluation of the effectiveness of controls inefficient.

Answer C is incorrect because the infrequency of transactions the availability of


bond contracts may make it inefficient to perform tests of controls.

36. Which of the following might be detected by an auditor’s review of the


entity’s sales cutoffs? A. Inflated sales for the year.
B. Lapping of year-end accounts receivable.
C. Unrecorded sales discounts.
D. Excessive goods returned for credit.

An auditor’s view of the client’s sales cutoff involves examining recorded sales
for several days (for example, one week) before and after the balance sheet date
and comparing them with sales invoices and shipping documents. By performing
this substantive procedure, the auditor may detect the recording of sale in a period
other that in which title passed. The completeness and cutoff assertions are
directly related to the auditor’s cutoff test.

Answer B is incorrect. Lapping is the concealment of a cash shortage resulting


from delays in recording cash receipts. This may be detected by confirming
receivable balances and by comparing bank validated deposit slips to the entries in
the accounts receivables subsidiary ledger.

Answer C is incorrect because unrecorded sales discounts may be revealed by


examining cash collections.

Answer D is incorrect because an auditor’s review of the client’s sales cutoff does
not include examination of sales returns.

37. An auditor most likely would review a client’s periodic accounting for the
numerical sequence of shipping documents and sales invoices to support
management’s financial statement assertion of A. Existence
B. Rights and obligations
C. Completeness
D. Valuation and allocation

An auditor’s test of the numerical sequence of shipping documents and sales


invoices may reveal omitted items. This procedure addresses management’s
completeness assertion that deals with whether all transactions, assets, liabilities,
and equity interests that should have been recorded.

Answer A is incorrect because existence is an assertion that all assets, liabilities,


and equity interests exist.

Answer B is incorrect because the rights and obligations assertion addresses


whether the entity holds or controls the rights to assets, and liabilities are the
obligations of the entity.
Answer d is incorrect because the valuations and allocation assertion concerns
whether assets, liabilities, and equity interests are included in the financial
statements at appropriate amounts and any resulting valuation or allocation
adjustments are appropriately recorded.

38. If the objective of a test of details of transactions is to detect overstatements of


sales, the auditor’s direction of
testing should be from the
A. Cash receipts journal to the sales journal.
B. Accounting records to the source documents.
C. Source documents to the account records.
D. Sales journal to the cash receipt journal.

The existence of source documents such as sales invoices and shipping documents
indicates that the sales are valid.
Thus, the proper direction of testing should be from the recorded sales
transactions to the source documents. This procedure may reveal overstatements
of sales that are likely to result from entries in the sales account with no
supporting documentation.

Answer A and D are incorrect because the cash receipts journal and the sales
journal are not source documents.

Answer c is incorrect because tracing a sample of source documents to the


accounting records may reveal unrecorded sales, not overstatements of sales.

39. Cutoff test designed to detect credit sales made before the end of the year that
have been recorded in the subsequent year provide assurance about
management’s assertion of
A. Accuracy
B. Classification
C. Rights and obligations
D. Cutoff

The cutoff assertion addresses whether transactions and events have been recorded
in the correct accounting period. This assertion is tested by examining recorded
sales for several days before and after the balance sheet date.

Answer A is incorrect because accuracy is an assertion that amounts and other


data relating to recorded transactions and events have been recorded appropriately.

Answer B is incorrect because the classification assertion addresses whether


transactions and events have been recorded in the proper accounts.

Answer c is incorrect because right and obligations assertion concerns whether the
entity controls or holds the right to assets, and liabilities are the obligations of the
entity.

40. An auditor most likely would limit substantive audit tests of the sales
transactions when control risk is assessed as low for the occurrence assertion
concerning sales transactions and the auditor has already gathered evidence
supporting

A. Beginning and ending inventory balances.


B. Cash receipt and accounts receivables.
C. Cutoffs of sales and purchases.
D. Shipping and receiving activities.

The occurrence assertion concerns whether transactions and events that have been
recorded have occurred and pertain to the entity.

Because cash receipt and accounts receivable are directly related to sales (a cash
sale is debited to cash and credited to sales; a sale on account is debited to
accounts receivable and credited to sales), evidence related to these accounts
provides assurance about the occurrence of sales transactions.

Answer A is incorrect because the beginning and ending inventory balances do not
have a direct relationship with sales transactions.

Answer C and D are incorrect because cutoffs of purchases and receiving


activities are not related to sales.

41. Tracing bills of lading to sales invoice provides evidence that


A. Invoice sales were shipped.
B. Recorded sales were shipped.
C. Shipments to customers were recorded as sales
D. Shipments to customers were invoiced.

Tracing shipping documents (such as bills of lading) to sales invoices provides


direct proof that the goods shipped were billed to customers.

Answers A and B are incorrect because in determining whether invoiced and


recorded sales were shipped, the proper direction of testing is from the sales
records to the shipping documents

Answer C is incorrect because to determine whether shipments were recorded as


sales, the auditor would have to trace the shipping documents to the sales records.

42. The auditor finds a situation in which one person has the ability to collect
receivables, make deposits, issue credit memos, and record receipt of
payments. Which of the following audit procedures would be most effective in
discovering fraud in this scenario?
A. A perform a detailed review of debits to sales discounts, sales returns and
allowances, or other debit accounts, excluding cash posted to the cash
receipt journal.
B. Take a sample of bank deposits and trace the detail in cash bank deposit
back deposit back to the entry in the cash receipts journal.
C. Send negative confirmations to all outstanding accounts receivable
customers.
D. Send positive confirmations to a random selection of customers.

A theft of cash collections can be concealed by debiting a noncash account (for


example, sales discounts, sales returns and allowances, or allowance for bad
debts). Thus, the most effective procedure is to review debits posted to these
accounts.

Answer A is incorrect because bank deposits will agree with the entries in the cash
receipts journal. The theft of cash is concealed by debiting a non-cash account
when recording cash collections

Answers C and D are incorrect because a stolen cash collection is recorded by


debiting a non-cash account and crediting accounts receivables. Thus, customers
would be unaware of the fraudulent act of the employee because the balance per
their records will agree with the amount provided in the confirmation request.

43. Which of the following most likely would give the most assurance concerning
the valuation and allocation assertion of accounts receivable?
A. Vouching amounts in the subsidiary ledger to details on shipping
documents.
B. Inquiring about receivables pledged under load agreements.
C. Assessing the allowance for bad debts for reasonableness.
D. Comparing receivable turnover ratios with industry statistics for
reasonableness.

Valuation and allocation is an assertion that assets, liabilities, and equity interests
are included in the financial statements at appropriate amounts and any resulting
valuation or allocation adjustments are appropriately recorded.

Management asserts that accounts receivable are stated at net realizable value
(gross accounts receivable minus allowance for bad debts, allowance for sales
returns, allowance for sales discounts, etc.). Hence, assessing the reasonable-
valuation of the accounts receivable.

Answer A is incorrect because vouching amounts in the subsidiary ledger to


details on shipping documents provides evidence about the occurrence of sales
transactions.

Answer B is incorrect because inquiring about receivables pledged under loan


agreements relates to presentation and disclosure assertion.

Answer D is incorrect because comparing receivable turnover ratios with industry


statistics for reasonableness provides evidence about the completeness assertion.

44. Which of the following is not a principal objective in auditing accounts


receivable?
A. To determine whether receivables are carried at their net realizable value.
B. To determine whether receivables are properly classified, described, and
disclosed in the financial statements, including notes, in accordance with
the applicable financial reporting framework.
C. To determine whether the entity has real claims in all receivables on the
balance sheet.
D. To determine whether the accounts are collected by the balance sheet date.
Accounts receivable are claims against customer arising from the sales of goods
or services. In auditing this account, the auditor should not anticipate collections
at the balance sheet date.

45. A large university has relatively ineffective internal control. To obtain


assurance that all tuition revenue has been recorded, the auditor should

A. Confirm a sample of tuition payments with the students.


B. Prepare a year-end bank reconciliation.
C. Compare business office revenue records with registrar’s office records of
students enrolled.
D. Observe tuition payment procedures on a surprise basis.

An auditor who seeks assurance that all tuition revenue has been recorded should
perform substantive procedures such as test of details and substantive analytical
procedures. For example, the auditor may obtain analytical evidence by
comparing business office revenue records with the registrar’s office records of
students enrolled.

Answers A and B are incorrect because confirming tuition payment with the
students and preparing year-end bank reconciliation will not detect unrecorded
stolen cash receipts.

Answer D is incorrect because observing tuition payment procedures is a test of


controls and does not provide evidence about the completeness of the revenue
account.

46. The process of obtaining and evaluating audit evidence through a direct
communication from a third party in response to a request for information
about a particular item affecting Assertions made by management in the
financial statements is called

A. Re-performance
B. External confirmation
C. Inquiry
D. Recalculation

According to PSA 500, confirmation is a specific type of inquiry which involves


obtaining a representation of information or of an existing condition directly from
a third party.

PSA 505 (External Confirmations) gives the following examples of situations


where external confirmations may be used:
• Bank balances and other information from bankers.
• Accounts receivable balances.
• Inventories held by third parties at bonded warehouse for processing or on
consignment.
• Property title deeds held by lawyers or financiers for safe custody or as
security.
• Investments purchased from stockbrokers but not delivered at the balance
sheet date.  Loans from ledgers.
• Accounts payable balances.
Answer A is incorrect because re-performance is the auditor’s independent
execution of procedures or controls that were originally performed as part of the
entity’s internal control, either manually or through the use of CAATs. An
example is reperforming the aging of accounts receivable.

Answer C is incorrect because inquiry consists of seeking information of


knowledgeable persons, both financial or nonfinancial, throughout the entity or
outside the entity.

Answer D is incorrect because recalculation consists of checking the


mathematical accuracy of documents or records. For example, the auditor may
obtain an electronic file from the entity and check the accuracy of the
summarization of the file by using CAATs.

47. The confirmation of customer’s accounts receivable rarely provides reliable


evidence about the valuation assertion because

A. Customers may not be inclined to report understatement errors in their


accounts.
B. Auditors typically select many accounts with low recorded balances to be
confirmed.
C. It is not practicable to ask the customer to confirm detailed information
relating to its ability to pay the account.
D. Recipients usually respond only if they disagree with the information on
the request.

PSA 505 states that external confirmation does not normally provide all the
necessary audit evidence relating to the valuation assertion since it is not
practicable to ask the debtor to confirm detailed information relating to its ability
to pay the account.

Answer A is incorrect because non-reporting by customers of understatement


errors in their account does not provide reliable evidence about the completeness
assertion.

Answer B is incorrect because auditors typically select accounts with material


balance since confirmation provides strong evidence regarding the existence of
accounts receivable as at a certain date.

Answer D is incorrect because a positive external confirmation request asks the


respondent to reply to the auditor in all cases, that is, whether or not the
respondent agrees with the information provided in the request.
48. Auditors may use positive and/or negative forms of confirmation requests. An
auditor most likely will use

A. The negative form for small balances.


B. The positive form, when the combined assessed level of inherent and
control risk for related assertions is acceptably low, and the negative form
when it is unacceptably high.
C. The positive form to confirm all balances regardless of size.
D. A combination of the two forms, with the positive form used for trade
balances and the negative form for other balances.

According to PSA 505, a negative external confirmation request ask the


respondent to reply only in the event of disagreement with information provided
in the request.

Negative confirmation request may be used when:

• The combined assessed level of inherent and control risk is low.


• A large number of small balances is involved.
• A substantial number of errors is not expected.
• The auditor has no reason to believe that respondents will disregard theses
requests.

Answer B is incorrect because the positive form is used when the combined
assessed level of inherent and control risk is high.

Answer C is incorrect because the negative form is used when a large number of
small balances is involved.

Answer D is incorrect because the type of confirmation request does not depend
on nature of the balances.

49. The following statements relate to the use of negative confirmation request.
Which is true?

A. Negative confirmation request are effective when detection risk is low.


B. Unreturned negative confirmation request indicate that alternative
procedures are necessary.
C. Unreturned negative confirmation request rarely provide significant
explicit evidence.
D. Negative confirmation request are effective when understatements of
account balances are suspected.
PSA 505 states that, “A negative external confirmation request ask the respondent
to reply only in the event of disagreement with the information provided in the
request. However, the auditor remains aware that there will be no explicit audit
evidence that intended third parties have received the confirmation request and
verified that the information contained therein is correct.”

Answers A and D are incorrect because positive confirmation request are more
effective when detection risk is low and when understatements of account
balances are suspected.

Answer B is incorrect because unreturned negative confirmation request indicate


that the recipients agree with the information provided in the request, thus
alternative procedures may be unnecessary.

50. In confirming accounts receivable, an auditor decided to confirm customers’


account balances rather than individual invoices. Which of the following most
likely would be more included with the client’s confirmation letter?

A. An auditor-prepared letter explaining that nonresponse may cause an


inference that the account balance is correct
B. An auditor-prepared letter requesting the customer to supply missing and
incorrect information directly to the auditor.
C. A client-prepared letter reminding the customer that a nonresponse will
cause a second request to be sent.
D. A client-prepared statement of account showing the details of the
customer’s account balance.

A client-prepared statements of account that shows the detail of the receivable


balance would make it easier for the customer to reconcile the account balance
and thus, may increase response rates.

Answers A and B are incorrect because a confirmation of the account balance


must be requested by the client, not the auditor.
Answer C is incorrect because a confirmation request should not contain any form
of threat to customers.

51. Which of the following statements would an auditor most likely add to the
negative form of confirmations of accounts receivable to encourage timely
consideration by the
recipient?

A. “This is not a request for payment; remittances should not be sent to our
auditors in the enclosed envelope”
B. “If you do not report any differences within 15 days, it will be assumed
that this statement is correct.”
C. “The following invoices have been selected for confirmation and represent
amounts that are overdue
D. “Report any differences on the enclosed statement directly to our auditors;
no reply is necessary if this amount
agrees with your records.”

Indicating a time limit on the negative form of confirmation request may


encourage timely consideration by the recipient.

Answer A and D are incorrect because the statements do not encourage a timely
response.

Answer C is incorrect because stating that the amount is overdue could discourage
a customer from responding promptly.

52. An auditor confirms a representative number of open accounts as of December


31 and investigates respondents’ exceptions and comments. By this procedure,
the auditor is most likely to learn of which of the following?

A. One of the cashier has been covering a personal embezzlement by lapping.


B. The credit manager has misappropriated remittances from customers
whose accounts have been written off.
C. One of the sales clerk has not been preparing charge slips for credit sales
to family and friends.
D. One of the computer processing control clerks has been removing all sales
invoices applicable to his account from data fil.

Lapping is the concealment of a cash shortage resulting from the delay in


recording cash receipts. For example, the theft of cash receipt from one customer
is concealed by crediting that customer’s account when another customer pay
his/her account. Lapped accounts can be detected through confirmation of
receivable balances because the customer will not agree with the balance provided
in the request.

Answer B is incorrect because written off accounts are no longer open accounts.

Answer C is incorrect because there will be no account balance to be confirmed if


a charge slip has not been prepared by the sales clerk.

Answer D is incorrect because there will be no account balance if the sales


invoices are not processed.
53. During the process of confirming receivables as of December 31, 200A, a
positive confirmation was returned indicating the “balance owed as of
December 31 was paid on January 6, 200B.” The auditor will most likely

A. Reconfirm the zero balance as of January 7, 200B


B. Verify that the amount was received.
C. Determine whether a cash discount was taken by the customer.
D. Determine whether there were any changes in the account between
January 1 and January 6, 200B.

When a response indicates that the balance was already paid, the auditor should
trace the customer’s remittance to verify that the payment was actually received
by the client.

Answer A is incorrect because there is no requirement to reconfirm the balance.

Answer C is incorrect because the auditor’s primary focus is the customer’s


confirmation of the year-end balance, not whether a cash discount was taken.

Answer D is incorrect because the auditor’s objective is to confirm the year-end


receivable balance, not the transactions subsequent to year-end.

54. A company was computerized sales and cash receipts journal. The computer
programs for these journals have been properly debugged. The auditor’s
examination of the accounting records revealed that the total of the accounts
receivable subsidiary accounts differs materially from the accounts receivable
control account. This discrepancy could indicate

A. Credit memoranda being improperly recorded.


B. Statements being intercepted prior to mailing.
C. Lapping of receivables.
D. Receivables not being properly aged.

Based on approved credit memoranda, sales returns and allowance are recorded
by crediting accounts receivable. The discrepancy between the subsidiary ledger
and the control account could be due to the recording of unauthorized credit
memoranda, assuming sales and cash receipts have been properly recorded.

Answer B is incorrect because, although the interception of customer statements


might be indicative of fraud, such action would not cause the ledger discrepancy.

Answer C is incorrect because lapping result from delays in recording cash


receipts from customers. It would conceal the theft of cash but would not cause
ledger discrepancy to arise.
Answer D is incorrect because no accounting entries are made when aging
receivables.

55. To reduce the risk associated with accepting fax responses to request for
confirmations of accounts receivable, an
auditor most likely would

A. Inspect the faxes for forgeries or alterations and consider them to be


acceptable if none are noted
B. Consider the faxes to be nonresponses and evaluate them as unadjusted
differences.
C. Verify the source and contents of the faxes in telephone calls to the
senders.
D. Examine the shipping documents that provide evidence for the existence
assertion.

PSA 505 (External Confirmations) states that the auditor should consider
validating the source of replies received in electronic format (for example, fax or
electronic mail). The auditor may verify the source and contents of a response in a
telephone call to the purported sender. Moreover, the auditor should consider
requesting the purported sender to mail the original confirmation directly to the
auditor.

Answer A is incorrect because faces may not be signed. IN addition, PSA 500
(Audit evidence) states that an audit rarely involves the authentication of
documentation, nor is the auditor trained as or expected to be an expert in such
authentication.

Answer B is incorrect because replies received in electronic format are considered


valid if the source and contents can be verified.

Answer D is incorrect because the auditor’s purpose is to verify the existence of


receivables by direct communication with the customers.
56. PSA 505 defines confirmation as “the process of obtaining and evaluating
audit evidence through representation of information or an existing condition
directly from a third party in response to a request for information about a
particular item affecting assertions in the financial statements or related
disclosures.” Two assertions for which confirmation of accounts receivable
balances provides primary evidence are
A. Completeness and valuation and allocation.
B. Valuation and allocation and rights and obligations.
C. Rights and obligations and existence.
D. Existence and completeness.
Confirmation of receivable balances directly with customers is most likely to be
effective for the rights and obligations and existence assertions. It provides
reliable evidence that the receivables exist and that the client has the right to
collect such receivables from customers.

Answers A, B, and D are incorrect because confirmation does not provide


sufficient evidence about the completeness and valuation and allocation
assertions.

57. For accounts receivable, negative confirmation is less effective than positive
confirmation because

A. Some recipients may report incorrect balances that require extensive


follow-up.
B. A majority of recipients usually lack the willingness to respond
objectively.
C. The auditor cannot infer that all nonrespondents have verified their
account information.
D. Negative confirmations do not produce evidence that is statistically
quantifiable.
When negative confirmations are used, unreturned requests indicate that the
intended recipients agree with the information provided in the requests. However,
no explicit evidence is provided that the intended recipients received their
requests and verified the information.

Answer A is incorrect because inaccurate reporting of balances by the intended


parties is always a possibility under both forms of confirmation.

Answer B is incorrect because the assumed lack of willingness to respond


objectively would affect both forms of confirmation.
Answer D is incorrect because both forms of confirmation provide quantifiable
audit evidence.

58. Which of the following procedures would an auditor most likely perform for
year-end accounts receivable confirmations when the auditor did not receive
replies to second requests?
A. Inspect the shipping records documenting the merchandise sold to the
debtors.
B. Review the cash receipts journal for the month prior to year-end.
C. Intensify the study of internal control concerning the revenue cycle.
D. Increase the assessed level of detection risk for the existence assertion.

When no response is received to a second positive confirmation request, the


auditor should perform alternative procedures such as examination of subsequent
cash receipts, examination of shipping documents or other client documentation to
provide audit evidence for the existence assertion, and examination of sales for
several days before and after the balances sheet date to provide audit evidence for
the cutoff assertion.

Answer B is incorrect because the auditor should examine subsequent cash


receipts.

Answer C is incorrect because nonresponses do not necessarily indicate that the


client’s internal control is weak.

Answer D is incorrect because the auditor assesses inherent and control risks, not
detection risk. The level of detection risk acceptable to the auditor depends on the
combined assessed level of inherent and control risks.

59. Which of the following is the most effective procedure for determining
collectability of an account receivable?
A. Confirmation of the account
B. Review of the subsequent cash collections
C. Review of authorization of credit sales to the customer and the previous
history of collections.
D. Examination of the related sales invoice(s).

The most effective procedure of determining collectability is to obtain evidence


that it was subsequently collected. Collectability relates to the valuation and
allocation assertion and is primarily considered in determining the adequacy of
the allowance for bad debts.

Answer A is incorrect because confirmation provides audit evidence that the


receivable exists and that the customer acknowledges the liability. However,
confirmation does not provide complete assurance that the amount is collectible.

Answer C is incorrect because while the client’s history of collections may be a


good indicator of collectability, it is not as good as subsequent cash collections.

Answer D is incorrect because examination of the related sale invoice(s) provides


audit evidence about the receivable’s validity, not collectability.

60. An auditor reconciles the total of the account receivable subsidiary ledger to
the general ledger control account as of December 31. By this procedure, the
auditor is most likely to learn which of the following?

A. An account balance is past due and should be written off.


B. A December invoice was improperly computed.
C. A December check from a customer was posted in error to the account of
another customer with a similar name.
D. An opening balance in the subsidiary ledger account was improperly
carried forward from the previous accounting period.
An auditor's reconciliation of the subsidiary ledger total to the general ledger will
identify erroneous balances in the subsidiary ledger at the end of the previous
period carried forward to the current period.
Answer A is incorrect because aging receivables will most probably detect
uncollectible accounts.
Answer B is incorrect because reconciliation of the subsidiary ledger and general
ledger control account will not detect an improper amount posted in both records
if such amount is based on the same erroneous document. The auditor should
consider recalculation of invoice amounts to detect the error.
Answer C because confirmation will most likely detect posting of customer
payment to a wrong account. This error will not affect the subsidiary ledger and
general ledger control balances.

61. Once a CPA has determined that accounts receivable have increased because
of slow collections in a tight money environment, the CPA is likely to

d) Review the entity's credit and collection policy.

e) Expand tests collectability.

f) Review the going concern ramifications.

g) Increase the balance in the allowance for bad debts account.


The auditor should determine the effect of slow collections of accounts receivable
on the allowance for bad debts account. The auditor should expand his/her test of
collectability such as a review of collections subsequent to the balance sheet date.
Answer A is incorrect because the auditor's review of the credit and collections
policy is just a part of expansion of the tests of collectability.
Answer C is incorrect because slow collections of accounts receivable does not
necessarily raise doubts about the ability of the client to continue as going
concern.
Answer D is incorrect because the allowance balance need not be increased
because of slow collections if the accounts are expected to be collectible as in
prior periods.

62. All of the following are examples of substantive procedures to verify the
valuation of net accounts receivables, except the
A. Comparison of the allowance for bad debts with past records.

B. Re-computation of the allowance for bad debts.

C. Inspection of the aging schedule and credit records of past due accounts.
D. Inspection of accounts for current versus noncurrent status in the statement
of financial position.
The inspection of accounts for current versus noncurrent status tests
management's assertion about presentation and disclosure, not valuation.
Answers A, B, and C are incorrect because comparison for allowance for bad
debts with past records, re-computation of the allowance for bad debts, and
inspection of the aging schedule and credit records of past due accounts are tests
of management's valuation assertion about the account's balance.

63. The most likely reason for the auditor to be concerned about the valuation of
cash is that A. The proof of cash cannot be reconciled.

B. The client uses a checking account.

C. Both currencies and negotiable securities are on hand.

D. The client has foreign currency accounts.


The audit objective of valuation is normally of minimum concern during the audit
of cash. However, the client's foreign currency must be converted to Philippine
pesos using the current exchange rate. Changes in the exchange rates and
restriction on the client's foreign accounts may affect the valuation of these
accounts.
Answers A and C because a proof of cash that cannot be reconciled and currency
and negotiable securities that are both on hand affect the auditor's concern about
the existence of cash, not its valuation.
Answer B is incorrect because most entities maintain checking accounts for cash
disbursements.

64. When counting cash on hand, the auditor must exercise control over all cash
and other negotiable assets to prevent
A. Theft
B. Deposits in transit
C. Substitution
D. Irregular endorsement
There should be simultaneous verification of cash and other negotiable assets to
prevent the likelihood of converting negotiable assets to cash to conceal a cash
shortage.
Answers A and D are incorrect because simultaneous verification does not directly
prevent theft or irregular endorsement except during the time the auditor performs
the verification procedures.
Answer B is incorrect because deposits in transit (collections recorded in the
client's book but are not yet credited by the bank) normally arise from cash
transactions.

65. The best evidence regarding year-end bank balances is documented in the A.
Bank reconciliations
B. Interbank transfer schedule
C. Cash in bank lead schedule
D. Cut-off bank statement
A bank reconciliation statement is a schedule that explains any differences
between the bank statement balance and the cash balance per the company's
records. The adjusted cash balance is the amount of cash reported in the financial
statements. Thus, a bank reconciliation statement provides direct evidence of the
year-end bank balances.
Answer B is incorrect because an interbank transfer schedule shows only the
transfer of funds between banks.
Answer C is incorrect because a cash in bank lead schedule shows the components
of the amount of cash included in the working trial balance.
Answer D is incorrect because a cutoff bank statements reports cash transactions
for a short period after the balance sheet date and is used by the auditor to verify
year-end reconciling items such as deposits in transit and outstanding checks.

66. Which of the following sets of information does an auditor usually confirm on
one form?
A. Accounts receivable and accrued interest receivable
B. Cash in bank and collateral for loans
C. Accounts payable and purchase commitments
D. Inventory on consignment and contingent liabilities.
The standard bank confirmation form is used by the auditors to confirm the cash
in bank balance and request loan information such as balance, maturity date,
interest rate, and description of collateral.
Answers A, C, and D are incorrect because accounts receivable and accrued
interest receivable, accounts payable and purchase commitments, and inventory
on consignments and contingent liabilities are pairs of information that are not
usually confirmed on one form.

67. An auditor ordinarily sends a standard confirmation request to all banks with
which the client has done business during the year under audit, regardless of
the year-end balance. The purpose of this procedure is to
A. Detect kiting activities that may otherwise not be discovered.
B. Provide the data necessary to prepare a proof of cash.
C. Request that a cutoff bank statement and related checks be sent to the
auditor.
D. Seek information about other deposit and loan amounts that come to the
attention of the institution in the process of completing the confirmation.
The bank confirmation form requests information about cash in bank and direct
liabilities of the entity to the bank.
Answer A is incorrect because kiting can be detected by preparing an interbank
transfer schedule for a few days before and after the balance sheet date to
determine whether fund transfer between banks have been recorded in the proper
periods.
Answer B is incorrect because the cash records and the monthend bank statement
provide data for the preparation of the proof of cash.
Answer C is incorrect because a separate letter should be prepared by the client to
request for a cutoff bank statement.

68. Which of the following items is not requested on a standard bank account
balance confirmation form? A. The principal paid on a direct liability
B. Maturity date of a direct liability
C. Description of collateral for a direct liability.
D. The interest rate of a direct liability.
The bank confirmation form requests information about account number and
description, balance, maturity date, interest rate, date through which interest is
paid, and description of collateral. The auditor is not concerned with the amount
paid on a direct liability.

69. Which of the following is not considered an objective of the audit of cash?
A. Cash is stated at its realizable value.
B. Compensating cash balances are reported as other current assets.
C. Cash is properly classified, described, and disclosed in the financial
statements, including notes, in accordance with the applicable financial
reporting framework.
D. The client has ownership rights in the reported cash.
Compensating balance are classified according to the appropriate classification of
the related borrowing, either current or noncurrent.

70. The purpose of a proof of cash is to


A. Validate that the client's bank did not make an error during the period
being examined.
B. Confirm that the client has properly separated the custody function from
the recording function with respect to cash.
C. Proof that the client's year-end balance of cash is fairly stated.
D. Determine whether any unauthorized disbursements or unrecorded
deposits were made for the given time period.
A proof of cash is a four-column schedule that reconciles bank balances for two
successive months (first and last columns) and cash receipts and disbursements
for the period (middle columns). It can be used to detect unauthorized
disbursements or unrecorded deposits for the period.
Answer A is incorrect because detection of bank errors is not the primary reason
why a proof of cash is prepared.
Answer B is incorrect because the auditor should obtain knowledge about internal
control relevant to cash transactions to determine whether the custody and the
recording functions are properly segregated.
Answer C is incorrect because a proof of cash reconciles cash balances and
transactions for two successive months of only one bank account. Other
substantive procedures must be performed to examine other cash accounts (for
example, cash on hand) to obtain audit evidence about the fairness of the cash
balance.

71. An internal auditor would be concerned about the possibility of fraud if


A. Only one person has access to the petty cash fund.
B. Cash receipts, net of the amounts used to pay petty cashtype expenditures,
are deposited in the bank daily.
C. The monthly bank statement reconciliation is performed by the same
employee who maintains the perpetual inventory records.
D. The accounts receivable subsidiary ledger and accounts payable subsidiary
ledger are maintained by the same person.
From the standpoint of a good internal control, all cash receipts should be
deposited intact daily. Using cash receipts to pay petty cash expenditures increases
the possibility of making unauthorized removal of cash before deposit. An imprest
petty cash fund should be maintained for small disbursements.
Answer A is incorrect because accountability for petty cash fund is established if
only one person, designated as the petty cash custodian, has access to the petty
cash fund.
Answers C and D are incorrect because the functions described are not
incompatible and thus, can be performed by the same person. Incompatible
functions are those that will allow a person both to commit and conceal errors or
fraud.

EXPENDITURES/ DISBURSEMENT CYCLE

72. Purchase cutoff procedures test the completeness assertion. An entity should
include goods in inventory if it
A. Has paid for it
B. Holds legal title to the goods
C. Has physical possession of the goods
D. Has sold the goods
A purchase cutoff test is a substantive procedure to obtain audit evidence about
whether purchases are recorded in the appropriate period. An entity should include
goods in the inventory and recognize a liability in the period in which acquired
title to the goods.
Answer A is incorrect because goods purchased on account need not be paid for
by the cutoff date.
Answer C is incorrect because title to the goods can pass before actual delivery to
the buyer, for example, in transit goods purchased under FOB shipping point term.
Answer D is incorrect because goods already sold should be excluded from
inventory.

73. In auditing accounts payable, an auditor's procedures will most likely focus
primarily on management's assertion on
A. Existence
B. Valuation and allocation
C. Completeness
D. Presentation and disclosure
The primary audit risk for liabilities, including accounts payable, is the possible
understatement of the account. Thus, in auditing accounts payable, the auditor will
most likely focus on the completeness assertion.
Answer A is incorrect because the risk that reported liabilities do not exist is
generally lower than the risk that the balance is understated.
Answer B is incorrect because the risk that accounts payable are not reported at
appropriate amounts is less than the risk that the balance is understated.
Answer D is incorrect because the risk that the accounts payable are not properly
presented and disclosed in the financial statement is not as high as the risk that
they are materially understated.

74. Which of the following primary audit test to determine if accounts payable are
valued properly?
A. Vouching accounts payable in supporting documentation.
B. An analytical procedure.
C. Verification that the accounts payable are recorded as current liability in
the balance sheet.
D. Examination of cash disbursements subsequent to year-end.
To determine if accounts payable are properly valued, the auditor will vouch a
sample of recorded accounts payable to the supporting documentation of purchase
- namely, purchase order, receiving order, and vendor invoice.
Answer B is incorrect because analytical procedures are performed to provide an
overall review of the accounts, not to test the valuation of specific accounts.
Answer C is incorrect because determining whether accounts payable are
classified as current liability addresses the presentation and disclosure assertion
rather than valuation.
Answer D is incorrect because examining subsequent cash disbursements may
reveal unrecorded liabilities but does not addresses the valuation of those
liabilities that are recorded on the company's books.

75. Which of the following procedures is least likely to be performed before the
balance sheet date? A. Search for unrecorded liabilities
B. Confirmation of accounts receivable
C. Attendance at the physical inventory count
D. Testing internal control over cash
The auditor searches for unrecorded liabilities by reviewing cash disbursements
made after year-end to determine if they pertain to unrecorded liabilities as of the
client's balance sheet date.
Answer B is incorrect because, depending on the combined assessed level of
inherent and control risks for receivables, confirmation may be made before, on,
or after the balance sheet date.
Answer C is incorrect because if the internal control over the inventory is
effective, attendance at the physical inventory count usually can be made during
or at the end of the period under audit.
Answer D is incorrect because the understanding of internal control is usually
obtained prior to the balance sheet date as part of the auditor's risk assessment
procedures.
76. An auditor performs a test to determine whether all merchandise for which the
client was billed was received. The
population for this test consists of all
A. Receiving reports
B. Vendor's invoices
C. Cancelled checks
D. Merchandise received
A supplier bills the buyer through an invoice that describes the item(s), amount
due, and payment terms. To determine whether all merchandise for which the
client was billed was received, the auditor's should trace vendor's invoices to the
related receiving reports.
Answer A is incorrect because tracing receiving reports to vendor's invoices
provides audit evidence about whether all goods received were billed by suppliers.
Answer C is incorrect because trading canceled checks to receiving reports
assures that goods paid for - not goods billed - were received.
Answer D is incorrect because if the direction of testing is from goods received,
merchandise received but not billed will not be detected.

77. Which of the following is a substantive procedure than an auditor would most
likely perform to verify the existence and valuation of recorded accounts
payable?

A. Confirming accounts payable balances with known suppliers who have


zero balances.

B. Investigating the open purchase order file to ascertain that pre-numbered


purchase orders are used and accounted for.

C. Receiving the client's mail, unopened, for a reasonable period of time after
year-end to search for unrecorded vendor's invoices.

D. Vouching selected entries in the accounts payable subsidiary ledger to


purchase orders and receiving reports.
Vouching a sample of entries in the accounts payable subsidiary ledger to
purchase orders and receiving reports provides audit evidence about whether
recorded liabilities exist at a given date and reported at appropriate amounts.
Answer A is incorrect because confirming payables with known suppliers who
have zero balances will probably detect unrecorded liabilities. Hence, this
procedure relates more to management's completeness assertion.
Answer B is incorrect because investigating the open purchase order file to
ascertain that pre-numbered purchase order files are used and accounted for is
most directly related to management's completeness assertion.
Answer C is incorrect because the auditor's search for unrecorded vendor's
invoices is most directly related to management's completeness assertion.

78. When using confirmation to provide evidence about the completeness


assertion for accounts payable, the appropriate population most likely is

A. Amounts recorded in the accounts payable subsidiary ledger.

B. Vendors with whom the entity has previously done business.

C. Invoice filed in the entity's open invoice file.

D. Payees' checks drawn in the month subsequent to the balance sheet date.
The completeness assertion for the account payable addresses whether all amounts
due to vendors that should have been recorded have been recorded. If
confirmation has to be used to verify this assertion, the population must be all
vendors with whom the entity has previously done business. This means that
confirmation requests will be sent not only to vendors with large year-end
balances, but also to those with small or zero balances as long as the entity has
made purchases of their goods during the year under audit.
Answers A and C are incorrect because, in testing the completeness assertion, the
auditor is concerned with accounts payable that have not recognized or invoices
that have not been filed.
Answer D is incorrect because the population for the confirmation process should
consist of vendor's accounts regardless of the balance -large, small, or zero- at the
confirmation date.

79. Unrecorded liabilities are most likely to be found during the review of which
of the following documents?

A. Bills of lading

B. Unpaid bills

C. Unmatched sales invoices

D. Shipping records
The auditor's review of unpaid bills may reveal payables that should have been
recorded or accrued but have not been recognized as liabilities as of the balance
sheet date.
Answers A, C, and D are incorrect because these records and documents relate to
the entities sales, not liabilities.

80. Which of the following audit procedures least likely to detect an unrecorded
liability?

A. Readings the minutes of the meetings of the board of directors.

B. Analysis and re-computation of interest expense.

C. Analysis and re-computation of depreciation expense.

D. Mailing of bank confirmation forms.


Analysis and re-computation of depreciation expense will provide audit evidence
about the management's valuation and allocation assertion concerning the entity's
property, plant, and equipment. Thus, this procedure do not relate to the auditor's
objective of detecting unrecorded liabilities.
Answer A is incorrect because the entity's board of directors usually authorizes
transactions that will result to incurrence of large amount of liabilities.
Answer B is incorrect because the auditor's computation of the current year's
interest expense based on the client's recorded liabilities may reveal that it is
unreasonably lower than the actual interest expense paid and accrued. The
auditor's investigation of the significant differences may lead to the detection of
unrecorded liabilities.
Answer D is incorrect because the bank confirmation form requests information
about the client's direct liabilities to the bank which may have not been recognized
on the company's books.

81. When title to merchandise in transit has passed to the audit client, the auditor
engaged in the performance of a purchase cutoff will encounter the greatest
difficulty in gaining assurance with respect to the

A. Quality

B. Quantity

C. Price

D. Terms
The quality of the merchandise can be determined through physical inspection of
the goods already received. Thus, the auditor will encounter the greatest difficulty
in gaining assurance about quality with respect to goods included in inventory but
have not yet arrived.
Answers B, C, and D are incorrect because supporting purchase documentation
will provide information to the auditor about the quantity, price, and terms of the
purchase.

82. Which of the following audit procedures is best for identifying unrecorded
trade accounts payable?

A. Reconciling vendor's statements to the file of receiving reports to identify


items received just prior to the balance sheet date.

B. Examining unusual relationships between monthly accounts payable


balances and recorded cash payments.

C. Investigating payables recorded just prior to and just subsequent to the


balance sheet date to determine whether they are supported by receiving
reports.

D. Reviewing cash disbursements recorded subsequent to the balance sheet


date to determine whether the related payables applied to prior period.
The auditor's search for unrecorded includes the following audit procedures.
• Examining cash disbursements made subsequent to the balance sheet date
and comparing them with recorded account payables as at year-end.

• Sending confirmation to vendors, including those having zero balances.

• Reconciling balances with vendor's documentation.


Answer A is incorrect because reconciling vendor's statements to the file of
receiving reports to identify items received just prior to the balance sheet date
does not provide audit evidence about whether the items have been recorded.
Answer B is incorrect because examining unusual relationships between monthly
accounts payable balances and recorded cash payments tests only the recorded
payables.
Answer C is incorrect because investigating recorded payables to determine if
they are supported by receiving reports tests only the recorded payables and does
not assure the auditor that all the goods received have been recorded.

83. In a payables application, checks are authorized and paid based on matching
purchase orders, receiving reports, and vendor invoices. Partial payments are
common. An appropriate audit procedure for verifying that a purchase order
has not been paid twice is to sort the
A. Check register file by purchase order, compute total amounts paid by
purchase order, compare total amounts paid with purchase order amounts,
and investigate any discrepancies between total amounts paid and
purchase order amounts.
B. Receiving reports file by vendor invoice amounts and investigate any
discrepancies between the total amounts received and vendor invoice
amounts.
C. Vendor invoice file by purchase order, compute total amounts paid by
purchase order, compare total amounts invoiced with purchase order
amounts, and investigate any discrepancies between the total amounts
invoiced with purchase order amounts.
D. Receiving report filed by purchase order, compute total amounts received
by purchase order, compare total amounts received with purchase order
amounts, and investigate any discrepancies between the total amounts
received and purchase order amounts.
Sorting the check register file by purchase order, computing the total amounts
paid by purchase order comparing the total amounts paid with purchase order
amounts, and investigating discrepancies between the total amounts paid and
purchase order amounts will provide reasonable assurance to the auditor that a
purchase order has not been paid twice.
Answers B, C, and D are incorrect because the audit procedures described do not
include examination of the check register file. Thus, they do not provide assurance
that no overpayments have been made by the entity.

84. Which of the following procedures relating to the examination of accounts


payable could the auditor delegate entirely to the client’s employee?
A. Mall confirmations for selected account balances.
B. Prepare a schedule of accounts payable.
C. Test footings in the accounts payable ledger.
D. Reconcile unpaid invoices to vendor’s statements.
The preparation of schedule of accounts payable merely involves listing of
vendor’s accounts and amounts taken from the accounts payable subsidiary ledger.
This procedure could be delegated entirely to the client’s employees. However,
the auditor should test and review this client-prepared schedule.
Answers A, C, and D, are incorrect because the procedures described should be
performed by the auditor.

85. In an audit of a purchasing department, which of the following usually is


considered a risk factor?
A. Purchase specifications are developed by the department requesting the
material.
B. Purchases are not rotated among suppliers included on an approved vendor
list.
C. Purchases are made from parties related to buyers or other company
officials.
D. Purchases are made against blanket or open purchase orders for certain
types of items.

Purchasing from parties related to buyers or other company officials is a risk


factor because this may heighten the possibility of fraud.

Answer A is incorrect because the department in need of the material normally


develops specifications.

Answer B is incorrect because rotation is not usually appropriate. Moreover,


maintaining an approved vendor list is a control to ensure that purchases are made
only from approved vendors.

Answer D is incorrect because blanket or open purchase orders are normally


prepared for materials that are frequently purchased.

86. The following statements compare confirmation of accounts payable with


vendors and confirmation of accounts receivable with customers. Which is
false?
A. As compared with the confirmation of accounts receivable, the
confirmation of accounts payable will tend to emphasize accounts with
zero balances at the balance sheet date.
B. Statistical sampling techniques are more appropriate in the confirmation of
accounts receivable than in the confirmation of accounts payable.
C. It is less likely that the confirmation request sent to the vendor will show
the amount owed than the request sent to the customer will show the
amount due.
D. Confirmation of accounts receivable with customer is a more widely
accepted auditing procedure than is confirmation of accounts payable with
vendors.
Statistical sampling techniques are appropriate for large populations having
similar characteristics. This is true for both confirmation of accounts receivable
with customers and confirmation of accounts payable with vendors.

Answer A is a correct statement. As compared with the confirmation of accounts


receivable, the confirmation of accounts payable will tend to focus on accounts
with zero balances at the balance sheet date to detect possible understatement of
payables.

Answer C is a correct statement. So that all purchases may be confirmed –


including those current shipments whose bills are still being processed by the
vendor, the confirmation request for accounts payable generally does not show the
amount owed by the client. For accounts receivable, often only payments of
customer in transit will be reconciled.

Answer D is a correct statement. Confirmation of accounts receivable is a more


widely accepted auditing procedure than is confirmation of accounts payable. This
is because externally generated vendor documentation is often available for
accounts payable, which may lessen the need to send confirmation requests to
vendors.

87. Which of the following procedures would an auditor least likely to perform
before the balance sheet date?
A. Confirmation of accounts payable
B. Identification of related parties
C. Assessment of control risk
D. Attendance at the physical inventory count

In auditing accounts payable, the most important assertion to verify is


completeness. Thus, accounts payable balances are best confirmed at year-end.

Answers B, C, and D are incorrect because identification of related parties,


assessment of control risk, and attendance at the physical inventory count may be
performed before the balance sheet date.

PRODUCTION CYCLE

88. Which of the following statements concerning the auditor’s attendance at the
physical inventory count is incorrect?
A. A financial statement audit should always include attendance at the
physical inventory count.
B. If the auditor is unable to attend the physical inventory count on the date
planned due to unforeseen circumstances, he/she should take or observe
some physical counts on an alternative date, and when necessary, perform
audit procedures on intervening transactions.
C. Where attendance is impracticable, due to factors such as the nature and
location of inventory, the auditor should consider whether alternative
procedures provide sufficient appropriate audit evidence of existence and
condition to conclude that reference to a scope limitation need not be
made.
D. Inventories that are under the custody and control of third parties (for
example, inventories located in public warehouses)may be verified by
obtaining direct confirmation from the custodians, provided that,
depending on the materiality of the amount involved, additional
procedures should be applied as deemed necessary.
PSA 501 (Audit Evidence – Specific Considerations for Selected Items) states,
“When inventory is material to the financial statements, the auditor should obtain
sufficient appropriate audit evidence regarding its existence and condition by
attendance at physical inventory counting unless impracticable.”
89. PSA 501 states that in planning attendance at the physical inventory count, the
auditor considers the risks of material misstatement related to inventory as
well as the nature of the internal control related to inventory. Which of the
following would the auditor also consider?
I. Whether adequate procedures are expected to be established and
proper instructions issued for the physical inventory count.
II. The timing of the count.
III. The locations at which inventories are held.
IV. Whether an expert’s assistance is to be sought.
A. I and IV only
B. II and III only
C. II, III, and IV only
D. I, II, III, and IV

90. According to PSA 501, when inventories are under the custody and control of
a third party, the auditor would ordinarily obtain direct confirmation from the
third party as to the quantities and condition of inventories held on behalf of
the entity. Which of the following would the auditor also
consider?
I. The integrity and independence of the third party.
II. Observing, or arranging for another auditor to observe, the physical
inventory count.
III. Obtaining another auditor’s report on the adequacy of the third party’s
internal control for ensuring that inventories are correctly counted and
adequately safeguarded.
IV. Inspecting documentation regarding inventories held by third parties
(for example, warehouse receipts) or obtaining confirmation from other
parties when such inventories have been pledges as collateral.

A. I, II, and IV only


B. I, III and IV only
C. II, III, and IV only
D. I, II, III and IV

91. In an audit of inventories, an auditor is least likely to verify that


A. The client has used proper inventory pricing.
B. Damaged goods and obsolete items have been properly accounted for.
C. The financial statement presentation of inventories is appropriate.
D. All inventory owned by the client is on hand at the time of the count.

Inventory owned by the client need not be on hand at the time of the count. For
example, some items purchased under FOB shipping point term may still be in
transit on the date of the count. Also, some goods may have been shipped to
customer on consignment basis.

Answers A and B are incorrect because the auditor should verify management’s
valuation and allocation assertion.

Answer C is incorrect because the auditor should verify management’s


presentation and disclosure assertion.

92. An auditor selected items for test counts while observing a client’s physical
inventory. The auditor then traced the test counts to the client’s inventory
listing. This procedure most likely obtained evidence concerning
management’s assertion
A. Existence
B. Rights and obligations
C. Completeness
D. Valuation and allocation

Completeness is an assertion of management that all assets, liabilities, and equity


interests that should have been recorded have been recorded. Tracing the test
counts to the client’s inventory listing assures the auditor that items included in
the observed physical inventory are reflected in the inventory records.

Answer A is incorrect because the direction of testing must be from the client’s
inventory listing to the inventory tags to obtain evidence that items included in
the listing have been counted, that is, they exist.

Answer B is incorrect because tracing the test counts to the inventory listing does
not provide evidence that the inventory is owned by the client.

Answer D is incorrect because the valuation and allocation assertion is verified by


determining whether inventory items are valued at the lower of cost or net
realizable value.
CHAPTER 8
Audit Sampling

1. In designing audit procedures, the auditor is required to determine appropriate


means of selecting items for testing to gather audit evidence. Which of the
following means is/are available to the auditor?
I. Selecting all items (100% examination).
II. Selecting specific items.
III. Audit sampling.

A. I and II only
B. III only
C. I and III only
D. I, II, and III

According to PSA 500 (Audit Evidence), the application of any one or


combination of the above means may be appropriate depending on the
particular circumstances, such as:
• The risks of material misstatement related to the assertion being tested;
and
• The practicality and efficiency of different means.
2. Which of the following should be considered by the auditor in deciding which
means (or combination of means) to use in selecting items for testing?
I. The risk of material misstatement related to the assertion being tested.
II. Audit efficiency.

A. I only
B. II only
C. Both I and II
D. Neither I nor II

3. It will be appropriate to audit all the items that make up a class of transactions
or account balance (100% examination), except

A. When the class of transactions or account balance consists of a large


number of small value items.
B. When the class of transactions or account balance consists of a small
number of large value items.
C. When there is a significant risk of misstatement and other selection
methods do not provide sufficient appropriate audit evidence.
D. When the repetitive nature of a calculation or other process performed
automatically by the client’s computer information system (CIS) makes a
100% examination cost effective.

According to PSA 500, a 100% examination may be appropriate when:


• The population constitutes a small number of large value items.
• There is a significant risk and other means do not provide sufficient
appropriate audit evidence.
• The repetitive nature of a calculation or other process performed
automatically by an information system makes a 100% examination
cost effective, for example, through the use of computer-assisted audit
techniques (CAATs).
4. PSA 500 states that the auditor may decide to select specific items from a
population based on such factors as the auditor’s understanding of the entity,
the assessed risk of material misstatement, and the characteristics of the
population being tested. Specific items that may be selected for testing usually
include the following, except A. Items that are of high value.
B. Items that are suspicious, unusual, risk-prone, or have a history of error.
C. All items whose values do not exceed a certain amount so as to verify only
a small proportion of the total amount of class of transactions or account
balance.
D. Items that provide information about matters such as the nature of the
entity, the nature of transactions, and the internal control.
According to PSA 500, specific items may include:

• High value or key items – items that are of high value or exhibit some
other characteristics like those that are unusual, suspicious, risk-prone,
etc.
• All items over a certain amount – items whose values exceed a certain
amount so as to verify a large proportion of the total amount of a class
of transactions or an account balance.
• Items to obtain information – items that provide specific information
like the nature of the entity, the nature of transactions, and internal
control.
5. Audit sampling involves the
A. Selection of all items mover a certain amount.
B. Application of audit procedures to less than 100% of items within a class
of transactions or an account balance such that all items have a chance of
selection.
C. Application of audit procedures to all items over a certain amount and
those that are unusual or have a history of error.
As defined in PSA 530 (Audit Sampling), audit sampling involves the
application of audit procedures to less than 100% or items within a population
of audit relevance such that all sampling units have a chance of selection in
order to provide the auditor with a reasonable basis on which to draw
conclusions about the entire population.
Answers A and D are incorrect because they involve selection of specific items
for testing to gather audit evidence.
Answer C is incorrect because it involves 100% examination.

6. Population, as defined in PSA 530 (Audit Sampling), means the entire set of
data from which a sample is selected and about which the auditor wishes to
draw conclusions. It is important for the auditor to ensure that the population
is
I. Appropriate to objective of the audit procedure.
II. Complete.

A. I only
B. II only
C. Both I and II
D. Neither I nor II
The auditor should ensure that the population is appropriate to the objective of
the audit procedure, which includes consideration of the direction of the test to
be applied. For example, the appropriate population to test for overstatement
of accounts payable will be accounts payable listing. However, of the auditor’s
objective is to test for understatement of accounts payable, the appropriate
population is not the accounts payable listing but the subsequent
disbursements, unpaid invoices, vendors’ statements, or other audit evidence
that will satisfy the objective of the test.
It is also important for the auditor to ensure that the population is complete.
For example, if the sample is to be drawn from the vouchers file, the auditor
should be satisfied that all vouchers have, in fact, been filed.

7. The two general approaches to audit sampling are: A. Stratification and value
weighted.
B. Random and nonrandom.
C. Statistical and nonstatistical.
D. Precision and reliability.
The two general approaches to audit sampling are statistical and
nonstatistical.
A. statistical sampling plan should have the following characteristics:

a) Random selection of a sample; and


b) Use of probability theory to evaluate sample results, including
measurement of sampling risk.
A nonstatistical sampling plan is any sampling plan which, according to the
standard, does not have the characteristics of statistical sampling.
Answers A, B, and D are incorrect because stratification and value weighted;
random and nonrandom; and precision and reliability are not general
approaches to audit sampling.
Stratification is the process of dividing a population into discrete
subpopulations (also called strata), each of which is a group of sampling units
which have similar characteristics (often monetary value).
Value weighted is a selection method in which the sampling unit is identified
as the monetary units that make up a transaction class or an account balance.
Random selection gives each sampling unit a chance of being selected.
Conversely, nonrandom selection does not give each sampling unit a chance
of being included in the sample.
Precision is the allowance for sampling risk. Reliability (also called
confidence level) is the degree to which the sample selected is expected to be
representative of the population. It is the mathematical complement of
sampling risk.

8. The principal methods of selecting samples are the use of I. Random number
tables or CAATs. II. Systematic selection.
III. Haphazard selection.

A. I and II only
B. II and III only
C. I and III only
D. I, II, and III
According to PSA 530, the principal methods of selecting samples are as
follows:

a) Use of computerized random number generator (through CAATs) or


random number tables.
b) Systematic selection, in which every nth item from a population of
sequentially ordered items is selected.
c) Haphazard selection, in which the sample is selected without
following a structured or organized approach, but also without
conscious bias. This selection method in inappropriate for statistical
sampling but may be useful for nonstatistical sampling plans.

9. An advantage of statistical sampling over nonstatistical sampling is that


statistical sampling helps an auditor to A. Minimize the failure to detect
errors and fraud.
B. Measure the sufficiency of the evidential matter obtained.
C. Eliminate the risk of nonsampling errors.
D. Reduce the level of audit risk and materiality to a relatively low amount.
Statistical sampling involves application of the laws of probability that
enables the auditor to design an efficient sample (i.e., a sample that is neither
too large nor too small), to measure the sufficiency of the audit evidence
obtained, and to evaluate the sample results.
Answer A is incorrect because, in some circumstances, a nonstatistical
sampling plan, may be more appropriate to minimize the failure to detect
errors and fraud.
Answer C is incorrect because nonsampling errors do not relate to audit
sampling. These are human errors like the auditor’s use of inappropriate
procedures or failure to recognize an error because of misinterpretation of
audit evidence obtained. In addition, nonsampling errors arise because of the
fact that most audit evidence is persuasive rather than conclusive.
Answer D is incorrect because both statistical and nonstatistical sampling may
be used to reduce audit risk. Moreover, statistical sampling is irrelevant to
materiality.

10. The likelihood of assessing control risk too high is the risk that the sample
selected to test controls
A. Does not support the tolerable error for some or all of management’s
assertions.
B. Contains proportionately fewer deviations form prescribed internal
controls than exist in the balance or class as a whole.
C. Does not support the auditor’s planned assessed level of control risk when
the true operating effectiveness of internal control justifies such an
assessment.
D. Contains misstatements that could be material to the financial statements
when aggregated with misstatements in other account balances or
transactions classes.
As defined in the standard, sampling risk arises from the possibility that the
auditor’s conclusion based on a sample may be different from the conclusion
reached if the entire population were subjected to same audit procedure. It
arises from the fact that a sample may not be representative of the population
from which it was drawn.

In performing tests of controls, the two aspects of sampling risk are:


1) The risk of assessing control risk too high is the risk that the assessed
level of control risk based on the sample is greater than the true
operating effectiveness of the control.
2) The risk of assessing control risk too low is the opposite of assessing
control risk too high. It is the risk that the auditor may be believe that a
control is operating effectively when it is not.
Answers A and D are incorrect because they relate more to substantive testing.
Answer B is incorrect because it relates to assessing control risk too low.
11. While performing a test of details during an audit, the auditor determined that
the sample results supported that the conclusion that the recorded account
balance was materially misstated. It was, in fact, not materially misstated. This
situation illustrates the risk of A. Assessing
control risk too low
B. Assessing control risk too high
C. Incorrect acceptance
D. Incorrect rejection
The two aspects of sampling risk in substantive testing are:

1) The risk of incorrect rejection is the risk that a sample supports the
conclusion that the account balance is materially misstated when,
unknown to the auditor, the account balance is not materially misstated
(i.e., it is fairly stated).
2) The risk of incorrect acceptance is the risk that a sample supports the
conclusion that the account balance is not materially misstated (i.e., it
is fairly stated) when, unknown to the auditor, the account balance is
materially misstated.
Answers A and B are incorrect because assessing control too high or too low
related to test of controls.
Answer C is incorrect because the risk of incorrect acceptance is the risk that
the auditor will conclude based on a sample that the account balance is not
materially misstated when, in fact, it is materially misstated.

12. The risk of incorrect acceptance and the likelihood of assessing control risk
too low relate to the A. Effectiveness of the audit.
B. Efficiency of the audit.
C. Tolerable misstatement.
D. Preliminary estimates of materiality levels.
If an auditor erroneously accepts an account balance a fairly stated, it is
unlikely that additional audit procedures will be performed. Thus, the
probability that the erroneous conclusion will be discovered is minimal,
thereby decreasing the effectiveness of the audit.
An auditor’s rejection of a fairly stated account balance is most likely to result
in performing extended substantive testing that will ultimately lead to the
acceptance of the balance. Hence, the risk of incorrect rejection affects the
efficiency of the audit.
Assessing control risk too low leads to an unjustified reduction in substantive
testing which, in turn, results in obtaining insufficient audit evidence, thereby
decreasing the effectiveness of the audit. Conversely, assessing control risk
too high results in an unjustified increase in substantive testing and thus
affecting the efficiency of the audit.
Answer B is incorrect because the risk of incorrect acceptance and the
likelihood of assessing control risk too low both relate to audit effectiveness,
not audit efficiency. As indicated above, audit efficiency is affected by the risk
of incorrect rejection and the likelihood of assessing control risk too high.
Answers C and D are incorrect because the tolerable error and preliminary
estimates of materiality levels are considered when the auditor plans a sample
for substantive testing. Moreover, the likelihood of assessing control risk too
low is an aspect of sampling risk in performing tests of controls.

13. Which of the following statements is true?


A. Statistical sampling is more convenient to use than nonstatistical sampling.
B. Statistical sampling aids the auditor in evaluating results.
C. Statistical sampling requires the auditor to make fewer judgmental
decisions.
D. Statistical sampling will be looked upon by the courts as providing superior
audit evidence.

14. The following are examples of nonsampling risk, except A. Failure to


recognize an error.
B. Obtaining an unrepresentative sample.
C. Use of an audit procedure inappropriate to achieve a given objective.
D. Failing to evaluate results properly.

15. The following are advantages of using statistical sampling, except


A. Statistical sampling provides a means for mathematically measuring the
degree of risk that results from examining only part of a population.
B. Statistical sampling allows the auditor to greatly reduce substantive testing.
C. Statistical sampling allows the auditor to measure the sufficiency of the
evidential matter obtained.
D. Statistical sampling aids in the design of an efficient sample.

16. Which of the following methods is most appropriate when performing tests of
controls?
A. Stratified random sampling
B. Unrestricted random sampling with replacement
C. Variable sampling
D. Attribute sampling

17. In the audit of inventory, attribute sampling may be applied to estimate the
A. Average price of inventory items
B.
Physical quantity of inventory items
C. Percentage of slow-moving inventory items
D. Peso value of inventory

18. The risk that the assessed level of control risk based on the sample is less
than the true operating effectiveness of
the control policy or procedure is the risk of A. Assessing
control risk too low.
B. Assessing control risk too high.
C. Incorrect acceptance.
D. Incorrect rejection.

19. An element of sampling risk is


A. Choosing a sample size that is too small to achieve the sampling objective.
B. Choosing an audit procedure that is inconsistent with the audit objective.
C. Failing to perform audit procedures that are required by the sampling plan.
D. Failing to detect a deviation on a document inspected by the auditor.

20. Statistical sampling provides a technique for A. Exactly defining materiality.


B. Greatly reducing the extent of substantive testing.
C. Eliminating judgment in testing.
D. Measuring the sufficiency of evidential matter.
21. The distinguishing feature of statistical sampling is that
it
A. Reduces the problems associated with the auditor’s judgment concerning
materiality.
B. Requires the examination of a smaller number of supporting documents.
C. Is evaluated in terms of two parameters: statistical means and random
selection.
D. Provides a means for measuring mathematically the degree of uncertainty
that results from examining only part of a population.

22. Which of the following is true if certain forms are not consecutively
numbered?
A. Systematic sampling may be appropriate.
B. Selection of a random sample probably is not possible.
C. Random number tables cannot be used.
D. Stratified sampling should be used.
C.
23. In examining cash disbursements, an auditor plans to choose a sample using
systematic selection with a random start. The primary advantage of this
selection technique is that population items
A. May occur more than once in the sample.
B. May occur in a systematic pattern, thus making the sample more
representative.
That are indicative of fraud will be included in the sample.
D. Do not have to be prenumbered in order for the auditor to use the
technique.

24. In attribute sampling, a 5% change in which of the following factors


normally will have the least effect on the size of a statistical sample?
A. Expected deviation rate.
B. Risk of assessing control risk too low.
C. Population size.
D. Tolerable deviation rate.

25. Given random selection, the same sample size, and the same tolerable
deviation rate for the testing of two unequal populations, the risk of assessing
control risk too low for the larger population is
A. Higher than the risk of assessing control risk too low for the smaller
population.
B. Lower than the risk of assessing control risk too low for the smaller
population.
C. The same as the risk of assessing control risk too low for the smaller
population.
D. Indeterminable relative to the risk of assessing control risk too low for the
smaller population.

26. If the size of the sample to be used in a particular test of attributes has not
been determined by utilizing statistical concepts, but the sample has been
randomly chosen
A. The auditor will have to evaluate the results by reference to the principles
of discovery sampling. B. May not use statistical evaluation.
C. The auditor has committed a nonsampling error.
D. No inferences can be drawn from the sample.

27. In determining the number of items to be selected in a sample for a particular


substantive test of details, the auditor should consider all of the following,
except
A. Tolerable misstatement
D.
B. Characteristics of the population
C. Deviation rate
D. Allowable risk of incorrect acceptance

28. Which of the following statements best describes the concept of sampling
risk?
A. A randomly chosen sample may not be representative of the population as
a whole on the characteristic of interest.
B. The documents related to the chosen sample may not be available for
inspection.
C. An auditor may fail to recognize errors in the documents examined for the
chosen sample/
An auditor may select audit procedures that are not appropriate to achieve
the specific objective.
According to the standard, sampling risk arises from the possibility that the
auditor’s conclusion based on a sample may be different from the conclusion
reached if the entire population were subjected to the same audit procedures.
It is the risk that, unknown to the auditor, the sample selected is not
representative of the population.
Answers B, C, and D are incorrect because they all relate to nonsampling risk,
that is, the risk that the auditor will reach an incorrect conclusion even if
100% of the population were examined.

29. In assessing sampling risk, the risk of incorrect rejection and the risk of
assessing control risk too high relate to the A. Effectiveness of the audit.
B. Efficiency of the audit.
C. Audit quality controls.
D. Selection of the sample.
Both the risk of incorrect rejection and the risk of assessing control risk too
high will typically lead to the performance of unnecessary audit procedures.
Thus, these risks relate to the efficiency of the audit.
Answer A is incorrect because the risk of incorrect acceptance and the risk of
assessing control risk too low both relate to the effectiveness of the audit.
Answer C is incorrect because the standards on quality control do not
specifically mention any of these risks.
E.
Answer D is incorrect because these risks do not directly relate to actual
selection of the sample.

30. An underlying feature of random-based selection of items is that each


A. Item must be systematically selected using replacement.
B. Item in the accounting population should have an opportunity to be
selected.
C. Stratum of the accounting population be given equal representation in the
sample.
D. Item in the accounting population be randomly ordered.
Under random-based selection, each item has known chance of being selected.
The auditor typically uses random number tables or a computerized random
number generator (through CAATs) in applying this method.
Answer A is incorrect because systematic selection involves selecting every n th
item from the population. Moreover, random selection may be used without
replacement.
Answer C is incorrect because each subpopulation or stratum need not be
given equal representation. Answer D is incorrect because while items are
selected at random, there is no requirement that the accounting population be
randomly ordered.

31. If certain forms are not consecutively numbered A. Selection of a random


sample probably is not possible.
B. Systematic sampling may be appropriate.
C. Random number tables cannot be used.
D. Stratified sampling should be used.
Systematic selection involves selecting every nth item from the population.
The number of items to skip is determined by calculating the sampling
interval (population size divided by sample size). Under this method, there is
no need to establish correspondence between population items and random
numbers.
Answers A and C are incorrect because the use of random selection and
random number tables is possible even if items in the population are not
consecutively numbered.
Answer D is incorrect because there is no requirement to use stratified
sampling if the forms are not consecutively numbered.
Stratification involves dividing the population into different homogenous
groups called subpopulation or strata. Different selection methods are then
applied to each subpopulation or stratum.

32. When performing a test of a control with respect to control over cash receipts,
an auditor may use a systematic sampling technique, with a start at any
randomly selected item. The biggest disadvantage of this type of sampling is
that the items in the population
A. Must be systematically replaced in the population after sampling.
B. Must be recorded in a systematic pattern before the sample can be drawn.
C. May occur in a systematic pattern, thus destroying the sample randomness.
D. May systematically occur more than once in the sample.
PSA 530 states that when using systematic selection, the auditor would need
to determine that sampling units within the population are not structured in
such a way that the sampling interval corresponds with a particular pattern in
the population.
Answer A is incorrect because items need not be replaced in the population.
Answer B is incorrect because systematic sampling is a selection method and
does not relate to the manner in which items in the population are recorded.
Moreover, the occurrence of a systematic pattern in the population destroys
the sample randomness.
Answer D is incorrect because there is no chance that the items will
systematically occur more than once in the sample since systematic selection
involves selecting every nth item from the population.

33. For which of the following audit tests would an auditor most likely use attribute
sampling?
A. Selecting accounts receivable for confirmation of account balances.
B. Examining invoices in support of the valuation of property, plant, and equipment
additions.
C. Making an independent estimate of the amount of FIFO inventory.
D. Inspecting employee time cards for proper approval by supervisors.
Attribute sampling is designed to test the rate of deviation from a prescribed control
procedure.
Answers A, B, and C are incorrect because they relate more directly to
variables sampling which is designed to test whether an account balance is
materially misstated.

34. Which of the following sampling methods would be used to estimate a numerical
measurement of a population, such as a peso value? A. Variable sampling
B. Attribute sampling
C. Random-number sampling
D. Stop-or-go sampling
A. variable sampling plan is designed to test whether an account balance is materially
misstated and therefore addresses numerical measurements such as a peso value.
Answer B is incorrect because attribute sampling deals with deviation rates, not numerical
measurements.
Answer C is incorrect because random-number sampling is a selection method
and may be used with either an attribute or a variable sampling plan.
Answer D is incorrect because stop-or-go sampling (also called sequential sampling) is
a form of attribute sampling.
In stop-or-go sampling, the sample is selected in several steps (i.e., not only a
single sample is tested). For each step, the auditor decided whether to stop the
test or to proceed to the next step.

35. Which of the following combinations results in a decrease in sample size in an attribute
sample?
Allowable Risk
Of Assessing
Control Risk Tolerable Expected Population
Too Low Deviation Rate Deviation Rate A. Increase
Increase Increase
B. Decrease Increase Decrease
C. Increase Increase Decrease
D. Increase Decrease Increase
To determine the sample size for tests of controls, the auditors considers the following
factors:
Factors Conditions Leading to
Smaller Larger
Sample Size Sample Size

1) Planned reliance Lower reliance Higher reliance on internal control1


on internal on internal
control control
2) Tolerable deviation Higher TDR Lower TDR rate (TDR)2
3) Allowable risk of Higher allowable Lower allowable assessing
control risk of assessing risk assessing risk too low control risk
control risk
too low too low
4) Expected population deviation rate
(EDR)3 Lower EDR Higher EDR
5) Number of sampling units in the Negligible effect on sample
size population unless population is small.
1 The auditor does not perform tests of controls when no reliance on internal controls
is planned.
2 The TDR is the rate of deviation from the prescribed control activity that the auditor
is willing to accept.
3 The EDR is the rate of deviation from the prescribed control activity that the auditor
expects to find in the population.

36. In determining the number of documents to select for a test to obtain assurance that all sales
returns have been properly authorized, an auditor should consider the tolerable rate of
deviation from the control activity. The auditor should also consider the

I. Likely rate of deviations.


II. Allowable risk of assessing control risk too high.
A. I only
B. II only
C. Both I and II
D. Either I or II
The auditor considers the following factors to determine the sample size for a test of
controls:

1) Expected deviation rate


2) Tolerable deviation rate
3) Allowable risk of assessing control risk too low

37. Which of the following factors is usually not considered in determining the sample size for a
test of controls?
A. Expected population deviation rate
B. Risk of assessing control risk too low
C. Tolerable deviation rate
D. Population size, when the population is large
A change in the number of sampling units in the population has a negligible
effect on the required sample size when the population is large. Therefore,
population size is often not considered in determining the sample size unless it
is small.
Answers A, B, and C are incorrect because the risk of assessing control risk
too low, the tolerable deviation rate, and the expected deviation rate are
considered in determining the sample size for a test of controls.

38. The sample size of a test of controls varies inversely with


Tolerable Expected
Deviation Rate (TDR) Deviation Rate (EDR)
A. No No
B. Yes Yes
C. No Yes
D. Yes No
The TDR is inversely related to the sample size – that is, as the TDR increases, the
sample size decreases.
The EDR has a direct effect on the sample size – that is, as the EDR increases, the
sample size increases.

39. In planning a statistical sample for a test of controls, an auditor increased the
expected population deviation rate (EDR) from the prior year’s rate because of
the results of the prior year’s tests of controls and the overall control
environment. The auditor most likely would then increase the planned
A. Risk of assessing control risk too low.
B. Sample size.
C. Allowance for sampling risk.
D. Tolerable deviation rate.
The EDR directly affects the sample size – that is, as the EDR increases, the
sample size increases. An increase in the EDR increases the degree of
assurance to be provided by the sample and therefore increases the sample
size.
Answer A is incorrect because the risk of assessing control risk too low is
determined based on the auditor’s judgment and does not necessarily increase
with the EDR.
Answer C is incorrect because the allowance for sampling risk is the
difference between the maximum deviation rate (also called upper precision
limit) and the sample deviation rate.
Answer D is incorrect because the tolerable deviation rate is determined based
on the auditor’s judgment. It is a function of the planned assessed level of
control risk and the level of assurance the evidence is expected to provide. It
does not necessarily increase with the EDR.

40. Which of the following factors does an auditor usually need to consider in
planning a particular audit sample for a test of controls?
A. Acceptable level of risk of assessing control risk too low.
B. Tolerable misstatement.
C. Number of sampling units in the population.
D. Total peso amount of the items to be sampled.
To determine the sample size for a test controls, the auditor considers the risk
of assessing control risk too low, the tolerable deviation rate, and the expected
population deviation rate.
Answer B is incorrect because the auditor considers the tolerable misstatement
in determining the sample size for substantive tests of details.
Answer C is incorrect because the number of units in the population has a
negligible effect on the sample size unless it is small.
Answer D is incorrect because the total peso amount of the items to be
sampled is irrelevant to the objective of a test of controls, that is, to determine
the operating effectiveness of prescribed controls.

41. Which of the following statements is true concerning statistical sampling in tests of controls?
A. For a given tolerable rate, a larger sample size should be selected as the expected
population deviation rate decreases.
B. As the population size doubles, the sample size also should double.
C. The expected population deviation rate has little or no effect on determining sample
size except for very small populations.
D. The population size has little or no effect on determining sample size except for very
small populations.
A change in the size of the population has a negligible effect on the required
sample size when the population is large.
Answer A is incorrect because the expected population deviation rate directly
affects the sample size. A smaller sample size should be selected as the
expected population deviation rate decreases.
Answer B is incorrect because, as previously stated, a change in the population
size has a negligible effect on the required sample size when the population is
large.
Answer C is incorrect because, as indicated in answer A, the expected
population deviation rate directly affects the sample size.

42. Which of the following statements is correct concerning statistical sampling in tests of
controls?
A. In determining the tolerable rate, an auditor considers detection risk and the sample
size.
B. Deviations from specific control activities at a given rate ordinarily result in
misstatements at a lower rate.
C. As the population size increases, the sample size should increase proportionately.
D. There is an inverse relationship between the expected population deviation rate and
the sample size.
Deviations from a specific control activity increase the risk of, but do not
always result in, misstatement. Hence, deviations from a specific control
activity at a given rate ordinarily result in misstatement at a lower rate.
Answer A is incorrect because the tolerable rate is a function of the planned
assessed level of control risk and the assurance sought from the audit
evidence.
Answer C is incorrect because as the population size increases, the sample size
increases at a decreasing rate. Thus, a large population size will have little or
no effect on the sample size.
Answer D is incorrect because the expected deviation rate is directly related to the
required sample size.

43. An auditor plans to examine a sample of 100 purchase orders for proper
approvals as prescribed by the client’s internal control. One of the purchase
orders in the chosen sample of 100 cannot be found, and the auditor is unable
to use alternative procedures to test whether the purchase order was properly
approved. The auditor should
A. Choose another purchase order to replace the missing purchase order in the sample.
B. Select a completely new set of 100 purchase orders.
C. Consider this test of controls invalid and proceed with substantive tests because internal
control is ineffective.
D. Treat the missing purchase order as a deviation for the purpose of evaluating the sample.
The auditor’s failure to apply the planned audit procedures or to perform
alternative procedures to selected items requires consideration of the reasons
for the limitation. Moreover, the auditor considers such items as deviations
from the prescribed control procedures for the purpose of evaluating the
sample.
Answer A is incorrect because choosing another purchase order is appropriate
only if the selected item has been found to be properly voided or canceled.
Answer B is incorrect because selecting a completely new set of 100 purchase
orders is unnecessary.
Answer C is incorrect because by treating the missing purchase order as a
deviation, the sampling plan could still be completed.

44. An attribute sampling plan may be used to test the effectiveness of controls.
The auditor’s evaluation of the sampling results ordinarily leads to a
conclusion concerning
A. The relation of the population deviation rate to the tolerable rate.
B. Monetary precision exceeding a certain predetermined amount.
C. The population value not being misstated by more than a predetermined amount.
D. Population characteristics occurring at least once in the population.
In an attribute sampling plan, the auditor’s concern is the occurrence rate of
deviations in the population. This statistical sampling plan enables the auditor
to make an estimate of the occurrence rate and to arrive at a conclusion
concerning the relation of the population deviation to the tolerable deviation
rate.
Answers B and C are incorrect because variables sampling is concerned with peso values.
Answer D is incorrect because determining if a population characteristic
occurs at least once in the population will involve examining items until one is
detected.

45. As a result of sampling procedures applied as tests of controls, an auditor


incorrectly assesses control risk lower than appropriate. Which of the
following is the most likely explanation for this situation?
A. The deviation rate in the auditor’s sample exceeds the tolerable rate, but the deviation
rate in the population is less than the tolerable rate.
B. The deviation rate in the auditor’s sample is less than the tolerable rate, but the deviation
rate in the population exceeds the tolerable rate.
C. The deviation rates of both the auditor’s sample and the population are less than the
tolerable rate.
D. The deviation rates of both the auditor’s sample and the population exceed the tolerable
rate.
If the sample deviation rate is less than the tolerable rate, the auditor will
conclude that the control tested is functioning effectively. However, if the true
population deviation rate exceeds the tolerable rate, the auditor’s assessment
of control risk would be lower than appropriate, that is, too low.
Answer A is incorrect because the auditor’s assessment of control risk is too
high if the sample deviation rate exceeds the tolerable rate, but the true
population deviation rate is less than the tolerable rate.
Answers C and D are incorrect because the auditor’s conclusions is correct if
the sample and population deviation rates are both greater or less than the
tolerable rate.

46. The diagram below depicts the auditor’s estimated maximum deviation rate
compared with the tolerable rate, and also depicts the true population
deviation rate compared with the tolerable rate.

Auditor’s Estimate True State of Population


Based on Deviation Rate Deviation Rate is
Sample Results Exceeds Tolerable Rate Less Than Tolerable

Rate I III
Maximum Deviation
Rate Exceeds
Tolerable Rate II IV
Maximum Deviation
Rate is Less Than
Tolerable Rate

As a result of testing internal controls, the auditor assesses control risk too
high and thereby increases substantive testing. This is illustrated by situation
A. I
B. II
C. III
D. IV
The auditor would assess control risk too high if, based on sample results, the
maximum deviation rate exceeds the tolerable rate, but the true population
deviation rate is less than the tolerable deviation rate. Moreover, assessing
control risk too high concerns the efficiency, not the effectiveness, of the audit
because it typically leads to the performance of additional audit procedures to
ultimately arrive at the correct conclusion.
Answers A and D are incorrect because the auditor will properly assess control
risk at a high level in situation I and at a low level in situation IV.
Answer B is incorrect because the auditor will assess control risk too low in situation
II.

47. In evaluating an attribute sample, the estimated range that is expected to contain
the population characteristic is the
A. Confidence level
B. Expected deviation rate
C. Precision
D. Upper deviation limit
Precision (also called confidence level) is the range within which the estimate
of the population characteristic is expected to fail. It is an interval around the
sample statistic that is expected to contain the true population value.
Answer A is incorrect because confidence level refers to the auditor’s measure
of how reliable the sample results should be.
Answer B is incorrect because the expected deviation rate is the rate of
deviation the auditor expects to find in the population.
Answer D is incorrect because the upper deviation limit, as it suggests, is the
upper limit of the precision or confidence interval.

48. Which of the following sample planning factors would influence the sample
size for a substantive test of details for a specific account?
Expected Amount Measure of
of misstatements Tolerable Misstatements
A. Yes Yes
B. No No
C. Yes No
D. No Yes
The auditor considers the following factors in determining the sample size for
substantive tests of details:

Conditions Leading to
Smaller Sample Size Larger Sample Size a. Reliance on Higher
reliance on Lower reliance on internal control internal control internal
control

b. Reliance on other Higher reliance to Lower reliance or substantive tests be


placed on other no reliance to be directed at the substantive tests placed
on other same assertion substantive tests

c. Measure of Larger measure of Smaller measure of tolerable error


tolerable error tolerable error
d. Expected size Smaller errors or Larger errors or and frequency lower
frequency higher frequency of errors

e. Population value Smaller monetary Larger monetary


significance to significance to the
financial the financial
information information

f. Acceptable Higher acceptable Lower acceptable level of risk level


of risk level of risk

g. Stratification Stratification ofNo stratification


the population of the population
when appropriate

h. Number of Negligible effect in sample size unless sampling units


population is small. in the population

49. An error that arises from an isolated event that has not recurred other than on
specifically identifiable occasions and is therefore not representative of
similar errors in the population is a/an A. Anomalous error
B. Isolated error
C. Scandalous error
D. Non-recurring error

50. Which of the following statements concerning projection of errors found in


the sample to the population is incorrect?
A. For tests of details, the auditor should project monetary errors found in the sample to the
population, and should consider the effect of the projected error on the audit objective and on
other areas of the audit.
B. Anomalous errors found in the sample should be included in the projection of errors to the
population.
C. For tests of controls, no explicit projection of errors is necessary since the sample deviation
rate is also the projected deviation rate for the population as a whole.
D. If a class of transactions or an account balance has been divided into strata, the error is
projected for each stratum separately.
PSA 530 states, “When an error has been established as an anomalous error, it
may be excluded when projecting sample errors to the population.”

51. In estimation sampling for variables, which of the following must be known
to estimate the appropriate sample size required to meet the auditor’s needs
in a given situation?
A. The estimated deviation rate in the population.
B. The qualitative aspects of misstatements.
C. The estimated population value.
D. The acceptable level of risk.
When determining the sample size for a substantive test of details, the auditor
should consider the acceptable level of sampling risk.
Answer A is incorrect because the estimated population deviation rate is considered in
attribute sampling.
Answer B is incorrect because the qualitative aspects of misstatements are
considered in the evaluation of the sample results.
Answer C is incorrect because estimation of the population value is the objective of
variables sampling.

52. In applying variables sampling, an auditor attempts to


A. Predict a monetary population value within range of precision.
B. Estimate a qualitative characteristic of interest.
C. Determine various rates of occurrence for specified attributes.
D. Discover at least one instance of a critical deviation.
Variables sampling is designed to estimate the value of a population, for example, an account
balance.
Answer B is incorrect because variables sampling involves estimating the
value of a population. Hence, the estimate is quantitative, not qualitative.
Answer C is incorrect because the rate of occurrence of deviations from a
prescribed control activity is determined when applying attribute sampling.
Answer D is incorrect because discovering at least one instance of a critical
deviation is the objective of discovery sampling, which is a form of attributes
sampling.

53. When planning a sample for s substantive test of details, an auditor should
consider tolerable misstatement for the sample. This consideration should
A. Not be changed during the audit process.
B. Be related to the auditor’s business risk.
C. Be related to preliminary judgments about materiality levels.
D. Not be adjusted for qualitative factors.
The tolerable misstatement is the maximum amount of misstatement that may
exist in an account balance without causing the financial statements to be
materially misstated.
The combined tolerable misstatement for an entire audit should not exceed the
auditor’s preliminary estimate of materiality for the financial statements taken
as a whole.
Answer A is incorrect because, as the audit progresses, the auditor may decide
to change the tolerable misstatement like when he/she discovers the incorrect
planning assumptions were used.
Answer B is incorrect because the auditor’s business risk is irrelevant to the determination of
tolerable misstatement.
Answer D is incorrect because qualitative factors should be considered when
determining the tolerable misstatement for the sample. For example, the
auditor should consider the nature and cause of misstatements and their
impact on other phases of the audit.

54. An auditor may decide to increase the risk of incorrect rejection when
A. The cost and effort of selecting additional sample items are low.
B. Increased reliability from the sample is desired.
C. Many differences (audit value minus recorded value) are expected.
D. Initial sample results do not support the planned level of control risk.

The risk of incorrect rejection is the risk that the sample supports the
conclusion that the recorded account balance is materially misstated, when, in
fact, it is fairly stated. This risk ordinarily result in the performance of
additional audit procedures that will lead the auditor to the proper conclusion.
Thus, the risk of incorrect rejection affects the efficiency, not the effectiveness
of the audit. If the cost and effort of selecting additional items are low, the
auditor may accept a higher risk of incorrect rejection.

Answer B is incorrect because a decrease in the risk of incorrect rejection is


required if increased reliability (confidence level) from the sample is desired.

Answer C is incorrect because the account balance is more likely to be


misstated if many differences are expected and therefore incorrect rejection is
less likely.

Answer D is incorrect because control risk relates to tests of controls whereas


incorrect rejection relates to substantive testing.

55. In statistical sampling methods used in substantive testing, an auditor most


likely would stratify a population into meaningful groups of
A. Probability-proportional-to-size (PPS) sampling is used.
B. The population has highly variable recorded amounts.
C. The standard deviation of recorded amounts is relatively small.
D. The auditor’s estimated tolerable misstatement is extremely small.
Stratification involves dividing the population into homogenous groups called
strata or subpopulations, thus reducing the effect of high variability of
amounts in the population. Because the variability of items within each
subpopulation or stratum is reduced, the auditor will be able to select a smaller
sample for each subpopulation.
Answer A is incorrect because the population is automatically stratified under PPS sampling.
Answer C is incorrect because the auditor would select a sample from the total
population when the standard deviation of recorded amounts is relatively
small.
Answer D is incorrect because a relatively large sample size will be required
when the tolerable misstatement is extremely small.

56. How would decreases in tolerable misstatement and assessed level of control
risk affect the sample size in a substantive test of details?
Decrease in Decrease in
Tolerable Misstatement Assessed Level
Of Control Risk
A. Increase sample size Increase sample size
B. Increase sample size Decrease sample size
C. Decrease sample size Increase sample size
D. Decrease sample size Decrease sample size
The tolerable misstatement is inversely related to the sample size – that is, as
the tolerable misstatement decreases, the sample size increases. Also, a
decrease in the assessed level of control risk will allow the auditor to accept a
higher level of detection risk and therefore a small sample size for substantive
testing.

57. Which of the following courses of action would an auditor most likely follow
in planning a sample of cash disbursements if the auditor is aware of several
unusually large cash disbursements?
A. Continue to draw new samples until all the unusually large disbursements appear in the
sample.
B. Increase the sample size to reduce the effect of the unusually large disbursements.
C. Stratify the cash disbursements population so that the unusually large disbursements are
selected.
D. Set the tolerable rate of deviation at a lower level than originally planned.
The auditor should stratify the population to ensure that the “unusually large
disbursements” will be tested. Sampling procedures will then be applied to
those smaller disbursements.
Answer A is incorrect because it will be inefficient to continue to draw new
samples to assure inclusion of all unusually large disbursements.
Answer B is incorrect because the auditor will tend to include in his/her test
those disbursements described as “unusually large.”
Answer D is incorrect because the existence of unusually large disbursements
in the population does not affect the tolerable deviation rate in an attribute
sampling application.
58. A number of factors influence the sample size for a substantive test of details
of an account balance. All other factors being equal, which of the following
would lead to a larger sample size?
A. Smaller measure of tolerable misstatement.
B. Smaller expected frequency of errors.
C. Greater reliance on analytical procedures.
D. Greater reliance on internal control.
The tolerable misstatement is inversely related to the sample size – that is, as
the tolerable misstatement decreases, the sample size increases.
Answer B is incorrect because the expected frequency of errors directly affects
the sample size – that is, as the expected frequency of errors decreases, the
sample size also decreases.
Answer C is incorrect because the auditor will select a smaller sample size for
a substantive test of details if greater reliance is to be placed on analytical
procedures directed at the same assertion.
Answer D is incorrect because as the degree of reliance on internal control
increases, the acceptable level of detection also increases. This means that the
auditor may restrict his/her substantive testing by selecting a smaller sample
size.

59. An auditor established a P900,000 tolerable misstatement for an asset with an


account balance of P15,000,000. The auditor selected a sample of every 20 th items
from the population that represented the asset account balance and discovered a net
overstatement of P52,500 (P55,500 overstatements minus P3,000 understatements).
Under these circumstances, the auditor most likely would conclude that
A. The asset account is fairly stated because the tolerable misstatement exceeds the net of projected
actual overstatements and understatements.
B. The asset account is fairly stated because the total projected misstatement is less than the
tolerable misstatement.
C. There is an unacceptably high risk that the actual misstatements in the population exceed the
tolerable misstatement because the total projected misstatement exceeds the tolerable
misstatement.
D. There is an unacceptably high risk that the tolerable misstatement is more than the sum of actual
overstatements and understatements.
The net overstatement of P52,500 represents 1/20 of the items in the
population. Thus, the projected misstatement will be P1,050,000 (P52,500 x
20), which exceeds the tolerable misstatement of P900,000. This circumstance
will lead to a conclusion that there is an unacceptably high risk that the actual
misstatements in the population exceed the tolerable misstatement.
Answers A and B are incorrect because the calculation above indicates that the
projected misstatement exceeds the tolerable misstatement.
Answer D is incorrect because even if the sum of actual overstatements and
understatements is to be projected to the population, the projected
misstatement will exceed the tolerable misstatement. Moreover, if the
tolerable misstatement exceeds the projected misstatement, the auditor will
simply accept the account balance as fairly stated.

60. Which of the following sample selection methods is not appropriate when using
statistical sampling?
A. Random selection
B. Systematic selection
C. Monetary unit sampling
D. Haphazard selection
Haphazard selection method involves selection of the sample without
following a structured approach. This technique is not appropriate when using
statistical sampling.

61. Which of the following sample selection methods cannot ordinarily be used in audit
sampling?
A. Value-weighted selection
B. Random selection
C. Block selection
D. Systematic selection
Block selection involves selection of a block(s) of contiguous items from
within the population. In most populations, items in a sequence are expected
to have similar characteristics to each other, but different characteristics from
items elsewhere in the population.
Because audit sampling involves drawing valid conclusions about the whole
population based on the sample, block selection ordinarily cannot be applied
in audit sampling.

62. In systematic selection, the number of sampling units in the population is divided by
the sample size to determine the
A. Sampling interval
B. Pattern that may exist in the population
C. Sampling risk
D. Nonsampling risk
In systematic selection, the number of sampling units in the population is
divided by the sample size to determine the sampling interval. For example,
20 will be the sampling interval in a population of 5,000 sampling units within
which a sample of 250 items will be selected (5,000 / 250). After selecting a
starting point within the sampling interval of 20, the auditor will then select
every 20th item.
63. In audit sampling, ____________ involves dividing the population into discrete
sub-populations which have an identifying characteristic.
A. Value-weighted selection
B. Stratification
C. Random selection
D. Block selection
The auditor may stratify a population by dividing it into discrete sub-
populations which have an identifying characteristic.
Stratification of the population may improve audit efficiency because each
stratum will contain homogeneous items that will allow selection of smaller
sample size without increasing sampling risk.

64. Which of the following statements relating to stratification is incorrect?


A. When performing tests of details of transactions and account balances, the population is often
stratified by monetary value.
B. The results of audit procedures applied to a sample of items within a stratum can be projected to
the entire population.
C. When verifying the valuation assertion for accounts receivable, account balances mat be
stratified by age.
D. Stratification reduces the variability of items within each stratum.
When using stratified sampling technique, misstatement is projected for each
stratum separately. The projected misstatements are then combined to
determine the possible effect on the entire population.

65. Audit efficiency may be improved when the sampling unit is defined as the
individual monetary units that comprise the
population. This technique is called
A. Stratification
B. Random selection
C. Systematic selection
D. Value-weighted selection
Value-weighted selection identifies the sampling nit as the individual
monetary units that make up the population. Under this method, audit effort
may be directed to the larger value items because they will have a greater
chance of being selected. This can result in smaller sample sizes and may thus
improve audit efficiency.
TRUE OR FALSE

1. Nonsampling risk is the risk that audit tests will not uncover existing
exceptions in a sample.
2. For a given tolerable deviation rate, a larger sample size should be selected as
the expected population deviation rate decreases.

3. The tolerable deviation rate for a test of controls is generally lower than the
expected rate of deviations in the related accounting records.

4. In statistical on nonstatistical sampling methods used in substantive testing, an


auditor most likely would stratify a population into meaningful groups if the
population contains both very high and very low recorded amounts.

5. If sample results indicate that the control is operating effectively, but in fact it
is not, control risk will be assessed too high.

6. To determine if a sample is truly representative of the population, an auditor


would be required to use systematic sample selection.

7. The risk of incorrect acceptance relates to the effectiveness of the audit.

8. One of the ways to eliminate nonsampling risk is through the use of attributes
sampling rather than variables sampling.
9. As the amount of misstatements expected in the population approaches
tolerable misstatement, the planned sample size will increase.

10. The auditor’s principal objective when using a sample of test of details of
balances is whether the transactions being
audited are free of misstatements

11. The computed upper deviation rate is the sum of the sample deviation rate and
an appropriate allowance for sampling risk.

12. When selecting a stratified sample, the sample size is determined for each
stratum and selected randomly from the entire unstratified population.

13. Statistical sampling provides a technique for measuring the sufficiency of


evidential matter.
14. Sampling risk is the risk that audit tests will not uncover existing exceptions
in a sample.

15. Auditors who prefer statistical sampling to nonstatistical sampling may do so


because statistical sampling helps the auditor eliminate subjectivity in the
evaluation of sampling results.
16. Sampling risk is the risk than an auditor will reach an incorrect conclusion
because a sample is not representation of the population.

17. The primary objective of using stratified sampling in auditing is to determine


the occurrence rate for a given characteristic in the population being studied.
18. A sample of all items of a population will eliminate sampling risk, but
increase nonsampling risk.

19. The use of inappropriate audit procedures is a significant cause of


nonsampling risk.

20. The use of an appropriate sample selection technique ensures a representative


sample.

21. The process which requires the calculation of an interval and then selects the
items based on the size of the interval is random sample selection.

22. Correspondence is established between the random number table and the
population by deciding the number of digits to use in the random number table
and their association with the population numbering system.

23. It is impossible to draw a six-digit random number from a table that is


separated into columns of five digits.
24. To determine the sample size for a test of controls, an auditor should consider
the tolerable deviation rate, the desired confidence level, and the expected
population deviation rate.

25. When selecting a three-digit number from a table that is separated into
columns of five digits, it is permissible to use the first three digits, the middle
three digits, or the last three digits.

KEY ANSWERS
1. D 10. C
6. C
2. C 11. D
6. C
3. A 12. A
6. D
4. C 13. B
6. B
5. B 14. B
32. C
15. B 50. B
33. D
16. D 51. D
33. A
17. C 52. A
33. C
18. A 53. C
36. A
19. A 54. A
37. D
20. D 55. B
37. D
21. D 56. B
37. B
22. A 57. C
37. A
23. D 58. A
24. C 41. D
59. C
25. A 42. B
60. D
26. B 43. D
61. C
27. C 44. A
62. A
28. A 45. B
63. B
29. B 46. C
47. C 64. B
30. B
48. A 65. D
31. B
49. A

TRUE OR FALSE

1. True

2. False
3. False

4. True

5. False
6. False
7. True

8. False
9. True

10. False
11. True
12. False
13. True
14. False
15. False
16. True

17. False

18. False
19. True
20. False

21. False
22. True

23. False
24. True

25. True
CHAPTER 9
COMPLETING THE AUDIT AND POST-AUDIT
RESPONSIBILITIES
Analytical Procedures

1. Analytical procedures used in the overall review stage of the audit generally
include:
A. Retesting controls that appeared to be ineffective during the assessment of
control risk.
B. Considering unusual or unexpected account balances that were not previously
identified.
C. Gathering evidence concerning account balances that have not changed from the
prior year.
D. Performing tests of transactions to corroborate management’s financial
statement assertions.

PSA 520 (Analytical Procedures) states, “The auditor shall design and
perform analytical procedures near the end of the audit that assist the auditor
when forming an overall conclusion as to whether the financial statements
are consistent with the auditor’s understanding of the entity.”

Analytical procedures used in the final review stage of the audit are intended
to corroborate the conclusions formed during the audit of individual
components or elements of financial statements. They assist in arriving at
the overall conclusion as to the reasonableness of the financial statements.
Moreover, analytical procedures may also identify a previously
unrecognized risk of material misstatement.

Analytical procedures applied as an overall review in the completion stage of the


audit typically include reading the financial statements and accompanying notes
and considering:

1) Unusual or unexpected account balances or relationships that were not


previously identified; and
2) The adequacy of evidence regarding previously identified unusual or
unexpected balances.
Answer A is incorrect because analytical procedures are not tests of controls.
Answer C is incorrect because there may be nothing unusual or unexpected
concerning account balances that have not changed from the prior year.
Answer D is incorrect because analytical procedures are not tests of transactions.

2. Analytical procedures performed in the overall review stage of an audit


suggest that several accounts have unexpected relationships. The result of
these procedures most likely indicate that:
A. The communication with the audit committee should be revised.
B. Irregularities exist among the relevant account balances.
C. Additional substantive tests of details are required.
D. Internal control activities are not operating effectively.
The auditor should perform analytical procedures in the final review stage of
the audit in order to assess the conclusions reached and evaluate the overall
financial statement presentation.
When analytical procedures disclosed significant fluctuations or relationships
that are consistent with other relevant information or that deviate from
predicted amounts, the auditor is required to investigate and obtain adequate
explanations and appropriate corroborative evidence. Thus, additional tests of
details are required to be performed.
The auditor’s investigation of unusual fluctuations begins with inquiries of
management. In turn, the auditor will perform the following:

a) Corroboration of management’s responses


b) If management is unable to provide an explanation or if the explanation is not
considered adequate, the auditor should consider the need to apply other audit
procedures based on the result of such inquiries.
Answer A is not correct because the auditor should first obtain explanations for
the unexpected relationships to determine if the communication with the audit
committee will be revised.
Answer B is incorrect because it is the auditor’s investigation, not the analytical
procedures performed, that may uncover the irregularities.

Related Party Transactions

3. The responsibility for the identification and disclosure of related parties and
transactions with such parties rests with the:
A. Auditor
B. Entity’s management
C. Financial Reporting Standards Council (FRSC)
D. Securities and Exchange Commission (SEC)

Management is responsible for the identification and disclosure of related


transactions with such parties. Management is required to implement adequate
internal control to ensure that related party transactions are appropriately
identified in the information system and disclosed in the financial statements.

4. The auditor should review information provided by those charged with


governance and management identifying
I. The names of all known parties
II. Related party transactions
A. I only
B. II only
C. Both I and II
D. Neither I nor II
The auditor should review information provided by those charged with
governance and management identifying the names of all known parties and
transactions with such parties.

5. Which of the following events most likely indicated the existence of related
parties?
A. Making a loan without scheduled terms for payment of the funds.
B. Discussing merger terms with a company that is a major competitor.
C. Selling real estate at a price that differs significantly from its book value.
D. Borrowing a large sum of money at a variable rate of interest.
The following suggest related party transactions:

• Transactions which have abnormal terms of trade, such as unusual prices,


interest rates, guarantees, and repayment terms.
• Transaction which lack an apparent local business reason for their
occurrence.
• Transactions in which substance differ from form.  Transactions
processed in an unusual manner.
• High volume significant transactions with certain customers or suppliers
as compared with others.
• Unrecorded transactions such as the receipt or provision of management
services at no charge.
Answer B is incorrect because the parties become related only after the merger
transaction has occurred.
Answer C is incorrect because usually, the real estate’s fair value is significantly
different from its book value.
Answer D is incorrect because large sums of money are normally borrowed at
variable rates of interest, particularly long-term borrowings.

6. Which of the following would not necessarily be a related party transaction?


A. A purchase from another corporation that is controlled by the corporation’s chief
shareholder.
B. A loan from the corporation to a major shareholder.
C. Sale of land to the corporation by the spouse of a director.
D. A sale to another corporation with similar name.
According to PSA 24 (Related Party Disclosures), a party is related to an entity if:

a) Directly, or indirectly through one or more intermediaries, the party:


i. Controls, is controlled by, or is under common control with, the entity
(this includes parents, subsidiaries, and fellow subsidiaries);
ii. Has an interest in the entity that gives it significant influence over the
entity; or
iii. Has joint control over the entity;
b) The party is an associate of the entity;
c) The party is a joint venture in which the entity is a venture;
d) The party is a member of the key management personnel of the entity or its
parent;
e) The party is a close member of the family of any individual referred to in (a)
or (d);
f) The party is an entity that is controlled, jointly controlled or significantly
influenced by, or for which significant voting power in such entity resides
with, directly or indirectly, any individual referred to in (d) or (e); or
g) The party is a post-employment benefit plan for the benefit of employees of
the entity, or of any entity that is a related party of the entity.
Two corporations having a similar name are not necessarily related.
Answers A, B, and C are incorrect because these transactions are considered
related party transactions.

7. Which of the following procedures should be performed by the auditor to


determine the completeness of information provided by those charged with
governance and management identifying the names of all known related
parties?

I. Review prior year’s working papers for names of known related parties.
II. Inquire as to the affiliation of those charged with governance and officers
with other entities.
III. Review minutes of the meetings of shareholders and those charged with
governance.
A. I and II only
B. II and III only
C. I and II only
D. I, II, and III
PSA 550(Related Parties0 requires the auditor to review information provided by
those charged with governance and management identifying the names of known
related parties. Moreover, the auditor is required to perform the following
procedures to determine the completeness of such information:

a) Review prior year’s working papers for the names of known


related parties.
b) Review the entity’s procedures for identification of related
parties.
c) Inquire as to the affiliation of those charged with governance
and officers with other entities.
d) Review shareholder records to determine the names of
principal shareholders or, if appropriate, obtain a listing of
principal shareholders from the share register.
e) Review of minutes of the meetings of shareholders and those
charged with governance and other relevant statutory records
such as register of directors’ interests.
f) Inquire of other auditors currently involved in the audit, or
predecessor auditors, as to their knowledge of additional
related parties.
g) Review the entity’s income tax returns and other information
supplied to regulatory agencies.

8. Which of the following statements concerning related party transactions is


correct?
A. In the absence of evidence to the contrary, related party transactions should be
assumed to be outside the ordinary course of business.
B. The audit procedures directed toward identifying related party transactions
should include considering whether transactions are occurring but are not being
given proper accounting recognition.
C. An auditor should determine whether a particular transaction would have
occurred if the parties had not been related.
D. An auditor should substantiate that related party transactions were consummated
on terms equivalent to those that prevail in arm’s-length transactions.
PSA 550 states that in examining the identified related party transactions, the
auditor should obtain sufficient appropriate audit evidence as to whether these
transactions have been properly recorded and disclosed.
Answer A is incorrect because the existence of related parties and transactions
with such parties are considered ordinary features of business.
Answer C is incorrect because, unless the transaction is routine, it is not
ordinarily possible for the auditor to determine whether a particular
transaction would have occurred if the parties had not been related.
Answer D is incorrect because related party transactions need not be
consummated on terms equivalent to those that prevail in arm’s-length
transactions.

9. An auditor searching for related party transactions should obtain an


understanding of each subsidiary’s relationship to the total entity because
A. This may permit the audit of intercompany account balances to be performed
as of concurrent dates.
B. This may reveal whether particular transactions would have taken place if
the parties had not been related.
C. The business structure may be deliberately designed to obscure related party
transactions.
D. Intercompany transactions may have been consummated on terms equivalent
to arm’s-length transactions.
While the existence of related parties and transactions with such parties and
transactions with such parties are considered ordinary features of business, the
auditor should be aware of them because a related party transaction may be
motivated by other than ordinary business considerations like profit sharing or
even fraud. Thus, the auditor should consider the possibility that the business
structure may be deliberately designed to obscure related party transactions.
Answer A is incorrect because the auditor is not required to conduct a
concurrent audit.
Answer B is incorrect because determining whether particular transactions
would have occurred if the parties had not been related is ordinarily not
possible, except for routine transactions.
Answer D is incorrect because related party transactions need not be
consummated on terms equivalent to arm’s-length transactions.

10. After determining that a related party transaction has, in fact, occurred an
auditor should
A. Obtain an understanding of the business purposes of the transaction.
B. Substantiate that the transaction was consummated on terms equivalent to
an arm’s-length transaction.
C. Add a separate paragraph to the auditor’s report to explain transaction.
D. Perform analytical procedures to verify whether similar transactions
occurred, but were not recorded.
After identifying related party transactions, the auditor should obtain sufficient
appropriate audit evidence to determine whether such transactions have been
properly recorded and disclosed. The auditor should become satisfied about their
purpose, nature, extent, and effect. Therefore, the auditor should obtain an
understanding of the business purpose of an identified related party transaction.
Answer B is incorrect because the auditor’s primary concern is to determine
whether a related party transaction has been properly recoded and disclosed, not
whether such transaction was consummated on terms equivalent to an arm’s-
length transaction.
Answer C is incorrect because no modification of the report is necessary if the
related party transaction has been properly recorded and disclosed. However, the
auditor may add an emphasis of matter paragraph to the audit report to emphasize
that the entity has had significant related party transactions.
Answer D is incorrect because management is responsible to ensure that the
related party transactions are appropriately identified in the entity’s information
system and disclosed in financial statements. Thus, the auditor is not responsible
for undisclosed, unrecorded related party transactions.

11. Which of the following audit procedure is most likely to assist an auditor in identifying related party
transactions?

A. Inspecting communications with law firms for evidence of unreported contingent


liabilities.
B. Reviewing accounting records for nonrecurring transactions recognized near the
balance sheet date.
C. Retesting ineffective controls previously reported to the audit committee.
D. Sending second requests for unanswered positive confirmations of accounts
receivable.

The following are examples of audit procedures which may identify the existence of
related party transactions:
Performing detailed tests of transaction balances.
Reviewing minutes of meetings of shareholders and those charged with
governance.
Reviewing accounting records for large or unusual transactions or balances,
paying particular attention to transactions recognized at or near the end of
reporting period.
Reviewing confirmations of loans receivable and payable and confirmations from
banks. Such a review may indicate a guarantor relationship and other related party
transactions.
Reviewing investment transactions, for example, purchase or sale of an equity
interest in a joint venture or other entity.
Answer A is incorrect because the purpose of inspecting communications with
law firms for evidences of unreported contingent liabilities is to identify potential
litigation, claims, and assessment that may require disclosure in the financial
statements.
Answer C is incorrect because the auditor should not retest ineffective controls
previously reported to the audit committee.
Answer D is incorrect because confirmation of accounts receivable do not
normally provide audit evidence about the existence of related party transactions.

12. For a reporting entity that has participated in related party transactions that
are material, disclosure in the financial statements should include:
A. A reference to deficiencies in the entity’s internal control.
B. A statement to the effect that a transaction was consummated on terms
equivalent to those that prevail in arm’s-length transactions.
C. The nature of the relationship and the terms and manner of settlement.
D. Details of the transactions within major classifications.
PAS 24 (Related Party Disclosures) provides that if an entity has participated in
material related party transactions, it should disclose the nature of the
relationship, information about the transaction, and outstanding balances
necessary for an understanding of the potential effect of the relationship on
financial statements.
As minimum, the disclosures should include:

o The amount of the transaction. o The amount of


outstanding balances, their terms and conditions, whether
secured or unsecured, and the nature of the consideration to be
provided in settlement.
o Provision for doubtful accounts related to the
outstanding balances. o The expense recognized during the
period in respect of doubtful accounts due from related parties.
Answer A is incorrect because the company is not required to make financial
statement disclosures about its internal control.
Answer B is incorrect because a statement to the effect that a transaction was
consummated on terms equivalent to those that prevail in arm’s-length
transactions is not required. This representation shall be made only if such terms
can be substantiated.
Answer D is incorrect because there is no requirement to segregate the transactions
into major classifications.

SUBSEQUENT EVENTS REVIEW

13. As used in PSA 560 (Subsequent Events), the term


“subsequent events” refer to
I. Events occurring between the date of the financial statements and the date
of auditor’s report.
II. Facts that become known to the auditor after the date of the auditor’s
report.
A. I only
B. II only
C. Both I and II
D. Neither I nor II
According to PSA 560, the term “subsequent events” refers to events occurring
between the date of financial statements and the date of auditor’s report, and facts
that became known to the auditor after the date of the auditor’s report.

14. Which of the following statements best describe the “date of the financial
statements?”
A. The date on which those with the recognized authority assert that they have
prepared the entity’s complete set of financial statements, including the
related notes, and that they have taken responsibility for them.
B. The date that the auditor’s report and audited financial statements are made
available to third parties.
C. The date of the end of the latest period covered by the financial statements.
D. The date on which the auditor has obtained sufficient appropriate audit
evidence on which to base the opinion on financial statements.
According to the standard, the date of the financial statements is the date of the
end of the latest period covered by the financial statements.
Answer A is incorrect because it is the date of the approval of the financial
statements.
Answer B is incorrect because it is the date of issuance of the financial statements.
Answer D is incorrect because it is usually the date of the auditor’s report.

15. The auditor is required to perform procedures designed to obtain sufficient


appropriate audit evidence to identify all events that may require adjustment
of, or disclosure
in, the financial statements up to the
A. Date of auditor’s report
B. Date of approval of the financial statements
C. Date the financial statements are issued.
D. Date of the financial statements
The standard requires the auditor to perform audit procedures designed to obtain
sufficient appropriate audit evidence that all events up to the date of auditor’s
report that may require adjustment of, disclosure in, the financial statements may
have been identified.

16. Which of the following procedures would an auditor most likely perform to
obtain evidence about the occurrence of subsequent events?
A. Inquiring as to whether any unusual adjustments were made after the date
of financial statements.
B. Confirming a sample of material accounts receivable after the date of
financial statements.
C. Comparing the financial statements being reported on with those prior
period.
D. Investigating personnel charges in the accounting department occurring
after the date of the financial statements.

The audit procedures to identify subsequent events ordinarily include the following:

• Reviewing the entity’s established procedures to identify subsequent


events.
• Reviewing minutes of meetings of shareholders, the board of directors,
and committees held subsequent to the date of the financial statements
and inquiring about matters discussed at meetings for which minutes
are not yet available.
• Reading the entity’s latest available interim financial statements,
appropriate budgets, and forecasts, and other related management
reports.
• Inquiring or extending previous oral or written inquiries, of the entity’s
lawyers concerning claims and litigation.
• Inquiring of management as to whether any subsequent events have
occurred which might affect the financial statements. Examples of
such inquiries are: o The current status of items that were accounted
for on the basis of preliminary or inconclusive data. o Whether new
commitments, borrowings, or guarantees have been entered into.
o Whether sales of assets have occurred or are planned.
o Whether the issue of new shares or debentures or other
agreement to merge or liquidate has been made or is planned. o
Whether any assets have been appropriated by the government or
destroyed, for example, by fire or flood. o Whether there have
been any developments regarding risk areas and contingencies.
o Whether any unusual accounting adjustments have been
made or are contemplated. o Whether any events have occurred or
are likely to occur in which will bring into question the
appropriateness of the accounting policies adopted by the entity,
for example, events that may call into question the validity of the
going concern assumption.
Answer B is incorrect because confirmation of receivables is ordinarily performed
before the date of the financial statements.
Answer C is incorrect because comparison of the financial statements being
reported on with those of the prior period is an analytical procedure that is
performed in the planning stage of an audit.
Answer D is incorrect because personnel charges occurring after the date of the
financial statements are not considered significant subsequent events that will
require adjustment of, disclosure in, the financial statements.

17. Which of the following procedures should an auditor ordinarily perform


regarding subsequent events?
A. Review the cut-off bank statements for several months after the year-end
B. Compare the latest available interim financial statements with the financial
statements being audited.
C. Send second requests to the client’s customers who failed to respond to
initial accounts receivable confirmation requests.
D. Communicate material weaknesses in internal control to the client’s audit
committee.
An analytical procedure that is designed to identify subsequent events in
comparison to the latest interim financial statements with those statements being
reported on.
Answer A is incorrect because the auditor reviews cutoff bank statements (usually
7 to 10 days from the balance sheet date) to verify year-end bank reconciliations.
Answer C is incorrect because confirmation or receivables typically do not relate to the
recording of subsequent events.
Answer D is incorrect because communicating material internal control
weaknesses to the audit committee is not a subsequent events procedure.

18. An auditor is concerned with completing various phases of the audit after
the balance sheet date. This subsequent period extends to the date of the
A. Delivery of the auditor’s report to the client
B. Auditor’s report
C. Final review of the audit working papers
D. Public issuance of financial statements
The auditor is required to perform procedures designed to identify all subsequent
events up to the date of the auditor’s report. Hence, the subsequent events period
extends from the date of the financial statements (i.e., the balance sheet date) to
the date of the auditor’s report.
Answers A and D are incorrect because the delivery of the auditor’s report to the
client and the public issuance of the financial statements both occur later than the
date of the auditor’s report.
Answer C is incorrect because the auditor is required to perform subsequent
events worked up to the date of the auditor’s report.

19. Which of the following statements best expresses the auditor’s


responsibility with respect to facts which become known to the auditor
after the date of the auditor’s report but before the date the financial
statements are issued?
A. The auditor should amend the financial statements.
B. If the acts discovered will materially affect the financial statements, the auditor
should issue a new report which contains either a qualified opinion or an adverse
opinion.
C. The auditor should consider whether the financial statements need amendment,
discuss the matter with management, and consider taking actions appropriate in the
circumstances.
D. The auditor should withdraw from the engagement.
PSA 560(Subsequent Events) provides that an auditor does not have any
responsibility to perform procedures or make any inquiry regarding the financial
statements after the date of the auditor’s report.
It provides further that during the period from the date of the auditor’s report to
the date the financial statements are issued, the entity’s management has the
responsibility to inform the auditor of facts which may affect the financial
statements.
If during that period the auditor becomes aware of a fact which may materially
affect the financial statements, he/she should consider whether the financial
statements need amendment, discuss the matter with management, and take the
action appropriate in the circumstances.
Answer A is incorrect because, if necessary, the entity’s management, not the
auditor, should amend the financial statements.
Answer B is incorrect because expressing a qualified opinion or an adverse
opinion is appropriate when management does not amend the financial statements
in circumstances where the auditor believes they need to be amended and the
auditor’s report has not been released to the entity. When the auditor’s report has
been released to the entity, the auditor would notify those persons ultimately
responsible for the overall direction of the entity not to issue the financial
statements and the auditor’s report to third parties.
Answer D is incorrect because the discovery of facts after the date of the auditor’s
report does not necessarily require withdrawal from the engagement.

20. After issuing a report, an auditor has no obligation to make continuing


inquiries or perform other procedures concerning the audited financial
statements, unless
A. Final determinations or resolutions are made of contingencies that had
been disclosed in the financial statements.
B. Information about an event that occurred after the date of the auditor’s
report comes to the auditor’s attention.
C. The control environment changes after the issuance of the report.
D. Information, which existed at the report date and may affect the report,
comes to the auditor’s attention.
PSA 560 provides that after the financial statements and the auditor’s report have
been issued, the auditor has no obligation to make any inquiry concerning such
financial statements.
But, if the auditor becomes aware of a fact which existed at the date of the
auditor’s report which, if known at that date, may have caused the auditor to
modify the auditor’s report, the auditor should consider whether the financial
statements need amendment, discuss the matter with management, and take the
action appropriate in the circumstances.
Answers A and C are incorrect because after the report is issued, the auditor has
no responsibility to consider changes in the entity’s internal control or final
determinations or resolutions of contingencies.
Answer B is incorrect because the auditor does not have reporting responsibility
for events that occurred after the date of the report.

21. Which of the following events occurring after the issuance of an auditor’s
report most likely would cause the auditor to make further inquiries about
the previously issued financial statements?
A. A technological development that could affect the entity’s future ability
to continue as a going concern.
B. The entity’s sale of a subsidiary that accounts for 30% of the entity’s
consolidated sales
C. The discovery of information regarding a contingency that existed
before the financial statements were issued
D. The final resolution of a lawsuit disclosed in the notes to the financial
statements.
If, after the report has been issued, the auditor becomes aware of the fact which
existed at the report date and which, if known at that date, may have caused the
auditor to modify the report, the auditor should consider whether the financial
statements need revision, discuss the matter with management, and take actions
appropriate in the circumstances.
Answers A and B are incorrect because the auditor has no reporting responsibility
for events occurring after the issuance of the auditor’s report if such events would
not affect the report.
Answer D is incorrect because the auditor has no responsibility to monitor
disclosed contingencies after the report is issued.

22. After an audit report containing an unmodified opinion on a client’s


financial statements was issued, the client decided to sell the shares of a
subsidiary that accounts for 30% of its revenue and 25% of its net income.
The auditor should
A. Describe the effects of this subsequently discovered information in a
communication with persons known to be relying on financial
statements.
B. Take no action because the auditor has no obligation to make further
inquiries.
C. Determine whether the information is reliable and, if determined to be
reliable, request that revised financial statements be issued.
D. Notify the entity that the auditor’s report may no longer be associated
with the financial statements.
After the report has been issued, the auditor has no responsibility to make any
further or continuing inquiry regarding the financial statements covered by the
report, unless facts that may affect the report come to his/her attention.
Answers A, C, and D are incorrect because these actions might be appropriate if
the auditor has discovered facts that existed at the date of the report.
ASSESSMENT OF GOING CONCERN ASSUMPTION

23. PSA 570 (Going Concern) states that a fundamental principle in the
preparation of financial statements is the going concern assumption.
Under this assumption, an entity is ordinarily viewed as continuing
business for the foreseeable future with neither the intention nor the
necessity of liquidation, ceasing trading or seeking protection from
creditors pursuant to laws and regulations. The responsibility to make an
assessment of an entity’s ability to continue as a going concern rests with
the
A. Auditor
B. Entity’s management
C. Securities and Exchange Commission (SEC)
D. Entity’s creditors
Under PAS 1(Presentation of Financial Statements), management is required to
make an assessment of an entity’s ability to continue as a going concern.

24. Which of the following statements best describes the auditor’s


responsibility concerning the appropriateness of the going concern
assumption in the preparation of financial statements?
A. The auditor’s responsibility is to make a specific assessment of the
entity’s ability to continue as a going concern.
B. The auditor’s responsibility is to predict future events or conditions that
may cause the entity to cease to continue as a going concern.
C. The auditor’s responsibility is to consider the appropriateness of
management’s use of the going concern as an assumption and consider
whether there are material uncertainties about the entity’s ability to
continue as a going concern that need to be disclosed in the financial
statements.
D. The auditor’s responsibility is to give a guarantee in the audit report that
the entity has the ability to continue a going concern.
PSA 570 states that the auditor’s responsibility is to consider the appropriateness
of management’s use of the going concern assumption in the preparation of
financial statements, and consider whether there are material uncertainties about
the entity’s ability to continue as a going concern that need to be disclosed in the
financial statements.
Answer A is incorrect because the entity’s management is required to make
specific assessment of the entity’s ability to continue as a going concern.
Answers B and D are incorrect because the auditor cannot predict future events or
conditions that may cause the entity to cease to continue as a going concern.
Hence, even if the auditor’s report does not make any reference to going concern
uncertainty, it cannot be viewed as a guarantee as to the entity’s ability to continue
as a going concern.

25. Which of the following conditions or events most likely would cause an
auditor to have substantial doubt about an entity’s ability to continue as a
going concern?
A. Cash flows from operating activities are negative.
B. Stock dividends replace annual cash dividends.
C. Significant related party transactions are pervasive.
D. Research and development projects re postpones.
PSA 570 gives the following examples of conditions or events which may give
rise to business risks, that individually or collectively, may cast significant doubt
about the entity’s ability to continue as a going concern.
Financial

• The net liability or current net current liability position


• Fixed-term borrowings approaching maturity without realistic prospects of
renewal or repayment; or excessive reliance on short-term borrowings to
finance long-term assets
• Indications of withdrawal of financial support by debtors and other
creditors
• Negative operating cash flows indicated by historical or prospective
financial statements
• Adverse key financial ratios
• Substantial operating losses or significant deterioration in the value of
assets used to generate cash flows
• Arrears or discontinuance of dividends
• Inability to pay creditors on due dates
• Inability to comply with the terms of loan agreements
• Change from credit to cash-on-delivery (COD) transactions with suppliers
• Inability to obtain financing for essential new product development or
other essential investments
Operational

• Loss of key management personnel


• Loss of major market, franchise, license, or principal supplier
• Labor difficulties or shortages of important supplies

Other

• Non-compliance with capital or other statutory requirements


• Pending legal or regulatory proceedings against the entity that may, if
successful, result in claims that are likely to be satisfied.
• Changes in legislation or government policy expected to adversely
affect the entity
Answer B is incorrect because payments of stock dividend will not cause going
concern problems since they do not require outflow of cash.
Answer C is incorrect because the auditor’s concern with respect to related party
transactions is the adequacy of disclosure in financial statements.
Answer D is incorrect because postponing R&D projects will not cast significant
doubt about the entity’s ability to continue as a going concern.

26. Which of the following audit procedures most likely would assist an
auditor in identifying conditions and events that may indicate substantial
doubt about an entity’s ability to continue as a going concern?
A. Confirming with third parties the details of arrangements to maintain
financial support
B. Comparing the entity’s depreciation and asset capitalization policies to other
entities in the industry
C. Reconciling the cash balance per books with the cutoff bank statement and
the bank confirmation
D. Inspecting the title documents to verify whether any assets are pledged as
collateral
The auditor typically performs the following procedures to identify conditions
and events that may cast significant doubt about an entity’s ability to continue as a
going concern:

o Analytical procedures o Subsequent events review


o Review of compliance with debt and loan
agreements
o Reading minutes of meetings o Inquiry of legal
counsel
o Confirmation with related and third parties of
arrangements for financial support
Answer B is incorrect because comparing the entity’s accounting policies to other
entities would not provide evidence about the entity’s continued existence.
Answer C is incorrect because bank reconciliation procedures would be performed to
test the existence of cash.
Answer D is incorrect because inspecting title documents to verify whether any
assets are pledged as a collateral relates to disclosure, not going concern, issues.

27. Which of the following conditions or events most likely would cause an
auditor to have substantial doubt about an entity’s ability to continue as a
going concern?
A. Restrictions on the disposal of principal assets present
B. Usual trade credit from suppliers denied
C. Significant related party transactions are pervasive.
D. Arrearages in principal stock dividends are paid.
Denial of usual trade credit from suppliers is an indication that the entity is facing
financial difficulties.
Answer A is incorrect because long-term debt arrangements ordinarily include
restrictions on the disposal of principal assets.
Answer C is incorrect because the existence of related party transactions concerns disclosure,
not going concern, issues.
Answer D is incorrect because the payment of stock dividends in arrears does not
indicate that the entity is financially distressed.

28. Which of the following audit procedures would most likely assist an
auditor in identifying conditions and events that may indicate there could
be substantial doubt about an entity’s ability to continue as a going
concern?
A. Confirmation of bank balances
B. Confirmation of accounts receivable from major customers
C. Reconciliation of interest expense with debt outstanding
D. Review of compliance with terms of debt agreements
The auditor’s review of compliance with terms of debt agreements may reveal
conditions of non-compliance because the entity is in weak financial condition.
Answers A, B, and C are incorrect because these procedures are less effective if
the auditor’s objective is to identify conditions or events that may cast significant
doubt about the entity’s ability to continue as a going concern.

29. Harold, CPA, believes there is substantial doubt about the ability of ABC
Co. to continue as a going concern for a reasonable period of time. In
evaluating ABC’s plans for dealing with adverse effects of future
conditions and events, Harold most likely would consider, as a mitigating
factor, ABC’s plans to
A. Postpone expenditures for research and development projects.
B. Purchase production facilities currently being leased from a related party.
C. Strengthen internal controls over cash disbursements
D. Discuss with lenders the terms of all debt and loan agreements.
If the auditor believes that there are conditions or events indicating that
substantial doubts exists about an entity’s ability to continue as a going concern,
he/she should inquire to the management concerning its plans to mitigate their
adverse effects. For example, management’s plans to dispose of assets, borrow
money, or restructure debt, reduce or delay expenditures, or increase capital. The
auditor should obtain sufficient appropriate audit evidence that management’s
plans are feasible and that the outcome of these plans will improve the situation.
Answer B is incorrect because the purchase of production facilities will worsen the
company’s weak financial condition.
Answer C is incorrect because improvements in internal control will not solve the
company’s going concern problems.
Answer D is incorrect because discussion with lenders is not a sufficient action to improve the
situation.

30. Harry, CPA, believes that there is substantial doubt about the ability of
ABC Corp. to continue as a going concern for a reasonable period of time.
In evaluating ABC’s plans for dealing with the adverse effects of
future conditions and events, Harry most likely would consider, as a mitigating
factor, ABC’s plans to
A. Purchase equipment and production facilities currently being leased.
B. Accumulate treasury shares at prices favorable to ABC’s historic price
range.
C. Negotiate reductions in required dividends being paid on preference
shares.
D. Accelerate research and development projects related to future
products.
The auditor should inquire of management concerning its plans to mitigate
adverse effects of identified conditions or events indicating that a substantial
doubt exists about an entity’s ability to continue as a going concern.
The auditor should consider management’s plans to dispose of assets, borrow
money or restructure debt, reduce or delay expenditures, and increase capital.
Management’s plans to negotiate reductions in required dividends being paid on
preference shares are intended to increase capital.
Answers A, B, and D are incorrect because leasing equipment and production
facilities, increasing capital, and postponement of R&D projects would be
mitigating factors.

31. When an auditor concludes that there is substantial doubt about a


continuing audit client’s ability to continue as a going concern for a
reasonable period of time, the auditor’s responsibility is to
A. Consider the adequacy of disclosure about the client’s possible inability
to continue as a going concern.
B. Issue a qualified or adverse opinion, depending upon materiality, due to
the possible effects on the financial statements.
C. Report to the client’s audit committee that management’s accounting
estimates may need to be adjusted.
D. Reissue prior year’s auditor’s report and add an emphasis of matter
paragraph specifically referring to “substantial doubt” and “going
concern”.
PSA 570 provides that if the use of the going concern assumption is appropriate
but a material uncertainty exists, the auditor considers whether the entity’s
financial statements:

a) Adequately describe the principal events or conditions that may


give rise to the significant doubt of an entity’s ability to
continue in operation and management’s plans to deal with
these events and conditions; and
b) Disclose clearly that there is material uncertainty related to
events or conditions which may cast significant doubt on the
entity’s ability to continue as a going concern, and, therefore,
that it may be unable to realize its assets and discharge its
liabilities in the normal course of business.

Answer B is incorrect because if adequate disclosure is made in financial


statements, the auditor may still express an unmodified opinion. However, the
auditor’s report should be modified by adding an emphasis of matter paragraph
that highlights the going concern issue.
Answer C is incorrect because the going concern issue is not directly related to
management’s accounting estimates that may need adjustment.
Answer D is incorrect because there is no need to change the prior year’s audit
report since the auditor does not intend to change his/her opinion on the prior
year’s financial statements.

OBTAINING WRITTEN REPRESENTATIONS FROM MANAGEMENT

32. Under PSA 580 (Written Representations), the auditor is required to obtain audit
evidence that management
I. Has fulfilled its responsibility for the fair presentation of the
financial statements in accordance with applicable financial
reporting framework.
II. Has provided the auditor with all relevant information and access
as agreed in the terms of audit engagement.
A. I only
B. II only
C. Both I and II
D. Neither I nor II
According to PSA 580, the auditor shall request management to provide a written
representation that it has fulfilled its responsibility for the preparation of financial
statements in accordance with the applicable financial reporting framework,
including where relevant, their fair presentation as set out in the terms of the audit
engagement.
The standard further provides that the auditor shall request management to provide a written
representation that:

• It has provided the auditor with all relevant information and access as
agreed in the terms of audit engagement; and
• All transactions have been recorded and are recorded and are reflected
in the financial statements.

33. The date of the written representation shall be:


A. After the date of the auditor’s report
B. After the date of approval of the entity’s financial statements
C. Before the entity’s financial statements are issued.
D. As near as practicable to, but not after the date of the auditor’s report on the
financial statements.

34. A purpose of a management representation letter is to reduce


A. The possibility of a misunderstanding concerning management’s
responsibility for the financial statements
B. The scope of an auditor’s procedures concerning related party transactions
and subsequent events
C. Audit risk to an aggregate level of misstatement that could be considered
material
D. An auditor’s responsibility to detect material misstatements only to the
extent that the letter is relied on
The auditor is required to obtain written representations from management
concerning matters material to the financial statements when other sufficient
appropriate audit evidence cannot reasonably be expected to exist. According to
the standard, the possibility of a misunderstanding between the auditor and the
management is reduced when oral representations are confirmed by management
in writing.
Answers B, C, and D are incorrect because management’s written representations
cannot be a substitute for audit evidence that can be obtained by performing audit
procedures.

35. When an audit is made in accordance with PSAs, the auditor should always
A. Observe the taking of physical inventory on the balance sheet date
B. Obtain certain written representations from the management
C. Employ analytical procedures as substantive tests to obtain evidence about
specific assertions related to account balances
D. Document the understanding of the client’s internal control and the basis for
all conclusions about the assessed level of control risk for financial
statement assertions

PSA 580 requires the auditor to obtain certain written representations from
management. They confirm oral representations – either unsolicited or in response
to specific inquiries – received from management.
Answer A is incorrect because attendance at the physical inventory taking can be
performed either during or after the end of the period covered by the audit like
when well-kept perpetual inventory records are periodically compared with
physical counts.
Answer C is incorrect because the auditor is required to apply analytical
procedures as risk assessment procedures to obtain an understanding of the entity
and its environment and in the overall review at the end of the audit. Analytical
procedures may also be applied as substantive procedures.
Answer D is incorrect because the auditor is not required to document the basis if
control risk is assessed at the maximum level for some assertions.

36. A written management representation letter is most likely to be an


auditor’s best source of corroborative information of a client’s intent to
A. Settle an outstanding lawsuit for an amount less than the accrued loss contingency
B. Discount a line of business
C. Terminate an employee pension plan
D. Make a public hearing of its ordinary share capital
Written management representations do not substitute for other audit evidence
that the auditor could reasonably expect to be available. However, in some
circumstances, audit evidence other than that obtained from inquiries may not be
reasonably expected to be available. For example, the auditor’s procedures mayb
provide little or no evidence to corroborate management’s intention to discontinue
a line of business. Accordingly, written representations will be necessary to
confirm management’s intent.
Answers A, C, and D are incorrect because other audit evidence is expected to be
available to corroborate management’s intent to settle an outstanding lawsuit,
terminate an employee pension plan, or make a public offering of an entity’s
ordinary share capital.

37. When considering the use of management’s written representations as


audit evidence about the completeness assertion, an auditor should
understand that such
representations
A. Constitute sufficient appropriate audit evidence to support the assertion
when considered in combination with a sufficiently low assessed level of
control risk.
B. Are not part of the audit evidence considered to support assertion
C. Replace a low assessed level of control risk as audit evidence to support
the assertion
D. Complement, but do not replace, substantive tests designed to support the
assertion.
Management representations cannot be a substitute for other audit evidence that
the auditor could reasonably expect to be available. Thus, they complement, but
do not replace, other audit evidence to corroborate management’s assertions.
Answer A is incorrect because, regardless of the assessed level of control risk, the
auditor should perform substantive tests to verify management’s assertions and
detect misstatements.
Answer B is incorrect because management’s written representations are considered
audit evidence.
Answer C is incorrect because management’s written representations
complement, but do not replace, substantive tests of account balances, transaction
classes, and disclosure complements of financial statements. Control risk is
assessed only to determine the acceptable level of detection risk, which, in turn, is
considered in substantive tests.

38. The written representations shall be in the form of representation letter


addressed to the
A. Entity’s management
B. Auditor
C. Entity’s chief executive officer
D. Entity’s chief financial officer
39. A written representation from a client’s management that, among other matters.
Acknowledges responsibility for the fair presentation of financial statements,
should normally be signed by
A. Chief financial officer and the chair of the board of directors
B. Chief executive officer and chief financial officer
C. Chief executive officer, the chair of the board of directors, and the client’s lawyer D. Chair of
the audit committee
A management’s representation letter would ordinarily be signed by the members
of the management who have primary responsibility for the entity and its financial
aspects based on their best knowledge and belief. These members are ordinarily
the senior executive officer and the senior financial officer.
However, the auditor is not precluded from obtaining written representations from
other members of the management. For example, the auditor may obtain written
representation about the completeness of the minutes of meetings of the board of
directors, stockholders, and other committees from the individual who has
custody of such minutes.

40. The following statements are ordinarily included in a management representation


letter, except
A. The completeness and availability of shareholders and directors
meetings
B. Sufficient appropriate audit evidence has been made available to permit
the expression of an unmodified opinion
C. There have been no irregularities involving management or employees
who have a significant role in internal control or that could have a
material effect on the financial statements
D. The financial statements are free of material misstatements, including
omissions
The auditor should perform audit procedures to gather sufficient appropriate audit
evidence on which the opinion is to be based.
Answers A, C, and D are incorrect because they include materials that are
ordinarily included in a management’s representation letter.

41. What type of opinion is the most appropriate when management does not provide
written representations about its responsibility for the presentation of financial
statements? A. Qualified opinion
B. Disclaimer of an opinion
C. Adverse opinion
D. Unmodified opinion

42. To which of the following matter would materiality limits not apply in obtaining
written management representations?
A. Reductions of obsolete inventory to net realizable value
B. The disclosure of compensating balance arrangements involving related
parties
C. Losses from purchase commitments at prices in excess of market value
D. The availability of minutes of shareholders’ and directors’ meetings
Written representations from management may be limited to matters that are
considered either individually or collectively material to the financial statements.
With respect to certain items, it may be necessary for the auditor to inform
management of his/her understanding of materiality.
Materiality limits do not apply to management’s representations concerning the
availability of minutes of shareholders’ and directors’ meetings since it is
independent of amounts in the financial statements.

43. Which of the following statements concerning management representations is


incorrect?
A. Representations by management can be a substitute for other audit
evidence that the auditor could reasonably be expected to be available
B. If the auditor is unable to obtain sufficient appropriate audit evidence
regarding a matter, which has, or may have, material effect on the
financial statements and such audit evidence is expected to be available,
this will constitute a limitation in the scope of audit, even if a
representation from management has been received on the matter
C. If a representation by management is contradicted by other audit
evidence, the auditor should investigate the circumstances and, when
necessary, reconsider the reliability of other representations by
management
D. The auditor’s working papers would ordinarily include a summary of
oral discussions with management or written representations from
management
According to the standard, representations by management cannot be a substitute
for other audit evidence that the auditor could reasonably expect to be available.
For example, a representation by management as to the cost of an asset is not a
substitute for the audit evidence of such cost that an auditor would ordinarily
expect to obtain.
44. If management does not provide one or more of the questioned written
representations, the auditor shall I. Discuss the matter with the management.
II. Reevaluate the integrity of the management and evaluate the effect
that this may have on the reliability of representations (oral or
written) and audit evidence in general
III. Take appropriate actions, including determining the possible effect
on the opinion in the auditor’s report
A. I only
B. I and II only
C. I and III only D. I, II, and III

45. Which of the following statements concerning written management


representations is incorrect?
A. The written representations shall be in the form of a representation
letter addressed to the entity’s chief executive officer and chief financial
officer.
B. The auditor shall request written representations from management with
appropriate responsibilities for the financial statements and knowledge
of the matters concerned.
C. The auditor may consider it necessary to request management to
provide a written representation that it has communicated to the auditor
all deficiencies in internal control of which management is aware of.
D. Audit evidence obtained during the audit that management has fulfilled
its responsibility for the presentation of the financial statements in
accordance with the applicable financial reporting framework is not
sufficient without obtaining confirmation from management that it
believes that it has fulfilled its responsibility.

PROCEDURES REGARDING LITIGATING CLAIMS

46. The primary source of information to be reported about litigation, claims, and
assessments is the
A. Independent auditor
B. Client’s management
C. Court records
D. Client’s lawyer
Management should adopt policies and procedures to identify, evaluate, and
account for litigation, claims, and assessments as basis for the preparation of
financial statements in accordance with the applicable framework.
Answer A is incorrect because the auditor’s responsibility is to perform audit
procedures in order to be aware of any litigation and claims involving the entity
that may result in a material misstatement of financial statements (PSA 501, Audit
Evidence – Additional Considerations for Specific Items).
Answer C is incorrect because there is no requirement to examine court orders.
Answer D is incorrect because the client’s lawyer corroborates the information
furnished by the client’s management.

47. Which of the following procedures is least likely to be performed by the


auditor to identify litigation and claims involving the entity which may
result in a material misstatement of financial statements?
A. Conform directly to the client’s lawyer that all claims have been recorded
in the financial statements.
B. Make appropriate inquiries of management including obtaining
representations.
C. Examine legal expense accounts.
D. Use any information regarding the entity’s business including information
obtained from discussions with any in-house legal department.
A loss contingency is recognized in the financial statements only if it is probable
and the amount can be reasonably estimated. Therefore, not all contingent claims
are recognized in the financial statements.
Answers B, C, and D are incorrect because they describe procedures that the
auditor would have to carry out to identify litigation and claims involving the
entity.

48. The primary reason an auditor requests that letters of inquiry be sent to a
client’s attorney is to provide the auditor with
A. A description and evaluation of litigation, claims, and assessments that existed in the balance
sheet date
B. The attorney’s opinions of the client’s historical experiences in recent similar litigation
C. Corroboration of information furnished by the management about litigation, claims, and
assessments
PSA 501 (Audit Evidence – Specific Cosideration for Selected Items) states that
when the auditor assesses a risk of material misstatement regarding litigation or
claims that have been identified or when the auditor believes they may exist, the
auditor should seek direct communication with the entity’s legal counsel. Such
communication will assist in obtaining sufficient appropriate audit evidence as to
whether potentially material litigation and claims are known and management’s
estimate of financial implications, including costs, are reliable.
Answers A and D are incorrect because, as stated in the standard, the letter would
ordinarily specify the following when it is considered unlikely that the entity’s
legal counsel will respond to a general inquiry:

o A list of litigation and claims o Management’s assessment of


the outcome of the litigation or claim and its estimate of the
financial implications, including costs involved
o A request that the entity’s legal counsel confirm the
reasonableness of management’s assessment and provide the
auditor with further information if the list is considered by the
entity’s legal counsel to be incomplete or incorrect.
Answer B is incorrect because the auditor is primarily concerned
with current litigation involving the entity, not the client’s
historical experiences in similar litigation.

49. The letter of audit inquiry should be


A. Prepared sent by the auditor
B. Prepared by management and sent by the auditor
C. Prepared and sent by management
D. Prepared by the auditor and sent by management
The letter of audit inquiry, which should be prepared by management and sent by
the auditor, requests the entity’s legal counsel to communicate directly with the
auditor.

50. An auditor should obtain evidence relevant to all of the following factors
concerning third-party litigation against a client except the
A. Jurisdiction in which the matter will be resolved.
B. Existence of a situation indicating an uncertainty as to possible loss
C. Profitability of an unfavorable outcome
D. Period in which the underlying cause for legal action occurred
When performing procedures after year-end concerning litigation and claims, the
auditor is primarily concerned with the impact of the matter on the fair
presentation of the client’s financial statements.
Accordingly, the auditor should obtain sufficient appropriate audit evidence about
the existence, amount, profitability of an unfavorable outcome, and timing of the
cause of the litigation or claims.
The auditor is least interested in determining the jurisdiction in which the
litigation will be resolved.
Answers B, C, and D are incorrect because the uncertainty as to the possible loss,
the probability of an outcome, and the period in which the underlying cause for
legal action occurred are matters that should be considered by the auditor to
determine the impact of litigation on the financial statements.

51. The refusal of a client’s lawyer to provide a representation on the legality of a


particular act committed by the client is ordinarily
A. Proper grounds to withdraw from the engagement
B. Insufficient reason to modify the auditor’s report because of the lawyer’s obligation of
confidentiality
C. Considered to be a scope limitation
D. Sufficient reason to issue a “subject to” opinion
PSA 501 states that where the entity’s legal counsel refuses to respond in an
appropriate manner and the auditor is unable to obtain sufficient appropriate audit
evidence by applying alternative audit procedures, the auditor would consider
whether there is a scope limitation which may lead to a qualified opinion or a
disclaimer of opinion.
Answer A is incorrect because the lawyer’s failure to corroborate information
furnished by management could lead to a qualified opinion or a disclaimer of
opinion but would not be considered proper grounds for the withdrawal from the
engagement.
Answer B is incorrect because the auditor could be precluded from expressing an
unmodified opinion when there is a material scope limitation.
Answer D is incorrect because “subject to” is not allowed to be used as a
qualifying phrase in an auditor’s report.

52. Management’s refusal to give the auditor permission to communicate with


the entity’s legal counsel is most
likely to lead to A. An
adverse opinion
B. A qualified opinion or an adverse opinion
C. An unmodified opinion
D. A qualified opinion or a disclaimer of opinion
If management refuses to give the auditor permission to communicate with the
entity’s legal counsel, this would be a scope limitation and should ordinarily lead
to a qualified opinion or a disclaimer of opinion.

53. In which of the following circumstances would an auditor most likely meet
with the client’s legal counsel to discuss the likely outcome of the litigation
and claims?
I. The auditor determines that the matter is a significant risk.
II. There is a disagreement between management and the entity’s legal counsel.
III. The subject matter of the litigation is complex.
A. I and II only
B. II and III only
C. I and III only
D. I, II, and III
PSA 501 states that, in certain circumstances, for example, where the auditor
determines that the matter is a significant risk, the matter is complex or there is
disagreement between management and the entity’s legal counsel, it may be
necessary for the auditor to meet with the entity’s legal counsel to discuss the
likely outcome of litigation and claims. Such meetings would take place with
management’s permission and, preferably, with a representative of management in
attendance.

54. Which of the following statements extracted from a client’s lawyer’s letter
concerning litigation, claims, and assessment most likely would cause the
auditor to request clarification?
A. “I believe that the action can be settled for less than the damages
claimed.”
B. “I believe that the company will be able to defend this action
successfully.”
C. “I believe that the plaintiff’s case against the company is without merit.”
D. “I believe that the possible liability to the company is nominal in
amount.”
The letter of audit inquiry requests the entity’s lawyer to evaluate the likelihood
of an unfavorable outcome, and if possible, to make an estimate of the amount or
range of potential loss. A statement that the action can be settled for less than the
damages claimed is unclear as to the amount or range of potential loss.
Answers B and C are incorrect because the responses clearly state that the entity
should not expect any liability to arise from the lawsuit. Answer D is incorrect
because the response is clear as to the amount or range of possible loss.

55. The auditor should consider the status of legal matters up to the
A. Balance sheet date
B. Date of the auditor’s report
C. Date of approval of the financial statements
D. Date of issuance of financial statements

TRUE OR FALSE

1. If potential loss on a contingent liability is remote, the liability is


usually disclosed in the auditor’s report but not disclosed in the
financial statements.
2. When obtaining evidence regarding litigation against a client, the CPA
would be least interested in determining the period in which the
underlying cause of litigation occurred.
3. The auditor has no responsibility for events occurring in the subsequent
period unless these events affect transactions recorded on or before the
balance sheet date.
4. The standard letter of inquiry to the client’s legal counsel should be
prepared on auditor’s stationery and assigned by an audit partner.
5. The auditor’s primary means of obtaining corroboration of
management’s information concerning litigation is a letter of audit
inquiry to the client’s lawyer.
6. When a client will not permit inquiry of outside legal counsel, the audit
report will ordinary contain an unmodified opinion with an emphasis of
matter paragraph.
7. An auditor has the responsibility to actively search for subsequent
events that occur subsequent to the date of auditor’s report.
8. A letter of representations from management provides written
documentation, which is a higher quality of evidence than
management’s oral responses to inquiries.
9. If a lawyer refuses to furnish corroborating information regarding
litigation, claims, and assessments, the auditor should consider the
refusal to be a scope limitation.
10. After an auditor has issued an audit report, there is no obligation to
make any further audit tests or inquiries with respect to the audited
financial statements covered by that report unless new information
comes to the auditor’s attention concerning an event that occurred prior
to the date of the auditor’s report that may have affected the auditor’s
report.
11. The date of the management representation letter should coincide with
the date of auditor’s report.
12. The management letter is used to allow management to corroborate oral
representations to the auditor.
13. Auditors should perform audit procedures relating to subsequent events
throughout the last day of field work.
14. The review of audit working papers by the audit partner is normally
related completed immediately as each working paper is completed.
15. Subsequent events which require adjustment to the financial statements
provide additional information about significant conditions/events
which did not exist at the balance sheet date.
16. Subsequent events which require adjustment to the financial statements
provide additional information about significant conditions/events
which existed at the balance sheet date.
17. The date of the management representation letter received from the
client should coincide with the balance sheet date.
18. Generally, loss contingencies that are judged to be remote should be
disclosed in the notes to financial statements.
19. To obtain evidence about an entity’s subsequent events, an auditor
would reconcile bank activity for the month after the balance sheet date
with cash activity reflected in accounting records.
20. The auditor is required to confirm directly with the client’s lawyer that
all claims have been recorded in the financial statements.
21. The auditor is required to confirm directly with the client’s lawyer that
all claims have been recorded in the financial statements.
22. A lawyer is responding to an independent auditor as a result of the audit
client’s letter of inquiry. The lawyer may appropriately limit the
response to matters to which the attorney has given substantive
attention in the form of legal consultation or representation.
23. Subsequent events for which the auditor has a responsibility to actively
search are defined as events which occur subsequent to the balance
sheet date.
24. The letter of representation is signed by high-level corporate officials,
usually the chief executive officer and chief financial officer.
25. Refusal by a client to prepare and sign the representation letter would
require the auditor to issue a qualified opinion or a disclaimer.

KEY ANSWERS

1. B 20. D 39. B 2. C 21. C 40. B 3. B


22. B 41. B 4. C 23. B 42. D 5. A 24.
C 43. A 6. D 25. A 44. D
7. D 26. A 45. A
8. B 27. B 46. B
9. C 28. D 47. A
10. A 29. A 48. C
11. B 30. C 49. B 12. C 31. A 50. A 13. C 32. C 51.
C 14. C 33. D 52. D 15. A 34. A 53. D
16. A 35. B 54. A
17. B 36. B 55. B
18. B 37. D
19. C 38. B

TRUE OR FALSE

1. FALSE 14. FALSE


2. FALSE 15. TRUE
3. FALSE 16. TRUE
4. FALSE 17. FALSE
5. TRUE 18. FALSE
6. FALSE 19. FALSE 7. FALSE 20. FALSE
8. FALSE 21. FALSE
9. TRUE 22. TRUE
10. TRUE 23. FALSE
11. TRUE 24. TRUE
12. FALSE 25. TRUE
13. TRUE

Chapter 10
THE AUDITOR’S REPORT ON FINANCIAL STATEMENTS

1. A major purpose of the auditor’s report on financial statements is to


a. Assure investors of the complete accuracy of the financial statements
b. Enhance the degree of confidence of intended users in the financial
statements
c. Deter creditors fom extending loans in high-risk situations
d. Describe the specific auditing procedures undertaken to gather evidence
for the opinion

PSA 200 (Overall Objectives of the Independent Auditor and the Conduct
of an Audit in Accordance with Philippine Standards on Auditing) states
that the purpose of an audit is to enhance the degree of confidence of
intended users in the financial statements. This is achieved by the
expression of an opinion by the auditor on whether the financial
statements are prepared, in all material respects, in accordance with an
applicable financial reporting framework.
Answer A is incorrect because an auditor’s opinion is not an assurance of
complete accuracy of the financial statements.
Answer C is incorrect because while the auditor’s report may contain
some information that users of financial statements may use to make
informed decisions, it is not a substitute for their judgment. Answer D is
incorrect because the Auditor’s
Responsibility section of the auditor’s provides only a general
description of how an audit is conducted.
2. If a company’s external auditor expresses an unmodified opinion as a result
of the audit of the company’s financial statements, readers of the audit
report can assume that
a. The external auditor found no fraud
b. The company is financially sound and the financial statements are accurate
c. Internal control is effective
d. The auditor concludes that the financial statements are prepared, in all material
respects, in accordance with the applicable financial reporting framework.
An UNMODIFIED OPINION is expressed when the auditor concludes
that the financial statements are prepared, in all material respects, in
accordance with the applicable financial reporting framework.
If the auditor concludes that, based on the audit evidence obtained, the
financial statements as a whole are not free from material misstatement or
is unable to obtain sufficient appropriate audit evidence to conclude that
the financial statements as a whole are free from material misstatement,
the auditor should express a modified opinion (qualified opinion, adverse
opinion or disclaimer of opinion) on the financial statements.
Answer A is incorrect because readers may only assume that fraud found
did not, in the auditor’s opinion, materially affect the fair presentation of
the statements.
Answer B is incorrect because the report contains an opinion on whether
the financial statements are prepared, in all material respects, in
accordance with the applicable financial reporting framework and not an
appraisal of the company. Also, an unmodified opinion provides
reasonable assurance that the statements are free of material misstatement,
not that they are accurate.
Answer D is incorrect because an unmodified opinion may be expressed
even though the auditee’s internal control system is ineffective.

3. When the financial statements contain material but not pervasive


misstatements because the accounting policies selected are not consistent
with the applicable financial reporting framework, the auditor should
a. Express a qualified opinion and describe the matter giving rise to
the modification in a separate paragraph
b. Express a qualified opinion and describe the matter giving rise to
the modification within the opinion paragraph
c. Disclaim an opinion and describe the matter giving rise to the
modification in a separate paragraph
d. Disclaim an opinion and describe the matter giving rise to the
modification within the opinion paragraph
A QUALIFIED OPINION shall be expressed when, based on sufficient
appropriate audit evidence obtained, the auditor concludes that
misstatements, individually or in the aggregate, are material, but not
pervasive, to the financial statements.
A qualified opinion is also appropriate when the auditor is unable to obtain
sufficient appropriate audit evidence on which to base the opinion, but the
auditor concludes that the possible effects of undetected misstatements on
the financial statements, if any, could be material but not pervasive.
When a qualified opinion is expressed, the auditor’s report should include
a separate paragraph with the heading “Basis for Qualified Opinion” that
provides a description of the matter giving rise to the expression of a
qualified opinion. This paragraph should be placed immediately before the
opinion paragraph.
If the material misstatement relates to specific amounts in the financial
statements (including quantitative disclosures), the “Basis for Qualified
Opinion” paragraph should include a description and quantification of the
financial effects of the misstatement, unless impracticable. If the financial
effects cannot be quantified, the auditor should so state in the paragraph.
Answer B is incorrect because a Basis for Qualified
Opinion paragraph is required.
Answer C and D are incorrect because a DISCLAIMER OF OPINION is
appropriate when the auditor is unable to obtain sufficient appropriate
audit evidence on which to base the opinion, and the auditor concludes
that the possible effects on the financial statements of undetected
misstatements, if any, could be both material and pervasive.

4. A CPA engaged to audit financial statements observes that the accounting


for a certain material item is not in accordance with the applicable
financial reporting framework, although the departure is prominently
disclosed in a note to the financial statements. The CPA should
a. Express an unmodified opinion but insert an Emphasis of Matter
paragraph emphasizing the matter by reference to the note
b. Disclaim an opinion
c. Not allow the accounting treatment for this item to affect the type
of opinion because the departure from the requirement of the
applicable financial reporting framework was disclosed.
d. Modify the opinion because of the departure from the requirement
of the applicable financial reporting framework
When financial statements contain material misstatements because the
accounting policies applied are not consistent with the applicable financial
reporting framework, the auditor should express a QUALIFIED OPINION
(if the effect is material but not pervasive) or ADVERSE
OPINION (if the effect is both material and pervasive). Answer A is
incorrect because an unmodified opinion is inappropriate when financial
statements are not free from material misstatement.
Answer B is incorrect because a disclaimer of opinion is appropriate when
the auditor is unable to obtain sufficient appropriate audit evidence and the
auditor concludes that the possible effects of misstatements could be both
material and pervasive.
Answer C is incorrect because a note disclosure of the departure from the
requirement of the applicable financial reporting framework does not
eliminate the need to modify the opinion.

5. Whenever there is a scope limitation, the appropriate response is to issue


a/an
a. Qualified opinion
b. Adverse opinion
c. Disclaimer of opinion
d. Unmodified report, a qualification of scope and opinion, or a disclaimer, depending on
materiality
6. When a qualified opinion is expressed, the implication is that the auditor
a. Does not believe the financial statements are presented in accordance with the applicable
financial reporting framework
b. Does not know if the financial statements are presented in accordance with the applicable
financial reporting framework
c. Believes the financial statements are presented fairly
d. Believes the financial statements are presented fairly “except for” a specific item of them
7. If a misstatement is immaterial to the financial statements of the company
for the current period, but is expected to have a material effect in future
periods, it is appropriate to express a/an
a. Qualified opinion
b. Unmodified opinion
c. Disclaimer of opinion
d. Adverse opinion
8. On January 2, 2016, the ABC CO. received a notice from its primary
suppliers that effective immediately all wholesale prices would be
increased 10%. On the basis of the notice, ABC revalued its December 31,
2015 inventory to reflect the higher costs. As a result, the statement of
financial position reflects inventory stated at an amount higher than its net
realizable value. The inventory constituted a material proportion of total
assets; however, the effect of the revaluation was material to current assets
but not to total assets or net income. In reporting on the company’s
financial statements for the year ended December 31, 2015, in which
inventory is valued at the adjusted amount, the auditor would most likely
a. Express an unmodified opinion provided the nature of the adjustment and the amounts
involved are disclosed in notes to the financial statements
b. Express a qualified opinion
c. Disclaim an opinion
d. Express an adverse opinion
PAS 2 (Inventories) requires that inventories should be valued at the lower
of cost and net realizable value. Because the effect of the misstatement is
material but not pervasive, the auditor should express a qualified opinion.
The inclusion of the matter in the notes does not overcome the need for a
modified opinion. Answer A is incorrect because an unmodified opinion is
inappropriate when financial statements are not free from material
misstatement.
Answer C is incorrect because a disclaimer of opinion is only appropriate
when there is no sufficient appropriate evidence to form an opinion.
Answer D is incorrect because an adverse opinion may not be appropriate
since the effect of revaluation was not material to total assets or net
income, i.e., not pervasive.
9. ABC CO. completed construction and began to lease a 500unit apartment
on May 28, 2014. During June, 250 units were leased, and an additional
150 units were leased in
July 2015.
During the month of May 2014, the company charged to expense
P230,000 for the cost of advertising, a grand opening party, and the
advertising agency fee for planning the campaign. At December 31, 2015,
the statement of financial position reflected P875,000 of initial direct costs
incurred by the company including commissions and legal fees paid in
negotiating the lease. These initial direct costs are shown as an addition to
the carrying amount of the leased asset and is being recognized as an
expense over the term of the lease on the same basis as the lease income.
During your audit of the company’s financial statements for the year
ended December 31, 2015 (conducted in accordance with PSAs), no facts
other than those described above came to your knowledge that would
cause your opinion to be other than that the financial statements were
presented fairly in accordance with
Philippine Financial Reporting Standards. What type of
opinion should your report contain? a. An adverse opinion
b. An unmodified opinion
c. A disclaimer opinion
d. A qualified opinion
Because the transactions were accounted for properly in accordance with
the requirements of PFRS on leases, an unmodified opinion is appropriate.

10. When an auditor modifies an opinion because of inadequate disclosure, the


auditor should describe the nature of the omission in a separate “Basis for
Modification” paragraph and modify the
Introductory Auditor’s Opinion
Paragraph Responsibility Paragraph
Paragraph
a. Yes No No
b. Yes Yes No
c. No Yes Yes
d. No No Yes
11. In which of the following situations would an auditor ordinarily choose
between expressing a qualified opinion or an adverse opinion?
a. The auditor did not observe the entity’s physical inventory and is
unable to become satisfied as to its balance by other auditing
procedures.
b. The financial statements fail to disclose information that is
required by the applicable financial reporting framework.
c. The auditor is asked to report only on the entity’s statement of
financial position and not on the other general purpose financial
statements.
d. Events disclosed in the financial statements cause the auditor to
have substantial doubt about the entity’s ability to continue as a
going concern. Material misstatements arising from omission of
disclosures required by the acceptable financial reporting
framework may result in either a qualified or an adverse opinion.
Answer A is incorrect because a disclaimer of opinion is appropriate when
the auditor is unable to obtain sufficient appropriate audit evidence on
which to base the opinion, and the auditor concludes that the possible
effects on the financial statements of undetected misstatements, if any,
could be both material and pervasive.
Answer C is incorrect because audits of single financial statements may
result in the expression of an unmodified opinion (PSA 805, Special
Considerations – Audits of Single Financial Statements and Specific
Elements,
Accounts or Items of a Financial Statement).
Answer D is incorrect because substantial doubt about the entity’s
continued existence normally results in the addition of an emphasis of
matter paragraph immediately after the opinion paragraph.
12. Yang, CPA, concludes that there is significant doubt about ABC
CO.’s ability to continue as a going concern. If ABC’s financial statements
adequately disclose its financial difficulties, Yang’s report should Include
Emphasis Specifically
Of matter Specifically use the
Paragraph use the words
Following words “Going “Significant
The Opinion Concern” Paragraph Doubt”
a. Yes Yes Yes
b. Yes Yes No
c. Yes No Yes
d. No Yes Yes
PSA 570 (Going Concern) gives the following illustration of an Emphasis
of Matter paragraph when the auditor is satisfied as to the adequacy of the
note disclosure:
Emphasis of Matter
Without qualifying our opinion, we draw attention to Note X in the financial
statements which indicates that the
Company incurred a net loss of Pxxx during the year ended December
31,20X1 and, as of that date, the Company’s current liabilities exceeded its
total assets by Pxxx. These conditions, along with other matters as set
forth in Note X, indicate the existence of a material uncertainty that may
cast significant doubt about the Company’s ability to continue as a going
concern.

13. Which sections of an auditor’s unmodified report on financial statements


should refer to Philippine Standards on Auditing (PSA) and Philippine
Financial Reporting Standards (PFRS)?
PSA PFRS
a. Management’s opinion
Responsibility auditor’s responsibility
b. Auditor’s management’s responsibility
Responsibility opinion
c. Opinion management’s responsibility
d. Auditor’s opinion
Responsibility
14. Without affecting the CPA’s willingness to express an unmodified opinion
on the client’s financial statements, corporate management may refuse a
request to
a. Authorize its attorney to confirm that a list of pending or threatened
litigation prepared by management includes all items known to the
attorney.
b. Change its basis of accounting for inventories from first-in, first-out
(FIFO) method to weighted average method.
c. Write down to salvage value certain equipment that is no longer
useful.
d. Allow the CPA to examine tax returns for years prior to that of the
financial statements being audited.
Even if management refuses to change its inventory costing method from
FIFO to weighted average, the financial statements would still comply
with the requirements of the PFRS. Therefore,the CPA may still express an
unmodified opinion.
15. In which of the following circumstances would an auditor most likely add
an Emphasis of Matter paragraph to the auditor’s report while expressing
an unmodified opinion?
a. The auditor is asked to report on a single financial statement (e.g., a
statement of financial position).
b. There is significant doubt about the entity’s ability to continue as a
going concern.
c. Management’s estimates of the effects future events are
unreasonable.
d. Certain transactions cannot be tested because of management’s
records retention policy. When there is significant doubt on the
entity’s ability to continue as a going concern, the auditor should
consider the adequacy of disclosure of the material uncertainty. If
adequate disclosure is made in the financial statements, the auditor
shall express an unmodified opinion and include an Emphasis of
Matter paragraph (after the opinion paragraph) in the report. Answer
A is incorrect because under PSA 805, an auditor who is engaged to
audit a single financial statement may express an unmodified opinion
without ding Emphasis of
Matter paragraph.
Answer C is incorrect because an unmodified opinion would not be
appropriate when financial statements contain unreasonable estimates.
Answer D is incorrect because a qualified opinion or a disclaimer of
opinion is proper when there is restriction on the scope of the audit.
16. If an auditor is satisfied that sufficient evidence supports management’s
assertions about an uncertainty and its presentation or disclosure, the
auditor should
a. Express a modified opinion with an Emphasis of Matter paragraph.
b. Express an unmodified opinion with an Emphasis of Matter
paragraph.
c. Express an unmodified opinion with an Other Matter paragraph.
d. Express a qualified opinion or disclaim an opinion, depending upon
the materiality of the loss.
According to PSA 706 (Emphasis of Matter Paragraphs and
Other Matter Paragraphs in the Independent Auditor’s Report), if the
auditor considers it necessary to draw users’ attention to a matter presented
or disclosed in the financial statements that, in the auditor’s judgment, is
of such importance that it is fundamental to users’ understanding of the
financial statements, the auditor shall include an Emphasis of Matter
paragraph in the auditor’s report provided that the auditor has obtained
sufficient appropriate audit evidence that the matter is not materially
misstated n the financial statements. The auditor shall include it
immediately after the opinion paragraph and indicate that the auditor’s
opinion is not modified in respect of the matter emphasized.
Answer A and D are incorrect because a modified opinion (qualified,
adverse or diclaimer) is appropriate when financial statements are not free
of material misstatement and/or when the scope of the audit is limited.
Answer C is incorrect because an Other Matter paragraph refers to a
matter other than those presented or disclosed in the financial statements
that, in the auditor’s judgment, is relevant to users’ understanding of the
audit, the auditor’s responsibilities or the auditor’s report.

17. During the year ended December 31, ABC Co. Reported its
property, plant and equipment at the lower of cost or market (LCM)
because their fair value had declined. The loss has been included in the
income statement and the adjustment has been fully disclosed in the notes.
If a CPA believes that the values reported in the financial statements are
reasonable, what opinion should be expressed?
a. An unmodified opinion
b. A “subject to” qualified opinion
c. An adverse opinion
d. A disclaimer of opinion
Under PAS 16 (Property,Plant and Equipment), a company may report its
property,plant and equipment using the cost model or the revaluation
model. Recording them at LCM is therefore not consistent with the
requirement of the PAS. Assuming that the effects are material, the auditor
should express either a qualified or adverse opinion.
Answer A is incorrect because a material misstatement in the financial
statement precludes an unmodified opinion.
Answer B is incorrect because the use of “subject to” in a qualified opinion
is prohibited.
Answer D is incorrect because a disclaimer is appropriate when the scope
limitation does not permit expression of an opinion.

18. Reference to “financial statements” in PSA 700


(Forming an Opinion and Reporting on Financial
Statements), means
a. A complete set of general purpose financial statements, including the
related notes.
b. A complete set of financial statements prepared to meet the financial
information needs of specific users.
c. A complete set of financial statements prepared in accordance with a
special purpose framework.
d. A complete set of financial statements prepared in accordance with
either a general or special purpose framework.
Reference to “financial statements” in PSA 700 means a complete set of
general purpose financial statements, including the related notes.
General purpose financial statements are financial statements prepared in
accordance with a general purpose framework, i.e., financial reporting
framework designed to meet the common financial information needs of a
wide range of users.
Answer B, C and D are incorrect because PSA 800 (Special
Considerations – Audits of Financial Statements Prepared in Accordance
with Special Purpose Frameworks) applies to financial statements
prepared in accordance with a special purpose framework, i.e., a financial
reporting framework designed to meet the financial information needs of
specific users.

19. Which of the following statements best describes a


compliance framework?
a. A compliance framework requires compliance with the
requirements of the framework and acknowledges explicitly or
implicitly that, to achieve fair presentation of the financial
statements, it may be necessary for management to provide
disclosures beyond those specifically required by the framework.
b. A compliance framework requires compliance with the
requirements of the framework and acknowledges explicitly that it
may be necessary for management to depart from a requirement of
the framework to achieve fair presentation of the financial
statements.
c. A compliance framework only requires compliance with the
requirements of the framework.
d. A compliance framework refers to a financial information needs of
specific users.
The term “fair presentation framework” is used to refer to a financial
reporting framework that requires compliance with the requirements of the
framework and:

i. Acknowledges explicitly or implicitly that, to achieve fair presentation


of the financial statements, it may be necessary for management to
provide disclosures beyond those specifically required by the
framework; or
ii. acknowledges explicitly that that it may be necessary for management
to depart from a requirement of the framework to achieve fair
presentation of the financial statements.
The term “compliance framework” is used to refer to a financial reporting
framework that requires compliance with the requirements of the
framework, but does not contain the acknowledgements in (i) or (ii) above.
(PSA 70,par. 7)

20. Which of the following should be considered when forming an


opinion on the audited financial statements?
I.Whether sufficient appropriate audit evidence has been obtained.
II. Whether uncorrected misstatements are material, individually or in aggregate.
III. The qualitative aspects of the entity’s accounting practices, including
indicators of possible bias in management’s judgments.

a. I only
b. I and III only
c. I and II only
d. I, II and III
21. When reporting on financial statements prepared in accordance with
a compliance framework, the auditor shall evaluate

A B C D
A. The overall presentation,
Structure and content of
The financial statements No Yes No Yes
B. Whether the financial
Statements, including the
Related notes, represent
The underlying transactions
And events in a manner that
Achieves fair presentation No Yes Yes No

The auditor’s evaluation should include consideration of (a) and (b) above
when reporting on financial statements prepared in accordance with a fair
presentation framework (PSA700,par 14).

22. Management and, when appropriate, those charged with governance


have responsibility for I. The preparation of the financial
statements
II. An adequate description of the framework applied in the preparation of the
financial statements
a. I only
b. II only
c. Both I and II
d. Neither I nor II
23. An audit client’s description that its financial statements are prepared
in accordance with a particular applicable financial reporting
framework is appropriate only if
a. The financial statements comply with all the requirements of that
framework that are effective during the period covered by the
financial statements.
b. The financial statements are in substantial compliance with that
framework.
c. The financial statements adequately disclose the significant
accounting policies selected and applied.
d. The terminology used in the financial statements, including the
title of each financial statement, is appropriate.
24. To distinguish it from reports that might be issued by others,such as
by officers of the entity,the board of directors, or from the reports of
other auditors who may not have to abide by the same ethical
requirements the independent auditor, the auditor’s report should
have an appropriate
a. Addressee
b. Title
c. Signature
d. Opinion
The auditor’s report should have a title that clearly indicates that it is the
report of an independent auditor. The title “Independent Auditor’s Report”
affirms that the auditor has met all of the relevant ethical requirements
regarding independence and, therefore, distinguishes the independent
auditor’s report from reports issued by others.

25. The audit report date on an unmodified report indicates


a. The last date on which users may institute a lawsuit against either
the client or the auditor.
b. The last day of the auditor’s responsibility for the review of
significant events occurring after the end of the reporting period.
c. The end of the reporting period.
d. The date on which the financial statements were filed with the
Securities and Exchange Commission.
26. The auditor’s report should be addressed
a. Only to the shareholders of the entity whose financial statements are being
audited.
b. Only to the board of directors of the entity whose financial statements are
being audited.
c. To the CEO or the CFO of the entity whose financial statements are being
audited.
d. Either to the shareholders or the board of directors of the entity whose
financial statements are being audited.
The auditor’s report is ordinarily addressed to those for whom the report is
prepared, often either to the shareholders or to those charged with
governance of the entity whose financial statements are being audited.

27. Which of the following is included in the introductory or opening


paragraph of the auditor’s report?
a. Identification of the financial statements audited, including the date
of and period covered by the financial statements.
b. A statement that the financial statements are the responsibility of the
entity’s management.
c. A statement that the audit was conducted in accordance with
Philippine Standards on Auditing.
d. A statement that the responsibility of the auditor is to express an
opinion on the financial statements based on the audit.
The introductory paragraph shall identify the entity whose financial
statements have been audited and shall state that the financial statements
have been audited. The introductory paragraph shall also:
a. Identify the title of each of the financial statements that compromise the
complete set of financial statements;
b. Refer to the summary of significant accounting policies and other
explanatory information; and
c. Specify the date and period covered by the financial statements.
PSA 700 (Forming an Opinion and Reporting on Financial Statements)
gives the following example of an independent auditor’s report:
Independent Auditor’s Report
(Appropriate Addressee)
Report on the Financial Statements
We have audited the accompanying financial statements of the ABC
Company, which comprise the statement of financial position as of at
December 31,20x1, and the statement of comprehensive income, statement
of changes in equity and statement of cash flows for the year then ended,
and a summary of significant accounting policies and other explanatory
information.
Management’s Responsibility for the Financial Statements
Management is responsible for the preparation and fair presentation of
these financial statements in accordance with Philippine Financial
Reporting Standards, and for such internal control as management
determines is necessary to enable the preparation of financial statements
that are free from material misstatement, whether due to fraud or error.
Auditor’s Responsibility
Our responsibility is to express an opinion on these financial statements
based on our audit. We conducted our audit in accordance with Philippine
Standards on Auditing. Those standards require that we comply with
ethical requirements and plan and perform the audit to obtain reasonable
assurance about whether the financial statements are free from material
misstatement.
An audit involves performing procedures to obtain evidence about the
amounts and disclosures in the financial statements. The procedures
selected depend on the auditor’s judgment, including the assessment of the
risks of material misstatement of the financial statements, whether due to
fraud or error. In making those risk assessments, the auditor considers
internal control relevant to the entity’s preparation and fair presentation of
the financial statements in order to design audit procedures that are
appropriate in the circumstances, but not for the purpose of expressing an
opinion on the effectiveness of the entity’s internal control. An audit also
includes evaluating the appropriateness of accounting policies used and
the reasonableness of accounting estimates made by management, as well
as evaluating the overall presentation of the financial statements.
We believe that the audit evidence we have obtained is sufficient and
appropriate to provide a basis for our audit opinion.
Opinion
In our opinion, the financial statements present fairly, in all material
respects, the financial position of ABC Company as of December 31,20x1,
and of its financial performance and its cash flows for the year then ended
in accordance with Philippine Financial Reporting Standards.
Report on Other Legal and Regulatory Requirements
(Form and content of this section of the auditor’s report will vary
depending on the nature of the auditor’s other reporting responsibilities.)

(Auditor’s signature)
(Date of the auditor’s report)
(Auditor’s address)

28. An entity’s management is responsible for the preparation and fair


presentation of the financial statements. Its responsibility includes the
following,
except
a. Designing, implementing, and maintaining internal control relevant
to the preparation and presentation of financial statements.
b. Making accounting estimates that are reasonable in the
circumstances.
c. Selecting and applying appropriate accounting policies.
d. Assessing the risks of material misstatement of the financial
statements.
The independent auditor – not the entity’s management – should make an
assessment of the risks of material misstatement of the financial
statements, whether due to fraud or error.

29. The opinion paragraph of the auditor’s report


I. Identifies the applicable financial reporting framework on
which the financial statements are based.
II. Expresses an opinion on the financial statements.
a. I only
b. II only
c. Both I and II
d. Neither I nor II

As illustrated in the standard, the opinion paragraph of the auditor’s report


reads:
“ In our opinion, the financial statements present fairly, in all material
respects, the financial position of ABC Company as of December 31,20x1,
and its financial performance and its cash flows for the year then ended in
accordance with Philippine Financial Reporting
Standards.”

30. The following statements relate to the date of the auditor’s report. Which
is false?
a. The auditor should date the report as of the completion date of the
audit.
b. The date of the auditor’s report should not be earlier that the date on
which the financial statements are signed or approved by
management.
c. The date of the auditor’s report should not be later than the date on
which the financial statements are signed or approved by
management.
d. The date of the auditor’s report should always be later than the date
of the financial statements (i.e., the balance sheet ate).
PSA 700 states that since the auditor’s opinion is provided on the financial
statements and the financial statements are responsibility of management,
the auditor is not in a position to conclude that sufficient appropriate audit
evidence has been obtained until the auditor obtains evidence that a
complete set of financial statements has been prepared and management
has accepted responsibility for them.

31. Which of the following statements best expresses the objective of the
traditional audit of financial statements?
a. To express an opinion on the fairness with which the statements
present financial position, financial performance, and cash flows in
accordance with Philippine Financial Reporting Standards.
b. To express an opinion on the accuracy with which the statements
present financial position, financial performance, and cash flows in
accordance with Philippine Financial Reporting Standards.
c. To make suggestions as to the form or content of the financial
statements or to draft them in whole or in part.
d. To assure adoption of sound accounting policies and the
establishment and maintenance of internal control.
The objective of the traditional financial statement audit is for the auditor
to express an opinion (or a disclaimer of opinion) on the fairness, in all
material respects, of the financial statements prepared and presented by the
client’s management.
The auditor’s report should clearly state whether the audit was conducted
in accordance with PSAs, and whether, in the opinion of the independent
auditor, the financial statements are presented in accordance with
Philippine Financial Reporting Standards.
Answer B is incorrect because the auditor expresses an opinion on the
fairness (not the accuracy) of financial statements.
Answer C is incorrect because the client’s management is responsible for
adopting sound accounting policies and establishing and maintaining
internal control.

32. Which of the following best describes why an independent auditor


is asked to express an opinion on the fair presentation of financial
statements?
a. It is a customary courtesy that all shareholders receive an
independent report on management’s stewardship in managing the
affairs of the business.
b. The opinion of an independent party is needed because a company
may not be objective with respect to its own financial statements.
c. It is difficult to prepare financial statements that fairly present a
company’s financial position, financial performance, and cash
flows without the expertise of an independent auditor.
d. It is management’s responsibility to seek available independent aid
in the appraisal of the financial information shown in its financial
statements.
The independent auditor’s opinion lends credibility to the financial
statements prepared and presented by an entity’s management.
Answer A is incorrect because, although it is customary courtesy that
all shareholders receive an independent report on management’s
stewardship function, it is not the primary reason why an independent
report is issued to accompany the financial statements.
Answer C is incorrect because the preparation of financial statements
is the sole responsibility of an entity’s management.
Answer D is incorrect because the independent auditor’s function is to
express an opinion on the fairness of financial statements, not to make
an appraisal of the financial information shown in an entity’s financial
statements.

33. How are management’s responsibility and the auditor’s responsibility


represented in the auditor’s report?
Management’s Auditor’s
Responsibility Responsibility
a. Implicitly Implicitly
b. Implicitly Explicitly
c. Explicitly Implicitly
d. Explicitly Explicitly

34. In which of the following circumstance would an auditor most likely add
an Emphasis of Matter paragraph to the auditor’s report while expressing
an unmodified opinion?
a. There is a substantial doubt about the entity’s ability to continue as a going
concern.
b. Management’s estimates of the effects of future vents are unreasonable.
c. No depreciation has been provided in the financial statements.
d. Certain transactions cannot be tested because of management’s records
retention policy.

According to the standard, the auditor should modify the auditor’s


report by adding an emphasis of matter paragraph to highlight a
material matter regarding a going concern problem.
PSA 570 (Going Concern) gives the following example of such a paragraph:
“Without qualifying our opinion, we draw attention to Note X in the
financial statements which indicates that the Company incurred a net
loss of Pxxx during the year ended December 31, 20x1 and as of that
date, the Company’s current liabilities exceeded its total assets by
Pxxx. These conditions, along with other matters as set forth in Note
X, indicate the existence of a material uncertainty which may cast
significant doubt about the Company’s ability to continue as a going
concern.”
Answer B is incorrect because financial statements containing
unreasonable management’s estimates are not fairly presented and will
preclude the auditor from expressing an unmodified opinion.
Answer C is incorrect because an auditor who is asked to report on one
financial statement may appropriately express an unmodified opinion
without adding an Emphasis of Matter paragraph in the audit report.
Answer D is incorrect because a scope limitation will require a
qualified opinion or a disclaimer of opinion.

35. An emphasis of matter paragraph of an auditor’s report describes an


uncertainty as follows:
Without qualifying our opinion, we draw attention to Note X to the
financial statements. The Company is the defendant in a lawsuit
alleging infringement of certain patent rights and claiming
royalties and punitive damages. The Company has filed a counter
action and preliminary hearings and discovery proceedings on both
actions are in progress. The ultimate outcome of the matter cannot
presently be determined and no provision for any liability that may
result has been made in the financial statements.
What type of opinion should the auditor express under these
circumstances?

a. Unmodified
b. “except for” qualified
c. “subject to” qualified
d. Disclaimer of opinion

The addition of an Emphasis of Matter paragraph because of a significant


uncertainty (other than a going concern problem) does not affect the
auditor’s opinion. As indicated above, the Emphasis of Matter paragraph
refers to the fact that the auditor’s opinion is not qualified.
Answer B is incorrect because an uncertainty does not necessarily lead to a qualified
opinion.
Answer C is incorrect because the use of the qualifying phrase “subject to” is not
allowed.
Answer D is incorrect because a disclaimer of opinion would be
appropriate in extreme cases, such as situations involving multiple
uncertainties that are significant to the financial statements.
36. An auditor’s responsibility to express an opinion on the financial
statements is
a. Implicitly represented in the auditor’s report
b. Explicitly represented in the “Auditor’s
Responsibility” section of the auditor’s report.
c. Explicitly represented in the “Management’s
Responsibility” paragraph of the auditor’s report.
d. Explicitly represented in the opinion paragraph of the auditor’s report.
The “Auditor’s Responsibility” section begins with “Our responsibility
is to express an opinion on these financial statements based on our
audit.”

37. The existence of audit risk is recognized by the statement in the auditor’s
report that the auditor
a. Is responsible for expressing an opinion on the financial statements, which are the
responsibility of management.
b. Realizes some matters, either individually or in the aggregate, are important while other
matters are not important.
c. Obtains reasonable assurance about whether the financial statements are free from
material misstatement.
d. Assess the accounting principles used and also evaluates the overall financial statement
presentation.
Audit risk is the risk that the auditor may give an inappropriate
opinion on financial statements that are materially misstated. The
existence of audit risk is recognized by the statement in the auditor’s
report that the auditor obtained reasonable (not absolute) assurance
about whether the financial statements are free of material
misstatement.
Answer A, B and D are incorrect because they don’t pertain to audit risk.

38. Which of the following statements is a basic element of the auditor’s


report?
a. The auditor is responsible for the preparation and fair presentation of the financial
statements.
b. The financial statements are consistent with those of the prior period.
c. An audit involves performing procedures to obtain audit evidence about the amounts and
disclosures in the financial statements.
d. The disclosures provide reasonable assurance that the financial statements are free of
material misstatement.
39. Which paragraphs of an auditor’s report on financial statements should
refer to Philippine Financial
40.
Reporting Standards?
a. Introductory and Opinion
b. Auditor’s Responsibility and Management’s
Responsibility
c. Introductory and Auditor’s Responsibility
d. Management’s Responsibility and Opinion

40.An independent auditor discovers that a payroll supervisor of the company


being audited has misappropriated P250,000. The company’s total assets and
income before tax are P350 million and P75 million, respectively. Assuming
no other issues affect the report, the auditor’s report will most likely contain
a/an

e. Unmodified opinion
f. Disclaimer of opinion
g. Adverse opinion
h. Scope qualification
As long as the misappropriation is accounted for properly, an
unmodified opinion will be appropriate because the financial
statements will be fairly presented in accordance with Philippine
Financial Reporting Standards. Moreover, the amount of
misappropriation is immaterial relative to the company’s assets and
income.

41. A client makes test counts on the basis of a statistical an. The
auditor observes such counts as are deemed necessary and is able to
become satisfied as to the reliability of the client’s procedures. In reporting
on the results of the audit, the auditor
a. Must qualify the opinion if the inventories were material
b. Can express an unmodified opinion
c. Must comment in an Emphasis of Matter paragraph as to the inability to
observe year-end inventories.
d. Is required to disclaim an opinion if the inventories were material.

The auditor may express an unmodified opinion and the audit


report need not refer to the failure to observe year-end physical
cunt and to the alternative procedures applied.
Answer A is incorrect because a qualified opinion is inappropriate
if there is no significant scope limitation and the financial
statements are fairly presented.
Answer C is incorrect because a comment in an Emphasis of
Matter paragraph concerning the omission of a procedure is not
required if the auditor has become satisfied by performing
alternative procedures.
Answer D is incorrect because, given no significant scope
limitation, a disclaimer of opinion is unnecessary.

42. A note to the financial statements of the ABC Bank indicates that all of the
records relating to the bank’s business operations are stored on magnetic
disks, and that no emergency backup systems or duplicate disks are stored
because the bank and its auditors consider the occurrence of a catastrophe to
be remote. Based upon this note, the auditor’s report should express
a. A qualified opinion
b. An unmodified opinion
c. An adverse opinion
d. A “subject to” opinion
The bank’s failure to have backup records does not affect the fairness of
the financial statements. Hence, an unmodified opinion is appropriate.

43. An auditor who uses the work of an expert may refer to the auditor’s expert
in the auditor’s report if the
a. Expert is employed by the entity
b. Expert’s work provides the auditor greater assurance of reliability
c. Auditor expresses a qualified opinion or an adverse opinion related to the work of the
expert
d. Auditor indicates a division of responsibility related to the work of the expert
Par.A42 of PSA 620 (Using the Work of an Auditor’s Expert) states, “It
may be appropriate in some circumstances to refer to the auditor’s expert
in an auditor’s report containing a modified opinion to explain the nature
of the modification.in such circumstances, the auditor may need the
permission o the auditor’s expert before making such a reference.”
Answer A is incorrect because an expert employed by the entity is a
management’s expert, not an auditor’s expert.
Answer B is incorrect because, although the work of an expert is expected
to provide greater assurance of reliability, reference to the auditor’s
expert’s work is justified only when the opinion as a result of the expert’s
work.
Answer D is incorrect because PSA 620 provides that if the auditor makes
reference to the work of an auditor’s expert in the auditor’s report when
such reference is relevant to an understanding of a modification to the
auditor’s opinion, the auditor shall indicate in the auditor’s report that such
reference does not reduce his/her responsibility for the opinion.

44. When would the auditor refer to the work of an appraiser in the auditor’s
report?
a. An adverse opinion is expressed based on a difference of opinion between
the client and the outside appraiser as to the value of certain assets.
b. A disclaimer of opinion is expressed because of a scope limitation imposed
on the auditor by the appraiser
c. a qualified opinion is expressed because of a matter unrelated to the work
of the appraiser
d. An unmodified opinion expressed and an Emphasis of Matter paragraph is
added to disclose the use of the appraiser’s work.
According to PSA 620, the auditor may refer to the auditor’s expert if a
modified report is to be issued as a result of the expert’s work.
Answer B is incorrect because the appraiser is not in a position to limit the
scope of the auditor’s work since the auditor can engage another appraiser
Answer C is incorrect because the auditor’s qualification must be related to the
expert’s work.
Answer D is incorrect because the auditor should not refer to the work of
an expert if an unmodified report is to be issued.

45.A modified opinion on the financial statements is necessary when


I. The auditor concludes, based on the audit evidence obtained, that
the financial statements as a whole are not free from material
misstatement.
II. The auditor is unable to obtain sufficient evidence to conclude that
the financial statements as a whole are free from material misstatement.

a. I only
b. II only
c. Either I or II
d. Neither I nor II

TRUE OR FALSE

1. PSA 700 (Forming an Opinion and Reporting on Financial Statements)


requires that the audit report must be titled and that the title must include
the word “independent.”
2. An auditor may be unable to express an unmodified opinion if an
immaterial departure from PFRS is present in the financial statements.
3. If financial statements fail to disclose a material fact, t auditors may
disclose the information in an emphasis of matter paragraph and issue an
unmodified opinion on the statements.
4. The opening paragraph of the standard unmodified audit report states that
the audit is designed to obtain reasonable assurance whether the
statements are free of material misstatement.
5. The client’s management is responsible for the fairness of the
representations made in financial statements.
6. The audit report date on an unmodified report indicates the last day of the
auditor’s responsibility for the review of significant events that occurred
subsequent to the date of the financial statements.
7. When the audited financial statements of the prior year are presented
together with those of the current year, the continuing auditor’s report
should cover only the current year.
8. If the financial statements contain a material departure from PFRS, the
auditor usually should no issue an unmodified opinion.
9. The introductory paragraph of the unmodified audit report states that the
financial statements are the responsibility of management.
10. If the auditor believes that there is minimal likelihood that resolution of an
uncertainty will have a material effect on the financial statements, the
auditor would issue an “except for” qualified opinion.
11. When there is a significant question about a company’s ability to continue
as a going concern, the report issued is usually modified with an emphasis
of matter paragraph.
12. If the statement of financial position of a company is dated December 31,
2015,the audit report is dated February 12, 2016, and both are released on
February 25, 2016, this indicates that the auditor has searched for
subsequent events occurred up to February 12, 2016.
13. The adverse effects of events causing an auditor to believe there is a
substantial doubt about an entity’s ability to continue as a going concern
would likely be mitigated by evidence relating to the feasibility of plans to
purchase leased equipment at less than market value.

KEY ANSWERS

1. B
2. D

3. A
4. D
5. D
6. D
7. B
8. B
9. B
10. C
11. B
12. A
13. B
14. B
15. B
16. B
17. C
18. A
19. C
20. D
21. A
22. C
23. A
24. B
25. B
26. D
27. A
28. D
29. C
30. C
31. A
32. B
33. D
34. A
35. A
36. B
37. C
38. C
39. D
40. A
41. B
42. B
43. C
44. A
45. C
46. B
47. C
48. D
49. A
50. D
51. B
52. C
53. C
54. D
55. C
56. B
57. A
58. D
59. A
60. B
61. D
62. A
63. B
64. C
65. A
66. C
67. C
68. A
69. D
70. A
71. D
72. B
73. D
74. A
75. D
76. B
77. C
78. B
79. D
80. A
81. B
82. D
83. C
84. A
85. D
86. C
87. D
88. D
89. A
90. C

TRUE or FALSE

1. True
2. False
3. False
4. False
5. True
6. True
7. False 8. True
9. False
10. False
11. False
12. True
13. False 14. False 15. False
16. False
17. False
18. False
19. False
20. False
21. False
22. True
23. True
24. True
25. True
46. Which of the following terms is used in the standard to describe the effects on
the financial statements of misstatements or the possible effects on the financial
statements, if any, that are undetected due to an inability to obtain sufficient
appropriate audit evidence?

A. Persuasive
B. Pervasive C. Material
D. Extensive

According to the standard, pervasive effects on the financial statements are those that, in the
auditor’s judgment:

1) Are not confined to specific elements, accounts or items of the financial statements;
2) If so confined, represent or could represent a substantial proportion of the financial
statements; or
3) In relation of disclosures, are fundamental to users’ understanding of the financial
statements.

47. The auditor shall express an adverse opinion when

A. The auditor, having obtained sufficient appropriate audit evidence,


concludes the misstatements, individually or in the aggregate, are material,
but not persuasive, to the financial statements.
B. The auditor is unable to obtain sufficient appropriate audit evidence on
which to base the opinion, but the auditor concludes that the possible
effects on the financial statements of undetected misstatements, if any,
could be material but not persuasive.
C. The auditor, having obtained sufficient appropriate audit evidence,
concludes that misstatements, individually or in the aggregate, are both
material and pervasive to the financial statements.
D. The auditor is unable to obtain sufficient appropriate audit evidence on
which to base the opinion, and the auditor concludes that the possible
effects on the financial statements of undetected misstatements, if any,
could be both material and pervasive.

48. A limitation on the scope of the audit may arise from

I. Circumstances beyond the control of the entity.


II. Circumstances relating to the nature and timing of the auditor’s work.
III. Limitations imposed by management.
A. I and II only
B. II and III only
C. I and III only
D. I,II, and III

49. The auditor’s inability to obtain sufficient appropriate audit evidence (limitation
on scope of the audit) arising from circumstances beyond the control of the
entity include when

A. The entity’s accounting records have been destroyed.


B. Management prevents the auditor from observing the counting of the physical
inventory.
C. The auditor determines that performing substantive procedures alone is not
sufficient, but the entity’s controls are not effective.
D. The timing of the auditor’s appointment is such that the auditor is unable to
observe the counting of the physical inventories.

50. A paragraph included in the auditor’s report that refers to a matter appropriately
presented or disclosed in the financial statements that, in the auditor/s judgment,
is of such importance that it is fundamental to user’s understand of the financial
statements is called

A. Explanatory paragraph
B. Other Matter paragraph
C. Basis for Modified Opinion paragraph
D. Emphasis of Matter paragraph

The following are example of circumstances where the auditor may consider it
necessary to include an Emphasis of Matter paragraph;
• An uncertainty relating to the future outcome of exceptional litigation or
regulatory action.
• Early application (where permitted) of a new accounting standard that
has a pervasive effect on the financial statements in advance of its
effective date.
• A major catastrophe that has, had, or continues to have, a significant
effect on the entity’s financial position.
51. When the auditor includes an Emphasis of Matter paragraph in the auditor’s
unmodified report, the auditor shall

A. Include it immediately before the Opinion paragraph.


B. Use the heading “Emphasis of Matter” or other appropriate heading.
C. Indicate that the author’s opinion is modified in respect of the matter emphasized.
D. Include it immediately after the Management’s
Responsibility paragraph.

52. Which of the following should be included in the opinion paragraph when an
author expresses a qualified opinion?

Subject to With Foregoing Explanation


A. Yes No
B. No Yes
C. No No
D. Yes Yes

Par. A22 of PSA 705 states, “When the auditor expresses a qualified opinion, it
would not be appropriate to use phrases such as “with the foregoing explanation”
or “subject to” in the opinion paragraph as these are not sufficiently clear or
forceful.”

53. In which of the following circumstance would an auditor usually choose between
expressing a qualified opinion or disclaiming an opinion?

A. Departure from the requirements of the applicable financial reporting framework.


B. Unreasonable justification for a change in accounting principle.
C. Inability to obtain sufficient appropriate audit evidence.
D. Inadequate disclosure of accounting policies.
A limitation on the scope of the auditor’s work whether imposed by the
client or by circumstances such as the inadequacy of the accounting records, the
timing of the auditor’s work, or the inability to obtain sufficient appropriate audit
evidence may require expression of a qualified opinion or a disclaimer of opinion.

54. An auditor decides to express a qualified opinion on an entity’s financial


statements because a major inadequacy in its computerized accounting records
prevents the auditor from applying necessary procedures. The opinion paragraph
of the auditor’s report should state that the qualification pertains to
A. A client-imposed scope limitation
B. A departure from PSAs.
C. Inadequate disclosure of necessary information.
D. The possible effects on the financial statements.

When there is a limitation on the scope of the auditor’s work that requires
expression of a qualified opinion or a disclaimer of opinion, the auditor’s report
should describe the limitation and indicate the possible adjustments to the
financial statements that might have been determined to be necessary had the
limitation not existed.
Answer A is incorrect because the opinion paragraph should state that the
qualification pertains to possible effect on the financial statements and not to the
scope limitation itself.
Answer B is incorrect because the audit was conducted in accordance with PSAs.
Answer C is incorrect because the issue is the auditor’s failure to apply
necessary procedures and not inadequate disclosure.

55. Sam, CPA, was engaged to audit the financial statements of Mantha Corp. after its
fiscal year had ended. The timing of Sam’s appointment as auditor and the start of
field work made confirmation of accounts receivable by direct communication
with the debtors ineffective. However, Sam applied other procedures and was
satisfied as to the reasonableness of the account balances. Sam’s auditor’s report
most likely contained a/an

A. Qualified opinion because of a scope limitation.


B. Qualified opinion because of a departure from PSAs.
C. Unmodified opinion.
D. Unmodified opinion with an Emphasis of Matter paragraph.
By applying alternative procedures, the CPA was satisfied about the
reasonableness of the account balances. Hence, no scope limitation exists and the
CPA may express an unmodified opinion. The auditor’s report need not refer to
the omission of the confirmation procedures or the performance of alternative
procedures.
Answers A and B are incorrect because no scope limitation exists and the CPA has
not departed from the PSAs.
Answers D is incorrect because there is no basis given to add emphasis of
matter paragraph in the auditor’s report.
56. In which of the following situations would an auditor ordinarily choose between
expressing a qualified opinion or an adverse opinion?

A. The auditor wishes to emphasize an unusually important subsequent event.


B. The financial statements fail to disclose information that is required by Philippine
Financial Reporting Standards.
C. Events disclosed in the financial statements cause the auditor to have substantial doubt
about the entity’s ability to continue as a going concern.
D. The auditor did not observe the entity’s physical inventory and is unable to become
satisfied as to its balance by other auditing procedures.

A disagreement with management concerning the


acceptability of the accounting policies selected, the method of their
application, or the adequacy of financial statement disclosures may
lead to either a qualified or an adverse opinion.
Answer A is incorrect because the auditor may include an Emphasis of
Matter paragraph in his/her unmodified auditor’s report to emphasize an
unusually important subsequent event.
Answer C is incorrect because the auditor, without qualifying the opinion,
should add an Emphasis of Matter paragraph to highlight a material matter
regarding a going concern problem.
Answer D is incorrect because a scope limitation does not lead to an adverse
opinion.

57. Under which of the following circumstances would a disclaimer of opinion not be
appropriate?

A. The financial statements fail to contain adequate disclosure concerning related party
transactions.
B. The auditor is engaged after fiscal year-end and is unable to observe the physical
inventories or apply alternative procedures to verify their balances.
C. The auditor is unable to determine the amounts associated with fraud committed by the
client’s management.
D. The client refuses to permit its attorney to furnish information requested in a letter of audit
inquiry.

A disclaimer of opinion should be expressed when the


possible effect of a limitation on the scope of the auditor’s work is so
material and pervasive that the auditor has not been able to obtain
sufficient appropriate audit evidence and, accordingly, is unable to
express an opinion on the financial statements. Inadequate disclosure
concerning related party transactions is not a scope of limitation.
58. When a publicly held company refuses to include in its audited financial
statements any of the segment information that the auditor believes is required,
the auditor should express a/an

A. Disclaimer of opinion because of the significant scope limitation.


B. Adverse opinion because of a significant uncertainty.
C. Unmodified opinion with an Emphasis of Matter paragraph emphasizing the matter.
D. Qualified opinion because of inadequate disclosure.

The auditor should express either a qualified or an adverse opinion if


material information that the auditor believes is required to be disclosed is not
disclosed in the financial statements.
Answer A is incorrect because inadequate disclosure is a departure from PFRS, not a
scope limitation.
Answer B is incorrect because the auditor should consider adding an
Emphasis of Matter paragraph if there is a significant uncertainty. However, this
does not ordinarily result in an opinion modification.
Answer C is incorrect because inadequate disclosure leads to either a qualified
opinion or an adverse opinion.

59. When a client will not permit inquiry of outside legal counsel, the audit report will
ordinarily contain a/an

A. Disclaimer of opinion.
B. Adverse opinion.
C. “Subject to” qualified opinion.
D. Unmodified opinion with an Emphasis of Matter paragraph.

A limitation on the scope of auditor’s work, whether imposed by the client


of by circumstances, will lead to a qualified opinion or a disclaimer of opinion.
Answer B is incorrect because a scope limitation will never lead to an adverse
opinion.
Answer C is incorrect because the standard clearly states that a qualified
opinion should be expressed as being ‘except for’ the effects of the matter to
which the qualification relates.
Answer D is incorrect because a scope limitation results in a qualified opinion
or a disclaimer of opinion.
60. Which of the following phrases would an auditor most likely include in the auditor’s
report when expressing a qualified opinion because of inadequate disclosure.

A. Do not present fairly in all material respects.


B. Except for the omission of the information included in the Basis for Qualified Opinion
paragraph.
C. With the foregoing explanation of these omitted procedures.
D. Subject to the departure from PFRS, as described above.

Inadequate disclosure, if material to the financial statements taken as a


whole, is a departure from PFRS and will lead to either a qualified or an adverse
opinion.
An auditor’s report qualified for inadequate disclosure should include an
emphasis of matter paragraph preceding the opinion paragraph that explains the
reason for the qualification.
The opinion paragraph states, “In our opinion, except for the omission of
the information included in the Basis for Qualified Opinion paragraph. The
financial statements present fairly…”
Answer A is incorrect because the phrase “do not present fairly” is used when the
auditor expresses an adverse opinion.
Answer C and D are incorrect because the phrases “with the foregoing
explanation” and “subject to” should not be used when expressing qualified
opinion.
61. An auditor’s report includes the following statement: “In our opinion, because of
the effects of the matters discussed in the Basis for Adverse Opinion paragraph,
the financial statements do not present fairly, in all material respects, the financial
position of ABC Company as of December 31, 20X1, and of its financial
performance and its cash flows for the year then ended in accordance with
Philippine Financial Reporting Standard.” This auditor’s report contains a/an

A. Adverse opinion with an Emphasis of Matter paragraph.


B. Adverse opinion with an Other Matter paragraph.
C. Adverse opinion with an Explanatory paragraph.
D. Adverse opinion with a Basis for Adverse Opinion paragraph.
A disagreement with management regarding the acceptability of the
accounting policies selected, the method of their application or the adequacy of
financial statement disclosures could lead to a qualified or an adverse opinion.
The auditor should express an adverse opinion when the effect of a
disagreement is so material and pervasive to the financial statements that a
qualification is not adequate to disclose the misleading or incomplete nature of
the financial statements. The statement quoted which includes the phrase “do not
present fairly” expresses an adverse opinion.

62. In which of the following circumstances would an auditor be most likely to


express an adverse opinion?

A. The financial statements are not in conformity with the Philippine Accounting
Standards (PAS) on capitalization of leases.
B. Tests of controls show that the entity’s internal control is so poor that it cannot be
relied upon.
C. The Chief Executive Officer refuses the auditor access to minutes of board of
directors’ meetings.
D. Information comes to the auditor’s attention that raises substantial doubt about
the entity’s ability to continue as a going concern.
The auditor should express an adverse opinion when the financial statements
are not presented fairly in accordance with Philippine Financial Reporting
Standards.
Answer B is incorrect because an ineffective internal control system is not a
basis for an adverse opinion.
Answer C is incorrect because a limitation on the scope of the auditor’s work
could lead to a qualified opinion or a disclaimer of opinion.
Answer D is incorrect because significant doubt about the entity’s continued
existence, if adequately disclosed in the financial statements, does not result in the
modification of the opinion but requires an Emphasis of Matter paragraph
following the opinion paragraph.

63. An auditor should disclose the substantive reasons for expressing an


adverse opinion in the Basis for Adverse Opinion paragraph.

A. Following the opinion paragraph.


B. Preceding the opinion paragraph.
C. Following the introductory paragraph.
D. Within the notes to the financial statements.
When the auditor’s report contains an adverse opinion, the opinion paragraph
refers to the Basis for Adverse Opinion paragraph(s) preceding the opinion
paragraph. This paragraph should contain a clear description of the substantive
reasons for the adverse opinion and, unless impracticable, a quantification of
the possible effect(s) on the financial statements.
64. When an auditor expresses an adverse opinion, the opinion paragraph
should include

A. The principal effects of the departure from the requirements of the PFRS.
B. The substantive reasons for the financial statements being misleading.
C. A direct reference to a separate paragraph disclosing the basis for the opinion.
D. A description of the uncertainty or scope limitation that prevents an unmodified
opinion.
When the auditor expresses an adverse opinion, the opinion paragraph of the
auditor’s report should include a direct reference to the Basis for Adverse
Opinion paragraph(s) preceding the opinion paragraph that clearly states all
the substantive reasons for the adverse opinion.
Answers A and B are incorrect because the principal effects of the departure
from the requirements of the PFRS and all the substantive reasons for the
adverse opinion are disclosed in the Basis for Modified Opinion paragraph(s).
Answer D is incorrect because an uncertainty or a scope limitation does not lead to an
adverse opinion.
65. There are two broad financial reporting frameworks for comparatives: the
corresponding figures and the comparative financial statements. Which of the
following statements is correct concerning these reporting frameworks?

A. Under the corresponding figures framework, the corresponding figures for the
prior period(s) are integral part of the current period financial statements.
B. Under the corresponding figures framework, the corresponding figures for the
prior period(s) are considered separate financial statements.
C. Under the comparative financial statements framework, the comparative
financial statements for the prior period(s) are intended to be read in conjunction
with the amounts and other disclosures relating to the current period.
D. Under the comparative financial statements framework, the amounts and other
disclosures for the prior period(s) form part of the current period financial
statements.
Under PSA 710 (Comparative Information – Corresponding Figures and
Comparative Financial Statements), the two financial reporting frameworks
are:
Corresponding figures where amounts and other disclosures for the preceding
period are included as part of the current period financial statements, and are
intended to be read in relation to the amounts and other disclosures relating to
the current period.
These corresponding figures are not presented as complete financial
statements capable of standing alone, but are an integral part of the current
period financial statements intended to be read only in relationship to the
current period figures.
Comparative financial statements where amounts and other disclosures for the
preceding period are included for comparison with the financial statements of
the current period, but do not form part of the current period financial
statements, the comparative financial statements are considered separate
financial statements.

66. The following statements relate to the auditor’s reporting responsibilities


regarding comparative information. Which is/are correct?

I. For corresponding figures, the auditor’s report only refers to the financial
statements of the current period.
II. For comparative financial statements, the auditor’s report refers to each
period that financial statements are presented.

A. I only.
B. II only.
C. Both I and II.
D. Neither I nor II.
Because the corresponding figures are included as part of the current period
financial statements and are intended to be read in conjunction with the
amounts and other disclosures relating to the current period, the auditor’s
report only refers to the financial statements of the current period.
But because the comparative financial statements are considered separate
financial statements, the auditor’s report refers to each period that financial
statements are presented.

67. PSA 710 states that the extent of audit procedures performed on the
corresponding figures is significantly less that for the audit of the current period
figures. The auditor’s procedures are ordinarily limited to ensuring that the
corresponding figures have been correctly reported and are appropriately
classified, the auditor should assess whether

I. Accounting policies used for the corresponding figures are consistent


with those of the current period or whether appropriate adjustment and/or
disclosures have been made.
II. Corresponding figures agree with the amounts and other disclosures
presented in the prior period or whether appropriate adjustments and/or
disclosures have been made.
A. I only.
B. II only.
C. Both I and II.
D. Neither I nor II.

68. In which of the following circumstances would an auditor’s report least


likely include specific reference to the corresponding figures?

A. When the auditor’s report on the prior period, as previously issued,


included a modified opinion and the matter which gave rise to the
modification is resolved and properly dealt with the financial statements.
B. When the auditor’s report on the prior period, as previously issued,
included a modified opinion and the matter which gave rise to the
modification is unresolved, and results in a modification of the auditor’s
report regarding the current period figures.
C. When the auditor’s report on the prior period, as previously issued,
included a modified opinion and the matter which gave rise to the
modification is unresolved, but does not result in a modification of the
auditor’s report regarding the current period figures.
D. When the auditor’s report on the prior period financial statements
containing a material misstatement included an unmodified opinion and
the prior period financial statements have not been revised and reissued,
and the corresponding figures have not been properly restated and/or
appropriate disclosures have not been made.
Par. A3 of PSA 710 states, “When the auditor’s report on the prior period, as
previously issued, included a qualified opinion, a disclaimer of opinion, or an
adverse opinion is resolved and properly accounted for or disclosed in the
financial statements in accordance with the applicable financial reporting
framework, the auditor’s opinion on the current period need not refer to the
previous modification.”

69. According to PSA 710, the incoming auditor may refer to the predecessor
auditor’s report on the corresponding figures in the incoming auditor’s report for
the current period. The incoming auditor’s report should indicate

I. That the financial statements of the prior period were audited by the
predecessor auditor.
II. The type of opinion issued by the predecessor auditor.
III. The date of the predecessor auditor’s report.
A. I and II only.
B. II and III only.
C. I and III only.
D. I, II and III.

70. When the prior period financial statements are not audited, the incoming
auditor should state in the auditor’s report that

I. The corresponding figures are unaudited.


II. The incoming auditor is not required to perform procedures regarding opening
balances of the current period.
A. I only.
B. II only.
C. Both I and II.
D. Neither I nor II.
PSA 710 states, “If the prior period financial statements were not audited, the
auditor shall state in an Other Matter paragraph in the auditor’s report that the
corresponding figures are unaudited. Such a statement does not, however,
relieve the auditor of the requirement to obtain sufficient appropriate audit
evidence that the opening balances do not contain misstatements that
materially affect the current period’s financial statements.”

71. J, CPA, audited ABC Company’s prior-year financial statements. These


statements are presented with those of the current year for comparative purposes
without J’s auditor’s report, which expressed a qualified opinion. In drafting the
current year’s auditor’s report, S, CPA, the incoming auditor, should

I. Not name J as the predecessor auditor.


II. Indicate the type of opinion issued by J.
III. Indicate the substantive reasons for J’s qualification.
IV. Indicate the date of J’s auditor’s report.

A. I, II and IV only.
B. II, III and IV only.
C. I, II and III only.
D. I, II, III and IV.
PSA 710 states that when the financial statements of the prior period were
audited by another auditor,

a) the predecessor auditor may reissue the audit report on the prior period with the
incoming auditor only reporting on the current period; or
b) the incoming auditor’s report shall indicate:
i. that the financial statements of the prior period were audited by the
predecessor auditor;
ii. the type of opinion issued by the predecessor auditor and, if the
opinion was modified, the reasons therefor;
iii. the date of that report.
It should be emphasized that the requirement in (b) above is to state the prior
year’s financial statements were audited by the predecessor auditor, not to
name the predecessor auditor.

72. When comparative financial statements are presented, the auditor’s


opinion on the financial statements shall refer to

A. Current period only.


B. Current period and those of the other periods presented.
C. Current and immediately preceding period only.
D. Periods presented plus on preceding period.
PSA 710 provides that when comparative financial statements are presented,
the auditor’s opinion shall refer to each period for which financial statements
are presented an on which the audit opinion is expressed.

73. Comparative financial statements include in the financial statements of the


prior year that were audited by a predecessor auditor whose report is not
presented, if the predecessor’s report was qualified, the incoming auditor should

A. Express an opinion only on the current year’s statements and make no reference
to the prior year’s statements.
B. Issue an updated comparative audit report indicating the division of
responsibility.
C. Request the client to reissue the predecessor’s report on the prior year’s
statements.
D. Indicate the substantive reasons for the qualification in the predecessor
auditor’s opinion.
When the predecessor auditor’s report is not presented, the incoming auditor’s report
shall indicate:

i. that the financial statements of the prior period were audited by the predecessor
auditor;
ii. the type of opinion issued by the predecessor auditor and, if the opinion was
modified, the substantive reasons therefor; and
iii. the date of the report.
Answer A is incorrect because, when comparative financial statements are
presented, the auditor’s opinion (or disclaimer of opinion) should be expressed
individually on the financial statements of each period presented.
Answer B is incorrect because the standard does not require indication of divided
responsibility.
Answer C is incorrect because, although the predecessor auditor may reissue
the audit report on the prior period with the incoming auditor reporting only
on the current period, the situation in the question is that the predecessor
auditor’s report is not presented.
74. Comparative financial statements include the prior year’s statements that
were audited by a predecessor auditor whose report is not presented. If the
predecessor’s report was unmodified, the incoming auditor should

A. Indicate in the auditor’s report that the predecessor auditor expressed an


unmodified opinion.
B. Express an opinion on the current year’s statements alone and make no
reference to the prior year’s statements.
C. Obtain a letter of representations from the predecessor auditor concerning any
matters that might affect the incoming auditor’s opinion.
D. Request the predecessor auditor to reissue the prior year’s report.
Answer B is incorrect because, according to PSA 710, when comparative
financial statements are presented, the auditor should issue a report in which
the comparative financial statements are specifically identified. The auditor’s
opinion (or disclaimer of opinion) should be expressed individually on the
financial statements of each period presented.
Answer C is incorrect because the incoming auditor is required to
communicate with the predecessor auditor prior to accepting the audit
engagement.
Answer D is incorrect because, although the predecessor auditor’s report may
be reissued, the question states that the predecessor auditor’s report is not
presented.

75. The predecessor auditor, who is satisfied after properly communicating


with the incoming auditor, has reissued his/her auditor’s report on prior year
financial statements. The predecessor auditor’s report should

A. Refer to the work of the incoming auditor in the scope and opinion paragraphs.
B. Refer to the report of the incoming auditor only in the scope paragraph.
C. Refer to both the work and the report of the incoming auditor only in the
opinion paragraph.
D. Not refer to the report of the work of the incoming auditor.
PSA 710 states that when the financial statements of the prior period were
audited by another auditor, the predecessor auditor may reissue that audit
report on the prior period with the incoming auditor only reporting on the
current period. The reissued report should not refer to the report or work of the
incoming auditor.
Answers A, B, and C are incorrect because the reissued report should not refer
to the report or the work of the incoming auditor.

76. An auditor expressed a qualified opinion on the prior year’s financial


statements because of a lack of adequate disclosure. These financial statements
are properly restated in the current year and presented in comparative form with
the current year’s financial statements. The auditor’s updated report on the prior
year’s financial statements should

A. Make no reference to the type of opinion expressed on the prior year’s financial
statements.
B. Express an unmodified opinion on the restated financial statements of the prior
year.
C. Be accompanied by the auditor’s original report on the prior year’s financial
statements.
D. Continue to express a qualified opinion on the prior year’s financial statements.
During the course of the audit of the current period, the auditor may become
aware of circumstances or events that materially affect the financial statements
of a prior period. For example, if the auditor expressed a qualified opinion on
the prior year financial statements because of the departure form PFRS and
such statements are restated in the current period to conform with the
requirements of the PFRS, the continuing auditor’s updated report should refer
to the restatement and express an unmodified opinion. Moreover, the report
should contain an Other Matter paragraph to disclose the substantive reasons
for the different opinion.
Answer A is incorrect because the auditor’s report should contain an Other
Matter paragraph to disclose the substantive reasons for the change and the
type of opinion previously expressed by the auditor.
Answer C is incorrect because the original auditor’s report should not be reissued.
Answer D is incorrect because the auditor should express a different opinion.

77. In performing the audit on the current period financial statements, the
incoming auditor, in certain unusual circumstances, may become aware of
material misstatement that affects the prior period financial statements on which
the predecessor auditor had previously reported without modification. In these
circumstances, the incoming auditor should.

I. Discuss the matter with management.


II. Request that the predecessor auditor be informed.
A. I only.
B. II only.
C. Both I and II.
D. Neither I nor II.
According to PSA 710, “If the auditor concludes that a material misstatement
exists that affects the prior period financial statements on which the
predecessor auditor had previously reported without modification, the auditor
shall communicate the misstatement with the appropriate level of management
and those charged with governance and request that the predecessor auditor be
informed. If the prior period financial statements are amended, and the
predecessor auditor agrees to issue a new auditor’s report on the amended
financial statements of the prior period, the auditor shall report only on the
current period.”

78. The following statements relate to unaudited prior year financial statements that
are presented in comparative form with audited current year financial statements.
Which is

incorrect?

A. The incoming auditor should state in the auditor’s report that the comparative
financial statements are unaudited.
B. The incoming auditor need not perform audit procedures regarding opening
balances of the current period.
C. Clear disclosure in the financial statements that the comparative financial
statements are unaudited is encouraged.
D. In situations where the incoming auditor identifies that the prior year unaudited
figures are materially misstated, the auditor should request management to revise
the prior year’s figures or if management refuses to do so, appropriately modify the
report.
According to PSA 710, a statement in the auditor’s report that the prior period
financial statements are unaudited does not relieve the auditor of the
requirement to carry out appropriate procedures regarding opening balances of
the current period.
79. A client is presenting comparative (two-year) financial statements. Which of the
following is correct concerning reporting responsibilities of a continuing auditor?

A. The auditor may issue either one audit report on both presented year, or two audit
reports, one on each year.
B. The auditor should issue on audit report, but only on the most recent year.
C. The auditor should issue two audit reports, one on each year.
D. The auditor should issue on audit report that is on both presented years.
When comparative financial statements are presented, the auditor should issue
a report in which the comparatives are specifically identified because the
auditor’s opinion is expressed individually on the financial statements of each
period presented.

80. When audited financial statements are presented in a document (e.g., annual
report) containing other information, the auditor

A. Should read the other information to consider whether it is inconsistent with the
audited financial statements.
B. Has no responsibility for the other information because it is not part of the basic
financial statements.
C. Has an obligation to perform auditing procedures to corroborate the other
information.
D. Is required to express a qualified opinion if the other information has a material
misstatement of fact.

81. An auditor concludes that there is a material inconsistency in the other


information in an annual report to shareholders containing audited financial
statements. If the auditor concludes that the financial statements do not require
revision, but the client refuses to revise or eliminate the material inconsistency,
the auditor may

A. Disclaim an opinion on the financial statements after explaining the material


inconsistency in an emphasis of matter paragraph.
B. Revise the auditor’s report to include an Other Matter paragraph describing the
material inconsistency.
C. Express a qualified opinion after discussing the matter with the client’s directors.
D. Consider the matter closed because the other information is not in the audited
statements.

82. PSA 720 (The Auditor’s Responsibility in Relation to Other Information in


Documents Containing Audited Financial Statements) states, “If, on reading the
other information, the auditor identifies a material inconsistency, the auditor
should determine whether the audited financial statements or the other
information needs to be revised.” What type of opinion should be expressed if the
client refuses to make the necessary revision in the financial statements?

A. Disclaimer of opinion.
B. Qualified opinion or disclaimer of opinion.
C. Unmodified opinion with an Other Matter paragraph describing the material
inconsistency. D. Qualified or adverse opinion.

83. Which of the following phrases would an auditor most likely include in the
auditor’s report when expressing a qualified opinion because of inadequate
disclosure?

A. Subject to the departure from generally accepted accounting principles, as


described above.
B. With the foregoing explanation of these omitted disclosures.
C. Except for the omission of the information discussed in the Basis for Qualified
opinion paragraph.
D. Do not present fairly in all material respects.

The Qualified Opinion paragraph states. “In our opinion, except for the omission
of information discussed in the Basis for Qualified Opinion paragraph, the
financial statement present fairly.
Answers A and B are incorrect. When the auditor expresses a qualified opinion. It
would not be appropriate to use phrases such as “with the foregoing explanation”
or “subject to” in the opinion paragraph as these are not sufficiently clear or
forceful. (PSA 705 (Revised and Redrafted), par. A22)
Answer D is incorrect because the phrase “do not present fairly” is used when
expressing an adverse opinion.

84. ABC CO.’s financial statements adequately disclose uncertainties that concern
future events, the outcome of which are not susceptible to reasonable estimation.
The auditor’s report should include

A. An unmodified opinion
B. A “subject to” qualified opinion
C. An “except for” qualified opinion
D. An adverse opinion

AASC ALERT Series


2 of 2011
reporting on information required by BIR Revenue Regulation (RR; 15-2010)
85. RR 15-2010 requires disclosure of specific information on various taxes in the
Notes to Financial statements that will accompany the income tax return to be
filed with the BIR. These disclosure requirements

A. Form part of the disclosure requirements under PFRS


B. Form part of the disclosure requirements under PFRS for SMEs
C. Form part of the disclosure requirements under PFRS and PFRS for SMEs
D. Do not form part of the disclosure requirements under PFRS and other Philippine
financial reporting frameworks such as PFRS for SMEs

86. Under the PSAs, the tax information required by RR 15-2010 that is presented as
part of the notes to financial statements is considered

A. Significant information
B. Other information
C. Supplementary information
D. Material information

Supplementary information, as defined in the glossary of the PSAs is


“information that is presented together with the financial statements that is not
required by the applicable financial reporting framework used to prepare the
financial statements, normally presented in either supplementary schedules or as
additional notes.”
The supplementary information may be different from the financial statements by:

• Disclosing the supplementary information in a separate note at the end of


the required notes in the financial statements which is clearly labeled as
“Supplementary information.”
• Removing any cross references from the financial statements to the
separate note on supplementary information.

87. The tax information disclosures under RR 15-2010 are required to be presented
A. Only in the consolidated financial statements.
B. Both in the consolidated financial statements and the separate financial statements
of the parent company and its subsidiaries.
C. Only in the consolidated financial statements and the separate financial statements
of the parent company.
D. Only in the separate financial statements of the parent company and the separate
financial statements of the subsidiaries.
The consolidated financial statements are not the financial statements to be
submitted to the BIR accompanying the income tax returns. Hence, the required
disclosures under RR 15-2010 are to be presented in the separate financial
statements only.
88. Where the supplementary information required by RR 15-2010 is not clearly
differentiated from the financial statements, such supplementary information shall
be

A. Addressed in a separate section in the auditor’s report under the sub-title “Report
on Other Legal and Regulatory Requirements.”
B. Addressed in a separate section in the auditor’s report under the sub-title “Report
on the Supplementary Information Required under RR 15-2010.”
C. Addressed in an emphasis-of-matter paragraph in the auditor’s report.
D. Covered by the auditor’s opinion on the financial statements.

89. When the supplementary information required under RR 15-2010 is not presented

A. The auditor’s “Report on the Financial Statements.” Would not be affected because
such supplementary information is not part of the basic financial statements.
B. The auditor is precluded from expressing an opinion on the financial statements.
C. The auditor’s report should contain a qualified opinion.
D. The auditor’s report should contain an adverse opinion.

90. The supplementary information required under RR 15-2010 is clearly


differentiated from the audited financial statements. How would the “Report on
the Supplementary Information” be affected if the auditor’s “Report on the
financial statements” contains an adverse opinion?

A. The auditor should express a qualified opinion on the supplementary information.


B. To attain consistency in reporting, the auditor should express an adverse opinion on
the supplementary information.
C. The auditor is precluded from expressing an opinion on the supplementary
information.
D. The auditor should express an unmodified opinion on the supplementary
information when his/her “Report on the Financial Statements” contains either an
adverse opinion or a disclaimer of opinion.

The auditor is precluded from expressing an opinion on the supplementary


information when his/her “Report on the Financial Statements” contains either an
adverse opinion or a disclaimer of opinion.
The auditor’s report on the supplementary information should state that, “Because
of the significance of the matter described in the Basis for Adverse (or Disclaimer
of) opinion paragraph, it is inappropriate to and we do not express as opinion on
the information referred to above.”
TRUE OR FALSE
1. PSA 700 (Forming an Opinion and Reporting on Financial Statements) requires
that the audit report must be titled and that the title must include the word
“independent”.
2. An auditor may be unable to express an unmodified opinion if an immaterial
departure from PFRS is present in the financial statements.
3. If financial statements fail to disclose a material fact, the auditors may disclose
the information in an emphasis of matter paragraph and issue an unmodified
opinion on the statements.
4. The opening paragraph of the standard unmodified audit report states that the
audit is designed to obtain reasonable assurance whether the statement are free of
material misstatement.
5. The client’s management is responsible for the fairness of the representation made
in financial statements.
6. The audit report date on an unmodified report indicates the last day of the
auditor’s responsibility for the review of significant events that occurred
subsequent to the date of the financial statements.
7. When the audited financial statements of the prior year are presented together
with those of the current year, the continuing auditor’s report should cover only
the current year.
8. If financial statements contain a material departure from PFRS, the auditor
usually should not issue an unmodified opinion.
9. The introductory paragraph of the unmodified audit report states that the financial
statements are the responsibility of management.
10. If the auditor believes that there is minimal likelihood that resolution of an
uncertainty will have a material effect on the financial statements, the auditor
would issue an “except for” qualified opinion.
11. When there is a significant question about a company’s ability to continue as a
going concern, the report issued is usually modified with an emphasis of matter
paragraph.
12. If the statement of financial position of a company is dated December 31, 2015,
the audit report is dated February 12, 2016, and both are released on February 25,
2016, this indicated that the auditor has searched for subsequent events that
occurred up to February 12, 2016.
13. The adverse effects of events causing an auditor to believe there is substantial
doubt about an entity’s ability to continue as a going concern would likely be
mitigated by evidence relating to the feasibility of plans to purchase leased
equipment at less than market value.
14. The auditor is required to issue a disclaimer of opinion in the event of a going
concern problem.
15. Audit report issued for financial statements of a company should refer to PFRS in
the auditor’s responsibility paragraph.
16. The audit report is normally addressed to the company’s chief executive officer or
chief financial officer.
17. The auditor is required to issue a disclaimer of opinion in the event of a material
uncertainty.
18. The financial statements most commonly audited by external auditors are the
statement of financial position, the statement of comprehensive income, and the
statement of changes in retained earnings.
19. A client-imposed scope limitation will generally result in an adverse opinion.
20. Whenever an auditor issues a qualified report, he or she must us the term “with
the foregoing explanation” in the opinion paragraph.
21. When an auditor does not confirm material accounts receivable, but is satisfied by
the application of alternative auditing procedures, he normally should issue an
unmodified opinion but disclose elsewhere in the report this departure from
customary procedure.
22. When an auditor expresses an adverse opinion, the opinion paragraph should
include a direct reference to a separate paragraph disclosing the basis for the
opinion.
23. Materiality is an essential consideration in determining the appropriate type of
report under a given set of circumstances.
24. An auditor would issue an adverse opinion if the statements taken as a whole do
not fairly present the financial position and financial performance of the company.
25. The predecessor auditor, after properly communicating with the successor auditor,
has reissued a report because the audit client desires comparative financial
statements. The predecessor auditor’s report should make no reference to the
report or the work of the successor auditor.

KEY ANSWERS

1. B 19. C 37. C 55. C 73. D 2. D 21. D 38. C 56. B 74. A 3. A


21. A 39. D 57. A 75. D 4. D 22. C 40. A 58. D 76. B 5.
D 23. A 41. B 59. A 77. C 6. D 24. B 42. B 60. B 78. B 7.
B 25. B 43. C 61. D 79. D 8. B 26. D 44. A 62. A 80. A 9. B
27. A 45. C 63. B 81. B
10. C 28. D 46. B 64. C 82. D 11. B 29. C 47. C 65. A 83. C 12. A
30. C 48. D 66. C 84. A 13. B 31. A 49. A 67. C 85. D 14.
B 32. B 50. D 68. A 86. C 15. B 33. D 51. B 69. D 87. D 16.
B 34. A 52. C 70. A 88. D 17. C 35. A 53. C 71. D 89. A 18. A
36. B 54. D 72. B 90. C

TRUE OR FALSE

1. True 6. True 11. False 16. False 21. False


2. False 7. False 12. True 17. False 22. True 3. False 8. True 13.
False 18. False 23. True
4. False 9. False 14. False 19. False 24. True
5. True 10. False 15. False 20. False 25. True

Chapter 11
Other reporting responsibilities PSA 800
Special Considerations – Audits of Financial Statements Prepared in Accordance with
Special Purpose Frameworks
1. Financial statement prepared in accordance with financial reporting framework
designed to meet the financial information needs of specific users are referred to
as
A. Special purpose financial statements B. Special
purpose framework
C. General purpose financial statements
D. Specific purpose financial statements
Special purpose financial statements are financial statements prepared in accordance with a
special purpose framework.
Special purpose framework is a financial reporting framework designed to meet
the financial information needs of specific users.
2. The following are examples of special purpose frameworks, except

A. A tax basis of accounting for a set of financial statements that accompany an


entity’s tax return.
B. The cash receipt and disbursements basis of accounting for cash flow information
that an entity may be requested to prepare for creditors.
C. Philippines Financial Reporting Standards (PFRS) promulgated by the Financial
Reporting Standards Council (FRSC).
D. The financial reporting provisions of a contract (for example, a financing
agreements).

The following financial reporting frameworks are often identified as the


applicable financial reporting framework in legislative and regulatory
requirements governing the preparation of general purpose financial statements.

• International Financial Reporting Standards (IFRS) promulgated by the


International Accounting Standards Board (IASB)
• International Public Sector Accounting Standards (IPSAs) promulgated by
the International Public Sector Accounting Standards Board.
• Philippine Financial Reporting Standards (PFRS) promulgated by the
Financial Reporting Standards Council (FRSC).
3. An auditor’s report on financial statements prepared in accordance with the
financial reporting provisions of a contract (that is, a special purpose framework)
to comply with the provisions of that contract should include all of the following,
except

A. An opinion as to whether the financial statements are presented fairly, in all material aspects,
in accordance with the financial reporting provisions of the contract.
B. A statement that indicated the basis of accounting used.
C. An opinion as to whether the basis of accounting used is appropriate under the
circumstances.
D. Reference to the note to the financial statements that describes the basis of presentation.

The auditor’s report on financial statements prepared in accordance with the


financial reporting provisions of a contract should include a statement that
indicated the basis of accounting used or should refer to the note to the financial
statements giving that information.
The opinion should state whether the financial statements are prepared, in all
material respects, in accordance with the identified basis of accounting. There is
no requirement to express an opinion on the propriety of the basis of accounting
used.
The following is an example of an auditor’s report on the complete set of financial
statements prepared in accordance with the financial reporting provisions of a
contract:
We have audited the accompanying financial statements of ABC Company, which
compromise the statement of financial potion as at December 31, 20x1, and the
statement of comprehensive income, statement of changes in equity and statement
of cash flows for the year ended, and a summary of significant accounting policies
and other explanatory information. The financial statements have been prepared
by management based on the financial reporting provisions of Section Z of the
contract dated January 1, 20x1.
4. When an auditor reports on financial statements prepared on an entity’s income tax
basis, the auditor’s report should

A. State the basis of presentation of the financial statements.


B. Disclaim an opinion on whether the statements were examined in accordance with
Philippine Standards on Auditing (PSAs).
C. Not express an opinion on whether the statements are presented in accordance
with the tax basis of accounting used.
D. Include an explanation of how the results of operations differ from the cash
receipt and disbursements basis of accounting.
The auditor’s report should state the basis of accounting used or should refer to
the note to the financial statements giving that information.
Answer B is incorrect because the audit should be conducted in accordance with Philippine
Standards of Auditing (PSAs).

Answer C is incorrect because the auditor should express an opinion on whether


the financial statements are prepared, in all material respects, in accordance with
the identified basis of accounting.

Answer D is incorrect because the auditor’s report only refers to the note to the
financial statements that explains the basis of accounting used.
5. An auditor is reporting in a statement of cash receipt and disbursements. This
statement is best referred to in the opinion paragraph by which of the following
descriptions?
A. “Results of operations arising from cash transactions.”
B. “Cash receipts and disbursements”
C. “Income statement resulting from cash transactions.”
D. “Statement of cash flows.”
The opinion paragraph of a report on a statement of cash receipts and
disbursements states, “In our opinion, the financial statement presents fairly, in all
material respects, the cash receipt and disbursements of ABC Company for the
year ended December 31, 20x1 in accordance with the cash receipts and
disbursements basis of accounting described in Note X.”
6. In an audit of special purpose financial statements, the auditor shall obtain an
understanding of
I. The purpose for which the financial statements are prepared.
II. The intended users.
III. The steps taken by management to determine that the applicable financial
reporting framework is acceptable in the circumstances.
A. I only
B. II and III only
C. I and II only
D. I, II, and III

7. An auditor’s report on financial statements prepared on the cash receipts and


disbursements basis of accounting should include all of the following, except

A. A statement that the audit was conducted in accordance with Philippine Standards on
Auditing.
B. A reference to the note to the financial statements that describes the cash receipts and
disbursements basis of accounting.
C. A statement that the cash receipts and disbursements basis of accounting is not a
comprehensive basis of accounting.
D. An opinion as to whether the financial statements are presented fairly, in all material
respects, in accordance with the cash receipts and disbursements basis of accounting.

Answer A, B and D are incorrect because the statement that the audit was
conducted in accordance with PSAs, a reference to the note to the financial
statements that describes the basis of presentation and an opinion as to whether
the statements are fairly presented should be included in the auditor’s report.
PSA 805
Special Considerations – Audits of Single Financial Statements and Specific
Elements, Accounts or Items of a Financial Statement
8. A CPA is permitted to accept a separate engagement (not in conjunction with an
audit of financial statements) to audit an entity’s

Schedule of Schedule of Accounts


receivable Profit Participation

A. Yes No
B. No Yes C. Yes
Yes
D. No No
An audit engagement to express an opinion on one or more components of a
financial statement (for example, accounts receivable, inventory, or a schedule of
profit participation) may be undertaken as a separate engagement or in
conjunction with an audit of the entity’s financial statements.
9. Which of the following statements is correct with respect to an auditor’s report
expressing an opinion on a specific element on a financial statement?
A. The auditor who was expressed an adverse opinion on the financial statements
as a whole can never express an unmodified opinion on a specific element in
theses financial statements.
B. The materiality determined for a specific element of a financial statement may
be lower than the maturity determined for the entity’s complete set of financial
statements.
C. Such a report can only be issued if the auditor is also engaged to audit the
entire set of financial statements.
D. The attention devoted to the specific element is usually less than it would be if
the financial statements as a whole were audited.

Answer A is incorrect because when an adverse opinion or a disclaimer of opinion


has been expressed on the entire set of financial statements, it would be
appropriate to express an unmodified opinion on the specific element only if:
• The auditor is not prohibited by law or regulation from doing so;
• That opinion is expressed in an auditor’s report that is not published together
with the auditor’s report containing the adverse opinion or disclaimer of
opinion; and
• The specific element does not constitute a major portion of the entity’s
complete set of financial statements.

Answer C is incorrect because an engagement to audit one or more components of


financial statements may be undertaken as a separate engagement or in
conjunction with an audit of the entity’s financial statements.

Answer D is incorrect because a special purpose audit engagement to report on


one or more components of financial statements will ordinarily be more extensive
than if the same component(s) were to be audited in connection with a report on
the financial statements taken as a whole.
10. An auditor may express an opinion on an entity’s accounts receivable balance
even if the auditor has disclaimed an opinion on the financial statements taken as
a whole provided the
A. Report on the accounts receivable is presented separately from the disclaimer of
opinion on the financial statements.
B. Auditor also reports on the current asset portion of the entity’s statement of financial
position.
C. Use of the report on the accounts receivable is restricted.
D. Report on the accounts receivable discloses the reason for the disclaimer of opinion on
the financial statements.
The standard states that when an adverse opinion or disclaimer of opinion on the
entire financial statements has been expressed, the auditor should report on
elements of the financial statements only if those elements are not so extensive as
to constitute a major portion of the financial statements.
Because an engagement to audit a financial statement element does not result in a
report on the financial statements taken as a whole, a separate report should be
presented containing the auditor’s opinion on the financial statement element
audited.
Answer B is incorrect because a report may be presented on one or more financial
statement elements audited.
Answer C is incorrect because the standards does not require restriction on the
distribution of the report on the financial statement element audited.
Answer D is incorrect because a separate report on the element audited should be presented.
Moreover, the report need not describe the reason for the disclaimer of opinion on the
financial statements.
11. When an auditor is requested to express an opinion on the rental and royalty income
of an entity, the auditor may
A. Accept the engagement provided the auditor will comply with relevant ethical
requirements, including those pertaining to independence, relating to financial
statement audit engagement and all PSAs relevant to the audit.
B. Accept the engagement provided distribution of the auditor’s report is limited to the
entity’s management.
C. Not accept the engagement unless also engaged to audit the full financial
statements of the entity.
D. Not accept the engagement because to do so would be tantamount to agreeing to
express a piecemeal opinion.

Answer B is incorrect because the standard does not require restrictions on the distribution and use
of the auditor’s report.
Answer C is incorrect because this engagement may be undertaken as separate
engagement or in conjunction with an audit of the entity’s financial statements.
Answer D is incorrect because the auditor is allowed to express an opinion on one
or more financial statement elements if he/she has not expressed an adverse
opinion or disclaimer of opinion on the financial statements taken as a whole or if
the elements to be reported on are not so extensive as to constitute a major portion
of the financial statements.
12. The following statements are ordinarily included in the separate auditor’s
report on an entity’s compliance with contractual agreements, except

A. “We conduct our audit in accordance with Philippine Standards on Auditing.”


B. “In our opinion, the financial statements of the Company are presented fairly,
in all material respects, in accordance with Philippine Financial Reporting
Standards.”
C. “An audit involves performing procedures to obtain audit evidence about the
amounts and disclosures in the financial statements.”
D. “We believe that the audit evidence we have obtained is sufficient and
appropriate to provide a basis for our opinion.”

The auditor’s report should include a statement that, in the auditor’s opinion, the
entity has complied with particular provisions of the contractual agreement.

PSA 810
Engagements to Report on Summary Financial Statements
13. An auditor may report on summary financial statements that are derived from
complete audited financial statements if the

A. Auditor indicated whether the information in the summary financial statements is


consistent with the audited financial statements from which it was derived.
B. Summary financial statements are distributed only to management and the board of
directors.
C. Auditor describes the additional review procedures performed on the summary
financial statements.
D. Summary financial statements are presented in comparative form with the prior
year’s summarized financial statements.
According to the standard, the auditor’s report on the summary financial statements shall
include the following basic elements:

1. A title clearly indicating it is the report of an independent auditor;

2. An addressee;

3. An introductory paragraph that;

a. Identifies the summary financial statements on which the auditor is


reporting, including the title of each statement included in the
summary financial statements;

b. Identifies the audited financial statements;

c. Refers to the auditor’s report on the audited financial statements,


the date of that report, and the fact that an unmodified opinion is
expressed on the audited financial statements;

d. If the date of the auditor’s report on the summary financial


statements is later than the date of the auditor’s report on the
audited financial statements, states that the summary financial
statements and the audited financial statements do not reflect the
effects of events that occurred subsequent to the date of the
auditor’s report on the audited financial statements; and
e. A statement indicating that the summary financial statements do
not contain all the disclosure required by the financial reporting
framework applied in the preparation of the audited financial
statements, and that reading the summary financial statements is
not a substitute for reading the audited financial statements.

4. A description is management’s responsibility for the summary financial statements, explaining the
management is responsible for the preparation of the summary financial statements in accordance
with the applied criteria.

5. A statement that the auditor is responsible for expressing an opinion on the summary financial
statements based on the procedures required by the PSA.

6. A paragraph clearly expressing an opinion.

7. The auditor’s signature.

8. The date of auditor’s report.

9. The auditor’s address.


14. In the auditor’s report on summary financial statements that are derived from
an entity’s audited financial statements, a CPA should indicate that the

A. CPA has audited and expressed an opinion on the complete financial statements.
B. CPA expressed limited assurance that the financial statements are presented in
accordance with PRFS.
C. Summary financial statements are not fairly presented in all material respects.
D. Summary financial statements are prepared in accordance with special purpose financial
reporting framework.
PSRE 2400
Engagements to Review Financial Statements
15. In a review engagement, the practitioner and the client should agree on the
terms of the engagement. The agreed terms would be recorded in an engagement
letter or other suitable form such as a contract. The engagement letter should
include all of the following, except

A. A provision that the engagement cannot be relied upon to disclose errors, fraud, or illegal acts.
B. A provision that any errors, fraud, or noncompliance with laws and regulations that come to the
practitioner’s attention need not be reported.
C. A sample of the report expected to be rendered.
D. The objective of the service to be performed.
Although a review engagement cannot be relied upon to disclose whether fraud,
errors, or noncompliance with the laws and regulations exist, the engagement
letter should indicate to the client that the practitioner will inform management or
the board of directors of any material matters that will come to the practitioner’s
attention.
Matters that would be included in the engagement letter include:  The objective of the service to be
performed.

• Management’s responsibility for the financial statements.

• The scope of the review, including reference to PSRE 2400.

• Unrestricted access to whatever records, documentation, and other information


requested in connection with the review.

• A sample of the report expected to be rendered.

• The fact that the engagement cannot be relied upon to disclose errors. Illegal acts, or
other irregularities, for example, fraud or defalcations that may exist.

• A statement that the audit is not to be performed and that an audit opinion will not be
rendered. To emphasize that point and to avoid confusion, the practitioner may also
consider pointing out that a review engagement will not satisfy any statutory or third
party requirements for an audit.
16. Which of the following procedures should a practitioner perform during an
engagement to review an entity’s financial statements?

A. Examining cash disbursements in the subsequent period for unrecorded liabilities.


B. Sending bank confirmation letters to the entity’s financial institutions.
C. Obtaining a client representation letter from members of management.
D. Communicating material internal control weaknesses during the assessment of control risk.
A review of financial statements consists principally of inquiries and analytical
procedures. When considered appropriate, the practitioner should obtain written
representations from the members of management who have reasonability for
financial and accounting matters.
Answers A, B and D are incorrect because examining subsequent cash
disbursements, sending confirmation requests to financial institutions, and
communicating material weaknesses in internal control are performed in an audit.
17. Which of the following procedures is a practitioner lease likely to perform during a
review engagement?

A. Comparing the financial statements with anticipated results in budgets and forecasts.
B. Studying the relationships of financial statement elements expected to conform to
predictable patterns.
C. Inquiring of management about actions taken at the board of directors’ meetings.
D. Observing the safeguards over access to and use of assets and records.
18. Which of the following inquiries or analytical procedures ordinarily is
performed in an engagement to review an entity’s financial statements?

A. Inquiries concerning the entity’s procedures for recording and summarizing


transactions.
B. Analytical procedures designed to test the accounting records by obtaining
corroborating evidential matter.
C. Analytical procedures designed to test management’s assertions regarding continued
existence.
D. Inquiries of the entity’s attorney concerning contingent liabilities.
19. Which of the following procedures is usually performed by the practitioner in a
review engagement of an entity?

A. Sending a letter of inquiry to the entity’s lawyer.


B. Confirming a significant percentage of receivables by direct communication with debtors.
C. Comparing the financial statements with statements for comparable prior periods.
D. Communicating material weaknesses in the design or implementation of internal control.
20. Which of the following procedures most likely would not be included in a review
engagement of an entity?

A. Assessing control risk.


B. Considering whether the financial statements are in accordance with PFRS.
C. Obtaining a management representation letter.
D. Inquiring about subsequent events.
21. When providing limited assurance that the financial statements of an entity
require no material modifications to be in accordance with PFRS, the practitioner
should

A. Confirm with the entity’s lawyer that material loss contingencies are disclosed.
B. Understand the accounting principles of the industry in which the entity operates.
C. Develop audit programs to determine whether the entity’s financial statements are fairly
presented.
D. Assess the risk that a material misstatement could occur in a financial statement
assertion.
22. Which of the following would not be included in a practitioner’s report based
upon a review of an entity’s financial statements?

A. A statement that the financial statements are the responsibility of the company’s
management.
B. A statement describing the principal procedures performed.
C. A statement that the review was conducted in accordance with PSA
D. A statement describing the practitioner’s conclusions based upon the results of the review.
The review report includes a statement that the review was conducted in
accordance with the Philippine Standard on Review Engagement 2400. Moreover,
the report indicates that a review consists principally of inquiries and analytical
procedures and provides less assurance than an audit.
23. The date of the review report should

A. Not be earlier than the date on which the financial statements were approved by
management.
B. Be earlier than the date on which the financial statements were approved by management.
C. Coincide with the date of the financial statements.
D. Not be later than the date of the financial statements.
According to the standard, a practitioner should date the review report as of the
date the review is completed, which includes performing procedures relating to
events occurring up to the date of the report. However, since the practitioner’s
responsibility is to report on the financial statements as prepared and presented by
the management, the practitioner should not date the report earlier than the date
on which the financial statements were approved by management.
24. During an engagement to review the financial statements of an entity, a
practitioner becomes aware of a material departure from PFRS. If the practitioner
decides to modify the review report because management will not revise the
financial statements, the practitioner should

A. Express negative assurance on accounting principle not conforming with PFRS.


B. Express positive assurance on accounting principles conforming with PFRS.
C. Express a qualified opinion.
D. Express a qualification of the negative assurance provided or give an adverse statement that
the financial statements are not presented fairly, in all material respects, in accordance with
PFRS.
If the practitioner becomes aware of a material departure from PFRS, he/she
should describe the nature of the departure in a separate paragraph, including,
unless impracticable, a quantification of the possible effect(s) on the financial
statements. In addition, the practitioner should either:

a. Express a qualification of the negative assurance provided in the report;


or

b. When the effect of the departure is so material and pervasive that a


qualification is believed to be inadequate to disclose the misleading or
incomplete nature of the financial statements, give an adverse statement
that the financial statements are not presented fairly, in all material
respects, in accordance with PFRS.
Answer A is incorrect because the practitioner should provide negative assurance
when his/her review procedures did not disclose material departure from PFRS.
The review report should state that nothing has come to the practitioner’s
attention based on the review that caused the practitioner to believe the financial
statements are not presented fairly, in all material respects, in accordance with
PFRS.
25. If there has been a significant limitation on the practitioner’s review of an
entity’s financial statements, the practitioner should describe the limitation and
I. Express q qualification of the negative assurance. II. Provide no
assurance.

A. I only
B. II only
C. Either I or II
D.Neither I or II
The standard provides that if there has been a material scope limitation, the
review report should describe the limitation and either:

a. Express a qualification of the negative assurance provided regarding the


possible adjustments to the financial statements that might have been
determined to be necessary had the limitation not existed; or

b. When the possible effect of the limitation is so significant and pervasive


that the practitioner concludes that no level of assurance can be provided,
not provide any assurance.
26. For the purpose of expressing negative assurance in the review report, the practitioner
should obtain sufficient appropriate evidence primarily through
A. Inquiry and confirmation
B. Analytical procedures and substantive tests of details of transactions and account balances
C. Confirmation and tests of controls
D. Inquiry and analytical procedures
27. PSRE 2400 (Engagements to Review Financial Statements), as amended by the AASC
in February 2008, applies to

A. Reviews of any historical financial information of an audit client.


B. Reviews of any historical financial information by a
practitioner other than the entity’s auditor.
C. Review of historical financial or other information by a
practitioner other than the entity’s auditor.
D. Review of historical financial or other information of an audit client.
PSRE 2400 (Engagement to Review Financial Statements) and PSRE
2410 (Review of Interim Financial Information Performed by the Independent
Auditor of the Entity) were amended by the AASC in February 2008. The
objective of the amendments made is to clarify to which engagements each of the
standards is to be applied. The effect of the amendments is summarized as
follows:

• PSRE 2400 applies to reviews of historical financial information by a practitioner other


than entity’s auditor.

• PSRE 2410 applies to reviews of historical financial information by the entity’s auditor.

• Reviews of other historical information fall under PSAE 3000 (Revised). Assurance
Engagements other than Audits or Reviews of Historical Financial Information.
28. A practitioner’s review of an entity’s financial statements does not provide
assurance that he/she will become aware of all significant matters that would be
disclosed in an audit. However, if the practitioner has become aware that
information coming to his/her attention may be materially misstated, the
practitioner should

A. Carry out additional or more extensive procedures as are necessary to achieve limited
assurance.
B. Withdraw immediately from the engagement.
C. Perform a complete audit and issue modified auditor’s report.
D. Downgrade the engagement to a compilation and issue the appropriate report.
According to PSRE 2400, if the practitioner has reason to believe that the
information subject to review may be materially misstated, he/she should carry
out additional or more extensive procedures as are necessary to be able to express
negative assurance or to confirm that a modified report is required.
PSRE 2410
Review of Interim Financial Information Performed by the independent Auditor of the
Entity

29. Which of the following statements concerning objective of an engagement to


review interim financial information is

correct?

A. To obtain reasonable assurance that the interim financial information is free from
material misstatement.
B. To enable the auditor to express a conclusion whether, on the basis of the review,
anything has come to the auditor’s attention that causes the auditor to believe that the
interim financial information is not prepared, in all material respects, in accordance
with an applicable financial reporting framework.
C. To provide a basis of expressing an opinion whether the interim financial information
is presented fairly, in all material respects, in accordance with an applicable financial
reporting framework.
D. The objective of a review of interim financial information is similar to that of an audit
conducted in accordance with PSAs.
According to PSRE 2410 “The objective of an engagement to review interim
financial information is to enable the auditor to express a conclusion whether, on
the basis of the review, anything has come to the auditor’s attention that causes
the auditor to believe that the interim financial information is not prepared, in all
material respects, in accordance with an applicable financial reporting
framework.”
Answer A is incorrect because a review of interim financial information, in
contrast to an audit, is not designed to obtain reasonable assurance that the interim
financial information is free from material misstatement. A review consists of
making inquiries, primarily of persons responsible for financial and accounting
matters, and applying analytical and other review procedures. It does not provide
all of the evidence that would be required in an audit.
Answer C is incorrect because a review of interim financial information does not
provide a basis for expressing an opinion whether the financial information is
presented fairly, in all material respects, in accordance with an applicable
financial reporting framework.
Answer D is incorrect because the objective of a review of interim financial
information differs significantly from that of an audit conducted in accordance
with PSAs.
30. Which of the following procedures ordinarily should be applied when an
independent auditor conducts a review of interim financial information of an
entity?

A. Verify changes in key account balances.


B. Perform cut-off tests for cash receipts and disbursements.
C. Read the minutes of the board of directs’ meetings.
D. Inspect the open purchase order file.
A review of interim financial information includes reading the minutes of
meetings of shareholders, those charged with governance, and other appropriate
committees to identify matters that may affect the interim financial information. It
also involves inquiring about matters dealt with at meetings for which minutes are
not available that may affect the interim financial information.
Answers A, B, and D are incorrect because verifying changes in key account
balances, performing cut-offs test, and inspecting open purchase order file are
verification procedures ordinarily performed in an audit.
31. An independent auditor who conducts a review of an entity’s interim financial
information should have an understanding of the entity and its environment,
including its internal control, as it relates to the preparation of both annual and
interim financial information. This enables the auditor to

I. Identify the types of potential misstatements and consider the likelihood of their
occurrence.

II. Select the inquiries, analytical and other review procedures.

A. I only
B. II only
C. Both I and II
D. Neither I nor II
31. The following procedures are ordinarily performed in an engagement to review interim
financial information, except

A. Tests of the accounting records through inspection, observation, or confirmation


B. Obtaining an understanding of the entity and its environment, including its
internal control, as it relates to the preparation of both annual and interim
financial information.
C. Inquiring of members of management responsible for financial and accounting
matters.
D. Communication with other auditors who are performing a review of the interim
financial information if the reporting entity’s significant components.
Procedures for performing a review of interim financial information are ordinarily limited to
making inquiries, primarily of persons responsible for financial and accounting matters, and
applying analytical and other reviews procedures. A review ordinarily does not require test of the
accounting records through inspection, observation, or confirmation.
Answer B, C and D are incorrect because they describe procedures that are
ordinarily performed in an engagement to review interim financial information.
The following procedures are ordinarily performed in conducting a review of interim
financial information:

• Reading the minutes of the meetings of shareholders, those charged with


governance, and other appropriate committees to identify matters that may
affect the interim financial information, and inquiring about matters dealt
with at meetings for which minutes are not available that may affect the
interim financial information.

• Considering the effect, if any, of matters giving rise to modification of the


audit or review report, accounting adjustments or unadjusted
misstatements at the time of the previous audit or reviews.
• Communicating, where appropriate, with other auditors who are
performing a review of the interim financial information of the reporting
entity’s significant components.

• Inquiring of members of management responsible for financial and


accounting matters.

• Applying analytical procedures to the interim financial information


designed to identify relationships and individual items that appear to be
unusual and that may reflect a material misstatement in the interim
financial information.

• Reading the interim financial information, and considering whether


anything has come to the auditor’s attention that cause the auditor to
believe that the interim financial information is not prepared, in all
material respects, in accordance with applicable financial reporting
framework.
PSRS 4400
Engagements on Agreed-upon Procedures
33. A report may be based upon applying agreed-upon procedures to specified
elements, accounts, or items of a financial statement. The users of the report
should participate in establishing the procedures to be performed. If the auditor
cannot discuss the procedures with all the parties who will
receive the report, he/she may
I. Discuss the procedures to be applied with appropriate representatives of the parties
involved.
II. Review relevant correspondence from the parties involved.
III. Distribute a draft of the type of report that will be issued to the parties involved.

A. I and II only
B. I and III only
C. II and III only
D. I, II and III
PSRS 4400 (Engagements on Agreed-upon Procedures) states, “In certain
circumstances, for example, when the procedures have been agreed to between
the regulator, industry representatives and representatives of the accounting
profession, the auditor may not be able to discuss the procedures with all the
parties who will receive the report. In such cases, the auditor may consider, for
example, discussing the procedures to be applies with appropriate representatives
of the parties involved, reviewing relevant correspondence from such parties or
sending them a draft of the type of report that will be issued.”
34. The auditor may accept an engagement to perform specified procedures on the
specific subject matter of specified elements, accounts, or items of a financial
statement if

A. The report does not list the procedures performed.


B. The financial statements are prepared in accordance with s special purpose framework.
C. Use of the report is restricted.
D. The auditor is also the entity’s continuing auditor.
PSRS 4400 states that the report is restricted to those parties that have agreed to
the procedures to be performed since others, unaware of the reasons for the
procedures, may misinterpret the results.
Answer A is incorrect because the report should include a listing of the specific
procedures performed.
Answer B is incorrect because the financial statements need not be prepared in
accordance with a special purpose framework.
Answer D is incorrect because the auditor need not be the entity’s continuing auditor.
35. The distribution of which of the following types of reports is unrestricted?

A. Identification of the purpose for which the agreed-upon procedures were performed.
B. An expression of positive assurance based on the specific procedures performed.
C. A statement that the auditor is independent of the entity.
D. A general description of the procedures performed.
According to PSRS 4400, the report on an agreed-upon procedures engagement
needs to describe the purpose and the agreed-upon procedures of the engagement
in sufficient detail to enable the users of the report to understand the nature and
extent of the work performed.
Answer B is incorrect because the report should include a statement that the
procedures performed do not constitute either an audit or a review and, as such,
no assurance is expressed.
Answer C is incorrect because the report should contain a statement that the
auditor is not independent of the entity if such is the case.
Answer D is incorrect because the report should include a listing of the specific
procedures performed.
37. An agreed-upon procedures engagement may involve the auditor in performing
certain procedures concerning

I. Individual items of financial data. II. A financial


statement.
III. A complete set of financial statements.
A. I and II only
B. II and III only
C. I and III only
D. I, II, and III
PSRS 4400 (Engagements to Perform Agreed-upon Procedures Regarding
Financial Information) states, “An engagement to perform agreed-upon
procedures may involve the auditor in performing certain procedures concerning
individual items of financial data (for example, accounts payable, account
receivable, purchases from related parties and sales and profits of a segment of an
entity), a financial statement (for example, a statement of financial position) or
even a complete set of financial statements.”
38. Negative assurance may be expressed when an accountant is engaged to report agreed-
upon procedures to specified

I. Elements of a financial statement. II. Accounts of a


financial statement.
A. I only
B. II only
C. Both I and II
D. Neither I nor II
According to PSRS 4400, the objective of an agreed-upon procedures engagement is for the
auditor to carry out procedures of an audit nature to which the auditor and the entity and any
appropriate third parties have agreed and to report on factual findings. The accountant does not
provide negative or other forms of assurance. Users of the report assess for themselves the
procedures and findings of the accountant and draw their own conclusions.
39. An accountant may accept an engagement to apply agreed-upon procedures that are not
sufficient to express an opinion on one or more financial statements provided that

A. The accountant is also the entity’s continuing auditor.


B. Distribution of the accountant’s report is restricted.
C. The financial statements are prepared in accordance with a special purpose financial reporting
framework.
D. The accountant’s report does not enumerate the procedures performed.

An accountant may accept an agreed-upon procedures engagement provided that


the parties involved have a clear understanding of the procedures to be performed.
This is to prevent misinterpretation of the results by those who are unaware of the
reasons for the procedures performed.
Answer A is incorrect because the accountant need not be the entity’s continuing auditor.
Answer B is incorrect because the financial statements need not be prepared using
the special purpose financial reporting framework.
Answer D is incorrect because the accountant’s report should include a listing of the specific
procedures performed.
40. Which of the following is least likely to be included in an agreed-upon procedures
engagement report?

A. Identification of the purpose for which the agreed-upon procedures were performed.
B. A summary of procedures performed.
C. Limited assurance on the information presented.
D. Use of the report is restricted.

The accountant’s report should include a statement that the procedures performed
do not constitute either an audit or a review and, as such, no assurance is
expressed.
According to PSRS 4400, the report of factual findings should contain:

a. Title;
b. Addressee (ordinarily the client who engaged the auditor to perform the agreed-upon
procedures);
c. Identification of specific financial or non-financial information to which the agreed-
upon procedures have been applied;
d. A statement that the procedures performed were those agreed-upon with the
recipient;
e. A statement that the engagement was performed in accordance with the PSRS
applicable to agreed-upon procedures engagements;
f. A statement that the auditor is not independent if such is the case;
g. Identification of the purpose for which the agreed-upon procedures were performed;
h. A listing of the agreed-upon procedures performed;
i. A description of the auditor’s factual findings including sufficient details of errors
and exceptions found;
j. A statement that the procedures performed do not constitute either an audit or a
review and, as such, no assurance is expressed.
k. A statement that had the auditor performed additional procedures, an audit or a
review, other matters might have come to light that would have been reported;
l. A statement that the report is restricted to those parties that have agreed to the
procedures to be performed;
m. A statement (when applicable) that the report relates only to the elements, accounts,
items or financial and nonfinancial information specified and that it does not extend
to the entity’s financial statements taken as a whole;
n. Date of the report;
o. Auditor’s address; and
p. Auditor’s signature

PSRS 4410
Engagements to Compile Financial Information

41. When performing a compilation engagement, the accountant is required to


A. Assess internal controls.
B. Make inquiries of management to assess the reliability and completeness of the
information provided. C. Verify matters and explanations.
D. Obtain a general knowledge of the business and operations of the entity

According to PSRS 4110 (Engagements to Compile Financial Information), “The


accountant should obtain a general knowledge of the business and operations of
the entity and should be familiar with the accounting principles and practices of
the industry in which the entity operates and with the form and content of the
financial information that is appropriate in the circumstances.”

The standard further provides that, “The accountant ordinarily obtains knowledge
of these matters through experience with the entity or inquiry of the entity’s
personnel.”

PSRS 4110, par. 13 provides that the accountant is not ordinarily required to:
a) make any inquiries of management to assess the reliability and completeness of the information
provided;
b) assess internal controls;
c) verify any matters; or
d) verify any explanations.

42. Independence is a requirement for which of the following engagements?

Compilation Review Agreed-upon Procedures


A. No Yes No
B. No No No
C. Yes No Yes
D. Yes Yes Yes

Independence is not a requirement for compilation and agreedupon procedures


engagements. However, where the accountant or auditor is not independent, a
statement to that effect would be made in the report.
43. Which of the following require compliance with the requirements of the Code of Ethics
for Professional accountants in the Philippines?

Compilation Review Agreed-upon Procedures


A. Yes Yes No
B. No No Yes
C. No No No
D. Yes Yes Yes

44. An accountant who performs a compilation engagement


A. Should read the compiled information and consider whether it appears
to be appropriate in form and free from obvious material
misstatements.
B. Should use his/her auditing expertise in testing the assertions
underlying the compiled financial information.
C. Include in his/her report a listing of the specific procedures performed.
D. Need not to obtain an acknowledgement from management of its
responsibility for the appropriate presentation of financial information.

45. Each page of the financial information compiled by the accountant should include the
following reference, except
A. “Unaudited”
B. “Compiled without Audit or Review”
C. “Refer to Compilation Report”
D. “Compiled, Negative Assurance Expressed”

According to PSRS 4110 (Engagements to Compile Financial Information), the


financial information compiled by the accountant should contain a reference such
as:
• Unaudited;
• Compiled without Audit or Review; or
• Refer to Compilation Report on each page of the financial information or on the front of the complete
set of financial statements.

46. The objective of a compilation engagement is


A. For the accountant to use accounting expertise, as opposed to auditing
expertise, to collect, classify, and summarize financial information.
B. For the auditor to carry out procedures of an audit nature to which the
auditor and the entity and any appropriate third parties have agreed
and to report on factual findings.
C. To enable an auditor to state, on the basis of the procedures which do
not provide all the evidence that would be required in an audit,
anything has come to the auditor’s attention that causes the auditor to
believe that the financial statements are not prepared, in all material
respects, in accordance with an identified financial reporting
framework.
D. For the auditor to provide a high, but not absolute, level of assurance
that the financial information is free of material misstatement.

PSRS 4110 (Engagements to Compile Financial Information) states, “The


objective of a compilation engagement is for the accountant to use accounting
expertise, as opposed to auditing expertise, to collect, classify, and summarize
financial information.” Answer B is incorrect because it describes the objective of
an agreed-upon procedures engagement.

Answer C is incorrect because it describes negative (or limited) assurance provided in a review
engagement.

Answer D is incorrect because an audit provides a high, but not absolute level of
assurance that the financial information is free of material misstatement.

47. When compiling an entity’s financial statements, an accountant would be least likely to:
A. Obtain an acknowledgment from management of its responsibility for the
financial statements.
B. Perform analytical procedures designed to identify relationships that appear to
be unusual. C. Plan the work.
D. Read compiled financial statements and consider whether they appear to
include adequate disclosures.

Analytical procedures are necessary in review and audit engagements; not in compilation
engagements.

Answer A is incorrect because, according to PSRS 4410, the accountant should


obtain an acknowledgement from management of its responsibility for the
appropriate presentation and of its approval of the financial information.

Answer C is incorrect because the work should be planned to ensure that an effective
engagement will be performed.

Answer D is incorrect because the accountant should read the compiled financial
statements and consider they are free from obvious material misstatements,
including:
• Mistakes in the application of PFRS.
• Nondisclosure of PFRS and any known departures thereof.
• Nondisclosure of any other significant matters of which the accountant has
become aware.
48. When compiling the financial statements of an entity, an accountant should
A. Understand the accounting principles and practices of the entity’s industry.
B. Inquire of key personnel concerning related parties and subsequent events.
C. Perform ratio analyses of the financial data of comparable periods.
D. Review agreements with financial institutions for restriction of cash balances.

PSRS 4410 states, “The accountant should obtain general knowledge of the
business and operations of the entity and should be familiar with the accounting
principles and practices of the industry in which the entity operates and with the
form and content of the financial information that is appropriate in the
circumstances.”

Answer B and D are incorrect because inquiries concerning related parties and
subsequent events, and procedures to obtain corroborating evidence about
restriction on cash balances are appropriate in an audit.

Answer C is incorrect because analytical procedures such as ratio analyses are appropriate in
review and audit engagements.

49. Which of the following should not be included in an accountant’s report based
upon the compilation of an entity’s financial statements?
A. A statement that a compilation of the company’s financial statements was made in
accordance with the PSA applicable to compilation engagements.
B. A statement that management is responsible for the financial statements.
C. A statement that the accountant has not audited or review the statements.
D. A statement that the accountant does not express an opinion but provides only a
negative assurance on the statements.

The accountant’s report should indicate that since no audit or review was performed, no
assurance is expressed.

PSRS 4410 gives the following example of a compilation report:

We have compiled the accompanying financial statements of ABC


Company based on information you have provided. These financial statements
comprises the statement of financial position of ABC Company as at December
31, 20X1, the statement of comprehensive income, statement of changes in
equity and statement of cash flows for the year ended, and a summary of
significant accounting policies and other explanatory information.

We performed this compilation engagement in accordance with Philippine


Standard on Related Services 4410, Compilation Engagements.
We have applied our expertise in accounting and financial reporting to
assist you in the preparation and presentation of these financial statements in
accordance with Philippine Financial Reporting Standards. We have compiled
relevant ethical requirements, including principles of
integrity, objectivity, professional competence and due care.

These financial statements and the accuracy and completeness of the


information used to compile them are your responsibility.
Since a compilation engagement is not an assurance engagement, we are
not required to verify the accuracy or completeness of the information you
provided to us to compile these financial statements. Accordingly, we do not
express and audit opinion or a review conclusion on whether these financial
statements are prepared in accordance with
PFRS.

50. In performing a compilation of financial statements of an entity, the


accountant decides that modification of the report is not adequate to
indicate deficiencies in the financial statements taken as a whole, and the
client is not willing to correct deficiencies. The accountant should
therefore
A. Express an adverse audit opinion.
B. Express a qualification of negative assurance.
C. Withdraw from the engagement.
D. Perform a review of the financial statements.

PSRS 4410 States that if the accountant becomes aware that information supplied
by management is incorrect, incomplete, or otherwise unsatisfactory, he/she
should request management to provide additional information or correct that
deficiencies. The accountant should withdraw from the engagement if the
management refuses to do so.

Answers A and B are incorrect because the accountant should not express any
form of assurance on complied financial statements.
Answer D is incorrect because the accountant has no responsibility to upgrade the engagement to a
review.

PSAE 3400
The Examination of Prospective Financial Information

51. “Prospective financial information” means financial information based on


assumptions about events that may occur in the future and possible actions
by the entity. It can be in the form of a projection, a forecast, or a
combination of both. A forecast
A. Presents estimates given one or more hypothetical assumptions.
B. It is based on assumptions reflecting conditions expected to exist and courses of action
expected to be taken.
C. Unlike a projection, may contain range.
D. Is based on the most conservative estimates.

According to PSAE (Philippine Standards on Assurance Engagements) 3400 [The


Examination of Prospective Financial Information], a “forecast” means
prospective financial information prepared on the basis of assumptions as to
future events which management expects to take place and actions management
expects to take as of the date the information is prepared (best-estimate
assumptions).

A “projection”, as defined in the standard, means prospective financial information prepared on


the basis of:
a) Hypothetical assumption about future events and management actions which
are not necessarily expected to take place, such as when some entities are in a
start-up phase or are considering a major change in the nature of operations; or
b) A mixture of best-estimate and hypothetical assumptions.

Answer A is incorrect because, as indicated above, a projection (not a forecast) is


based on hypothetical assumptions about future events and management actions
which are not necessarily expected to take place.

Answer C is incorrect because both forecasts and projections can be expressed in


terms of a range.

Answer D is incorrect because a forecast is based on the entity’s best-estimate assumptions.

52. The party responsible for assumptions identified in the preparation of


prospective financial statements is usually A. The client’s management.
B. The client’s independent auditor.
C. The reporting accountant.
D. A third-party lending institution.

According to PSAE 3400, management is responsible for the preparation and


presentation of prospective financial information, including the identification and
disclosure of the assumptions on which it is based. The auditor examines and
reports on the prospective financial information to enhance its credibility whether
it is intended for use by third parties or internal purposes.

Answers B, C, and D are incorrect because the party responsible for assumptions
identified in preparation of prospective financial information is usually the
entity’s management.
53. Given one or more hypothetical assumptions, a responsible party may
prepare, to the best of its knowledge and belief, an entity’s expected
financial position, result of operations, and cash flows. Such prospective
financial
statements are known as A.
Partial presentations
B. Financial projections
C. Financial forecast
D. Pro forma financial statements

Financial projections are prepared on the basis on hypothetical assumptions which are not
necessarily expected to take place.

54. An examination of a financial forecast is a professional service that


involves
A. Assuming responsibility to update management on key events for one year
after the report’s date.
B. Compiling or assembling a financial forecast that is based on management’s
assumptions.
C. Limiting the distribution of the accountant’s report to management and board
of directors.
D. Evaluating the preparation of a financial forecast and the support underlying
management’s assumption.

Prospective financial information includes financial forecasts and projections. The


examination of prospective financial information requires the accountant to obtain
sufficient appropriate evidence as to whether:
a) Management’s best estimate assumptions on which the prospective
financial information is based are not unreasonable and, in the case of
hypothetical assumption, such assumptions are consistent with the
purpose of the information;
b) The prospective financial information is properly prepared on the basis
of the assumptions.
c) The prospective financial information is properly presented and all
material assumptions are adequately disclosed, including a clear
indication as to whether they are best-estimate assumptions or
hypothetical assumptions; and
d) The prospective financial information is prepared on a consistent basis
with historical financial statements, using appropriate accounting
principles.

Answer A is incorrect because the accountant does not have responsibility as to


update management on key events after the report’s date.
Answer B is incorrect because an examination of a financial forecast entails
evaluation of its preparation and the support underlying management’s
assumptions.

Answer C is incorrect because there is no requirement to restrict distribution of forecasts.

55. The accountant should not accept, or should withdraw from, an


engagement to examine prospective financial information when
I. The assumptions are clearly unrealistic.
II. The accountant believes that the prospective financial information will be inappropriate
for its intended use.

A. I only
B. II only
C. Either I or II
D. Neither I nor II

PSAE 3400 states, “The auditor should not accept, or should withdraw from, an
engagement when the assumptions are clearly unrealistic or when the auditor
believes that the prospective financial information will be inappropriate for the
intended use.”

56. When the examination of prospective financial information is affected by


conditions that preclude application of one or more procedures considered
necessary in the circumstances, the auditor should withdraw from
engagement or
A. Disclaim the opinion
B. Express an adverse opinion
C. Express a qualified opinion
D. Issue an unmodified report
A limitation on the scope of the accountant’s examination of prospective financial
information may lead to either withdrawal of the accountant from the engagement
or disclaimer of opinion.
57. When the accountant believes that the presentation and disclosure of the
prospective financial information is no
adequate, the auditor should
I. Express a qualified or adverse opinion.
II. Withdraw from the engagement.

A. I only
B. II only
C. Either I or II
D. Neither I nor II
PSAE 3400 states, “When the auditor believes that the presentation and disclosure
of the prospective financial information is not adequate, the auditor should
express a qualified or adverse opinion in the report on the prospective financial
information, or withdraw from the engagement as appropriate.”

58. The report on examination of prospective financial information should include


A. A statement that the accountant is responsible for the prospective
financial information, including the assumptions on which it is based.
B. A statement of positive assurance as to whether the assumptions
provide a reasonable basis for the prospective financial information.
C. Appropriate caveats concerning the achievability of the results
indicated by the prospective financial information.
D. A statement that the examination was conducted in accordance with
PSAs.
According to PSAE 3400, the report on an examination of prospective financial information
should include the following:
a) Title;
b) Addressee;
c) Identification of the prospective financial information;
d) A reference to the Philippine Standard on Assurance Engagements applicable to the
examination of prospective financial information;
e) A statement that management is responsible for the prospective financial information
including the assumptions on which it is based;
f) When applicable, a reference to the purpose and/or restricted distribution of the
prospective financial information;
g) A statement of negative assurance as to whether the assumptions provide a reasonable
basis for the prospective financial information;
h) An opinion as to whether the prospective financial information is properly prepared
on the basis of the assumptions and is presented in accordance with the relevant
financial reporting framework;
i) Appropriate caveats concerning the achievability of the results indicated by the
prospective financial information;
j) Date of the report which should be the date the procedures have been completed;
k) Auditor’s address; and
l) Signature.

59. Before accepting an engagement to examine the prospective financial


information, the auditor would consider I. The intended use of the information.
II. The nature of assumptions.
I. The period covered by the information.
A. I only
B. II only
C. I and III only
D. I, II, and III

PSAE 3400 states that before accepting an engagement to examine prospective


financial information, the auditor would consider, among other things:
• The intended use of the information.
• Whether the information will be general or limited distribution.
• The nature of the assumptions, that is, whether they are best-estimate or hypothetical
assumptions.
• The elements to be included in the information.
• The period covered by the information.

60. Which of the following statements concerning an examination of prospective


financial information is incorrect?
A. The auditor should consider the period of time covered by the financial
information.
B. The auditor should obtain a sufficient level of knowledge of the business
to be able to evaluate whether all significant assumptions required for
the preparation of the prospective financial information have been
identified.
C. The auditor need not obtain written representations from management.
D. The auditor should consider the extent to which reliance on entity’s
historical financial information is justified.

According to PSAE 3400, “The auditor should obtain a written representations


from the management regarding the intended use of the prospective financial
information, the completeness of significant management assumptions and
management assumptions and management’s acceptance of its responsibility for
the
prospective financial information.”

TRUE OR FALSE

1. A CPA firm can issue a compilation report only if the engagement partner
has no direct or material indirect financial interest in the client.
2. A compilation of financial statements provides negative assurance regarding
the financial statements.
3. A review engagement is not currently an acceptable form of association with
prospective financial statements.
4. The statement that “Nothing came to our attention which would indicate that
these statements are not fairly presented” expresses a disclaimer of opinion.
5. When a practitioner examines projected financial statements, the
practitioner’s report should include a separate paragraph that describes the
limitation on the usefulness of the presentation.
6. The concept of limited assurance is provided for in agreedupon procedures
engagement.
7. Use of agreed-upon procedures report is restricted to the specified users.
8. An audit review requires less evidence related to internal control than a
review.
9. A practitioner who reviews the financial statements of an entity should issue
a report stating that a review provides only limited assurance that a financial
statements are fairly presented.
10. An accountant’s report issued after compiling the financial statements of an
entity should state that a compilation is limited to presenting in the form of
financial statements information that is the representation of management.
11. Before performing a review of an entity’s financial statements, a practitioner
should obtain a sufficient level of knowledge of the accounting principles
and practices of the industry in which the entity operates.
12. An accountant’s report issued after compiling the financial statements of an
entity should state that a compilation is substantially less scope than an audit
in accordance with PSA, the objective of which is the expression of an
opinion.
13. Negative assurance is not permissible in reports based upon a review
engagement.
14. Compilation reports are normally dated as of the client’s balance sheet date.
15. An accountant’s report issued after compiling the financial statements of an
entity should state that a compilation consists principally of inquiries of
company personnel and analytical procedures.
16. Accepting an engagement to examine an entity’s financial projection most
likely would be appropriate if the projection were to be distributed to a bank
with which the entity is negotiating for a loan.
17. A non-audit engagement in which the accountant undertakes to present, in
the form of financial statements, information that is the representation of
management, without undertaking to express any assurance on the
statements is called an agreed-upon procedures engagement.
18. A CPA must be independent to issue a review report.
19. A practitioner’s report on agreed-upon procedures that is in the form of
procedures and findings should contain an acknowledgement of the
practitioner’s responsibility for the sufficiency of the procedures.
20. When performing compilation services, the accountant is required to obtain
an understanding of the client’s internal control.
21. An agreed-upon procedures engagement is one in which the auditor and
management or a third party agree that the auditor will apply his/her
judgement to determine procedures to be performed.
22. The CPA may issue a report on whether the summary financial statements
derived from the audited financial statements are consistent, in all material
respects, with those financial statements, in accordance with PFRS.
23. When an accountant compiles a financial forecast, the accountant’s report
should include a caveat that the prospective results of the financial forecast
may not be achieved.
24. General use statements are prepared for use by known contractual parties.
25. Financial projections can be provided for general use.

KEY ANSWERS

1. A 19. C 37. D
2. C 20. A 38. D 3. C 21. B 39. B 4. A 22. C 40. C 5. B 23.
A 41. D 6. D 24. D 42. A 7. C 25. C 43. D 8. C 26. D 44. A 9. B
27. B 45. D
10. A 28. A 46. A 11. A 29. B 47. B 12. B 30. C 48. A 13. A
31. C 49. D
14. A 32. A 50. C 15. B 33. D 51. B 16. C 34. C 52. A 17. D
35. D 53. B
18. A 36. A 54. D
55. C 5. True 18. True
56. A 6. False 19. False
57. C 7. True 20. False
58. C 8. False 21. False
59. D 9. False 22. True
60. C 10. True 23. True
11. True 24. False
TRUE OR FALSE 12. False 25. False
13. False
1. False 14. False
2. False 15. False
3. True 16. True
4. False 17. False

You might also like